You are on page 1of 543

% HAESE MATHEMATICS

dINIOM
Mathematics
Core Topics SL
for use with
Mathematics: Analysis and Approaches SL
Mathematics: Applications and Interpretation SL

for use with SNOLLNTOS


IB Diploma Programme

Charlotte Frost Joseph Smuall


Bradley Stevenion Michael Mampusti
MATHEMATICS: CORE TOPICS SL WORKED SOLUTIONS

Charlotte Frost B.Sc.


Bradley Steventon B.Ma.Sc.
Joseph Small B.Ma.Sc.
Michael Mampusti B.Ma.Adv.(Hons.)

Haese Mathematics
152 Richmond Road, Marleston, SA 5033, AUSTRALIA
Telephone: +61 8 8210 4666, Fax: +61 8 8354 1238
Email: info@haesemathematics.com
Web: www.haesemathematics.com

National Library of Australia Card Number & ISBN 978-1-925489-82-8


© Haese & Harris Publications 2019
Published by Haese Mathematics.
152 Richmond Road, Marleston, SA 5033, AUSTRALIA
First Edition 2019

Artwork by Brian Houston, Charlotte Frost, and Yi-Tung Huang.


Typeset in Australia by Deanne Gallasch. Typeset in Times Roman 10.
This book is available on Snowflake only.

The textbook has been developed independently of the International Baccalaureate Organization (IBO). The
textbook is in no way connected with, or endorsed by, the IBO.
This book is copyright. Except as permitted by the Copyright Act (any fair dealing for the purposes of private
study, research, criticism or review), no part of this publication may be reproduced, stored in a retrieval system,
or transmitted in any form or by any means, electronic, mechanical, photocopying, recording or otherwise,
without the prior permission of the publisher. Enquiries to be made to Haese Mathematics.
Copying for educational purposes: Where copies of part or the whole of the book are made under Part VB of
the Copyright Act, the law requires that the educational institution or the body that administers it has given a
remuneration notice to Copyright Agency Limited (CAL). For information, contact the Copyright Agency
Limited.
Acknowledgements: While every attempt has been made to trace and acknowledge copyright, the authors and
publishers apologise for any accidental infringement where copyright has proved untraceable. They would be
pleased to come to a suitable agreement with the rightful owner.
Disclaimer: All the internet addresses (URLs) given in this book were valid at the time of publication. While
the authors and publisher regret any inconvenience that changes of address may cause readers, no responsibility
for any such changes can be accepted by either the authors or the publisher.
FOREWORD
This book gives you fully worked solutions for every question in Exercises, Review Sets, Activities, and
Investigations (which do not involve student experimentation) in each chapter of our textbook
Mathematics: Core Topics SL.
Correct answers can sometimes be obtained by different methods. In this book, where applicable, each
worked solution is modelled on the worked example in the textbook.
Be aware of the limitations of calculators and computer modelling packages. Understand that when your
calculator gives an answer that is different from the answer you find in the book, you have not necessarily
made a mistake, but the book may not be wrong either.
We have a list of errata for our books on our website. Please contact us if you notice any errors in this
book.

CF BS JS MM

e-mail: info@haesemathematics.com
web: www.haesemathematics.com
TABLE OF CONTENTS

Chapter 1 STRAIGHT LINES


Chapter 2 SETS AND VENN DIAGRAMS 62
Chapter 3 SURDS AND EXPONENTS 92
Chapter 4 EQUATIONS 121
Chapter 5 SEQUENCES AND SERIES 153
Chapter 6 MEASUREMENT 231
Chapter 7 RIGHT ANGLED TRIANGLE TRIGONOMETRY 283
Chapter 8 NON-RIGHT ANGLED TRIANGLE TRIGONOMETRY 334
Chapter 9 POINTS IN SPACE 370
Chapter 10 PROBABILITY 395
Chapter 11 SAMPLING AND DATA 453
Chapter 12 STATISTICS 473
Chapter 1
STRAIGHT LINES
CEUOTS——
1 a y=3x+7 has gradient m =3 and y-intercept ¢ = 7.
b y= —2x —5 has gradient m = —2 and y-intercept ¢ = —5.
¢ y=2x—1 has gradient m =32 and y-intercept ¢ = —1%.
d y=11—4x has gradient m = —4 and y-intercept ¢ = 11.
e y=—6—x has gradient m = —1 and y-intercept ¢ = —6.
f y=2— %2 has gradient m = —% and y-intercept ¢ = 2.

g y= h; z_ %x + % has gradient m = % and y-intercept ¢ = é

h y= 2z6_ 3 2o — & has gradient m =% and y-intercept ¢ = —3.

i y= 3 _SSZ =2 — 22 has gradient m = —2 and y-intercept ¢ = 2.

2 a The equation of the lineis y—1=3(x —4)


Ly—1=3x—-12
y=3r—11

b The equation
of the line is y —5 = —2(z
— (—3))
y—5=-2(x+3)
y—H=-2x-6
y=-2x—-1

¢ The equation
of the line is y — (—3) i( —4)
y+3=1x-1
y:%z—4

d The equation of the line is y — (=7) = —2(z — (-2))


y+7=-2(z+2)
)
y+7=-5x—3 4
—__2 25
y=—3T—%
e The equation of the line is y =2z —9.

f The equation of the line is y = fix +4.


6 Chapter 1 (Straight lines) Exercise 1A

AY

16

12

T
0 >
012
3 4

b Yes, the variables are linearly related as the points all lie on a straight line.
8—5:3.
¢ The line passes through (0, 5) and (1, 8), so the gradient is —
The y-intercept is 5.
d The gradient is 3 and the y-intercept is 5, so the equation is y = 3z + 5.
e When 2 =10, y=23(10)+5
=35

5 . y-step 104
L The gradient of the road is et

As a percentage, % x 100% =~ 7.88%.

5 a PondP: Pond Q:

Tnetmmi
[ 0 1[2]3]1]
Amount of
[Emeamme
o] 12 [3]1]
Amount of
el 10120 30 fl i 5120355065

LA L AL
60 s 60 1
*
.
40 40
.
*

20 . Pl .

0. ¢ (rqinuFes») OT t (minu}esl
0 2 3 4 0 2 3 4

b The points on the graph of pond Q all lie on a straight line, so pond Q is being filled at a
constant rate.
-5
¢ i The line passes through (0, 5) and (1, 20), so the gradient is ot = 15. This means
that the amount of water increases by 15 L each minute.
The A-intercept is 5. This means that the amount of water in the pond initially was 5 L.
ii The gradient is 15 and the A-intercept is 5, so the equation is A = 15¢ + 5.
Chapter 1 (Straight lines) Exercise 1A 7

ili When t=8, A=15(8)+5


=125
There is 125 L of water in the pond after 8 minutes.

The line passes through (0, 90) and (1, 80), so £y (9)
- _ .
809 100
the gradient is

This means that the balance in the account decreases


by $10 each year.
The y-intercept is 90. This means that the initial
balance was $90.
The gradient is —10 and the y-intercept is 90, so the
equation is y = —10x + 90. 20
The account runs out of money when y =0 z (years,
~10z+90 =0 % 5 3 4
10z =90
=9
The account will run out of money after 9 years.

The line passes through (0, 46) and (1820, 0), y


. . 0 — 46 23
so the gradient is =——.
1820 -0 910 46

23
The gradient is — == and the y-intercept is 46, so
910

the equation is y = —%z + 46. 1850 >

When ¢ =0, H =150+ 120(0)


=150
The helicopter took off from a height of 150 m.
The height of the helicopter above sea level increases by 120 m each minute after taking off.
When ¢ =2, H =150+ 120(2)
=390
The helicopter is 390 m above sea level after 2 minutes.
When the helicopter is 650 m above sea level, H = 650
150 + 120t = 650
120t = 500
t = 500 _ 4%
120
The helicopter is 650 m above sea level after 4% minutes, or 4 minutes 10 seconds.

y=—4x+6
dr+y=6 {adding 4z to both sides}
y=>bxr—3
—5z+y=-3 {subtracting 5z from both sides}
o br—y=3 {multiplying both sides by —1}
8 Chapter 1 (Straight lines) Exercise 1A

< y=—-3z+3
cody=-3x+5 {multiplying both sides by 4}
3x+4y=5 {adding 3z to both sides}
d y=—%z——%
co 9y =-22+38 {multiplying both sides by 9}
2¢ 49y =8 {adding 2z to both sides}
e y=32z—-1
coby=3z—-1 {multiplying both sides by 5}
—3r+5y=—1 {subtracting 3z from both sides}
3r—by=1 {multiplying both sides by —1}

f y=%z+3
6y = bz
+ 18 {multiplying both sides by 6}
—bx
+ 6y = 18 {subtracting 5z from both sides}
Sr — 6y = —18 {multiplying both sides by —1}

10 a dSxt+y=2
y=—-br+2 {subtracting 5z from both sides}
b 3z+Ty=2
Ty=-3z+2 {subtracting 3z from both sides}
y=—%x+% {dividing both sides by 7}
¢ dr+3y=-1
{subtracting 42 from both sides}
{dividing both sides by 3}

{subtracting 22 from both sides}


{multiplying both sides by —1}
e 3r—13y=—4
—13y= -3z —4 {subtracting 3z from both sides}
3,44
Y=132+13 {dividing both sides by —13}

f 100—-3y="7
—3y=—-10c+7 {subtracting 102 from both sides}
y=%r-3 {dividing both sides by —3}

11 ax+by=d
by = —axr+d

Ly = 7%1 + % which has the form y = ma + ¢

The gradient of the line is 7(;‘


Chapter 1 (Straight lines) Exercise 1A

12 A: y=—x+3 has gradient —1 B: y+2=3(z-1)


. y=3x—>5 has gradient 3
C 3rx—y=-2 D: z4+y=14
y=3x+2 has gradient 3 . y=-—x+4 has gradient —1

gradient of A = gradient of D and


gradient of B = gradient of €
A and D are parallel, and B and € are parallel.

13 A z+2y=1 B: 2x+y=-3
y= 7%;}; -+ % has gradient 7% CLy=—2x-3 has gradient -2

C y—7=2x+4) D: y =2z — 7 has gradient 2


y=2x+15 has gradient 2

7% and 2 are negative reciprocals.


€ and D are both perpendicular to A.

14 a Since the line has gradient —4, the general form of its equation is 4x +y =d
Using the point (1, 2), the equationis 4a +y =4(1) +2
which is 4z 4y =6.
b Since the line has gradient %, the general form of its equation is = — 2y =d
Using the point (3, —5), the equation is = — 2y =3 — 2(—5)
whichis 2 —2y = 13.

¢ Since the line has gradient 7§, the general form of its equation is 5z + 3y = d
Using the point (—2, 6), the equation is 5z + 3y = 5(—2) + 3(6)
which is 5z + 3y = 8.
d Since the line has gradient Z, the general form of its equation is 7z — 6y = d
Using the point (—1, —4), the equationis 7z — 6y = T7(—1) — 6(—4)
which is 7z — 6y = 17.

15 a The line has gradient 311 ( 12) = ? =2, and passes through the point A(—2, 1).

-, the equation of the line is y — 1 =2(z — (—2))


y—1=2zx+4
L y=2x+5
B . 5—2 3 B
b The line has gradient T ol —1, and passes through the point A(7, 2).

*. the equation of the lineis y—2=—(x —7)


y—2=—x+7
Ly=-x+9
. . —17—-13 —30 .
¢ The line has gradient T = —5, and passes through the point A(—5, 13).

. the equation of the line is y — 13 = —5(xz — (—5))


y—13=—-bxr—25
y=—5r —12
10 Chapter 1 (Straight lines) Exercise 1A

d The line has gradient %(:1) = _—2 = %, and passes through the point P(6, —4).

-, the equation of the line is y — (—4) = Z(z — 6)


y+4= %a‘ —4
y=3%z-8
) . 2-(=5) 7 .
e The line has gradient 3(-2) =z and passes through the point M(—2, —5).

-, the equation of the line is y — (=5) = %(z —(-2))


L y+b=L(x+2)
Lyt+s=Lfa+ i
y=gr—%
f The line has gradient 9 77(7? N % = 72, and passes through the point R(5, —1).

~. the equation of the line is y — (—1) = —2(z —5)


) L ytl=—gz+
— 5y %2

v par
16 a The line has gradient SseEry MU
2-(-3) 5
Since the line has gradient 2, the general form of its equation
is 2z—y=d.
Using the point (=3, —4), 2z —y=2(-3) —(-4)
20 —y=-2

b The line has gradient sl 2 o=


6— (—4) 10
Since the line has gradient —1—30, the general form of its equation
is 3z 410y =d.
Using the point (—4, 2), 3z + 10y = 3(—4) + 10(2)
3z + 10y =8

¢ The line has gradient E ki Bt |


—1-(6 5
Since the line has gradient 7%, the general form of its equation
is 8z + 5y =d.
Using the point (—1, —1), 8z + 5y =8(—1)+5(—1)
. 8x+5y=-13
Chapter 1 (Straight lines) Exercise 1A 11

Line 2 is parallel to y = %x+ 1, which has gradient %.


line 2 has gradient 2 and passes through (3, 1).
line 2 has equation y—1=2(x —3)
Ly—l=3r—
y=3r—
5
The y-intercept of line 2 is —3.

The line is parallel to y = 3z — 2, which has gradient 3.


the line has gradient 3 and passes through (1, 4).
the equation of the line is y — 4 =3(z — 1)
y—4=3rx-3
Ly=3x+1

The line is parallel to 2z — y = —3, which has gradient 711 =2.

the line has gradient 2 and passes through (3, —1).


the equation of the line is y — (—1) = 2(z — 3)
L yYy+1=2x-6
20 —y="7
The line is perpendicular to y = —2x + 1, which has gradient —2.
the line has gradient 3 and passes through (-1, 5).
the equation of the lineis y — 5= 1(z — (—1))
y—5=3(x+1)
y—5=73r+1
y=go+y
The line is perpendicular to x + 2y = 6, which has gradient 7%.
the line has gradient 2 and passes through (-2, —1).
the equation of the line is y — (—1) = 2(z — (-2))
L y+1=2x+2)
Ly+1=2x+4
2r—y=-3

When x =3, we have b When x = —6, we have


y=43) -1 y=2(-6)—6
=1 v SEl
So, (3,11) does lie on the line. —_10 x
So, (—6, —2) does not lie on the line.
1
Substituting x = —4 and y = —8 into d Substituting * = —5 2 and y = 2 into
the LHS gives 7(—4) — 3(—8) the LHS gives 6(—3) +10(2) = —=3+20
=—-28+24 =17 v
=—4 Vv So, (—%,2) does lie on the line.
So, (—4, —8) does lie on the line.
12 Chapter 1 (Straight lines) Exercise 1B

20 a Substituting =2 and y = 15 into the b Substituting = :é and y =3 into the


equation gives 15=14(2) +¢ equation gives 3 =m(3) — 2
c+8=15 -
c=17
L 2_bog
2 2
m—5=6
m =11
¢ Substituting x =t and y =4 into the
equation gives 4 =2t — 1
2, _ 16
3t=7%
Lt=8

21 a Substituting * =6 and y = —3 into b Substituting * = —8 and y = —5 into


the equation gives the equation gives
2(6)+5(-3) =k 7(=8)—(=5) =k
o k=12-15 S k=-56+5
k=-3 oo k=-51
¢ Substituting x =k and y =0 into the
equation gives
3k —4(0)+36
=0
3k+36=0
3k =—-36
k=-12

22 a Line I has gradient _2:21 = %3 =1

Line 2 is perpendicular to /ine I, so its gradient is 1.


line 2 has gradient 1 and passes through (2, 4).
.. line 2 has equation y—4=2—2
z—y+2=0

b When y=0, 2—0+2=0


= —2
the z-intercept of line 2 is —2.

1 a For y=%a+1:
o the y-interceptis c=1
o the gradientis m =1
Chapter 1 (Straight lines) Exercise 1B 13

b For y=3z—2:
e the y-interceptis ¢ = —2
o the gradientis m=3=2

¢ For y=—3a+4:
e the y-interceptis ¢ =4
o the gradientis m = —5 = 52 T
>

3 T +4

d For y=—2z+5:
e the y-interceptis ¢ =5
o the gradientis m = —2 = 52

e For y=32-1: .
Ay
y:zz—l
e the y-interceptis ¢ = —1
3| z
e the gradient is m =
wles

—2
4
2
v

f For y=—4ux:
e the y-interceptis ¢ =0
o the gradientis m = —4 = 52
14 Chapter 1 (Straight lines) Exercise 1B

g For y=—x+4:
o the y-interceptis c=4
o the gradientis m =—-1= =%

h For y=§x—3: Ay
3
o the y-interceptis ¢ = —3
e the gradientis m = 2
il o

¥ o %
=gr—3
5

i For y=-22-1: Ay
2
e the y-interceptis ¢= —1
o the gradientis =5 x
m = —
wlwt

3
y ‘;’x 1
3
2|

. —5

2 a For 3z+2y=12:
When =0, 2y=12
y==6
So, the y-intercept is 6.
When y =0, 3z=12
Mg
So, the z-intercept is 4.

b For z+3y=6: Ay
When =0, 3y=6
6
y=2 Ly 2
>
So, the y-intercept is 2.
When y =0, z=6. A

So, the z-intercept is 6.


Chapter 1 (Straight lines) Exercise 1B 15

For 2x — 5y = 10:
When =0, —5y=10
Loy=-2
So, the y-intercept is —2.
When y =0, 2z=10
. x=5
So, the z-intercept is 5.

For 4z —y=8:
When =0, —y=38
-
coy=-8
So, the y-intercept is —8.
When y =0, 4z=38
L e’
So, the z-intercept is 2.

For 5z + 8y = 40: Ay
When =0, 8y=40 SRS + 8y
= 40
y=>5
So, the y-intercept is 5. - 4 i

When y =0, 5z=40


. =28 v

So, the z-intercept is 8.

For 3x —4y = —24:


When z =0, —4y=-24
y==6
So, the y-intercept is 6.
When y =0, 3z=-24
Lor=-8
So, the z-intercept is —8.

For 2z 4 5y = 15: YA
When =0, 5y=15
L y=3 3 2z+ 5y =15
So, the y-intercept is 3. - < 5
x
>

When y =0, 2z=15 2


Y
L=
w5

So, the z-intercept is


vz
16 Chapter 1 (Straight lines) Exercise 1B

h For 6z + 4y = —36:
When z =0, 4y = —36
y=-9
So, the y-intercept is —9.
When y =0, 6z 36
.z 6
So, the z-intercept is —6.

For Tz 44y = 42:


When z =0, 4y =42
21
y=%5

So, the y-intercept is 52 1 -

When y =0, 7z=42


. x=6
So, the z-intercept is 6.

3 Y= ij + 2 has gradient m = 7% and y-intercept ¢ = 2.

b When z =38, we have AY

y=-30)+2
—6+2
—4 v
So, (8, —4) does lie on the line.
When z =1, we have

y=-3(1)+2
__3
=—-5+2
=2 5 %

So, (1, 3) does not lie on the line.


When = = —2, we have

y=-3(-2)+2
v
NI~

So, (=2, %) does lie on the line.

4 a For 2z —3y =18:


When z =0, —3y=18
. y=—6
So, the y-intercept is —6.
When y =0, 2z=18
e
So, the z-intercept is 9.
Chapter 1 (Straight lines) Exercise 1B 17

b 1 Substituting z =3 and y = —4 into ii Substituting z =7 and y = —2 into


the LHS gives 2(3) — 3(—4) the LHS gives 2(7) — 3(—2)
=6+12 =144+6
=18 v =20 x
So, (3, —4) does lie on the line. So, (7, —2) does not lie on the line.
¢ If (=3, c¢) lies on the line then 2(—3) —3c =18
—6—3c=18
—3c =24
. e=-8

5 C=5t+10 dollars
a When t=4, C=5(4)+10
=30
The cost of hiring the trailer for 4 hours is $30.
b LC(S)
40
4, 30)
20
=5t + 10
t (hours)
0 >
0 2 3 4

6 a x serves of nigiri at $4.50 each and y serves of sashimi at $9 each adds up to a total of $45.
4.52
+ 9y = 45
b When z =4, 4.5(4)+ 9y =45 ¢ When y=1, 4.5z+9(1) =45
. 1849y =45 45 +9=45
Oy =27 . 4.5 =36
Ly=3 cLr=8
Hiroko bought 3 serves of sashimi. .. Hiroko bought 8 serves of nigiri.
18 Chapter 1 (Straight lines) Exercise 1C

d £y
5KNd.52 + 9y = 45

1 a The midpoint M of [AB] is (3—;—5, v; 7) or (4,4).

b The gradient of [AB] is L5Ly 30s aiqs


5-3 2

< The gradient of the perpendicular bisector is 7%, the


negative reciprocal of the gradient of [AB].
d The perpendicular bisector has gradient 7% and passes
through (4, 4).
its equation is = + 3y = 4 + 3(4)
which is z + 3y = 16.

The midpoint M of [AB] is (_5;1, _2;4) or (3,3).


The gradient of [AB] is i P S
1-5 -4 32
the gradient of the perpendicular bisector is 2.
the equation of the perpendicular
bisector is 2z —y =2(3) — 3
whichis 2z —y=3.

The midpoint M of [AB] is (_1;— 5 %) or (2,4).


3-5 -2 1
The gradient of [AB] is
5-(-1) 6 3
the gradient of the perpendicular bisector is 3.
the equation of the perpendicular
bisector is 3z —y =3(2) —4
whichis 3z —y=2.

The midpoint P of [MN] is (%, £ ot 1) or (4, —1).

The gradient of [MN] is ! ; (;3) = % =1

the gradient of the perpendicular bisector is 1.


the equation of the perpendicular
bisectoris =z —y=4—(-1)
whichis z —y =5.
Chapter 1 (Straight lines) Exercise 1C 19

flmm@mmwflmmm<“%4l%§)m(&g
6-2 4 1
The gradient of [MN] is
-7 -8 2
the gradient of the perpendicular bisector is 2.
the equation of the perpendicular
bisector is 2z —y =2(3) — 4
which is 2z —y = 2.
e . (049 0+0 9
The midpoint M of [OP] is <T’ T) or (5, 0).
0—
The gradient of [OP] is = 0.
0(0,0) -~
So, [OP] is horizontal, and the perpendicular bisector is
the vertical line passing through (%, 0).
the equation of the perpendicular bisector is = = 5.

ol o
The midpoint M of [AB] is (3 +;_1), %) or (1,

The gradient of [AB] is


3-6 _-3_3
—1-3 —4 4
. ] . —
the gradient of the perpendicular bisector is —3.
the equation of the perpendicular
bisector is 4a + 3y = 4(1) + 3(2)
which is 4z + 3y = £
or 8z + 6y = 35.

The midpoint M of [PQ] is <6;—2, 71; 5) or (4, 2). Q(2,5)%,

) L B—(=1)
5-5
_ 6
Ei
_ 3o
The gradient of [PQ] is

the gradient of the perpendicular bisector is %


the equation of the perpendicular
tori
bisector is 2x — 3y = 2(4) — 3(2 y=2(4) - 3(2) .
which is 2z —3y =2. ' P(6, 1)

Substituting 2 =1 and y =0 into the LHS gives 2(1) —3(0)=2


So, R(1,0) does lie on the line.

PR=\/(1-67+(0-(-1))2 QR=/(1-22+(0-52
YS - VT
= /26 units = V/26 units

PR=QR = /26 units


R is equidistant from P and Q.
20 Chapter 1 (Straight lines) Exercise 1C

h a AB=/(-2—(-4))2+(2-(-3))?
VP
= V29 units
BC=+(3—-(-2))2+(4-2)?
_JiE
= /29 units
CD=+/(1 -3+ (-1 —4)2 v
= 2P+ (5
= V/29 units

AD= /I — (DR + (1 (3P


= /52422
= V29 units
All side lengths are equal, .. ABCD is a rhombus.
T. = . (—4+3 —3+4 1 1
b The midpoint M of [AC] is ( RS ) or (75, 5).

) .o4—(=3)
The gradient of [AC] is o) 1

the gradient of the perpendicular bisector is —1.


-, the equation of the perpendicular bisector is z +y = 7% + %
whichis z+y=0
or y=—2x.
¢ B 2=—(-2) v D —1=—(1) v
5 a i3z—-2y+1=0 il The perpendicular bisector has
2y =3z +1 gradient —2.
. y=32x+3 has gradient 3
b The equation of the perpendicular bisector is 2z + 3y =2(3) + 3(5)
which is 2z 43y =21
or 2x+3y—21=0.

6 a The midpoint of [AB] is (zl ;IZ, LE ;yz)

The gradient of [AB] is 22—


o — T

2—
the gradient of the perpendicular bisector is — .
Y2—y
the equation of the perpendicular bisector is

(w2 =2z + (Y2 — Y1)y = (22 — 21) (zl ;ZQ) +(y2 — 1) (yl ;y2>
2 2 2 2
Ty — &y Yo —Yr
(22 —21)w+ (g2 —y1)y =

(w2 —21)7 + (y2 — Y1)y


Chapter 1 (Straight lines) Exercise 1C 21

b We can find the perpendicular bisector of any two points A(zy, y1) and B(xs, y2) by
substituting in the values of x1, x2, y1, and ys.

7 a i The midpoint of [AB] is (#, 2—;5> or ( ).

rojon
5

I~
5—2
The gradient of [AB] is

the gradient of the perpendicular bisector is —1.


the equation of the perpendicular bisector is z +

nI~
Il
+
o
<
whichis = +

O
Il
<
i The midpoint
of [AC] is (IT+2 2+§*1)) or (2, s

o=
The gradient of [AC] is ;1 *12 _ 3
the gradient of the perpendicular bisector is %
the equation of the perpendicular bisector is = — 3y = 5 — 3(%)

nlee
whichis = —3y=0.

ili The midpoint of [BC] is (#, 5+;71)> or (3,2).


—1-5
The gradient of [BC] is =3

the gradient of the perpendicular bisector is 7§.


the equation of the perpendicular bisector is = + 3y = 3 + 3(2)
whichis x4+ 3y =
b Ay

C(F, —

The perpendicular bisectors all intersect at (5, 2).


A, B, and C are all equidistant from this point.
¢ The perpendicular bisectors of each pair of points will meet at a single point. As the three
points are equidistant from the point of intersection, a circle centred at the point of intersection
that passes through one of them will pass through all of them.
. S . [(—841 —6+45 7 R
o ) or (=1,-1).
8 a i The midpoint of [PQ] is (

The gradient of [PQ] is


5-(=6) _ 11
1—(—8) 9
the gradient of the perpendicular bisector is —%4
the equation of the perpendicular bisector is 9z + 11y = 9(7%) +11 (7%)
which is 9z 4 11y = —37.
22 Chapter 1 (Straight lines) Exercise 1D.1

il The midpoint of [PR] is (_8 sk _6+T(_2)) or (=2, —4).


. o2-(-6) 1
The gradient of [PR] is T-(=®) 3

the gradient of the perpendicular bisector is —3.


the equation of the perpendicular bisector is 3z +y = 3(—2) + (—4)
which is 3z +y = —10.

i The midpoint of [QR] is (# 5”*


>T g
2)) or (52022).
. . —2-5 7
The gradient of [QR] is 1= 3
. . . .3
the gradient of the perpendicular bisector is .
the equation of the perpendicular bisector is 3z — 7Ty = 3(5) — 7( )

wolee
o
which is 3z — 7y = —3.

We draw the graphs of y = 32+ 2 and


y =x — 2 on the same set of axes.
The graphs meet at the point (-2, —4).
the solution is = = —2, y= —4.

Check:
Substituting these values into:
o y=23r+2 gives —4=3(—-2)+2 v
o y=x—-2 gives -4=-2-2
Chapter 1 (Straight lines) Exercise 1D.1 23

We draw the graphs of y = —4x +1 and


y =3z — 6 on the same set of axes.
The graphs meet at the point (1, —3).
the solutionis z =1, y=—3.

Check:
Substituting these values into:
o y=—4dz+1 gives -3=—-4(1)+1 v
e y=3r—6 gives -3=3(1)-6

We draw the graphs of y =2x —5 and


y =42 +4 on the same set of axes.
The graphs meet at the point (6, 7).
the solutionis z =6, y=7.

Check:
Substituting these values into:
o y=2r—5 gives 7=2(6)—5 v
o y=3x+4 gives 7=1(6)+4 v

We draw the graphs of y =2 —1 and


2x + 3y = 12 on the same set of axes.
The graphs meet at the point (3, 2).
the solutionis =z =3, y=2.

Check:
Substituting these values into:
e y=x—1 gives 2=3-1 v
o 2+ 3y =12 gives 2(3)+3(2) =12 v
24 Chapter 1 (Straight lines) Exercise 1D.1

b We draw the graphs of = +3y =9 and


x — 2y =4 on the same set of axes.
The graphs meet at the point (6, 1).
the solutionis = =6, y=1.

Check:
Substituting these values into:
o r+3y=9 gives 6+3(1)=9 v
o r—2y=4 gives 6—2(1)=4

We draw the graphs of 3z — 2y =30 and


4x +y = —4 on the same set of axes.
The graphs meet at the point (2, —12).
the solution is = =2, y = —12.

Check:
Substituting these values into:
e 3z —2y =230 gives
3(2) - 2(-12)=30 v
o 4dxr+y=—4 gives
42)+(-12)=—-4 v

We draw the graphs of y = —2x+5 and


y = —2x — 1 on the same set of axes.
The lines are parallel, so there are no solutions.
Chapter 1 (Straight lines) Exercise 1D.2 25

We draw the graphs of = — %y = —2 and


y =4z + 8 on the same set of axes.
The lines are coincident. There are infinitely many
solutions.

We draw the graphs of 3z + 4y = —24 and


3z + 4y = —12 on the same set of axes.
The lines are parallel, so there are no solutions.

y=x+2 LA (1)
2 4+3y=21 .. (2
Substituting (1) into (2) gives 2z + 3(z +2) =21
2z 43z +6 =21
S5z =15
z=3
Substituting x = 3 into (1) gives y =342
LYy=>5
The solutionis = =3, y=>5.

Check: (1) 5=3+2 V


2) 23)+3(5)=6+15=21 v
26 Chapter 1 (Straight lines) Exercise 1D.2

b y=2x-3 ..(1)
4z —3y=7 .. ((2)
Substituting (1) into (2) gives 4z —3(22—3) =7
L dr—6x+9=7
—2r=-2
pte—1

Substituting = =1 into (1) gives y=2(1)—3


Ly=-1
The solutionis z =1, y=—1.
Check: (1) —1=2(1)-3=2-3
@ 4(1)-3(-1)=4+3=7 v
¢ bz+3y=19 ... (1)
y=6-2z ..(2)
Substituting (2) into (1) gives 5z + 3(6 — 2z) = 19
. br+18—6z=19
—xr =1
Lor=-1
Substituting = = —1 into (2) gives y =06 —2(—1)

The solutionis = = —1, y=38.


Check: (1) 5(-1)+3(8)=-5+24=19
2) 8=6-2(-1)=6+2 v

d y=3z+1 ..(1)
y=T7r—1 ...(2)
Substituting (1) into (2) gives 3z +1 =7z —1
—4r
= -2
=1
Substituting = = & into (1) gives y=3(3) +1
y=15+1
y=2%
The solutionis = =3, y =21

Check: (1) 22 =3(3)+1=14+1 v


@ 23=7(3)-1=34-1 v
Chapter 1 (Straight lines) Exercise 1D.2 27

e y=6x—8 e (D)
3r+2y=-6 .. (2
Substituting (1) into (2) gives 3z + 2(6z —8) = —6
o 3z+4+ 122 —16=—6
15z =10
. z:%

Substituting « = 2 into (1) gives y=6(2) -8


L y=4-8
Cy=-4

The solution is = = 2, Y= —4 .

Check: (1) —4=6(%)-8=4-8 v


@ 3(3)+2(-4)=2-8=-6
fde—-Ty=1 ... (1)
y=11-3z ... (2)
Substituting (2) into (1) gives 4z —7(11—3z) =1
LA —TT+ 21z =1
25z =18
T = ;—g = 3%

Substituting = = % into (2) gives y =11 — 3(;—?)


y=2p 24

v-#
y=1s
1
The solution is = = 3%, y=1 5.
o

Check () 4(3) ~7(18) =1(B) - 7(#) =¥ - W =F=1


|

25 25 25

@ 18=1-33%)=U-3(F)=F-F 3
=% v
arx=y—3 e (D
br—2y=9 .. (Q2)
Substituting (1) into (2) gives 5(y —3) —2y =9
hy—15—-2y=9
L 3y=24

Substituting y = 8 into (1) gives = =8—3

The solutionis =z =5, y=38.

Check: (1) 5=8-3 «


) 5(5)—-28)=25-16=9
28 Chapter 1 (Straight lines) Exercise 1D.2

b 22—3y=-8 .. (1)
r=3—-1 ..(Q)
Substituting (2) into (1) gives 2(3y — 1) — 3y = —8
L by—2—-3y=-8
. 3y=—6
Loy= )

Substituting y = —2 into (2) gives = =3(-2)—1


Lr=—T
The solution is = = -7, y= —2.

Check: (1) 2(-7)—3(-2)=—-14+6=—-8


Q@ -7T=3(-2-1=-6-1 v
z=4y+3 .. (1)
=947y .. (2
Substituting (1) into (2) gives 4y +3 =9+ Ty
—3y==6
Soy=-2
Substituting y = —2 into (1) gives = =4(-2)+3
or=-5
The solution is = = —5, y = —2.
Check: (1) —5=4(-2)+3=-8+3 V
2 —5=94+7(-2)=9-14 v
y=5z—-3 .. (1)
T=2y+3 .. (2)
Substituting (1) into (2) gives z = 2(5z —3)+3
=100 —-6+3
—9r =-3
La=3
Substituting = = % into (1) gives y = (—) -3
- y=§_

__4 g
coy=-1%
The solutionis = =1, y=—11.

Check: (1) —1%:5(%)_3:%_3:_% v

@ 3=2(-13)+3=2(-3) +3=-3+3
Chapter 1 (Straight lines) Exercise 1D.2 29

e 3v+4y=-13 ... ()
r=28y—2 e (2)
Substituting (2) into (1) gives 3(8y — 2) + 4y = —13
24y —6+4y=-13
28y =7
Y= _%

Substituting y = —% into (2) gives z =8(—%) -2


. g W
Sor=—4
The solution is = = —4, y= 7%.

Check: (1) 3(—4) + 4(7%) = _12—-1=-13 v

@ -4=8(-1) -2=-2-2 V
f x=-5y—2 e (1)
Te+4y=-10 ...(2)
Substituting (1) into (2) gives 7(—5y —2) + 4y = —10
—35y — 14+ 4y = —10
31y =4
=_4
31
= N ] B )
Substituting y = —47 into (1) gives 2 = —5(—5) -2
r=2-2
T = 71}—%
11
Sy is = = 157,
The solution 4
y= —357.

ay=Llz+s5 . (1
3w4dy=5 .. (2
Substituting (1) into (2) gives 3z +4(3z+5
o3+ 27+ 2

Substituting = = —3 into (1) gives y = 3(-3)+5

The solution is = = —3, y=3%.

Check: (1) 33=3(-3)+5=-3+5=1 v


@ 3(-3)+4(33) =—9+4(3)=-9+14=5 v
30 Chapter 1 (Straight lines) Exercise 1D.2

b a=-3y e (D
do—by=-24 .. (2

—3y — by = —24
—8y = —24
Y=

Substituting y = 3 into (1) gives x = ,%(3)

r=-21

The solution is = = —2%, y =3.


Check: (1) —2%=-3(3 )=-2 v
4

@ 4(-24)-503)=4(-%) S
-15=-9-15 24

¢ x4+6y=—-6 ..(1)
y=3r-5 .. (2
Substituting (2) into (1) gives x +6(3z —5) = —6
L x+22—-30=-6
e e 04
Lox=38
Substituting = =8 into (2) gives y=1(8)—5
L y=8-5
Loy=-2%
The solutionis = =8, y = —2%.
Check: (1) 8+6(—23) =8+6(—%) =8—14=—6
2 -22=108)-5=8-5=-1I v
y=—3z+3 .. (1)
Sr+dy=14
. (2)
Substituting (1) into (2) gives 5z +4(—4z +3) = 14
b —22+12=14
Y, mbgA=

Substituting = = 2 into (1) gives y=—3(%)+3


1+3
Chapter 1 (Straight lines) Exercise 1D.3 31

e 3x+Ty=6 .. ()
r=5y-1 ..
Substituting (2) into (1) gives 3(§y 7 1) +T7y=6
S5y—3+T7y==6
12y =9
y=1%
Substituting
y = £ into (2) gives z=5(3) -1
La=35-1
La=1
The solutionis =1, y= 3I
Check: (1) 3(3)+7(8)= +H=0 v @ F=3)-1-3-1 ¢
f 3x44y=10
... (1) N
y = %z +2 e (2)
Substituting (2) into (1) gives 3z +4(3z +2) = 10
3z +3r+8=10
6z =

Substituting @ = 5 into (2) gives


1=

Wl

v
Il


+
<

The solution is @ =%, y = 21.


lo
—~
~—

Ny
Do
el

<o
ol
Q
T

<
w

o
Il

Il

Il

+
-
Q
Q

o~
wlw

3r—y=5
1
B {4x+y:9
The coefficients of y are the same size but opposite in sign.
We add the LHSs and the RHSs to get an equation which contains = only.
3r—y=5 ..()
de+y=9 ..(Q
Adding, Tx =14
T dre.2
Substituting = =2 into (1) gives 3(2) —y =5
6—-—y=>5
—y=—1
y =

The solutionis =z =2, y=1.


Check: In(2): 4(2)+1=8+1=9 v
32 Chapter 1 (Straight lines) Exercise 1D.3

b 5r —2y =17
3x+2y=7
The coefficients of y are the same size but opposite in sign.
We add the LHSs and the RHSs to get an equation which contains z only.
br—2y=17 ... (1)
3r+2y=7 .2
Adding, 8x =24
Lr=3
Substituting = =3 into (1) gives 5(3) — 2y =17
15— 2y =17
—2y=2
Ly=-1
The solutionis = =3, y=—1.
Check: In(2): 3(3)+2(-1)=9-2=7
—4x
+ 3y =31
4o —y=-21
The coefficients of = are the same size but opposite in sign.
We add the LHSs and the RHSs to get an equation which contains y only.
—4x + 3y =31 e (D)
dr— y=-21 ..(2)
Adding, 2y =10
y=>5
Substituting y =5 into (1) gives —4x + 3(5) =31
—4x +15 =31
—4x =16
=4
The solution is = = —4, y = 5.
Check: In(2): 4(-4)—-5=-16-5=-21
6x +5y =9
—6x + Ty = —45
The coefficients of x are the same size but opposite in sign.
We add the LHSs and the RHSs to get an equation which contains y only.
6z +5y=9 v (D)
—6z+ Ty =—-45 ... (2)
Adding, 12y — 36
Soy=-3
Substituting y = —3 into (1) gives 6z +5(—3) =9
6 —15=9
bz =24
r=4
The solutionis =z =4, y=—3.
Check: In(2): —6(4)+7(-3)=-24—-21=-45
Chapter 1 (Straight lines) Exercise 1D.3 33

2¢ —3y =18
{ br+ 3y =24
The coefficients of y are the same size but opposite in sign.
We add the LHSs and the RHSs to get an equation which contains = only.
20 —3y =18 ... (1)
Sx+3y=24 ... ()
Adding, Tx =42
=6
Substituting « = 6 into (1) gives 2(6) — 3y = 18
12-3y=18
—3y=6
Ly =-2
The solutionis = =6, y= —2.
Check: In (2): 5(6)+3(-2)=30-6=24

{ —dz 46y = —21


4o — 2y =11
The coefficients of z are the same size but opposite in sign.
We add the LHSs and the RHSs to get an equation which contains y only.
—dz+6y=-21 .. (1)
dr —2y =11 e (2)
Adding, 4y = —-10
y=-2;
Substituting y = —24 into (1) gives —4x +6(—23) = —21
—dz+6(-5) = -21
—4r — 15 =-21
—4o = —6

The
o
solution is = =13, y=—21.
v=13
Check: In (2): 4(13) —2(—23) =4(3)—2(-3)=6+5=11

3r+y=16 ... (1)


Tx—2y=7 ... (2)
To make the coefficients of y the same size but opposite in sign, we multiply (1) by 2.
L 6r+2y =32 {(1)x2}
T —2y=717
Adding, 13x =39
z=3
Substituting = =3 into (1) gives 3(3) +y = 16
9+ y=16
y="1
The solutionis = =3, y=T1.
Check: In(2): 7(3)—2(7)=21-14=7
34 Chapter 1 (Straight lines) Exercise 1D.3

b dz+3y=—14 .. (1)
—z+5y=15 ... (2)
To make the coefficients of = the same size but opposite in sign, we multiply (2) by 4.
4+ 3y=-14
—4x+20y =60 {(2) x 4}
Adding, 23y = 46
Cy=2

Substituting y =2 into (1) gives 4z + 3(2) = —14


e 4+6=-14
4o = —-20
L r=-5
The solutionis = = —5, y=2.
Check: In(2): —(=5)+5(2)=5+10=15

¢ br—2y="7 e (D
20—y—4=0
20 —y=4 ...
To make the coefficients of y the same size but opposite in sign, we multiply (2) by —2.
Sr—2y="7
—dz+2=-8 {@2)x-2}
Adding, T =-1
Substituting = = —1 into (1) gives 5(—1) —2y=7
59y =7
—2y =12
y=—6
The solution is = = —1, y = —6.
Check: In(2): 2(-1)—(—6)=-2+6=4

d 3z—-Ty=-27 ... (1)


—6z+5y =18 ... (2)
To make the coefficients of = the same size but opposite in sign, we multiply (1) by 2.
6x — 14y = =54 {(1) x 2}
—6z + by =18
Adding, —9y = —36

Substituting y =4 into (1) gives 3z — 7(4) = —27


. 3r —28=-27
3r=1
L r=1 3
The solution is = =3, y=4.
Check: Tn(2): —6(§)+5(4)=-2+20=18
Chapter 1 (Straight lines) Exercise 1D.3 35

e 9z+2y=-24 .. (1)
—Tr+4y=27 ... (2)
To make the coefficients of y the same size but opposite in sign, we multiply (1) by —2.
—18z —4y =48 {(1) x -2}
—Te+4y =27
Adding, —2bx =175
=3

Substituting = = —3 into (1) gives 9(—3) + 2y = —24


2742y = -2
2y =
y=135
The solution is = = -3, y=15.1

Check: In (2): —T7(=3)+4(13) =21+4(3)=21+6=27


f 3e—7Ty=-8 ... (1)
9r+11ly =16 ... (2)
To make the coefficients of x the same size but opposite in sign, we multiply (1) by —3.
—9r+2ly =24 {(1) x -3}
9z + 11y = 16
Adding, 32y — 40
y=1;
Substituting y = 14 into (1) gives 3z — 7(14) = -8
3x—-7(5)=-8
3z — 8 =-38
3r = %

The solution is = =4, y=14.


oot
Check: In(2: 9(3)+11(1) =3+11(§) =3+F=8=16 v
a dex+3y=14 .. (1)
3r—4y=23 .. (2
To make the coefficients of y the same size but opposite in sign, we multiply (1) by 4 and
2) by 3.
(@ 162+ 12y =56 {(1) x 4}
92 —12y =69 {(2) x 3}
Adding, 25z =125
Loax=5
Substituting =5 into (1) gives 4(5) + 3y = 14
L 20+ 3y =14
3y=—6
=2
The solutionis = =15, y=—2.
Check: Tn(2): 3(5)—4(-2)=15+8=23 «
36 Chapter 1 (Straight lines) Exercise 1D.3

b 2x—-3y=6 .. ()
hr—4dy=1 .. (2
To make the coefficients of y the same size but opposite in sign, we multiply (1) by —4 and
() by 3.
—8x + 12y = —-24 {(1) x —4}
150 — 12y =3 {(2) x 3}
Adding, Tx =-21
Lr=-3
Substituting = = —3 into (1) gives 2(—3) — 3y =6
—6-3y=6
-3y =12
Ly=—4
The solution is = = -3, y = —4.
Check: In (2): 5(—3)—4(-4)=-154+16=1
¢ Sx+6y=17 .. (1)
3r—Ty=42
... (2)
To make the coefficients of y the same size but opposite in sign, we multiply (1) by 7 and
(2) by 6.
35w 42y =119 {(1) x 7}
18z — 42y = 252 {(2) x 6}
Adding, 53x =371
=T
Substituting = =7 into (1) gives 5(7) + 6y = 17
. 3546y =17
. by =-—18
Ly=-3
The solutionis =z =7, y=—3.
Check: In(2): 3(7)—7(-3)=21+21=42 v

d 22+410y=-5 .. (1)
3r—Ty=9 e (2)
To make the coefficients of x the same size but opposite in sign, we multiply (1) by 3 and
(2) by —2.
. 6z +30y=-15 {(1)x 3}
—6a+ 14y = —18 {(2) x -2}
Adding, 4dy= 33
L oy=-3

Substituting y = —2 into (1) gives 2z +10(—%) =5

L 2r—B=-5
. 2x =3
r=1%
The solution is = =13, y=—3.
Check: In(2): 3(11) —7(-3)=3(3)+2 =L 42 =3%=9
Chapter 1 (Straight lines) Exercise 1D.3 37

e 4o +2y=-23 .. ()
br—Ty=-5 .. (2)
To make the coefficients of = the same size but opposite in sign, we multiply (1) by 5 and
(2) by —4.
202+ 10y = —115 {(1) x 5}
—202+28y =20 {(2) x -4}
Adding, 38y = —95

Substituting y = —23 into (1) gives 4z +2(-24) = —23


Az +2(—-5) =-23
odr—5=-23
4r =—18
T = — 1

The solution is = = —43, y = —23.

Check: Tn(2): 5(—43) —7(-24) =5(-3) - 7(-5) =—2+8=-2L=-5 v


fde—-Ty=9 .. (1
br—8y=-2 ..(2)
To make the coefficients of = the same size but opposite in sign, we multiply (1) by 5 and (2)
by —4.
20z — 35y =45 {(1) x 5}
—20z+32y =8 {(2) x —4}
Adding, 3y — 53
y=-173
Substituting y = —173% into (1) gives 4a — 7(*17%) =9
4z -7(-2)53\ =9
_

do+ 3L =9
S
gy 3443
z=—282
The solution is @ = —28%, y = —173.
Check:
In(2): 5(—28%) —8(—172) =5(-5) —8(-%) =20424 __6__9 o
38 Chapter 1 (Straight lines) Activity 1 Parallel and coincident lines

ACTIVITY 1

1 a Wedraw the graphs of y =42+7 and 2y—8x =1


on the same set of axes.
We can see that the lines are parallel and therefore
do not meet.

y=4 =1

~T Tz

b i y=4dx+7 ..
2y—8x =1 .. (2
Substituting (1) into (2) gives 2(dz+7) — 8z =1
L8 +14-8x=1
14 =1 which is false
there are no solutions.
il We rearrange the second equation, so the system is now: y=4x+7 .. (1)
2y=8r+1 .. ()
To make the coefficients of y the same size but opposite in sign, we multiply (1) by —2.
—2y=—-8x—14 {(1)x -2}
2y= 8xr+1
Adding, 0= —13 which is false
there are no solutions.
¢ This system of simultaneous equations has no solutions.

2 a We draw the graphs of y = —2z 4+ 5 and


4z 4+ 2y = 10 on the same set of axes.
We can see that the lines are coincident.

b i y=-20+5 .. ()
lr+2y=10 .. (2)
Substituting (1) into (2) gives 4a + 2(—2z +5) = 10
o4
—4x+10=10
10 = 10 which is always true
there are infinitely many solutions.
Chapter 1 (Straight lines) Activity 2 Simultaneous equations using technology 39

ii We rearrange the second equation, so the system is now: y=—2z+5 ... (1)
2y=—4x+10 ... (2)
To make the coefficients of y the same size but opposite in sign, we multiply (1) by —2.
—2y= 4dxz—-10 {(1)x -2}
2y = —4x 4+ 10
Adding, 0= 0 which is always true
there are infinitely many solutions.
¢ This system of simultaneous equations has infinitely many solutions.

ACTIVITY 2

PART 1: GRAPHING

y=4xr—3 et R Form) [EXE]:Show coordinates


1 a Graph Func :Y
y=—-x+5

TYPE ] TOOL JI

So, the solution is =1.6, y=3.4.

b y=-3r—4 i [EXE]:Show coordinates


y=2x+7 Graph Func Yomaeir

7
INTSECT
DD ERLW

So, the solution is = = —2.2, y=2.6.

. y=2xr—3 EoEdiea] [EXE]:Show coordinates


y=10— 6z Graph Func :V Yoo10-6x
11

INTSECT
TYPE ] TOOL I

So, the solution is = = 1.625, y =0.25.


d We rearrange the second equation, so the system is now:

y=12—"5z ok G [EXE] Show coordinates


Graph Func
y=7-=x

So, the solution is = = 1.25, y =5.75.


40 Chapter 1 (Straight lines) Activity 2 Simultaneous equations using technology

e We rearrange the second equation, so the system is now:


y=3x—2
y="Tr+4

INTSECT

So, the solution is = = —1.5, y = —6.5.

f We rearrange both equations, so the system is now:


y=38r—16 ElE o ) [EXE]:Show coordinates
y=—22-10 Graph Func :Y=
[—1]
=]

=]
[—1
[—1 INTSECT
DELETE] TYPE J TOOL

So, the solutionis = =0.6, y = —11.2.

g We rearrange both equations, so the system is now:


—3_5,
¥y=3—1 B WeOfdins] fa [EXE]:Show coordinates
Graph Func :V=

DELETE] TYPE ] TOOL X=4.666666667 ¥=-4.33

So, the solution is = ~ 4.67, y ~ —4.33.

h We rearrange both equations, so the system is now:

=10-3z B Gomdls] fal [EXE]:Show coordinates


Graph Func :V= VI=10-3x &y
%T —4 YIE|1)0—3x [—1 V2= (129)%—4
y=
[
|S
[—1
r—1 INTSECT
() (R RE G

So, the solutionis = =4.2, y= —2.6.

i We rearrange both equations, so the system is now:


-3
y=gr+3 f Gomis] E [EXE]:Show coordinates
y=8— %96 Graph Func : Y Y1=(3.6)x+3et 13 y

vigdx+s
8
Y258—§x
g
INTSECT
G DELETE] TYPE ] TOOL. X=1.530612245 “¥=3.018357347

So, the solution is x ~ 1.53, y ~ 3.92.


Chapter 1 (Straight lines) Activity 2 Simultaneous equations using technology 41

2 a We rearrange the second equation, so the system is now:

{y:2~5l‘”6 f EoEiE
. 13y
y=25c+6 Y182.5x+6
Y2.5x+6
: [—1
[—1
[—1

Since the equations are identical, the lines are coincident.


there are infinitely many solutions.
b We rearrange the first equation, so the system is now:

{y=—0-6r+0-8 B Eomm ]
y=—-0.6z+2 Y{E;—’.ngr‘lg T —
Y28- . 6x+2 =]
: [—1
[—1
Y6 : [—1
(SELECT) W73 RAZGT=) MW HODIFY) (DR AW)

Since the equations have the same gradient, the lines are parallel.
there are no solutions.
¢ We rearrange both equations, so the system is now:
wolo

W=

(Hath)Rad Forn])
8

Graph Func :Y
Y=
wlon

W=
8

TYPE ] TOOL JI

Since the equations are identical, the lines are coincident.


there are infinitely many solutions.

PART 2: USING THE EQUATION SOLVER FUNCTIONALITY

3r+4y=1 ) el Forn) Hat g Forn]


1 a o2y
oy — =T anX+bnY=Cg
3 @n i X+bn Y=Cn
1[ Y[ -2}
2| 1

[REPEAT]

So, the solutionis = =3, y = —2.

b r+dy=-2 8 EDlies) T06D


—3z+2y =13 8n X+bn Y=Ca
]

[REPEAT]

So, the solution is = = —4, y=0.5.


42 Chapter | (Straight lines) Activity 2 Simultaneous equations using technology

b6z +y=13 Wt Dedlars])_(d/cead B Tk


< . IT an X+bn Y=Cn an X+bn Y=Cn
2z — 3y =16 a b ’ m}
l[m Y[ -3.6.
2

[SOLVE] [CLEARICEDIT]

So, the solution is = =2.75, y = —3.5.

r+3y=1 ok Epalen)_ @kl


. an X+bn Y=Cn an X+bn Y=Cn
—3x+Ty =21 a b %
1[- Y[ 1.5}
2 -3

SOLVE) (EERCLEAR
EDT)

So, the solutionis = —3.5, y =15

e | Abe ol il 8 o Y=Cn
5.70 — 3.4y = 12.6 e an X+bn
;[
5.7
-2.3
-3.4
-1.3}
12.8.
-

DELETE

So, the solutionis = =4, y=3.

3.6 — 0.7y = —11.37 e T e


192 +2.7y =—123 [2XDV=C X+bn Y=Cn
an X[m}
c
1[ -0.7 -11.37] Y 3.9
2 4.9 2.7 -1.28

DELETE

So, the solution is = = —2.4, y=3.9.

2 a We rearrange the first equation, so the system is now:

-y=-3 Thision) @06 B ok


3w—y=1 an X+bn Y=Cn
i
an X+bn Y=Cn
2 3
=
PN

So, the solution is = =4, y=11.


Chapter 1 (Straight lines) Activity 2 Simultaneous equations using technology 43

b We rearrange the first equation, so the system is now:

{3I+y=1 8 Gobais) Gk B Goibdion] @oRd


4z — 3y = —6 an X+bin V=be1 an X+bn Y=Cn
X
1[ Y[1.6923]
2| a4

-0.2307692308
[JIEE)3 REPEAT]

So, the solution is x ~ —0.231, y ~ 1.69.

¢ We rearrange the second equation, so the system is now:

3r+5y=3 § Gobas GOk [l


2 —y="7 an X+bn Y=Cn an X+bn Y=Cn
b X M]
1[ Y[—1.153
2 2

[IEAY3 REPEAT]

So, the solution is = ~2.92, y~ —1.15.

d We rearrange the first equation, so the system is now:


r—y=-15
5.8¢ —4y = —6

(SOLVE) REIER CLEARI(EDIT]

So, the solutionis = =0, y=1.5.

e We rearrange both equations, so the system is now:

4.5z —y =4.75 8 Golblion) @oEs e Dedarn))_(2/c)


—y=13 an X+bn Y=Cn an X+bn Y=Cn
T—y=1 a b X
1[.@ v[—u,au]
2| 1

0.9857142857
(SOLVE) ERCLEARIEDIT]

So, the solution is 2~ 0.986, y ~ —0.314.

f We rearrange both equations, so the system is now:


br—y=0 8 G @i Esiileillon]) (TeiRed
T+3y =12 an X+'.2n Y—Cg a,,x)(+bn Y=Cn
1[1 -1 v[ 3,75]
2 1

[JNAY3 REPEAT]

So, the solution is = = 0.75, y =3.75.


44 Chapter 1 (Straight lines) Exercise 1E

0.22+09y =5 Eblon) GRED B WD) @k


3 ol an X+bna Y=Cn an X+bn Y=Cn
0.6z 4+ 2.7y = -3 b
No Solution

[SOLVE) EERCLEARICEDIT]

Both lines have the gradient —3,2 so the lines are parallel.
there are no solutions.

1 Let £x be the cost of a plate and £y be the cost of a bowl.


5r+2y=>53 .. (1)
3x+8y=93 ... ()
20z — 8y = —212 {(1) x —4}
3z + 8y =93
Adding, —17z - 119
=T
Substituting = =7 into (1) gives 5(7) + 2y = 53
3542y =53
L2y =18
y=9
So, a plate costs £7 and a bowl costs £9.
Check: In (2): 3(7)+8(9)=214+72=93

2 Let 2 minutes be the time taken to play the waltz once and y minutes be the time taken to play the
sonatina once.
L4 4+3y=33 ... ()
6r+y=25 .. (2)
4z + 3y =33
—18z -3y =-75 {(2) x -3}
Adding, —14zx = —42
Lor=3
Substituting = =3 into (1) gives 4(3) + 3y = 33
1243y =33
L 3y=21
y="71
So, it takes 3 minutes to play the waltz and 7 minutes to play the sonatina.
Check: In(2): 6(3)+7=1847=25
Chapter 1 (Straight lines) Exercise 1E 45

3 Let z m be the length of the short cable and y m be the length of the long cable.
L 2x4+5y=26 .. (1)
3v+4y =243 .. ()
—6x — 15y = =78 {(1) x =3}
6r+ 8y=486 {(2)x2}
Adding, —Ty =—-29.4
Ly=4.2
Substituting y = 4.2 into (1) gives 2z + 5(4.2) = 26
L2 +21=26
.2z =5
L ax=25
So, the short cables are 2.5 m long and the long cables are 4.2 m long.
Check: Tn (2): 3(2.5)+4(42)=75+168=243 «

L4 Let x be the number of points awarded


for hitting the red region and y be the
number of points awarded for hitting the
blue region.

a 52+3y=68 .. (1)
68 points 56 points
20+ 6y =56 ... (2)
—10z — 6y = —136 {(1) x —2}
2z + 6y = 56
Adding, —8x =-80
Lor=10
So, 10 points are awarded for hitting the red region.

b Substituting = =10 into (1) gives 5(10) + 3y = 68


. 50+ 3y =68
. 3y=18
L y==6
So, 6 points are awarded for hitting the blue region.
46 Chapter 1 (Straight lines) Exercise 1E

5 Let « be the number of 3 litre paint cans sold and y be the number of 5 litre paint cans sold.
o Bu4by=71 .. (1)
36z + 48y = 768 ... (2)
—360 — 60y = —852 {(1) x —12}
362+ 48y = 768
Adding, —12y = -84
L oy=7

Substituting y = 7 into (1) gives 3z + 5(7) =71


L3z +35="T1
. 3z =36
Lox=12
So, the store sold 12+ 7 =19 paint cans that day.
Check: In (2): 36(12) 4 48(7) =432+ 336 =768

Let $2 per hour be Lidia’s rate of pay before 5 pm and $y per hour be Lidia’s rate of pay after
5 pm.
On Monday Lidia worked from 2 pm to 7 pm which is 3 hours before 5 pm and 2 hours after 5 pm.
On Tuesday Lidia worked from 11 am to 8 pm which is 6 hours before 5 pm and 3 hours after
5 pm.
Lo3r+2y=110 ... (1)
6z +3y =195 ... (2)
—6z —dy = —220 {(1) x —2}
6z + 3y = 195
Adding, —y=-25
Ly=25
Substituting y = 25 into (1) gives 3z + 2(25) = 110
. 3z +50=110
. 3z =060
]
So, Lidia is paid $20 per hour before 5 pm and $25 per hour after 5 pm.
Check: In (2): 6(20) +3(25) =120+ 75 =195
On Wednesday, Lidia worked from noon to 6 pm which is 5 hours before 5 pm and 1 hour after
5 pm.
So, on Wednesday Lidia earned 5 x $20 + 1 x $25 = $100 + $25 = $125.
Chapter 1 (Straight lines) Exercise 1E 47

7 a Let x be the number of seconds Kenenisa ran at 6.5 ms™ !, and y be the number of seconds
Kenenisa sprinted at 7.7 ms™1.
Kenenisa took 12 min 37.35's = 12 x 60 + 37.35 = 757.35 s to run 5000 m.
6.5z + 7.7y = 5000 ... (1)
T4y ="75735 .. (2)
6.5z + 7.7y = 5000
—6.5z — 6.5y = —4922.775 {(2) x —6.5}
Adding, 1.2y = 77.225
L y=3 ~644
So, Kenenisa sprinted for approximately 64.4 seconds, for a total distance of
64.4 x 7.7 ~ 496 m.
Let x be the number of seconds Kenenisa ran at 6.3 ms™—! , and y be the number of seconds
Kenenisa sprinted at 7.5 ms~!.
Kenenisa took 26 min 17.53 s = 26 x 60 4 17.53 = 1577.53 s to run 10000 m.
6.3z + 7.5y = 10000 ... (1)
z+y=1577.53 ... (2)
6.3z 4+ 7.5y = 10000
—7.5z — 7.5y = —11831.475 {(2) x —7.5}
Adding, —1.2z = —1831.475
x =329 ~ 1526.2
Kenenisa began sprinting after approximately 1526.2 seconds (= 25 min 26 s) or after
1526.2 x 6.3 ~ 9615 m.

y=2+2 .. (1)
z+y=9 e (2)
y=2 .. 3)
Substituting (1) into (2): =+ (z+2)=9
2 +2=9
20 =17
o1
Substituting * = into (1): y=2+2=24
NI~

So, (1) and (2) intersect at (5, 4t).


Substituting y = 2 into (1) and (2) gives us the remaining points of intersection (0, 2) and
(7, 2).
The triangle has base length 7 — 0 = 7 units and height 3 —2 = Z units.
area =4 x7x %
_1
=7
= 127 19l units 2
48 Chapter 1 (Straight lines) Exercise 1E

b bxr—2y=18 e (D)
2x 45y =13 e (2)
8xr—9y =114 ... (3)

2%z —10y =90 {(1) x5}


dr+10y =26 {(@2)x 2}
Adding, 292 —116
Lax=4
Substituting = =4 into (1) gives 5(4) — 2y = 18
20— 2y =18
—2y = -2
y=
So, (1) and (2) intersect at A(4, 1).
Also, -8z —20y=-52 {(2)x —4}
8o — 9y=11.4
Adding, 29y = —40.6
Ly=1

Substituting y = % into (2) gives 2z + 5(%) =13


L 2x4+7=13
2z =6
=3
So, (2) and (3) intersect at B(3, %)
Also, 45z — 18y = 162 {(1) x 9}
—16x + 18y = —22.8 {(3) x =2}
Adding, 29z =139.2
Sor=2=438
Substituting = = 4.8 into (3) gives 8(4.8) — 9y =114
384 -9y=114
9y =27
.. y i 3

So, (1) and (3) intersect at C(Z, 3).


Now, 2z + 5y =13 has gradient —2 and 52 — 2y = 18 has gradient 3, so [AB] and [AC]
are perpendicular.
ABC is a right angled triangle.

AB=1/(3- 42+ (Z-1)° AC= /(B -4+ 3-1)2


SN 2
- Jar 2

_ /2 _ Jus
% 25
_ V29 _ 2v/29
5 5
- areaofAABC:%x@ x@
_= 5529 —= 1514 units
nite?
Chapter 1 (Straight lines) Review set 1A 49

o[z
a7 [ ]
12

L
T 23 13

b Yes, the variables are linearly related as the points all lie on a straight line.
17—-20
¢ The line passes through (0, 20) and (1, 17), so the gradient is -3.
The y-intercept is 20.
d The gradient is —3 and the y-intercept is 20, so the equation is y = —3z + 20.
e When =7, y=-3(7)+20
=-1

2 a The graph passes through (0, 50) and 400 wedkly fhcome (£1)
(2000, 200)
.. gradient = el 300
2000 -0
— 150
23000 200
=45 = 0.075
The weekly income of a salesperson increases 100
by £0.075 for each pound increase in sales
that week. weekly sFles £S5)
The I-intercept is 50, which means that the 0 >
. . 1 2 4
weekly income of a salesperson is £50 before ) 000 Q009000 R
any sales are made.
b The line has gradient 0.075 and [-intercept 50.
the line has equation I = 0.075S + 50.

¢ When S =3400, I =0.075(3400)+ 50


=305
the salesperson’s weekly income is £305.

3 a The equation of the line is b y=—%z+4


y—2=—%(z—6) o3y =—x+12
L y—2=-ir+2 sox+3y—12=0
y=—2r+4
50 Chapter 1 (Straight lines) Review set 1A

4 a Line2is parallel to y—4 = 2(x—1), which


has gradient .
line 2 has gradient % and passes through
(6, 3).
line 2 has equation y —3 = 2(z — 6)
y—3= %x -9
L y=3z-6
2y =3x — 12
L3 —2y =12

b When y=0, 3z-2(0)=12


3 =12
r=4
the z-intercept of line 2 is 4.

5 a When x =25, we have b When 2 = —3, we have


y=-5+3 3(=3)+8y=-5
=-2 Vv S —94+8y=-5
So, (5, —2) does lie on the line. co 8y =
Loy = % v

So, (=3, %) does lie on the line.

6 a For y=2z-3: y
e the y-interceptis ¢ = —3
o the gradientis m = % - ) =
xT

=2z|— 3

b For y:72w+1:
e the y-interceptis c¢=1
e the gradientis m = 7% = _TS

v
Chapter 1 (Straight lines) Review set 1A 51

For 5z 4+ 3y = 30:
When =0, 3y=230
y=10
So, the y-intercept is 10.
When y =0, 5z =30
=6
So, the z-intercept is 6.

For y=2a+1:
e the y-interceptis c=1
e the gradientis m = 23

For 3z —4y =72: Ay


When =0, —4y=7T2 24
y=-—18 z
So, the y-intercept is —18.
3z —4y="72
When y =0, 3z=72
Lo =24
So, the z-intercept is 24.

For 2z 4+ 5y = —20:
When =0, 5y=-20
Ly=-—4
So, the y-intercept is —4.
When y =0, 2z 20
LT 10
So, the z-intercept is —10.

The midpoi
M ofnt
[AB] is (% 2*;‘4)) or (5, —1). A(5.2)
The gradient of [AB] is 42 %6 which is undefined.

So, [AB] is a vertical line, and hence the perpendicular <—QW>


bisector of [AB] is a horizontal line through (5, —1). i
the equation of the perpendicular bisector is y = —1. i
52 Chapter 1 (Straight lines) Review set 1A

b The midpoint
M of [AB] is (% %) or (5,3). B(2.5)e._
The gradient of [AB] is I o = 2
2-8 —6 3

the gradient of the perpendicular bisector is %


the equation of the perpendicular bisector is 3z — 2y = 3(5) — 2(3)
whichis 3z —2y=09.

We draw the graphs of y =2+ 1 and


y = —2x + 10 on the same set of axes.
The graphs meet at the point (3, 4).
the solution is =z =3, y = 4.

Check:
Substituting these values into:
e y=x+1 gives 4=3+1
o y=—2x+10 gives
4=-2(3)+10=-6+10 v

We draw the graphs of = + 3y =6 and


x —y = —6 on the same set of axes.
The graphs meet at the point (—3, 3).
the solutionis = —3, y=3.

Check:
Substituting these values into:
e z+3y=06 gives
—-3+3(3)=-3+9=6 V
e v —y=—6 gives -3—-3=—-6 v

9 a y=3c+4 ..(D)
20 —y=-5 ..(2)
Substituting (1) into (2) gives 2z — (3z +4) = —5
20 -3z —4=-5
—x=-1
=1

Substituting « =1 into (1) gives y=3(1)+4


y=71
The solutionis z =1, y=T7.

Check: (1) 7T=3(1)+4=3+4 v


@ 20)-7=2-7=-5 v
Chapter 1 (Straight lines) Review set 1A 53

b z=2y-5 ..
Jz+4y=5 ... (2)
Substituting (1) into (2) gives 3(2y —5) +4y =5
. 6y—15+4y=5
10y = 20
L y=2
Substituting y = 2 into (1) gives x =2(2)—5
i = 1
The solutionis =z = -1, y=2.
Check: (1) —-1=2(2)-5=4-5 V
2) 3(-1)+4(2)=-348=5 Vv

B 3r4+2y="7
10
br —2y =17
The coefficients of y are the same size but opposite in sign.
We add the LHSs and the RHSs to get an equation which contains x only.
x+2y=7 .. (1)
br—2y=17 ... (2)
Adding, 8z =24
x=3
Substituting x =3 into (1) gives 3(3) + 2y
942y
2y =—2
y=-—1
The solutionis =z =3, y=—1.
Check: In(2): 5(3)—2(-1)=15+2=17

b 2x+T7y=13 e (D)
—4r+3y=25 .. (2
To make the coefficients of = the same size but opposite in sign, we multiply (1) by 2.
odr+ 14y =26 {(1) x 2}
—4dx+ 3y=25
Adding, 17y =51
Ly=3
Substituting y = 3 into (1) gives 2z + 7(3) = 13
L 2x4+21=13
o 2r=-8
L x=—4
The solution is = = —4, y=3.
Check: In (2): —4(—4)+3(3)=16+9=25
54 Chapter 1 (Straight lines) Review set 1A

a y=—%x+4 has gradient m =—% and y-intercept ¢ =4.


b When = =6, we have ¢ Substituting x =k, y =5 into the
y=—%(6)+4 equation gives 5= —%k+4
=1 v coo—3k=1
So, (6, 1) does lie on the line. o k=-2

a Let €x be the cost of a table and €y be the cost of a chair.


w44y =200 ... (1)
8y =460
32+ ... (2) B B B B8

ewe o
3z + 8y = 460
BEm
B
@] o % E %
Adding, T =60
€200 €460
So, each table costs €60.

b Substituting 2 = 60 into (1) gives 60 + 4y = 200


4y =140
L y=35
So, each chair costs €35.
Check: In (2): 3(60) + 8(35) = 180 + 280 = 460 v

13 Let 2 be the number of working batteries and y be the number of faulty batteries.
Lx4+y=37 .. ()
2 +5y=83 .. ()
—2z -2y =-T4 {(I)x -2}
2z + 5y = 83
Adding, 3y=9
.‘ y - 3

There were 3 faulty batteries.


Chapter 1 (Straight lines) Review set 1B 55

14 a
.i The
.
mliiponzt of [AC]
.is (
(34(—4)
>
2+(—3))
5
D(—3.3
(—3.3) 5(3’ )

or (~3.-3).
The gradient of [AC] is 8-2_
—4-3
55
-7 7
the gradient of the perpendicular bisector is — %
the equation of the perpendicular .
bisector is 7z + 5y = 7(—%) +5(—3%) C(-4,-3)
which is 7z + 5y = —6.

iiii dnoi of [BD] is:


s oint 24+ (—3) —4+3 ) [
The midp ( o 3 or (_5, _5).

The gradient of [BD] is 3-(y _ 7 __7


—3-2 -5 5
the gradient of the perpendicular bisector is %
the equation of the perpendicular bisector is 5z — Ty = 5(7%) - 7(7 )

wl—=
whichis bz — Ty = 1.

b AC=+/(-4-3)2+(-3-2)? BD = /(-3 -2)


(3 2
- (—4)
+)?
= /(=7
+ (-5
2)2 =4/(-5)2+72
=1/49 + 25 =+/25+49
= /74 units = /74 units

Now, [AC] and [BD] both have midpoint (-4, —%), and the gradients of [AC] and [BD] are
negative reciprocals of each other.
[AC] and [BD] are perpendicular bisectors of each other, and equal in length.
ABCD is a square with diagonals [AC] and [BD].

1 a The graph passes through (0, 5) and (1, 15), so the Aspeed (yms—1)
gradient is —— 5 _ 10. This means that the speed of oS
the pebble increases by 10 ms~" each second. 30
The y-intercept is 5. This means that the initial speed
was b ms~L 2
b The gradient is 10 and the y-intercept is 5, so the equation 10
is y =10z +5. time (@ seconds)
¢ When z=8, y=10(8)+5 % 7 3 4 5
=85
The speed of the pebble after 8 seconds is 85 ms~!.

2 a Since the line has gradient %, the general form of its equation is 5z —y =d
Using the point (2, —1), the equation is 5z —y = 5(2) — (—1)
which is 5z —y =11.
56 Chapter 1 (Straight lines) Review set 1B

b Since the line has gradient 7%, the general form of its equation is = + 4y = d
Using the point (—3, —4), the equation is x + 4y = —3 + 4(—4)
which is z + 4y = —19.

3 a The line is parallel to y = 3z — 8, which has gradient 3.


the line has equation y = 3z + c.
Substituting =2, y=7, weget 7=3(2)+c
c=1
the line has equation y = 3z + 1.

b 224+5y="7
or y=1%—2x has gradient —2.
the line perpendicular to 2z + 5y = 7 has gradient %
Since the line has gradient %, the general form of its equation is 5z — 2y = d.
Using the point (—1, —1), the equationis bz — 2y = 5(—1) — 2(—1)
which is bz — 2y = —3.

L a Substituting x =2, y =k into the equation gives k=5 2) -3


k=T

b Substituting = = %, y —=23 into the equation gives 5(%) + 9(—%

5 a For 2z —3y=18:
When =z =0, —3y=18
h y=—6

So, the y-intercept is —6.


When y =0, 2z=18
=9
So, the z-intercept is 9.

b For y:gwf?;:
o the y-interceptis ¢ = —3
o the gradientis m = £
Chapter 1 (Straight lines) Review set 1B 57

¢ For 3z + 2y = 30:
When =0, 2y=30
Ly=15
So, the y-intercept is 15.
When y =0, 3z=30
=10
So, the z-intercept is 10.

For y= -2z +5:


e the y-interceptis ¢ =25
e the gradientis m = —2 = _TZ

For y=—dat & "


o the y-intercept is ¢ =3
_ 3
. .
m = 7% = Tl 4 —
e the gradientis
- >
4

y=—3T+3

For b5x 4 2y = —30:


When =0, 2y=-30
y=—15
So, the y-intercept is —15.
When y =0, 5xr=-30
. x=—6
So, the z-intercept is —6.

6 The midpoint M of [PQ] is (‘32+ 5 2+ (—6)) or (1, —2).

= 6 2 w B
The gradie; nt of [PQ] is 5 )-8 - -1

the gradient of the perpendicular bisector is 1.


the equation of the perpendicular
bisectoris z—y=1—(-2)
whichis z—y=3
y=x—3
58 Chapter 1 (Straight lines) Review set 1B

7 a 1 xz-5y+6=0
L bhy=x+4+6
y=1x+2 which has gradient <.
il 2—5y+6=0 has gradient %, so its perpendicular bisector has gradient —5.
b The perpendicular bisector has gradient —5, so the general form of its equation is 5z +y = d.
Using the point (4, 2), the equation is Su+y=>5(4)+2
which is Sr +y = 22.

a yh We draw the graphs of y =32+ 1 and


z —y =3 on the same set of axes.
The graphs meet at the point (—2, —5).
the solution is =z = —2, y = —5.

Check:
Substituting these values into:
o y=3x+1 gives
-5=3(-2)+1=-6+1 Vv
o r—y=23 gives
—2—(-5)=-2+5=3 V

We draw the graphs of 2z +y =6 and


z — 2y =8 on the same set of axes.
The graphs meet at the point (4, —2).
the solutionis =z =4, y=—2.

Check:
Substituting these values into:
e 2x+y==6 gives
2(4)+(-2)=8-2=6 Vv
e 1 —2y=28 gives
4-2(-2)=4+4=8 V

a y=6x+2 e (1)
3r—2y=-7 e (2)
Substituting (1) into (2) gives 3z —2(6x +2) = —7
3v—120 —4=-T7
-9z = -3
=1
T
Substituting @ = 1 into (1) gives y=6(1)+2

The solution is z =%, ¢


Check: (1) 4 =6(- 1
@ 3(3)-24)=1-8=-7
Chapter 1 (Straight lines) Review set 1B 59

b y==L1z+5 .. (1)
dr+3y=4 ... (2
Substituting (1) into (2) gives 4a + 3(3x +5) =4
- 4x+%x~ 15 =
Lr=-11
=2
Substituting = = —2 into (1) gives y = 3(-2)+5
y=4
The solution is = = -2, y=4.
Check: (1) 4:%(72)+5:71+5 v
2) 4(-2)+3(4)=-8412=4

a 3x+2y=8 ..(1)
br—4dy =17 .. (2)
To make the coefficients of y the same size but opposite in sign, we multiply (1) by 2.
b +4y =16 {(1)x2}
br —4y =17
Adding, 1lz =33
Lr=3
Substituting « =3 into (1) gives 3(3)+2y =8
L9+ =38
2y =—1
y=—3%
The solution is = =3, y=—1.
Check: Tn (2): 5(3)—4(-3)=15+2=17 v

b dx+6y=-15 ... (1)


3v — by =22 e (2)
To make the coefficients of y the same size but opposite in sign, we multiply (1) by 5 and
(2) by 6.
20z 430y =—75 {(1)x 5}
182 — 30y =132 {(2) x 6}
Adding, 38z =57
Loa=13
Substituting 2 = 13 into (1) gives 4(13)+6y = —15
646y =—15
6y
= —21
y=-33
The solution is = =14, y = —31.

Check: In (2): 3(13) —5(—3%) =45 +17


60 Chapter 1 (Straight lines) Review set 1B

11 a y==2r—%§ hasgradient
m =2
b i When z = —2, we have il When = =4, we have

y=3(-2)-3% y=34)-3%
. 28 —8_8
=737 3 3. 3
=—4 v =0 x
So, (—2, —4) lies on the line. So, (4,5) does not lie on the line.

C Ay

(4,5)"

12 Let 2 be the number of individual tickets sold and y be the number of books of 10 tickets sold.
x4+ 10y =500 ... (1)
3z +20y = 1350 ... (2)
—3z — 30y = —1500 {(1) x =3}
3z + 20y = 1350
Adding, —10y = —150
y=15
So, 15 books of 10 tickets were sold.

13 Let 2 be the number of one hour lessons and y be the number of two hour lessons given that week.
cox+2y=25 .. (1)
30x + 50y =690 ... (2)
=30z — 60y = =750 {(1) x —30}
30x + 50y = 690
Adding, —10y = —60
y==6
The piano teacher gave 6 two hour lessons that week.

15 a | The midpoint
of [AB] is (3+;*1>,%) or (1, 5).
The gradient of [AB] is SO 2 Ta sl
-1-3 —4 2
the gradient of the perpendicular bisector is —2.
the equation of the perpendicular B(-1, 4).-“'
bisector is 2z +y =2(1)+5
whichis 2z 4+y=7.
Chapter 1 (Straight lines) Review set 1B 61

il The midpoint of [AC] is (% %) or (2,3). #A(3,6)


The gradient of [AC] is
0— 6_=6_,
1— 3 -2 [2.3)
the gradient of the perpendicular bisector is —%.
the equation of the perpendicular !
bisector is x + 3y =2+ 3(3) C(l,O)'f
which is x4+ 3y = 11.

iii The midpoint of [BC] is (‘12“, —420) or (0,2). B(-L4),


: . 0—-4 —4
The gradient of [BC] is D -7 - -2

the gradient of the perpendicular bisector is %


the equation of the perpendicular
bisector is = —2y =0—2(2)
which is = —2y = —4.

1,0)
All three perpendicular bisectors intersect at (2, 3).
A, B, and C are all equidistant from this point.
B lies on the perpendicular bisector of [AC], which means AB = BC.
So, triangle ABC is isosceles.
Chapter 2
SETS AND VENN DIAGRAMS

1 abseD b 6¢G ¢ d ¢ {all English vowels}


{2,5} C{1,23,4,5, 6} e {3,861 ¢{1,23,4,5,6}

2 a {factors of 10} = {1, 2, 5, 10}


There are 4 elements in this set.
this is a finite set.
b {multiples of 10} = {...., —20, —10, 0, 10, 20, ....}
This set has an endless number of elements.
this is an infinite set.

¢ {perfect squares} = {1, 4, 9, 16, 25, ....}


This set has an endless number of elements.
this is an infinite set.

3 a A = {factors of 8} b A = {composite numbers less than 20}


={1, 2,4, 8} =1{4,6, 8,9, 10, 12, 14, 15, 16, 18}
n(A) =4 n(A) =10
< A = {letters in the word AARDVARK}
={A,R,D,V,K}
n(A) =5
d A = {months of the year}
= {January, February, March, April, May, June, July, August, September, October,
November, December}
n(A) =12
e A = {prime numbers between 40 and 50}
= {41, 43, 47}
n(A) =3

4 §=1{1,2,450912} and T ={2 59}


a 1 n(S)=6 it n(T)=3
b i 4isanelement of S. ii 4 is not an element of T'.
4 € S is a true statement. . 4 €T is a false statement.
iii 1 is not an element of 7.
1¢ T is a true statement.
iv Every element of 7" is also an element of S.
T C S is a true statement.
v T'CS {fromiv}
However, 1, 4, and 12 are all elements of .S but not elements of 7', so T # S.
T C S is a true statement.
Chapter 2 (Sets and Venn diagrams) Exercise 2B 63

5 S={1,2} and T ={1,2,


3}
a subsets of S: @, {1}, {2}, {1, 2}
subsets of T &, {1}, {2}, {3}, {1, 2}, {1, 3}, {2, 3}, {1, 2, 3}
b Yes, every subset of S is also a subset of 1".
¢ There are 4 subsets of S and 8 subsets of 7.
-, the fraction of the subsets of 7" which are also subsets of S =

ool
ol
6 Each of the four elements of {p, q, r, s} can be included or not included in a subset.
the set has 2 x 2 x 2 x 2 = 16 subsets.

7 a A= {prime numbers between 30 and 40}


= {31, 37}
B = {even numbers between 30 and 40}
= {32, 34, 36, 38}
C = {composite numbers between 30 and 40}
= {32, 33, 34, 35, 36, 38, 39}
D = {multiples of 21 between 30 and 40}
=y
b i n4)=2 il n(D)=0
< i The elements of B and the elements of C' are not in A.
Every set is a subset of itself. The empty set @ is a subset of all other sets.
So, A and D are subsets of A.
ii Every element of B is also an element of C, so B C C.
However, 33, 35, and 39 are all elements of C' but not elements of B, so B # C.
.. BccC.
The empty set @ is a subset of all other sets, and since C' # @, then D C C.
So, B and D are proper subsets of C'.
d i 33 is an element of C'. ii 37 is an element of A.
33 € C is a true statement. o, 37¢ A is a false statement.
iii 35 is not an element of B.
35 € B is a false statement.
8 A=1{2,4,6,2} and B=1{2,3,506, z+1}
A C B which means every element of A must also be an element of B. The elements 2 and 6 are
in both A and B, so the element 4 in A must be represented by =+ 1 in B.
Ld=z+1
r=3

1 a A={6,7,9,11,12} and B = {5,8, 10,13, 9}


i ANnB = {9} since 9 is the element common to both sets.
ii Every element which is in either A or B is in the union of A and B.
AUB ={5,6,7,8,9, 10, 11, 12, 13}
64 Chapter 2 (Sets and Venn diagrams) Exercise 2B

b A={1,2,3,4} and B={56,7,


8}
i AN B =@ since there are no elements which are common to both sets.
ii Every element which is in either A or B is in the union of A and B.
AUB=1{1,2,3,4,5,6,7, 8}
¢ A={1,3,57 and B=1{1,2,3,4,56,7,8,
9}
i AnB={1,3,5, 7} since 1, 3, 5, and 7 are the elements common to both sets.
ii Every element which is in either A or B is in the union of A and B.
AUB=1{1,23,4,5,6,7,38,9}
d A=1{0,3,58,14} and B={1,4,5,8, 11,13}
i AN B = {5, 8} since 5 and 8 are the elements common to both sets.
il Every element which is in either A or B is in the union of A and B.
AUB=1{0,1,3,4,5,8, 11, 13, 14}
2 a A=1{3,5"7,9} and B={24,6,38}
Sets A and B have no elements in common.
. these sets are disjoint.
b P=1{3,56,7,8 10} and Q= {4,9, 10}
The element 10 is common to both sets.
these sets are not disjoint.

3 A = {even numbers between 20 and 30}


= {22, 24, 26, 28}
B = {odd numbers between 20 and 30}
= {21, 23, 25, 27, 29}
C = {composite numbers between 20 and 30}
= {21, 22, 24, 25, 26, 27, 28}

a Sets A and B have no elements in common.


sets A and B are disjoint.
b ANC = {22, 24, 26, 28}
Every element in A is also an element of C', so A C C.
However, 21, 25, and 27 are all elements of C' but not elements of A, so A # C.
AcC
¢ Every element which is in either B or C is in the union of B and C.
- BUC = {21, 22, 23, 24, 25, 26, 27, 28, 29}
n(BUC) =9

L a True, RNS =@ tells us that R and S have no elements in common, and hence are disjoint.
b True, every element of AN B is an element of A, and every element of AN B is an element
of B.
n(ANB) <n(A) and n(ANB) < n(B).
¢ True, if ANB = AUB then there are no elements that are in only A or only B.
A=B.
Chapter 2 (Sets and Venn diagrams) Exercise 2C 65

1 U={1,23,45,6,7,8,9}
a The complement of A = {2, 3, 6, 7, 8} is the set of all elements of U that are not elements
of A.
A'=1{1,4,5,9}
b P = {prime numbers} {composite numbers} = {4, 6, 8, 9}
={2,3,5,7}
P’ is the set of all elements of U that are not elements of P.
P ={1,4, 6,8, 9} # {composite numbers} as 1 is neither prime nor composite.
2 U = {months of the year}
= {January, February, March, April, May, June, July, August,
September, October, November, December}

a M = {months starting with J}


= {January, June, July}
M’ is the set of all elements of U that are not elements of M.
M’ = {February, March, April, May, August, September, October, November, December}
b A = {months containing the letter A}
= {January, February, March, April, May, August}
A’ is the set of all elements of U that are not elements of A.
A" = {June, July, September, October, November, December}

3 U = {whole numbers between 10 and 20 inclusive}


= {10, 11, 12, 13, 14, 15, 16, 17, 18, 19, 20}

a A = {factors of 120} b B = {multiples of 3}


= {10, 12, 15, 20} = {12, 15, 18}
¢ A’ is the set of all elements of U that are not elements of A.
A’ = {11, 13, 14, 16, 17, 18, 19}
d B’ is the set of all elements of U that are not elements of B.
B = {10, 11, 13, 14, 16, 17, 19, 20}
e ANB = {12, 15} since 12 and 15 are the elements common to both sets.
f Every element which is in either A or B is in the union of A and B.
AU B = {10, 12, 15, 18, 20}
g A'NB = {18} since 18 is the element common to both sets.
h Every element which is in either A’ or B is in the union of A’ and B.
A'UB = {11, 12, 13, 14, 15, 16, 17, 18, 19}
i ANB’ = {10, 20} since 10 and 20 are the elements common to both sets.
i Every element which is in either A or B’ is in the union of A and B’.
AUB = {10, 11, 12, 13, 14, 15, 16, 17, 19, 20}
Ik A'n B’ = {11, 13, 14, 16, 17, 19} since these are the elements common to both sets.
| Every element which is in either A’ or B’ is in the union of A’ and B'.
A UB = {10, 11, 13, 14, 16, 17, 18, 19, 20}
66 Chapter 2 (Sets and Venn diagrams) Exercise 2D

4 a U={2,3,4,56728}, A={3.57}, B={2478}


i n(U)=7 ii n(A)=3 i n(A")=n(U)-n(4)
—7_

iv n(B)=4 v n(B')=n(U) —n(B)


=7-4
=3
b For any set S within a universal set U, n(S)+n(S’) = n(U).

5 n(U)=15 n(P)=6, n(Q)=4


a n(P)+n(P') = n(U) b n(Q) +n(@) = n(V)
L 6+n(P)=15 Son@) +4=15
L on(P)=9 son(Q) =11

1 First noticethat NCZ, ZCQ, and Q CR.


—2 ¢ NorZ asitisnotan integer. —2 € Q since —3 and 8 are integers, .. —2 € R.
1.8 ¢ NorZ as it is notan integer. 1.8 € Q since 1.8 = %, 18 and 10 are integers,
1.8 R,
18¢ NorZ asitisnot an integer. 1.8 € Q since 1.8 =4, 17 and 9 are integers,
18R
—17¢ N as —17<0. —17€Z asitisaninteger, .. —17 € Q and R.
V64 €N as /64 =8, which is an integer, .. /64 € Z, Q, and R.
2 ¢ Q aswisirrational, . Z¢ZorN. Z €R asitcan be placed on the number line.
V=3 ¢ R as it cannot be placed on the number line, .. /=3¢ Q, Z, or N.
-3 ¢Q as V3 is irrational, .. —/3 ¢ Z or N. —V/3 eR as it can be placed on the number
line.

So, the table is:


Chapter 2 (Sets and Venn diagrams) Exercise 2D 67

—-7T€Z but —7¢Z". b 2 is not an integer.


—7 €Z" is a false statement. o 2¢ 7 is a true statement.
—/4=—2 which is an integer. d /3 is irrational.
—V/4 €7 is a true statement. - V3 €Q is a false statement.
% € Q since 7 and 9 are both integers. f 0.201 is not an integer.
% € Q is a true statement. . 0.201 € Z is a false statement.

gt @, so —— € Q since 700 and 31 are integers.


0.31 31 0.31
7
— € is a true statement.
0.31 Q

V-1 =+v1=1 which is real.


V| —1| €R is a true statement.

Every element of Z* is also an element of N.


7+ C N is a true statement.
N C Z since every element of N is also an element of Z.
However —1, —2, —3, .... are all elements of Z but not elements of N, so N # Z.
N C Z is a true statement.

N#Z" as 0N but 0¢ Z*.


N =Z% is a false statement.
7~ C Z since every element of Z~ is also an element of Z.
7~ CZ is a true statement.

Q contains fractions such as %, which are not integers.


Q C Z is a false statement.
{0} is the set containing the element 0 only, which is in Z.
{0} C Z is a true statement.
7Z C Q since every element of Z is also an element of Q.
Z C Q is a true statement.
Every element which is in either Z* or Z~ is in the union of Z* and Z™~. 0 however is in Z
but not in Z* nor Z~.
7+t UZ~ =17 is a false statement.

The set of integers between 10 and 20 = {11, 12, 13, 14, 15, 16, 17, 18, 19}.
The number of elements is a particular defined value.
this set is finite.
The set of integers greater than 5 = {6, 7, 8, 9, 10, ....}.
This set has an endless number of elements.
this set is infinite.
The set of all rational numbers between 0 and 1 = {3, %, i %, et
This set has an endless number of elements.
this set is infinite.
68 Chapter 2 (Sets and Venn diagrams) Exercise 2E

5 U=7Z=/{0,+1, £2, £3, +4, ...}


7t ={1,2,3,4, ..}
The complement of ZT is the set of all elements of U that are not elements of ZT. This includes
{0} and Z~.
the complement of Z* is Z~ U {0}.

1 a A={ze€Z|-1<2<T7}
i The set of all = such that 2 is an integer between —1 and 7, including —1 and 7.
it A={-1,0,1,2,3,4,5,6, 7} iii n(4)=9
b A={zeN|-2<2<8}
i The set of all = such that x is a natural number between —2 and 8.
it A={0,1,23,4,56,7} ili n(4)=8
¢ A={z|0<2<1}
i The set of all real such that x is greater than or equal to 0, and less than or equal to 1.
ii It is not possible to list the elements of A.
ili A is an infinite set, so n(A) is undefined.
d A={z€Q|5<2<6}
i The set of all = such that z is a rational number greater than or equal to 5, and less than
or equal to 6.
ii It is not possible to list the elements of A.
iii A is an infinite set, so n(A) is undefined.

2 a {ze€Z'|x<5} can be represented by: <l dddddl,

b {zeN|z <5} can be represented by:


0 45 =

¢ {xeZ|—-2<x<3} can be represented by: . Lddelly


-2 0 3 z

d {zx€Z'|3<x<T} can be represented by: S ‘_3_3_# \-

e {zeR|xz >3} can be represented by: T


4—3—;

f {z €R|xz <6} can be represented by: -0


-
6 T
g {reR|2< 2 <6} can be represented by: —
2 6 T

h {z]3.6 <2z <10.2} can be represented by: —eo


Chapter 2 (Sets and Venn diagrams) Exercise 2E 69

{z |2 <3} U {x|x>6} can be represented by: o0 o—>


T3 6 =
{z|2<2} U {z|x>4} can be represented by: e o—>
2 4 =
{z ]2z <3} U {z|7<x <12} can be represented by: <« o—o
il | =
3 7 12 %

{zeZ"|x<6} U {x€Z"|8<x< 11} can be represented by:


<Lososseioode»
0 5 10 T

The set of all integers between —100 and 100 can be represented by
{r€Z|-100 <z < 100}.
The set of all real numbers greater than 1000 can be represented by {z € R | = > 1000}.
The set of all rational numbers between 2 and 3 inclusive can be represented by
{reQ|2<z<3}

- *r—0
4—‘—» -lL
8 T —1 4 T

can be represented by {z |z > 8}. can be represented by {z | -1 <z < 4}.


+—0 o—>
-—
-3 0 4 x —5 1 T

can be represented by can be represented by


{reZ|-3<xz<4} or {z|z<-5} U {z|z>1}
{zeZ|-2<z<3}.
+—0
1
-—
T 0 4 6 T
ol

can be represented by {a |z < 2}. can be represented by


{zxeN|jz<4} U {zeN|z =6}
e >~ <+——0 o—————o0
1
-— L me Tl _sue) s TN
-2 3 T -2 0 2 T

can be represented by can be represented by


{z|z< -2} U {z|z>3}. {z]z<-2} U {z]0<z<2}
——o o
11

1 4 6 T

can be represented by
{z|1<2<4} U {z]|z>6}

A=, B={257,9}
The empty set @ is a subset of all other sets.
ACB
70 Chapter 2 (Sets and Venn diagrams) Exercise 2E

b A={2578,9}, B={809} ¢ A={zeR|2<z<3}, B={zeR}


Only two elements of A are in B. Every element of A is in B.
A¢ B .. ACB
d A={2z€Q|3<2<9}, B={zcR|0<z<10}
Every element of A is in B.
. ACB
e A={z€Z|-10<x<10}, B={2€Z|0<2z<5}
The elements —10, —9, —8, ..., —1, and 6, 7, 8,9, 10 are iAnbut not in B.
A¢B
fA={zcQ|0<2<]}, B={yecQ|0<y<2}
The element
0 is in
A but not in B.
A¢B

6 a U=Z and C={zcZ|x< -5}


C'={z€Z|z>-4}
b U=Q and C={zcQ|2z<2} U {z€Q|z>8}
C'={reQ|2<z<8}
7 U={2€Z|0<2<8}, A={2ecZ|2<a<T}, B={reZ|5<x<8}
a A=1{2,34,567} b A'={0,1,8}
¢ B=1{56,7,8} d B’ =1{0,1,23, 4}
e ANB=1{56,7} f AUB=1{2,3,4,56,7,8}
g ANB ={2,3 4}
8 U=2%, P={zeZt|9<a<16}, Q=1{2 4,5, 11,12, 15}
a P =1{9,10, 11,12, 13, 14, 15} b PNQ = {11, 12, 15}
¢ PUQ=1{24,5,9, 10, 11, 12, 13, 14, 15}
d n(PUQ)=10 and n(P)+n(Q)-n(PNQ)=T7+6-3=10
n(PUQ) =n(P)+n(Q)—n(PNQ)
9 U={zecZ|0<x<40}, P = {factors of 28}, @ = {factors of 40}
a P=1{1,24,714,28}, Q =1{1,24,5,8,10, 20, 40}
b PNQ=1{1,2 4}
¢ PUQ={1,2,4,5,7,8, 10, 14, 20, 28, 40}
d n(PUQ)=11 and n(P)+n(Q)—n(PNQ)=6+8-3=11
n(PUQ) =n(P)+n(Q) —n(PNQ)
10 U={2€Z]|30<z<60}, M = {multiples of4 between 30 and 60},
N = {multiples of 6 between 30 and 60}
a M = {32, 36, 40, 44, 48, 52, 56}, N = {36, 42, 48, 54}
b MNN = {36, 48}
¢ MUN = {32, 36, 40, 42, 44, 48, 52, 54, 56}
d n(MUN)=9 and n(M)+n(N)—n(MNN)=7+4-2=9
n(MUN)=n(M)+n(N)—n(MNN)
Chapter 2 (Sets and Venn diagrams) Exercise 2E 71

M U=7 R={z€Z|-2<z<4}, S={z€Z|0<z<T}


a R=1{-2-1,0,1,2,3,4}, S=1{0,1,2 34,5, 6}
b RNS=1{0,1,2 3,4}
¢ RUS=1{-2-1,0,1,23,4,56}
d n(RUS)=9 and n(R)+n(S)—n(RNS)=7+7-5=9
n(RUS) =n(R)+n(S) —n(RNS)

12 U=%Z, C={yeZ|-4<y<-1}, D={yeZ|-7<y<0}


a C={-4, -3 -2 -1}, D={-T7,—6, =5, -4, -3, -2, —1}
b CND={-4,-3 -2, -1}
¢ CUD={-7,-6, -5, 4, -3, -2, —1}
d n(CUD)=7 and n(C)+n(D)—n(CND)=44+T7T—-4=T7
n(CUD) =n(C)+n(D)—n(CND)

13 U=7*+, P={2€Z|5<a<17}, Q={2€Z|10<2<20}, R={zecZ|15< 2 <23}


a P={56,713809, 10,11, 12, 13, 14, 15, 16, 17},
Q = {10, 11, 12, 13, 14, 15, 16, 17, 18, 19, 20},
R = {15, 16, 17, 18, 19, 20, 21, 22, 23}
b i PNQ=1{10,11,12, 13, 14, 15, 16, 17}
i PNR={15,16,17}
i QN R = {15, 16, 17, 18, 19, 20}
iv PUQ={56,7809,10,11,12, 13, 14, 15, 16, 17, 18, 19, 20}
v PUR={56,78,9,10, 11,12, 13, 14, 15, 16, 17, 18, 19, 20, 21, 22, 23}
vi QUR = {10, 11, 12, 13, 14, 15, 16, 17, 18, 19, 20, 21, 22, 23}
¢ i PNnQNR={15 16,17}
i PUQUR={56,7,8,9, 10,11, 12, 13, 14, 15, 16, 17, 18, 19, 20, 21, 22, 23}
14 U={ze€Z"|z<40}, A = {multiplesof4 less than 40}, B = {multiples of 6 less than 40},
C' = {multiples of 12 less than 40}
a A={4,8 12, 16,20, 24, 28, 32, 36}, B = {6, 12, 18, 24, 30, 36}, C = {12, 24, 36}
b i AnB={12 24, 36} i BNC = {12, 24, 36}
il AnC = {12, 24, 36} iv AnNBNC= {12, 24, 36}
v AUBUC = {4, 6, 8, 12, 16, 18, 20, 24, 28, 30, 32, 36}
< n(AUBUC) =12 and
n(A) +n(B) +n(C) —n(ANB) —n(BNC) —n(ANC)+n(ANBNC)
=9+4+6+3-3-3-3+3
=12
n(AUBUC) = n(A)+n(B)+n(C)—n(ANB)—n(BNC)—n(ANC)+n(ANBNC)

15 U={zeZ" |z <31}, A= {multiples of6 less than 31}, B = {factors of 30},


C = {primes < 30}
a A=1{6,12,18 24,30}, B={L2 35,6, 10,15, 30},
C =1{2,3,5,7, 11, 13, 17, 19, 23, 20}
72 Chapter 2 (Sets and Venn diagrams) Exercise 2F

b i AnB= {6, 30} il BNC =1{2,3,5}


i AnC =g iv ANBNnC=g2
v AUBUC =1{1,2,3,5,6,7,10, 11, 12, 13, 15, 17, 18, 19, 23, 24, 29, 30}
< n(AUBUC) =18 and
n(A) +n(B) +n(C) —n(ANB) —n(BNC)—n(ANC)+n(ANBNC)
=5+84+10-2-3-0+0
=18
n(AUBUC) =n(A)+n(B)+n(C)—n(ANB)—n(BNC)—n(ANC)+n(ANBNC)

1T a U={23456T} b U=1{23,57,11,13},
A=1{2,4,6}, B=1{5T} A={2,3,7, B=1{3511}
ANB=g AN=B{3}

U 3 13
- A 6 )

¢ U={1,2,3,4,5,6, T}, d ={1,3,4,5 7},


A=1{2,4,56} B={1467} ={3,4,5,7}, B={35}
ANB = {4, 6} NB={3,5} =B and B#A,
so BCA.
A e B

"k

2 U={zecZ|1<x<10}, A= {oddnumbers
< 10}, B = {primes
< 10}
a A=1{1,3,579}, B={23571)
b ANB=1{3,57, AUB=1{1,23,5,7,9}
Chapter 2 (Sets and Venn diagrams) Exercise 2F 73

3 U={zeZ|1<x<9}, A= {factorsof
6}, B = {factors
of 9}
a A=1{1,2,3,6}, B=1{L309)
b ANB={1,3}, AUB=1{1236,9}

4 U = {even numbers between 0 and 30}, P = {multiples of 4 less than 30},


@ = {multiples of 6 less than 30}
a P=1{4,8,12 16,20, 24, 28}, Q = {6, 12, 18, 24}
b PNQ={12,24}, PUQ ={4,6, 8, 12, 16, 18, 20, 24, 28}
<
P4 3 Q

16 20
28
UL 2 10 14 22 26

5 U={ze€Z"|z<30}, R = {primes less than 30}, S = {composites less than 30}


a R=1{2357 11,13 17,19, 23, 29},
S =1{4,6,8,9,10, 12, 14, 15, 16, 18, 20, 21, 22, 24, 25, 26, 27, 28}
b RNS=0
RUS ={2,3,4,5,6,7,8,9, 10, 11, 12, 13, 14, 15, 16, 17, 18, 19, 20, 21, 22, 23, 24,
25, 26, 27, 28, 29}

910 12 14
15 16 18 20
2122 24 25
' 26 27 28
1 30
74 Chapter 2 (Sets and Venn diagrams) Exercise 2F

7 a U = {parallelograms}, b U = {polygons},
R = {rectangles}, S = {squares} Q = {quadrilaterals}, T = {triangles}
RNS=S and S#R, so SCR. QNT =9

R Q r

U U

8 i . a A={bd e h} f he
b B=1{ i j}
¢ 9,0 c
A'={f,a, j, k}
2 d B'={a,b,
d, g,ck} e ANB={e,h}
i : k f AuB={bd, f, hiej}
g (AuB) ={a,c,9,k} h AnB ={a,c g, k}
i AUB ={a, b cd, f, g, j, k}
9 a 1 A={a,bcdhj
A B i fa. b, c ¥
a i B={a,cfde
gk}
i C={a,be f,i,1} v ANB={a,cd}
Avfi v AuBb= {f,
c de a g,
h j, k}
v vi BAC={ae,f} il ANBN=C{a}
U O vii AUBUC e, f, ¢,
={a,b, 4, k, L}
g, h,i,d,

b n(ANC)=2 and n(AUBUC)=12 and


n(A) + n(B) +n(C) —n(ANB) —n(ANC)—n(BNC)+n(ANBNC)
=6+7+6-3-2-3+1
=12
n(AUBUC) =n(A)+n(B)+n(C) —n(ANB) —n(ANC) —n(BNC)+n(ANBNC)
10 U={z<cZ" |40 <z <60}
A = {multiples of 2}
= {40, 42, 44, 46, 48, 50, 52, 54, 56, 58, 60}
B = {multiples of 3}
= {42, 45, 48, 51, 54, 57, 60}
C' = {multiples of 5}
= {40, 45, 50, 55, 60}

@@ [T
11 a Ais shaded " is shaded ¢ ANDB is shaded
Chapter 2 (Sets and Venn diagrams) Exercise 2F 75

d ANDB' is shaded e A'UB is shaded f AUB' is shaded

i
Ul

g (ANB)" is shaded h (AUB)’ is shaded

g
&
12 a A is shaded b B is shaded ¢ A’ is shaded
e
g d
Ul

B’ is shaded
hS
D
&
o

go
@a
f
Ul
AQ

o0
AUB
QB

is shaded
0

w
>~

S
&

Ul Ul Ul

g A'NB is shaded h AUB’ is shaded i (ANB)’ is shaded

( ) ( ) A@ OB
Ul Ul Ul

13 a Ais shaded b B is shaded ¢ A’ is shaded


l

Ul Ul
S

d B’ is shaded e ANDB is shaded f AUB is shaded

A
l

S
S
76 Chapter 2 (Sets and Venn diagrams) Exercise 2F

g A'NB is shaded h AUB’ is shaded i (ANB)" is shaded

U Ul Ul

14 a Ais shaded b B’ is shaded ¢ BNC is shaded

@ A B A B

U ¢ U g U B
d AUB is shaded e ANBNC is shaded f AUBUC is shaded

@ A B A B

U ¢ U ¢ U ¢
g (ANBNCQC) is shaded h (AUB)NC is shaded i (BNC)UA is shaded
A B A B A B

U g U Y U <
i AAN(BUCQC) is shaded

U @ C
Chapter 2 (Sets and Venn diagrams) Exercise 2G 77

a n(B)=2+5=7 b n(A)=5+9=14
= B ¢ n(AUB)=T+2+5=14
@ d n(A, butnot B) =7 e n(B,butnot A) =5
©) f n(neitherA nor B) =9
U

a n(X)=3+2=5 b n(XNY)=6
¥ Y ¢ n(XUY)=8+6+3=17
@ d n(X,butnotY)=38 e n(Y,butnot X)=3
f n(neither
X norY) =2
" @)
a n(B)y=b+c b n(d)=c+d
4 B ¢ n(AnNB)=b d n(AUB)=a+b+ec
@ e n((ANB))=a+c+d f n((AUB))=d

v (d)d

a i nPNQ)=a
ii n(P)=2a+a=3a
il Q
n(Q)=a+a+4
=2a+4
iv n(PUQ)=2a+a+a+4 U (@=5)
=4a+4
v n(Q) =2a+ (a—>5)
=3a—5
vi n(U)=2a+a+a+4+(a—5)
=ba—1
b i n(U) =29 il n(U) =31
. ba—-1=29 oo ba—1=31
. 5a =30 s 5a =32
. a=6 s a=2-64
It is not possible to have a non-integer number of elements, as we have in ii.
n(U) can be equal to 29, but not equal to 31.

a n(AUB)=a+b+c and
A B n(A)+n(B) —n(ANB)=a+b+b+c—b
@ =a+b+c
n(AUB) =n(A)+n(B) —n(ANB)
U b n(ANB')=a and n(A)—n(ANB)=a+b—-b=a
n(ANB') =n(A) —n(ANB)
78 Chapter 2 (Sets and Venn diagrams) Exercise 2H

6 h - n(AUB)=a+b and n(A)+n(B)=a+b


n(AUB) =n(A) +n(B) for disjoint sets A and B.

7 n(U) =26, n(A)=11, n(B)=12, n(ANB)=28


We are given n(ANB)=38 A B
. on(ANB)=11-8=3
and n(4A'NB)=12-8=4
oA NB)=26-8-3-4=11
a n(AUB)=3+8+4=15 U )
b n(B, butnot A) =4

8 n(U) =32, n(M)=13, n(MNN)=5 n(MUN)=26


We are given n(MNN)=5 M N
o n(MAN)=13-5=38
Also, n(MUN) =26
. on(M'AN)=26-5-8=13
n(M'NN')=32-26=6 U (6)
a n(N)=5+13=18
b n((MUN))=6

9 n(U) =50, n(S)=30, n(R)=25 n(RUS)=48


We are given n(RUS) =48 R S
son(RNS)=50-48=2
Also, n(RUS)=n(R)+n(S)—n(RNS)
oo 48 =25430—n(RNS)
. n(RNS)=T7 U (2)
n(RNS)=25-7=18
and n(R'NS)=30-7=23
a n(RNS)=T17
b n(S, but not R) =23

1 a Let LH represent those with long hair and Br represent


LH Br
those that are brown.
n(LH N Br) =2 @
n(
n(LHNB r )
r')=5- 2=3 ! 3
and n(LH' NBr)=8-2=6
n(LH'NBr)=14-2-3-6=3
Chapter 2 (Sets and Venn diagrams) Exercise 2H 79

b i n(LH)=6+3=9
9 cavies do not have long hair.
il n(LHNBr')=3
3 cavies have long hair and are not brown.
iii n((LHUBr)')=3
3 cavies are neither long-haired nor brown.

2 a Let R represent those days on which it rained and B U


Umb represent those days on which Murielle took her m
umbrella.
n(RNUmb) =5
. n(RNUmMb)=9-5=4 &
and n(R'NUmb)
=8 —-5=3
n(RNUmb)=14-5-4-3=2
b i n(RNUmY)=4
Murielle did not take her umbrella and it rained on 4 days.
il n(RNUmMb)=2
Murielle did not take her umbrella and it did not rain on 2 days.

3 Let S represent those who play singles and


D represent those who play doubles. S D
Let n(SND)==z
. n(SND)=28—z and n(S'ND)=16—=
n(S’ND’') =0 since every member in the club must
play either singles or doubles. (0)
But n(U) =31, so (28—z)+x+ (16 —xz) =31
M4 —x=31
Lor=13
13 members play both singles and doubles.

4 Let D represent those who work day shifts and


N represent those who work night shifts. D N
Let n(DNN)==z
n(DNN')=47—z and n(D'NN)=29-=x
n(D'NN') =0 since every person in the factory must
work either day shifts or night shifts. (0)
But n(U) =56, so (47—z)+z+ (29—=z)=>56
76 —x = 56
oz =20
20 people work both day shifts and night shifts.
80 Chapter 2 (Sets and Venn diagrams) Exercise 2H

5 Let F represent the stalls which sell food and


C represent the stalls which sell craft. B ¢
Let n(FNC)=z
L nFNC)=21-z and n(F'NC)=14-=x
But n(U)=38, so (21—z)+z+(14—xz)+7=238
L 42-z=38 U )
=4
a n(FNC)=z=14
4 stalls sell both food and craft.
b n(FNC)+n(F' NC)=(21—-2)+ (14 —z)
=35—2z
=35-2(4) {as z =4}
=27
27 stalls sell food or craft but not both.

6 Let S represent the movies seen by Sandra and


R represent the movies seen by Robert. S R
Let n(SNR)==z
~ n(SNR)=13-2 and n(S'NR)=14-=x
But n(U) =86, so (13—z)+xz+
(14 —x)+ 69 =86
L 96-z=8 U (69
xz=10
a n(SNR)=z=10
10 movies have been seen by both Sandra and Robert.
b n(SNR)=14—2
=14-10
=4
4 movies have been seen by Robert but not Sandra.

7 a Let B,. C, and P repr.esent the s‘tudents studying Biology, B c


Chemistry, and Physics respectively.
n(BNCNP)=1 @
n(PNC) =18 W@
L n(PNCNB)=18-1=17
n(BNC)=4 w
. n(BNCNP)=4-1=3 U P (14)
n(PNB)=3
. n(PNBNC)=3-1=2
Also, n(B)=22, so n(BNC'NP)=22-3-1-2=16
n(C)=26, so n(CNB'NP)=26-3-1-17=5
n(P)=25, so n(PNB'NC)=25-2-1-17=5
Now n(U) =63
o n((BUCUP))=63-16-3-1-2-5-17-5=14
Chapter 2 (Sets and Venn diagrams) Exercise 2H 81

b i n(BNC'NP)=16
16 students study Biology only.
il n(PUC)=5+2+14+17+3+5=233 @
33 students study Physics or Chemistry. a&
iii n((BUPUC)) =14
14 students study none of Biology, Physics, or
Chemistry.
P (14)
iv n(PNC")=5+2=7 U
7 students study Physics but not Chemistry.

8 Let P, A, and IV represent the students who went paragliding,


abseiling, and white water rafting respectively. E
n(PNANW)=5
n(ANW) =7 QW
Son(ANWNP)=7-5=2 @@
n(PNW) =8
L n(PNWnNA)=8-5=3 W)
n(PNA)=11
L n(PNANW)=11-5=6
Also, n(P)=19, so n(PNANW)=19-6-5-3=5
n(A) =21, so n(ANP' NW)=21-6-5-2=8
and n(W)=16, so n(WNP NA)=16-3-5-2=6
Now n(U) =36
L n((PUAUW))=36-5-6—-5-3-8-2-6=1
a n(PUA)=5+6+5+3+8+2=29
29 students went paragliding or abseiling.
b n(WnP NA)=6
6 students only went white water rafting.
¢ n((PUAUW)) =1
1 student did not participate in any of the activities mentioned.
d n(PNANW)+n(ANWNP)+n(PNWNA)=6+2+3=11
11 students did exactly two of the activities mentioned.

9 a Let F, C, and S represent the students who


can play the flute, clarinet, and saxophone F ¢
respectively
t n(FNCNS )=
n(FNCNS)=4- W
FmC’mS)_4—z @@
and )
n(F'NCNS)=6-=z

5 (3)
82 Chapter 2 (Sets and Venn diagrams) Exercise 2H

Now n(F)=11, so n(FNC'NS)=11-rx—(4—z)—(4d—z)=2+3


n(C)=15 so n(F'nCnNnS)=15-a—(4—2)—(6—z)=2+5
and n(S)=12, so n(F'NnC'NS)=12—z—(4—2)—(6—z)=x+2
Now n(U) =29
(z+3)+(d—a)+z+@d—2)+(x+5)+6—2)+(z+2)+3=29
Lo +27=29
=2
So, the Venn diagram is:

@
(T
ms 3)

b i n(FNCNS)=2
2 students can play all of the instruments mentioned.
it n(F'NC'nS)=4
4 students can play only the saxophone.
ili n(F'NnCNS)=4
4 students can play the saxophone and the clarinet, but not the flute.
v n(FNC'NS)+n(F'NnCNS)+n(F NC'NS)=5+7+4=16
16 students can play exactly one of the clarinet, saxophone, or flute.

10 a Let Cr, Ca, and S represent the farms which


have crops, cattle, and sheep respectively. ar Ca
n(CrNnCans)=
s n(CrnCans’) =
n(CrfiCa'flS)78fz v
and n(Cr'NnCanS)=4—-=z @@

U S (2)

Now n(Cr)=15, so n(CrNCad'NS)=15-2—(T—2)—(8—z)==x


n(Ca)=9, so n(Cr'NCanS)=9—-z—(7T—2)—(4—2)=2-2
and n(S)=11, so n(Cr'NCdNS)=11-z—-8-z)—(4-z)=2-1
Now n(U) =21
e+ (T-z)+z+B-z)+(z-2)+{d-z)+(z-1)+2=21
. r+18=21
=3
Chapter 2 (Sets and Venn diagrams) Exercise 2H 83

So, the Venn diagram is:


Cr, Ca

b i nCrnCdns)=3 il n(Cr'N(Cau8))=1+14+2=4
3 farms have only crops. 4 farms have only animals.
ili n(CrnCans)+n(CrnCdNS)=4+5=9
9 farms have exactly one type of animal, and crops.

1 a Let R, T, and S represent the members


who use rowers, treadmills, and spin-bikes
respectively.
Let n(RNTNS)==x

Now n(R)=134, so n(RNT'NS)=134—z—-23—41=T0—z


n(T)=92, so n(RRNTNS)=92—2-23-28=41—2x
and n(S)=144, so n(R'NT'NS)=144—2—41-28=T75—x
Now n(U) =300
(70 —2) + 23+ z + 41 + (41 — 2) + 28
+ (75 — ) + 60 = 300
. 338 — 2z = 300
L 2x =238
. x=19
So, the Venn diagram is:

b 1 n(RNTNS)=19
19 members use all three types of cardio equipment.
il n(RNTNS)+n(RNT'NS)+n(RNTNS)=23+41+28=92
92 members use exactly two of the three types of cardio equipment.
84 Chapter 2 (Sets and Venn diagrams) Review set 2A

¢ n(RNT')=51+41=92
92
Percentage of members who use rowers but not treadmills = 360
~ 30.7%

a Let L represent the nations with a life expectancy of more than 75 years, .S represent the nations
with mean years of schooling greater than 10, and I represent the nations with a gross national
income more than $18 000 USD per capita.
n(LNSNI) =37
n(LNI) =50 fi
son(LnS' NI)=50-37=13
n(SNT) = 44 WA
Con(l'NSNI)=44-37=7
n(LNS) =43 v
n(LNSNI')=43-37=6 = I

Also, n(L)=69, so n(LNS'NI")=69—-6-37—13=13


n(S) =58, so n(L'NSNI')=58—-6-37—-7=38
and n(I)=61, so n(L'NS'NI)=61-13-37-7=4
Now n(U) =100
S n((LUSUI))=100-13-6—-37—-13-8—-7—4=12
b n((LUSUI))=12
12 nations were not in any of L, S, or I.

c I nIL'NnSNI')=8 it n((LUI)NS)=13+13+4=30
8 nations were in .S only. 30 nations were in L or I but not S.
ili n((SNI)NL)=17
7 nations were in S and I but not L.

1 a A = {letters in the word VENN} B = {letters in the word DIAGRAM}


={V,E, N} ={D,1 A, G, R, M}
b n(A)=3, n(B)=6
¢ ANB =g, ‘VENN’ and ‘DIAGRAM’ have no letters in common.
d i Ved ii GeB
V ¢ A is a false statement. .. G € B is a true statement.
il AUB={V,E,N,D,L
A, G,R, M}
. n(AUB)=9
n(A)+n(B)=3+6=9=n(AUB)
n(AUB) = n(A)+n(B) is a true statement.
Chapter 2 (Sets and Venn diagrams) Review set 2A 85

2 U = {multiples of 6 less than 70}


= {6, 12, 18, 24, 30, 36, 42, 48, 54, 60, 66}
A= {6, 62 66} = {6, 36, 66}
A’ is the set of all elements of U that are not elements of A.
A = {12, 18, 24, 30, 42, 48, 54, 60}

3 a N CQ since every element of N is also an element of Q.


However %, %, %, %, .... are all elements of Q but not elements of N, so N # Q.
N C Q is a true statement.
b 0€Z but 0¢Z+
0 € ZT is a false statement.
¢ 0 can be written as % or %, and so on, and 0, 3, and 7 are integers.
0 € Q is a true statement.
d R contains irrational numbers such as = and v/5 which are not in Q.
R C Q is a false statement.

A a {xeN|ax <6} can be represented by: b6 1p


0 6

b {z€R|—-3<x<2} can be represented by: b


-3 2 T

¢ {2]0<2<4} U {a|z>10} can be represented by: —o —

5 U={xeZ"|x<30}, P = {factorsof24}, Q = {factors of 30}


a i P={1,23468 12 24}
i Q=1{1,23, 56,10, 15, 30}
i PNQ={L23,6)
v PUQ=1{1,234,5, 68, 10, 12, 15, 24, 30}
b n(PUQ)=12 and n(P)+n(Q)—n(PNQ)=8+8—-4=12
n(PUQ)=n(P)+n(Q) -n(PNE)
6 U = {the letters in the English alphabet}, A = {the letters in “springbok™},
B = {the letters in “waterbuck”}
a 1 AU B = {the letters in “springbok™ or “waterbuck”}
={s,p, 7,4, n, 0,0, 0k, w,a,t e u,c}
il AN B = {the letters common to both “springbok” and “waterbuck’’}
={r, b, k}
iii AN B’ = {the letters in “springbok” but not “waterbuck”}
={s,p, i, n, g, o}
86 Chapter 2 (Sets and Venn diagrams) Review set 2A

7 a N’ is shaded b MNN is shaded ¢ MNN' is shaded

M M M

U U U

a The region shaded in blue can be represented by C’.


b The region shaded in red can be represented by
(ANB)U(ANC) or AN(BUQC).

9 U={xeZ"|x<10}, P = {odd numbers less than 10}, @ = {even numbers less than 11}
a P={1,351"179} Q=1{24,6,8,10}
b P and @ are disjoint, as P and ) have no elements in common.
<

P‘

10 a Number of members in the club


T B =15+7+4+5+8+6+11
= 56 members

b i Number of members who only play badminton


W = 8 members
ii Number of members who do not play tennis
=8+6+11
= 25 members
S iii Number of members who play both tennis and
U squash, but not badminton = 5 members
Chapter 2 (Sets and Venn diagrams) Review set 2A 87

¢ SN(T'UB) is shaded

d n(SN(TUB)") =11 members

11 a Let S represent those who were absent for at least


one day due to sickness, and H represent those who
missed some school because of family holidays.
Let n(SNH)==z
“ TL(S n H/) =229—z (296)

and n(S'NH)=111-z :
But n(U) =564, so (229—z)+x+ (111 —z)+ 296 = 564
636 — x = 564
r="T2

So, the Venn diagram is:


S, @ H

B (296)

b i n(SNH)=T2
72 students missed school for both illness and holidays.
il n(S'NH)=39
39 students were away for holidays but not sickness.
il n(SUH)=157+72+39
= 268
268 students were absent during Term 1 for any reason.

12 Let R represent the meals which contain rice and


O represent the meals which contain onion. K Q
Let n(RNO)=x
~ n(RNO)=14-z
and n(R'NO)=17T—=z
n(R'NO’) =0
.since every
:
main course
i
contains rice or
U ©
onion.
But n(U)=23, so (14—z)+xz+ (17—xz) =23
o 3l—z=23
. z=8
8 dishes contain both rice and onion.
88 Chapter 2 (Sets and Venn diagrams) Review set 2B

13 Let S, D, and C represent the students who could swim,


drive, and cook respectively.
n(SNDNC)
=1
n(SNC)=9
L n(SND'NC)=9-1=8
n(SND)=5
L n(SNDNC)=5-1=4 -
n(DNC)=6
L nSNDNC)=6-1=5
Also, n(S)=15 so n(SND'NC")=15-4-1-8=2
n(D)=12, so n(S'NDNC)=12—4—1-5=2
and n(C)=23, so n(S'ND'NC)=23-8-1-5=9
Now n(U) =38
. n((SUDUC))=38-2-4-1-8-2-5-9=7
a n(SND'NC)=9
9 students could only cook.
b n((SUDUQO))=T
7 students could not do any of these things.
¢ n(SNDNC)+n(S"NDNC)+n(SND'NC)=4+5+8=1T7
17 students had exactly two of these life skills.

1 The subsets of {1, 3,5} are @, {1}, {3}, {5}, {1, 3}, {1,5}, {3,5}, {1,3,5}.

2 a SandT are disjoint, so they do not have any common elements.


sSNT=92
b n(S)=s and n(T)=t, where S and 7" are disjoint.
n(SUT)=s+t
3 a The set of real numbers between 5 and 12 can be represented by {z € R |5 <z < 12}. This
set has an endless number of elements, so it is an infinite set.
b The set of integers between —4 and 7, including —4, can be represented by
{x € Z| —4 < x < T7}. The number of elements in this set is a particular defined value, so it
is a finite set.
¢ The set of natural numbers greater than 45 can be represented by {z € N | 2 > 45}. This set
has an endless number of elements, so it is an infinite set.

4 a o——9 b
-— 44
-l 4 ¢ 1
2 5 z 4 9 x

can be represented by {z |2 < x <5} can be represented by


{reZ|4d<ax<9} or
{xe€eZ|5<x<8}.
Chapter 2 (Sets and Venn diagrams) Review set 2B 89

< -—e [—

-3 1 x

can be represented
by {z |z< -3} U {z|x>1}.

5 S={z€Z|2<2<T}
a S=1{3,4,56,7} b ¢ n(S)=5
2 G

6 a A=1{2,4,6,8} and B={zr€Z|0<z<10} ={1,2,3,4,5,6,7, 8,9}


Every element of A is also an element of B.
ACB
b A=g2 and B={z|2<2<3}
The empty set @ is a subset of all other sets.
ACB
¢ A={zeQ|2<a2<4} and B={zeR|0<z<4}
The element
4 is in A but not in B.
A¢B
d A={z|z<3} and B={z|z<4}
Every element of A is also an element of B.
ACB

7 a U = {the 7 colours of the rainbow}


= {red, orange, yellow, green, blue, indigo, violet}
X = {red, indigo, violet}
X' is the set of all elements of U that are not elements of X.
X' = {orange, yellow, green, blue}
b U={zeZ|-5<a<5b}
={-5-4,-3,-2,-1,0,1,2 3,4, 5}
X ={-4,-1,3,4}
X' is the set of all elements of U that are not elements of X.
X' ={-5,-3,-2,0,1,2, 5}
¢ U={2recQ} and X={2zcQ|az<
-8}
X' is the set of all elements of U that are not elements of X.
X ={zeQ|z>-8}

8 U={zeZ|0<x<10}
=1{1,2,3,4,56,7,8,9}
a 1 A= {the even integers between 0 and 9}
={2,4, 6, 8}
il B = {the factors of 8}
={1,2,4,8}
ANB ={2,4,8}
i (AUB)Y ={3,5,7,9}
90 Chapter 2 (Sets and Venn diagrams) Review set 2B

Q)
B A B

vl 5 7

9 U={xeZ"|x<40}, A= {factorsof 40}, B = {factors of 20}


a A={1,24,528, 10,20,40}, B={1,2 4,5, 10, 20}
b B C A since every element of B is also an element of A.
However, 8 and 40 are both elements of A but not elements of B, so B # A.
BCA

A 3.6 7 9 11
12 13 14 15 16
17 18 19 21
22 23 24 25 26
40 27 28 29 30 31
8 32 33 34 35 36
37 38 39
U

10 P= {.L €Z|3<x <10}, Q=1{2,9,15}, R = {multiples of3 less than 12}


a =1{3,4,5,6,7,8,9}
b ( ) =7
¢ The number of elements in P is a particular defined value, so P is a finite set.
d | The elements 2 and 15 are in @) but not in P.
Q ¢ P which also means Q ¢ P.
iil R={3,6,9} and all of these elements are also in P.
However, 4, 5, 7, and 8 are all elements of P but not elements of R, so R # P.
RcCP
e i PnQ@={9} i RNQ
= {9} il RUQ=1{2,3,6,9,
15}

&)
[ [
11 a (AUB)'NC is shaded b C'N B is shaded ¢ B'N(ANC) is shaded
Chapter 2 (Sets and Venn diagrams) Review set 2B 91

12 a Let M represent those who drive a manual car and


S represent those who have a car with a sunroof. S
n(MnS)=5 @
o n(MNnS)=13-5=38
and n(M'NS)=15—-5=10 (4)
n(MUS)) =4
b i Number of members in the club =8+ 5+ 10+ 4
= 27 members
il n(MnNS)=8
8 members drive a manual car without a sunroof.
il n(M')=10+4=14
14 members do not drive a manual car.

13 Let T represent those who forgot their towel and


H represent those who forgot their hat. 7 H
Let n(IT'NH)==x
on(TnNH)=11-2
and n(T'NH)=23-x
’ ’ : . U ©
n(I'"NH)=0 since every student left something at
home.
n(U)=30, so (11—z)+z+(23—2)=30
.34 —-x=30
Lor=4
4 students had neither a hat nor a towel.

14 Let A, C, and E represent the delegates who could speak


Arabic, Chinese, and English respectively.
n(ANCNE)=2
n(ANC)=
n(ANCNE) = 12 2=10
n(CNE)=16
A NCNE)=16-2=14
n(ANE)=17
~ nANC'NE)=17T-2=15
Also, n(A)=28, so n(ANC'NE)=28—-10—-2—-15=1
n(C)=27, so n(A'NCNE)=27T-10—2-14=1
and n(E)=39, so n(A/'NC'NE)=39-15-2-14=38
Now n(U) =58
. n((AUCUE))=58-1-10-2-15-1-14-8=7
a n(AnCnNnE)=1 b n((AUCUE))=7
1 delegate speaks Chinese only. 7 delegates speak none of these languages.
¢ n((AuC))=8+7=15
15 delegates speak neither Arabic nor Chinese.
Chapter 3
SURDS AND EXPONENTS

1 a (V2x3)?=(V/6)?
=6
=2x3 Vv

2x3=(V2xV2)x (V3xV3) v {since 2=v2xVv2 and 3=+3x3}


(V2x v2) x (VBxV3) = (vV2x V3) x (V2x V3) v
(V2xV3) x (V2x V3) = (V2xV3)* v
every step of this argument is valid, and we can deduce that /2X 3 = /2 x /3.
b (Vaxb?=axb {definition of square root}
= (vVax va) x (Vb
x Vb) {definition of square root}
= (Vax
Vb) x (Vax Vb) {changing order of multiplication}
= (Vaxvb)? {definition of perfect square}
since square roots are non-negative, /a X b= \/E X \/5 forany a >0, b>0.

§=gig v {since 2=v2x+v2 and 3=+3x3}

VExVE_VEL
V3 x V3 3 V3
VT,
2
2 e 2
3 V3 V3
every step of this argument is valid, and we can deduce that % = %

{definition of square root}

= g z ‘/;E {definition of square root}

% X % {multiplication of fractions}

since square roots are non-negative, \/% = forany a >0, b> 0.


SIS
Chapter 3 (Surds and exponents) Exercise 3A 93

1 a V11 x v11 b V3xV5 < (V3)?


11 =v3x5 =\/§><\/§
=15 =

d V5
x V6 e V2 x V6 f 2V2
x V2
5%x6 =v2x6 =2xx/§><\/§
V30 =12 =2x2

S 3vx2
2¢/2 h 3VTx2V7 i (3V5)?
:3><ZX\/§><\/§ =3x2xV7TxV7 =3V5 x3V5
6x2 =6x7 =3x3xvV5xV5
12 =42 =9x5
=45

i 3v2
x V5 Kk —2v3x3V5 I 2V
x V12
=3xv2xV5 =-2x3xV3xV5 =2x6x V12
3XV2x5 =—6xv3x5 =2xV6x12
0 = —6V15 =272
V3?2
x /10 = V& x V15 =22 x V72
10 =—V36 x V15 =Vix V72
:

=—V36x15 =VixT72
éfi

= —+/540 = V288

2 a b @ c @
V3 V5
TR

VB —Vvi
=2

d
2
3
SR T Es

Ealeel b

3 ol
o)

. X
Il

Il
TEE
94 Chapter 3 (Surds and exponents) Exercise 3A

V5
2\/fi><\/§

V5
22x 10 x V2
V5
4 x 10><\/§
=%

f_r @ fi f_r
4><10><2
—L
=t

rm frmffrfiffr 2

rWJJhVFmflrfirwr r#
©

w
-
T

Qfi


o0

3@
-


5%
oLt
g
=

2V2 +3v2 b 2v2—-3v2 C 5V5


— 3v5
=52 2
Chapter 3 (Surds and exponents) Exercise 3A 95

d 5vV5 4+ 3v5 e 3v5—5V5 f o TW3+2/3


8v5 =-2v5 =9V3
9v6 — 121/6 h V2HV2+V2
—3v6 =3v2

6 a 43 - V12 b 3v2
+ /50
43 -VEX3 =3vV2+/25x2
413-2%x3 =3V2+5%xV2
43 -2V3 =3v2+5V2
23 =8V2
3vV6+ 24 d 227 +2V12
3V6+VEX6 =2/9%x3+2/4x3
3v6+2 x V6 =2x3xV3+2x2x3
3v6 + 26 =6V3+4V3
5v6 =10V3
V75— V12 f V2+/8-V32
V25 x3—V4x3 =V2+VIx2-16x2
5xvV3-2x+3 =V24+2xV2-4xV2
53 —2V3 =V2+2V2-4V2
3V/3 =2

V2(3
- V72) b Va(V5 +1)
V2 x3+V2x (=V2) =V5xV5+V5x1
3v2 -2 =54+5
V10(3 +2V10) d - 4)
VT(3VT
10 x 3+ /10 x 2V/10 = VT % 3VV7 Tx+(-4)
3v10+2 x 10 =3xT7T-4V7T
3v10 420 =2—1 47
—V3(5+V3) f 2v/6(V6 —7)
~VBx54+(—V3)xV3 =2V6 x V6 +2V6
x (=7)
-5v3-3 =2x6—14V6
12
— 14V6
—VB(VB—5) —3v2(4— 6v/2)
—VB X VB4 (—VB) x (-5) —3V2 x 44 (=3v2) x (-6v/2)
—8+5V8 —12V2+x3x V2 6
x V2
—84+10v2 —12V2+18 x 2
—12v/2 +36
96 Chapter 3 (Surds and exponents) Exercise 3A

a (5+V2)[d+V2) b (7T+2V3)4+3)
=20+ 5V2 +v2(4)
+ V2(V2) =28+ 7V3 +2V/3(4)
+ 2v3(V3)
=20+5vV2+4v2+2 =284+ 7V3+8V3+6
=2249V2 =34+15V3
¢ (O-VDE+2VT) d (V3+1)(2-3V3)
=36+ 9(2v7) — V7(4) — VT(2VT) =V3(2) + V3(-3v3) +2-3V3
=36+ 18V7 — 4V/7 — 14 =2v/3-9+2-3V3
— 224 14V7 =-7-V3
e (VB-6)(2v8-3) o (2V5-7)(1-4V5)
= V8(2v/8) + VB(—3) — 6(2v/8) — 6(—3) 2V + 2v/5(=4v5) — 7 — 7(—4V5)
=16—3v8 - 12V8 + 18 =2V5-8x5-7+28/5
=34-15V8 =30v5—-40—7
=34 — 302 =30v5
— 47

a (3+v2)? b (6—3)?
=32+2(3)(V2) + (V2)? =62 +2(6)(-v3) + (V3)?
=9+6V2+2 =36-12v3+3
=11+6v2 =39-12V3
¢ (VB+1)? d (vV8-3)?
= (VB)? +2(v5)(1) + 12 = (VB)? +2(VB)(-3) + 3
=5+2V/5+1 =8-6V8+9
=6+2V5 =17-6V8
=17-12V2
e (4+2V3)? o (3Vh+1)?
= 4% +2(4)(2v3) + (2v3)° = (3V5)* +2(3v5)(1) + 17
16 4 16v/3 + (4 x 3) =(9x5)+6V5+1
=16+ 16V3 + 12 —454+6v5+1
=28+ 163 =46 465
g (7-2/10)? h (5V6 —4)?
=72+ 2(7)(~2V10) + (2V10)? = (5v/6)? + 2(5v/6)(—4) + 42
=49 — 28V/10+ (4 x 10) = (25 x 6) — 406 + 16
=49 — 2810+40 =150
— 40v/6 + 16
= 89— 28V10 = 166
— 40V/6
i (—2+2v2)?
= (=2)2 +2(=2)(2v2) + (2v2)?
=4-8V2+(4x2)
=4-8/2+8
=12-8/2
Chapter 3 (Surds and exponents) Exercise 3B 97

10 a @B+vVHB-V7) b (V2+5)(V2-5) c (A4—V3)(4+3)


=3
- (V7)? =(v2)* -5 =4 - (V3)?
=9-7 =2-925 =16-3
=2 =-23 =13
d 2v2+1D@VvV2-1) e (4+3VR)(4—3VB) f (9v3-5)(9V3+5)
=(2v2)2-1? =42 - (3v8)? =(9V3)? -5
=(4x2)-1 =16— (9 x 8) = (81 x3)—25
=8-1 =16—72 =243 - 25
=7 =56 =218

EXERCISE 3B :
c i d £
V3 V3
9 V3 11, v
V3o V3 V3o V3
9v3 113
T3 g
=3v3
6 12
A "7
_6 V2 _12 V2
V2 V2 V2 V2
_ 6v22 _ v2
=32 =6v2

5 15
k Z I %

_5 V5 _15 V5
VB VB V5 VB
_ 565 _ 15 5
=5 =3v5

[ L p L
3v5 V7
1 5 _ TV
V5 V5 VTOVT
_ 5 _ T
3x5 7

_\5 —V7
98 Chapter 3 (Surds and exponents) Exercise 3B

21 2!

T 7 AS %7

2% Blw
_2n V7 -2 VI

EE
X
VTVT VII VI

w
_ 2T _ 2Vl

[\

5 w
7 11

=37

SI
no

S
S
Chapter 3 (Surds and exponents) Exercise 3B 99

. 1+V2 _ (1+\/§)(1+\/§) W V3 (VB (43


1—
1-v2 )\ 1+v2 4-3 4—v3)\4+3

S
(1++2)? _ V3(4+V3)
- (v2)? S 2 (VB2
12 4 2(1)(v2) + (vV2)? 4343
1-2 T 16-3
142242 4v/3+3
—1 13
342v2
-1

=-3-2V2
—2V2 (=22 [(1+V2 1+v5 (1+\/5> <2+\/5>
T \1-v2/\1+v2 2-5 2-v5/\2+V5
S
T

—2v2(1
4+ /2) _2+v54+2V5+5
12 - (v2)? T 22 (VB2
—2v/2
- (2x2) _7+3V56
1-2 T 4-5
_ —2v2 -4 _7T+3V5
-1 T
=2V/2+4 =-7-3V5

_ (VB2 [(V3+1 I VIO-7 [(VI0-7)[V10—-4


T \VB-1/)\VB+1 VIO+4 \VI0+4)\VI0-4
3+v3+2v3+2 10— 4/10 — 7v/10 + 28
(V3)2 —12 - 10— 16
5433 _==38—11/10
3—1
5+3v3 _=
—38+11/10
2
_(3+VB\[(4—+5 n 6-V2_ (6-V2) (542
T \4+v5)\4- 5 5-v2 \5-v2)\5+v2
12-3V5+4/5—-5 _30+6V2-5V2-2
42— (V5)? T 82— (V22
7+ V5 _28+V2
16 —5 T 252
745 _28+V2
11 23
100 Chapter 3 (Surds and exponents) Exercise 3B

S
Vs
—2
(V45
\V7T-2)\V7T+2
_ T+ 2V/T+5V7+10
[VT+2
P4—\/fi 2= (5R) (55 411 )
41T+
\4+V11
11 — 12 — 311
G e
T+ TVT _ V-1
T4 16
— 11
_ 1T+ 7VT V-1
3 5

3 a (V2-1)2=
(V22 +2(v2) (1) +12 b (3++v2)2=32+2(3)(V
+ (vV2)*
2)
=2-2V2+1 =9+6v2+2
=3-2V2 =11+6v2
. V2-1_ [vV2-1\[+V2-1 d 57\/§:<57\/§)<6——\/§)
+1 \V2+1)\v2-1 6—+2 6—-v2/)\6+2
S

_ 2o _30+5v2-6v2 -2
S VR 62 — (v2)?
_ (V22 +2(/2)(=1) +1° _28-V2
2-1 36 —2
_ 2-2V241 28— /2
1 34
=3-2V2 28 V2
34 34
~ - 42
1 1
f LB et"s s Pt
¢ VZ+1)2 (V2)2+2(V2)(1) +12 (3-+v22 324+2(3)(-v2)
+ (vV2)?
1 . 1
242V2+1 T 9-6v2+2
1 1
34+2v2 T 11-6v2
1 3-2V2 -~ 1 11+6v2
3+2v2 )\ 3-2v2 T \11-6v2 ) \11+6v2
3-2V2 _11+6V2
32 — (2v/2)2 T 112 — (6v2)2
3—-2V2 o 1146V2
9—(4x2) T 121- (36 x 2)
3-2V2 _11+46V2
1 12172
=3-2V2 11462
49
=4
1
+2v2
6
Chapter 3 (Surds and exponents) Exercise 3C 101

1 1 3-2V2 h e 1 2V/2
+ 7
3+2v2 \3+2/2/)\3-2/2 2Z—7 \2v2-7)\2v2+7
. 3-2V2 2247
- (V22 T evar-n
3-2V2 2247
T 9-(4x2) T (4x2)—49
_3-2V2 V24T
S4 T 8—49
—3-2V2 :2\/'——7
—41
——F -V

2l =9, 22=4, 22=8 21=16, 2°=32, 20=¢64


31=3, 32=9, 33=27, 3*=81, 3°=243 36="729
41 =4, 4%2=16, 4> =64, 4* =256, 4°=1024, 4% =4096
51=5 52=25 5%=125 5*=625
6! =6, 62=236, 6°=216, 6*=1296
V=7, T2=49, 7 =343, T7*=2401

-1)° b (-1)° < (-p*


= (=1) x (=1) x (=1) x (1) x (1) =(-1)°x (-1) =1
=1x1x(-1) =(-1) x (-1)
-1 -1
-1t e (-1)8 £ 18 g —(-1)8
ol bl =—(1%) =—(1)
=-1 =-1

-2)° i 2 i (=2
=(=2) x (=2) x (=2) x (=2) x (-2) =—(2) =—(-2)° x (-2)
=4x4x(-2) =-32 =32x(-2)
=-32 = —64

_p)t I —(-5)*
= (=5) x (=5) x (=5) x (-5) = —(=5) x (=5) x (=5) x (-5)
=25x25 =-25x25
=625 = —625
47 = 16 384 b 7% =2401 ¢ —5°=-3125
o

(—5)5 = —3125 e 86 =1262144 f(—8)% =262144


o

—86 = 262144 h 2.13° ~ 902.436 0396 i —2.13% ~ —902.436 0396


w

(—2.13)? ~ —902.436 0396


102 Chapter 3 (Surds and exponents) Exercise 3D

5 a 91-0T and =~9l —=0T b 672=0027 and —62 —0.027


¢ 374 —001234567 and 3l4 — 0.012345679 d 17°=1 and (0.366)° =1
. 1
We notice that ¢™" = — and a’ =1 for a#0.
a

6 3'=3, 32=9, 33=27, 3'=81, 3°=243, 3°=1729, 37 =2187, 3% =6561,


So, the last digit of the powers of 3 follow the pattern 3,9,7,1,3,9,7, 1, ....
3101 = 31 %38 x 3% x .. x 31 x 3! But 3' =81 whichendsinal
25 of these o3t x 3 x 3t x...x3" endsinal
25 of these
3101 endsina 3

7 Th=7, 72=49, 7 =343, 7 =2401, 7°=16807, 7°=117649, 77 =823543,


78 = 5764801,
So, the last digit of the powers of 7 follow the pattern 7,9, 3,1,7,9, 3, 1, ...
T =7 T x T x L x T x T But 7! =2401 which ends ina 1
54 of these ST T x T x . x Tt endsinal
54 of these
77 endsina 7

8 1=1°
3+5=8=2
T4+9+11=27=3°
13+154+17+19 =64 = 4°
21 + 23425427 +29 = 125 = 5°
31433+
35+37+39+41 =216 =¢°
49451 +53+55=2343="7°
57 + 59 + 61 654 67+ 69+ 71 =512 =8
T3+ 75477+ 79+ 81 + 83+ 85 + 87 +89 =729 = 9°
91+ 93 + 95 + 97 + 99 + 101 + 103 + 105 + 107 + 109 = 1000 = 10°
111 4 113 + 115 + 117 4+ 119 + 121 + 123 + 125 + 127 + 129 + 131 = 1331 = 11°
133 + 135+ 137 + 139 + 141 + 143 + 145+ 147 + 149 + 151 + 153 + 155 = 1728 = 12°

a 5%5=21+23+25+27+29
b 73 =434+454+47+49+51
453+ 55
¢ 123 =133 + 135+ 137 + 139 + 141 + 143 + 145 + 147 + 149 + 151 + 153 + 155
Chapter 3 (Surds and exponents) Exercise 3D 103

e f 32x37! Sx ™ zF x 22
ot CS+m k+2

S 0303
Il |
8

2
Il
104 Chapter 3 (Surds and exponents) Exercise 3D

(z_2s)3 dkfds g 32><37><34 h (j4)3z

— 3?2SX3 _dk — 32+7 34 = jixae


— sz B — 324744 _ e
— k-3 — 313

P .
116 x 11 27
A k (13°)
c\5d
1 w? =w
=116 x 11¢ 7t = 13¢%54 _w?
=z _135cd w

wP
Tl
= Tp—1

b i:% ¢ 8=2x2x2
—9-2 =2

e 32=2x2x2x2x2 f i:is
5 32 Fu 2
=9

h %:2% i 64=232x2
=2° x 2!
=9 1 o

k 128 =64 x 2 | L:;


96y ol 128 2
_ o7 =277

I-1 ¢ 27=3x3x3 d ~-21


9 32 3 27 33
=32 =L _3-3

l=i1
3 3 g 81=3x3x3x3
o
=371

=30
1=3 i .
]243_8i><31 A
ST_1
=3"x3 _g-5
=3° e

2% 2% =2l
x 2¢ b 4x20=2%x2° ¢ 8x20=2"x2t
=21+a — 92+b — 93+t

(21;+1)2 — 92(a+1) 1-n\~1 _=2 9—(1-n) 2¢_


===2¢
e (2"
: 221+2 =2n71
= 28—2
Chapter 3 (Surds and exponents) Exercise 3D 105

2" gm—(-m) 4 22 . gwtl 1


277", h 21—n o 21—n i 2z - 21 ‘
- 22m — 92—(1-n) =9t
= 21+n
4 _ (22);0
ol—z — 9l—=x
_ 22.7:7(171)

— gdu—1

9x 3P =32 x3P b 27%


= (3%)° ¢ 3x9"=3"x(3%)"
= 32tp _ 33 _ 3l 32n
_ glt2n

27 x 3% = 3% x 3¢ e 9x 27t = 3% x (3%t e ot
3 31
_ g3+d B 32 X 33t : 3y71
_ g2+t

3 3! h 2 32 .92 (3%)e
3y 3Y 2_7t ol (33)t ’ 3l—a ~ 3l—a

3l-v 32 3%
: fi ~ 3l-a
_ g2t _ 32a—(1-a)
s 33(171
gn+1 (32)n+1

32n—1 32n—1
32n+2

T 32n—1
_ 32n+2—(2n—1)

=33

49 ¢ 25 d 4
=95 i 72 — (52)3 — (22)5

_ 52x3 _ 92x5
_ 56 _ 9l0

167 f 27t g 5% x 25 h 4™ x 8"


— (2t — (33)t = 5a % 52 — (22)7:, X (23)n

— 9ixp _ g3xt _ gat2 — 92n 4 93n


— otp — 33t = 22n+3n

i 25n

8m
25
= k 2e+2
31 1 9 t42
16m - (32)t+2
(52)1) = 2z+2—(z—1)
_@yr
@) 5 — grt2—atl = 32(t+2)
2 _ 53 _ 32t+4
93m
S
24n

== 23m—4n . 52;0—4
106 Chapter 3 (Surds and exponents) Exercise 3D

81 16 59+ % 125
m 322"
— (25)277-
3utT ° ra 252
_ 5atl 53
_ 3 _ Yk
— 95(2-7) T ogytl T (22)F - (52)2(1
— 9l0—5r — 34-(+1) 4k 5a+143
— g4yt T : 52(2a)
— 33~y — 94k—2k 5a+4

— 92k ~ Tpda
_ patd—da
: 54—3&

b (3n)? < (5m)? d (mn)?


=32 xn? =5 xm? — m3nd
=9n? = 1256m®
4
3 2

t(3) s (f)
L _r
m?2 - *

9
T m?

b 37! L2
_1 1
; =7
Eufeit
49

5\0 7\~1
t (3 s (%)
=1 _ (4)\!
= (%)
_4
7

io2t+2 1 -1
ko (12)2\ 3 1 52 451 4571
ol

=2+1 /53

T Ah =(5)
Bl
Il

3 (=23) 53
=(3)
le % | <%

=11 (=30}

T
Il
=1
ol

— 2125

1 1
b T
=924

e A_2
27 33
Chapter 3 (Surds and exponents) Exercise 3D 107

gk' - (32)k
60 (2x3)P
10 2x5 75 3x52
=32 x 97l x 571 2Px 3P
T 3x52
=92P x 3Pt x 572

13 a (2ab)? =22 x a® x b? b (—2a)%=(-2)? xd?


= 4a*p? = 4a®
¢ (60%)2 =6 x (b*)? d (—2a)* = (-2)% x a®
= 365 =—8d°
e (—3m2n?)® = (=3)3 x (m?)® x (n?)3 f(—2abh)* = (—2)* x a* x (b*)*
= —27m5nS = 16a*p'°
2a\? ) m\? m*
g (T) =1, provided a #0, b#0 h (E) =

- mt
T 8lnd
: <fi>3 By 02} (—2)3 x (a2)3
2.4 123 ! 2 )3
_ oy 845
8 =

(et _ cx @ (=Y e
b b2 ¢ (¢3)%
1648 _9pt
= =5

14 a 2 (a® + ) b 2%(z® — 22 +3)


=a? x2® + 2% x ot =a% xa? +a% x (—2z) +2° x 3
— 23 g2l R
=z +2a? =z — 223 4 322

c z(z® + 1) (22 - 1) d (a® — 2?)(2? +2)


= af(a?)* - 17) =2 x2?
+ 2% — 2% x 2%
— 22 2

=z(z* - 1) = g3+2 4 943 — 2212 _ 942


=£L‘XI4—I =2 —at + 2% — 222
— z1+4_z

:15,‘1

e (2% — x)? f 22z -2+


1
= (2*)? +2(@®)(—x) + 2* =a?xz—2%+a%xa”
3X2 g3+l 4 g2 = g2+l 92 4 p2-1

=% — 2% 4 22 =2 2% 4z
108 Chapter 3 (Surds and exponents) Exercise 3D

g 7 2? + 2% — ) h (a® +271)?
=z x2d a2 xa? 27 (—a) = (@2 +2(z*) (a7 Y) + (z71)?
— 1B 12 14l — 22X2 4 9p2-1 | —1x2
:IQ--z—zo :.Z'4+2£L‘+l‘72

2?+z—1

i (@ +z Y (@? -zt
— ($2 2 (w71)2

= g2X2 _ p—1x2
— gt g2

5 4
15 2 o b 20t x3w S
4 X P2 —=2X3IxXw 4 xXw 12 i -2
r= 4b° =6 x wit! Frs. ol
— 4p?
= 6uw® =P
2s 47 233
d 5¢7 x 6¢* B i)d f 1060
—5x6 7 4 d 6ab


. :+C4 e =
d2+7
— =
18 X —
a2 B3
=
30 x &> 6 a b
e _ & =3xa? 1t x bt
s = 3ab?
=d*

24m2n? t5 x 18 .
e h = i 58t x4t
6m2n (t2) 2, 4143
24 m2 nt 1548 =5Hx4dxs“xt

=% w2 =53 = 20s2¢*
— 4 xm2=2 x ptl 113
=4xm®xn® b
=4x1xn® =7
=4n
i (k45 ) 12225 | @) x b5 BEXA P
k3 x kO 8zy? b2 x 6O p2+6
4x5 2 5 12 o 5
_k - 0
f3+6 8 T Yy b8
520 = % x z2 7t x P72 _ pl240
k9 B 3xy3 b8
T T BT
— g1 =

16 a x_3:i3 b 2av’3:2><i3 c ab™?


T T
9 1
_2
== =ax—b
a
Chapter 3 (Surds and exponents) Exercise 3D 109

d (ab)~* (2ab71)2 (5m?)~*


=22 xa®x (b71)? 1
~ (a2 =da® x b~? " Gm2)?
=4a* x b2 Dt
52 % m2X2
T a?p?
1
25m4

a?b—1t
(3a™2b)? c2

=32 x (a2)2 x b? _a
2
-1
=9 x a2 x B2 == xb
=9xa*xb?

9x
— xb?

w
N Q
110 Chapter 3 (Surds and exponents) Exercise 3D

1 5
17 a —an =g b am =5a""™ < b—n ="

d 1 —9—(n-9)
—(n—:
e 1
=3~ —(2—
f 3
=3¢~ (- =S
on—3 32—n ad—m
_ 9—n+3 — q—2+4n e 3a—4+m

- 23—n — 3n—2 — 3am—4

a”
8 = _=¢ n
x bm h a” "
= —n—(2+n)

= a"pm —g—n—2n
a72n72

1 2 1
18 a 5 =z2 b Z =271 ¢ x+=-=zx+a!
xT xT x

2 1 3 4 5
d 22— = =22 2273 e —+= =x71 43272 f == =4~ ! —5273
x3 z x? P
4 5 B i 3 2 5 _ _
g Tt——+
= =Tz —4z" '+ 522 h 2 -S4+
2 =3p71 — 2272 45074
x x2 z x?

3 3-2 5—
19 a i b L < 2z
x T T

<% ' ] 2T _5_=z2


z =z z T 2

=1+3z7! =3z 1-2 =5z"2 —g!72


572
— 7!

d z;Q e z2+5 f m2+x72


xT xT x

= x 4+ =2 T2 5
= 22z 4z2_=Z2
23 3 T +z T +z x
=g'73 42273 =z+5z7" =z+1-2z7"
=z 242073

g 222 — 3z +4 h z3—3200+5 i 5 —x — z2
T T x

=222 3z
4= 4 =Sz3 -5+
3z 5 5 x
e N z?
T x x &I &I T x x x

=2 —3+4z! =z -3z 45272 =5z


' —1—-z

8 + 5z — 23 16 — 3z + 2> 5z — 3224+
x +6
xT k xT 2 ! xT 2
8 bx 223 16 3z a3 5zt 322 T 6
T
s
x x
~m &I
=tz &I T
~m
T TETz €T &I T T

=8z~ +5— 222 =16z72 -3z ' 4z =522


— 3+ 146272
Chapter 3 (Surds and exponents) Exercise 3E 111

20 4+2
+ lx b 5 — 4295 B 6+§;x
T T T

4 2x 5 4x 6 3z
* z=1 T2 2 T3 + z—3
Tt
=4y + 221~ — 52 _ 4plm(-2) — 6% + 3213
=4z + 222 = 5a% — 423 = 62> + 324

z2+3 24z —4
1‘71 ZE72

z2 3 22 x 4
=t =t
=2V 1 3¢ =227 (D L 1m0 _gy?
2%+ 3 =zt +2° — 42?

% — 3z +6 3 — 6z
+ 10
z—3 z—2

x x T T €T x

1 Cis not in scientific notation. D is not in scientific notation.


0.3 x 10° 21 x 10
=3x 107! x 10° =21 x 10" x 10"
=3 x10* =21 x 10*?
It should be 3 x 10%. It should be 2.1 x 10'2.
N
a 259 =2.59 x 100 b 259000= 2.59 x 100000
=2.59 x 10? =2.59 x 10°
FY VYV YV VYN

2590000000 = 2.59 x 1000 000000 2.59 = 2.59 x 1


=2.59 x 10° =2.59 x 10°
~ Y
0.259 = 2.59 + 10 0.000259 = 2.59 = 10000
=259 x107¢ =259 x107*

40.7 = 4.07 x 10
Y

4070 = 4.07 x 1000


=4.07 x 10! =4.07 x 10°
Y FN VNV

0.0407 = 4.07 = 100 407000 = 4.07 x 100000


=4.07x1072 =4.07 x 10°
FYVV VYV Y laaaas)

407000000 = 4.07 x 100 000 000 0.000040 7 = 4.07 <+ 100 000
=4.07 x 108 =4.07x107°
112 Chapter 3 (Surds and exponents) Exercise 3E

FY VYV VY Al

3 a 47450000 = 4.745 x 10000000 b 0.003 = 3+ 1000


=4.745 x 107 kg =3x10%m
VNV YN VYV YV Y

¢ 2599000 = 2.599 x 1000000 d 0.00000047 = 4.7 < 10000000


= 2.599 x 10° hands =47x10""m

4 a 4% 10° b 5 x 102 < 2.1


x 10°
Y VXY Y'Y
= 4.000 x 1000 =5.00 x 100 =2.100 x 1000
= 4000 =500 = 2100
d 7.8 x 104 e 3.8 x 10° f 8.6 x 10t
YY) Y YY) ~
= 7.8000 x 10000 = 3.80000 x 100000 =8.6x10
= 78000 = 380000 =86
g 4.33 x 107 h 6% 107
NYYY YV Y Y Y YY)

=4.3300000 x 10000 000 =6.0000000 x 10000000


= 43300000 = 60000000

5 a 4x1073 b 5x 1072 c 21x1073


T
= 0004. +- 10 103
=005. +e 10 2 T
=0002.1 + 10 s

=0.004 =0.05 =0.0021


d 78 x 1074 e 3.8x107° f 8.6 x 1071
YN VN B n
=00007.8 +10* = 000003.8 +~ 10° =08.6 + 10
=0.00078 =0.000038 =0.86
g 4.33x 1077 h 6x1077
YV VY VI FYV
VYVY I

= 00000004.33 = 107 = 00000 006. + 107


= 0.000000433 = 0.0000006

6 a 7.4% 109 b 1.12x 1072


YYYYYYYYY AR}

= 7.400000000 x 1 000000000 =001.12 + 10°


= 7400000000 people =0.0112 kg
¢ 5x1077 d 7.3 x 10°
YV VYV VYV VY

= 00000005. + 107 = 7.300000 x 1000000


=0.0000005 m = 7300000 kg

7 a [ygggy| can be represented b [38E-gy| can be represented

as 4.5 x 107. as 3.8 x 1074,


4.5 x 107 3.8 x 1071
Y'YV Y'Y VY

=4.5000000 x 10000 000 =00003.8 + 10*


= 45000000 = 0.00038
Chapter 3 (Surds and exponents) Exercise 3E 113

¢ [pIEQS]| can be represented d [ggE-g3]| can be represented

as 2.1 x 10°. as 4 x 1073,


2.1 x 10° 4%x1073
YY) YN P
= 2.10000 x 100000 =0004. = 10%
= 210000 =0.004

¢ |6.1E03 can be represented f can be represented

as 6.1 x 103. as 1.6 x 1076,


6.1 x 10° 1.6 x 107¢
[aaA) YN
= 6.100 x 1000 =0000001.6 + 10°
= 6100 =0.0000016

9 [3.9Egy| can be represented h [g§3E-g2| can be represented

as 3.9 x 10%. as 6.7 x 1072,


3.9 x 10 6.7 x 1072
YY) N
= 3.9000 x 10000 =006.7 =+ 102
= 39000 =0.067

8 Using technology:
a 680000 x 73000000 b 0.0006 <+ 15000 c (0.0007)*
=4.964 x 10'3 =4x1078 =343 x10710

d (3.42 x 10°) x (4.8 x 10%) e (6.42 x 1072)?


= 1.6416 x 10*° =4.12164 x 1073
3.16 x 1010
g (98x1071) = (7.2x107°)
6 x 107
~ 1.36 x 10
~5.27x 10718
1
i (1.2 x 10%)®
3.8 x 105
=1.728 x 10°
~263x107°

9 Using technolo, 6x 10¢ =17.5x 107 peanuts.


g & §x10-1

10 Using technology, 4.6 x 1077 4 2.15 x 1076 = 2.61 x 107 m.

11 a Minimum distance to travel from Earth to Venus, then to Mercury


=3.8x10° + 7.7 x 10°
=1.15%x10"m
b We have assumed that we will always be on the side of the planet that is closest to the next
planet, at the time when the planets are closest. It could take a very long time for these ideal
conditions to occur.
114 Chapter 3 (Surds and exponents) Exercise 3E

12 a1 Distance travelled = speed x time


=2.9979 x 10° ms™! x 60 s {1 minute = 60 seconds}
= 1.79874 x 10'"® m
~ 1.80 x 10" m
il Distance travelled
= speed x time
=2.9979 x 10° ms™! x (60 x 60 x 24) s {1 day = 60 x 60 x 24 seconds}
~2.59 x 10 m
b 1 year ~ 365.25 days
One light-year = speed X time
=2.9979 x 10% ms™! x (60 x 60 x 24 x 365.25) s
~9.46 x 10" m
¢ 4.22 light-years ~ 4.22 x 9.46 x 10'°
~3.99 x 10" m

d Diameter (light-years
Milky Way 100000
M87 980000
Hercules A 1500000

i 980000 light-years &~ 980000 x 9.46 x 10> m


~9.27 x 10 m
The diameter of M87 is approximately 9.27 x 102! m.
i 1500000 light-years ~ 1500000 x 9.46 x 10'® m
~ 142 x 102 m
The diameter of Hercules A is approximately 1.42 x 10%? m.
1.42 x 1022 m
Scale factor of diagram =~ {26 cm = 0.26 m}
0.26 m
~5.46 x 102
So, Hercules A is about 5.46 x 10?2 times wider than the diagram.
ili 100000 light-years ~ 100 000 x 9.46 x 10'®> m
~9.46 x 10* m
The diameter of the Milky Way is approximately 9.46 x 10% m.
Time taken to cross the Milky Way
__ distance
speed
946 x10° m
{100000 kmh™! = 100000000 mh~—*}
~ 100000000 mh—1
~ 9.46 x 10'? hours
~ 9.46 x 10'? + 24 + 365.25 years
~ 1.08 x 10° years
~ 1.08 billion years
Chapter 3 (Surds and exponents) Exercise 3E 115

Particle Mass (kg) Charge (coulombs)


electron | 9.10938356 x 1073! | —1.6021766208 x 10~
proton | 1.672621898 x 10727 | +1.6021766208 x 1071°
neutron | 1.674927471 x 10727 0

a Writing numbers in a form involving a power of 10 allows us to write very small numbers
without having to write and count lots of zeros.

mass of one neutron _ 1.674 927471 x 10~27 kg


mass of one electron 9.10938356 x 10—31 kg
~ 1.839 x 10°
~ 1839
A neutron is approximately 1839 times more massive than an electron.

mass of one proton _ 1.672621898 x 10727 kg


mass of one electron 9.10938356 x 10—31 kg
~ 1.836 x 10°
~ 1836
A proton is approximately 1836 times more massive than an electron.

i mass of one neutron _ 1.674 927471 x 10~27 kg


mass of one proton 1.672621 898 x 10—27 kg
~ 1.001
A neutron is approximately 1.001 times more massive than a proton.
¢ An atom of silver has 47 protons and has no charge
it must have the same number of electrons as protons
it has 47 electrons.
Total mass of neutrons
= total mass of atom — mass of 47 protons — mass of 47 electons
=1.7911934 x 1072 — 47 x 1.672621898 x 1027 — 47 x 9.10938356 x 10~
=1.7911934 x 107 — 7.8613229206 x 10725 — 4.2814102732 x 10~%
=1.004 632966 91268 x 10~2° kg
mass of total number of neutrons
Number of neutrons =
mass of one neutron

1.004 632 966 912 68 x 10725 kg


1.674927471 x 10—27 kg
~ 59.98
~ 60 neutrons
So, the atom of silver has 47 electrons and 60 neutrons.

d 350 coulombs _ 350


charge of one electron ~ —1.6021766208 x 10—19
~ —2.18 x 102!
~ 2.18 x 10?! electrons are transferred by 350 coulombs of charge.
116 Chapter 3 (Surds and exponents) Review set 3A

1 a 7/5-35 b 2654
=45 =2V6-v9%6
=26 - (V9 x V6)
=2V6-3V6
=6
¢ 5V/314-V3) d (1+v2)(2+V2)
=5V3x4-5/3x3 =2+v2+2V2+
(V2)?
=20V/3-5x%x3 =243V2+2
=20V/3—15 =4+3V2
e (6-5V2)7? f B3+V5)EB-V5)
=62 —2(6)(5v2) + (5v/2)* =32 - (V5)?
=36 — 60v2 + 25 x 2 =9-5
=36 — 60v/2 + 50 =4
=86 — 60v/2

2 a —(-1)°=-1 b —(-3)3=—(-27) ¢ 3'-3"'=1-1


=27 _2 3

3 a xt x 2? b 2 H" or (2=YH7 < (ab®)®


= pi+2
4 -9 - 1x7 :(%)7 :abxbsxb

=z® =277 1 — Sp18


-
Lol
128

4 a 37 3 b Ty1 c a\!
<Z>
-1 =Ly B
T 33
-
v
27
Tz
.
===
Lo
()
b
xr = -
a

: 4 92
5 a 27=3° b 9° =(3%) T = T
_ 32t
= 227(77171)

o 23—7n
Chapter 3 (Surds and exponents) Review set 3A 117

6 15zy?
o 15
_?xxxgv? b T4° = e4° 36¢°h°
o7 1236 X9 3 X 55°

=5 xaxy’? _ 5 =3x g% x h572


1 P
=5ry2 J =3¢°h3
5x = jG_B
7 =i
1
i
0
3 r Rt m?\ _ 3 N2 2 312 2
iy <Z> e ° (5_n) B S (]Z.x)z X) qz
B i
provided m £0, n#0 25 x
i A
T 64s3 = 25p°q

22 +8 B 22 8 A+x+a3 4 T ot k™% —a—(a+6)


e et e s ¢ ek —2x—6
— g8t — dg? 4 (5D 4 3-(-2) =k

=42? + 2% + 2°

- 2,12
9 a a*b’® x ab? b 6a1° + 92%° < 5((; 2%)2
T
4 2 5 2 5
=a xa°xb’xb _6z2y 5x (a2 xy
= g2 pot2 gz y° 52 x (22)2
— 57 =8 xa! 2 x b saty?
= %gjflyo - 254

_ 2 = é x 274 x y2
" _ %LL‘U@/Q

_y
5

10 a 4.6 x 101 b 1.9 x 10° c 3.2x 1073


Y'Y YYYYYVYVYVYY — 1'9 >< 1 Y . 3

= 4.600 00000000 x 100000 000 000 =0003.2 + 10


= 460000000 000 =19 =0.0032

11 a 12.74 million metres = 12.74 x 105 m b 000012 m = 1.2+ 10000 m


=1.274x 10" m =12x10"*m

i height of pil
12 Number of sheets of paper required = &
thickness of one sheet
0.1 m
- 10 cm=0.1 m
32x10=4m { }

=3.125 x 10?
=312.5
So we would require 313 sheets of paper.
118 Chapter 3 (Surds and exponents) Review set 3B

distance from Earth to Neptune 4.3 X 109 km


13
distance from Earth to Saturn 1.5 x 109 km
~ 2.87
Neptune is approximately 2.87 times further from Earth than Saturn is from Earth.

1 a 4/11-5V11 b V32-3V2
=11 =VI6x2—3V2
=16
x V2 —3v2
=4/2-3V2
=2
¢ (T+2V3)(5-3V3) d (6+2v2)(6-2V2)
=Tx5-Tx3V/3+2V/3x5-2V/3x3V3 =62 — (2v2)?
=35-21V/3+10V/3 -6 x3 =36 (4x2)
=35-11V/3-18 =36-8
=17-11V3 =28

2 2 V3 VT _NT VB
2 8 N GEG R
_2v3 _VTX5
3 5

_VE 5

‘ Felwmlian) |
VB+2 \VB+2/)\VB-2 Mamaew)n)
4+v7T \4+v7)\4-VT
47
_ 3(v3-2)
(V3)? -2 42— (V72
_ 3V/3-6 4A-T
3—-4 16 -7
_3V/3-6 4T
1 9

=6-3V3

38 Dot
— mt
by provided
=1 y # 0
< (&)=w2
T 2
w2

:722:2
,w2
Chapter 3 (Surds and exponents) Review set 3B 119

— = k"2 b 11" x 1174 =114 ¢ 9x3*=32x3


k _ g2+b

1
=11 qq—1 b a _
z=ab 59 gk 4
¢ o =gk

== b 35 x 81 =3k x 3 ¢ Z2=8)
a 3\a

— 31€+4
= 00" i _gia x5t
— 5311—17

270 -3 = 1 b 70=1
0 ¢ 37143
-1 =143
1_1

e é = 1_30 (= 3%)

240\ 23 x abx3 _5\2 1622 x 25 162210


<8b2> TS X528 i (5d1>< (d 5 )2 C TEp xS
8a18 = (5d* (= ) 7
( _ 162

= b1ars = (5d7")? 88
al8 — 52—4x2 _ %g—s

~ 6486 — 92548 =224


25
- #
72 x g3 b 2(ab) 2 < 2ab?2
=x
—2+(-3) =
1 1
= 2a X <—>
-5 R (ab)? 4
o 2 3,20
ixS = a2 b

10 = b 81T x gl
33 = (34)1—z x (32)1—21

= (32)@ - 34—4z x 32—4z

33 . 34—41+(2—4z)

= @ = 36—81

_ 332

1 1.43 x 10° km b 8.2x107% m


= 1/43000 x 100000 km = 0000000082 = 10° m
= 143000 km = 0.000000082 m

12 Time taken = . b Time taken = distance


speed speed
_ 3740 m . 21x105m
T 191 x 108 ms—1 191 x 108 ms—!
~1.96x107° s ~1.10x1072%s
~0.0110 s
120 Chapter 3 (Surds and exponents) Review set 3B

height of dime
13 Number of sheets of gold leaf required
thickness of one sheet
1.35 x 1073 m
1.8 x 10=7 m
=75x10°
= 7500 sheets
Chapter 4
EQUATIONS

a 22=4 b 322 =48


r=+V4 2*=16 {dividing both sides by 3}
o ) r = ::\/E

xr=+4

¢ 42?=4 d 522=35
2 =1 {dividing both sides by 4} 2?2 =7 {dividing both sides by 5}
T = ::\/I T = ::\/7

e ==1

e 222=-10
. 2#?=-5 {dividing both sides by 2}
which has no real solutions as 2% cannot be negative.

f 622=0 g 422 -5=15


Ca?=0 {dividing both sides by 6} . 42 =20 {adding 5 to both sides}
=40 o x2=5 {dividing both sides by 4}
xr = LT = ::\/g

h 7-322=19
—32% =12 {subtracting 7 from both sides}
2> =-4 {dividing both sides by —3}
which has no real solutions as 2 cannot be negative.
:i %z 2 - é =1

122 =9 {adding § to both sides}


- x?= 2 {multiplying both sides by 2}

xTr = ::\/g

— 10
T=E
Th= ::%

a (z-32%=16 b (z+1)=9 (v +4)2=-25


CE—3=::\/E CE+1—::\/§ has no real solutions
r—3=44 r+1=43 as (x+4)% cannot
r=3=+ o r=—1=x be negative.
122 Chapter 4 (Equations) Exercise 4B

d e (z+4)?=13 f (z-72%=0
Lox+4=4 T — :::\/(_]

iA
Ta=x

S h 1Bz+1)2=7 i (z—fi)2=2
Lo Br+1)2=14 LT — 2=::\/§

c. 3CE+1=::\/_4 2 ‘1':\/_::\/5

3r=—-1+14 L x=2V20r0
2x =8
or —2
71::\/fi
T =
o x=4or—1

I (@+v2)°’ =1 k (c—v3)?% =2
:L’”\/§:::\/T 20— V3 =%V2 2+ 1=+V7
LT 2==+1 2.L:\/_::\/§ 2.L:71::\/?

r=—V2+1 T =
B3 T =
N ::\/7

3 2?=n
B I=::\/T_l

a 22 =n has two real solutions when n > 0.


b 22 =n has one real solution when n = 0.
¢ 2% =n has no real solutions when n < 0.

1 a a3=27 b 2'=16 ¢ 25=-10


z =27 z =16 has no real solutions
r=3 r =42
as 29 cannot be
negative.

d 2°=-13 e 2°+8=0 f 22=14


z=v/—13 8= -8 =7
x=v/-8 r=7
r=-2

g 5z'=30
|~
Il
5

I4=
_ 3/ 8
r=127
Il
5

sl
I

L:::\4/6
3
o=

8
T= V8
Il
8

M
n
Chapter 4 (Equations) Exercise 4B 123

j 32°= k 42 +5=-19
o
.5 £7
_1
3 4t =24
L xd=-6
T = 51 3 z= /-6

a (2—1P=17
r—1=17
Lox=1+V17

d (z+5)*=-16
has no real solutions
as (z+5)* cannot
be negative.

3z —-1=4V1
3r—1==+1

3r=1+

3r=2o0r0
,ngoro

a If 32°—24=0 ¢ If (20+3)°+1=0
B
then 323 =24 then (z —1)* =11 then (224 3)° = —
oot =8 Lor—1=4V11 2r+3=v-1
z=8 r=1+V11 2z +3=-1
=2 L 2x=—4
the zeros of (z—1)* — 11
the zero of z=-2
are 1+ v/11.
323
— 24 is 2. the zero of
(2w +3)5+1 is —2.

a 27 -1
_ 1
=5 b -3 1
. 27
=6 a3 =27
z=/-27
z=-3

d 272=49 7%
= —64
2 =1 .3 _ 1
B 61
_ 4./ wneal/iag
T=*1 == 61
_ 1 2oa it
e T s g
124 Chapter 4 (Equations) Exercise 4C

-3
h (z+1)2=-4 (2w-5)7" =3
: 2
(x+1)P2=-1 1 (2253 =5
(x-372=%
i which has no real 2z —-5=15
T—o==y25 solutions as (@ + 1)? 20=5+ V5
e ot % cannot be negative.
5+ /5
xT
r=3+ %
2
=24 or

1 a 3x=0 b ax8=0 —Ty =


=0 . a=0 y=0
d ab=0 e 2zy=0 a’>=0
.a=0or b=0 x=0o0r y=0 a =

g 2yz=0 h 2abc =0
v xz=0 o y=0 or 2=0 . oabc=0
. a=0o0or b=0 or ¢c=0

z(z—5)=0 2e(x+3)=0
=0 o x—-5=0 . 2x=0 o 2+3=0
. rx=0o0rb . x=0o0r -3

(z+1)(z—3)=0 (z—=4)(z+T7)=0
wrx+1=0o0 z—-3=0 wrx—4=0o0r z+7=0
L x=—1lor3 o rx=4or—7

3x(7T—x)=0 —2z(x+1)=0
. 3r=0o0r 7T—2=0 —2x=0 or z+1=0
o x=0o0r7 o x=0o0r -1

4(z+6)2x—-3)=0 (2z+1)(20—1)=0
(x+6)(22—-3)=0 L 2rx+1=0or 2r—1=0
. r+6=0 or 2r—3=0 o 2r=-1or 2z=1
or 2r=3 . ! 1
rx=—6 . T=—350r5
. x=-6or3
11(z+2)(x—7)=0 —6(x—5)(3z+2)=0
S (@ +2)(x—-7)=0 S (@=5)Bz+2)=0
L rz+2=0o0r z-7=0 L x—=5=0o0r 3z+2=0
Lx=-2o0r7 L rx=5 or Jx=-2
. wx=5o0r—3
2}z +5)=0 45-2)*=0
L 22=0o0r z+5=0 5-z)2=0
. x=0o0r -5 =0 {null factor law}
Chapter 4 (Equations) Exercise 4D.1 125

—3(3z-1)2=0 n z(e+1)(z—-2)=0
(Bz-1)2=0 rz=0o0 x4+1=0o0or z—2=0
. 3x—1=0 {null factor law} wox=0,—-1,0r2
L 3r=1
L r=3%
(z-1)(z+2)(x—3)=0
wrx—1=0o0or 24+2=0 or x—3=0
x=1,-2,0r3

3(x+2)(z+4)(2x—-1)=0
L (@+2)(z+4)22-1)=0
L r+2=0o0r 24+4=0 or 2xr—1=0
o rx=—-2o0r—4 or 2r=1
L w=-2—4,0r3
a
370 b 32—
z

a=0, b#0 o3y =0, z#0


coay=0, z#0
x=0o0r y=0, 2z#0

ZLE d ~Z _p
zy 2y
. 2=0 which is impossible —x=0, y#0
there are no solutions. =0, y#0

42 +72 =0 b 622 +2x =0


(A +T7)=0 L 223z +1)=0
x=0 or dx+7=0 . 2x=0or 3z+1=
=0 or 7% =0 or —%
322 —T7x=0 d 222
— 11z =0
Lox(3z—=7)=0 (2 —-11)=0
. x=0o0r 3z—-7=0 Lax=0or 2xr—11=0
. x=0
_ or 37 .x=0or 4
3z% =8z f 9z
= 62
o032 -8 =0 S 627 —92=0
©ox(3z—8)=0 o 3x(22-3)=0
. x=0or 3z—-8=0 . 3x=0or 2z-3=
.
_
x=0 or 38 . x=0 or %

22 =5 +6=0 b 22 —20+1=0
(z—2)(z-3)=0 (x-1)=0
x=2 or 3 =1
126 Chapter 4 (Equations) Exercise 4D.1

c 2?2 +20—-8=0 d 22+T +12=0


S+ E—-2)=0 (z+3)(z+4)=0
x=—4 or 2 o rx=-3 or —4

e 2 =22+8 22421 =10z


ot =20 -8=0 oa?—10z+21=0
S (z+2)(z—-4)=0 (z—=3)(z—7)=0
L rx=-2or 4 rx=3 or 7

g 94 2% = 6z 2?4+ r=12
Lo —6x+9=0 ot —12=0
s (z=3)%=0 (z+4)(z—3)=0
Tl rx=—-4 or 3

i z? + 8z =33
oz 48:-33=0
oo (e+11)(z—3)=0
. x=-—11 or 3

3 a 922—-1204+4=0 222 — 132 —7=0


o (Bz—-2)2=0 2z +1)(z—-7)=0
.. z=3 2 La=-3%or 7

< 322 = 16z


+ 12 32?4
bx =2
© 322 — 162 —12=0 322 +52—-2=0
Bz +2)(z—6)=0 (Br—1)@+2)
=0
L x=-—5 2 or 6 .. —1
x=g3 or _ 2

e 2%
4+ 3 =5z 322 +8z+4=0
22 =52 +3=0 Br+2)(z+2)=0
2z —-3)(z—1)=0 L x=—5 2
or —2
z=3% or 1

g 322 =10z +8 4% 442 =3


322 — 102 —-8=0 oAz 44z —3=0
Bz +2)(z—4)=0 (22 —1)(2z+3)=0
z=-% or 4 .L =1 1
x=35 or _3 —3

i 4z =11z +3 1222 =11z + 15


o 4x? —11z—-3=0 1222 —1lz —15=0
(4z+1)(z—3)=0 . (4z+3)(3z —5) =0
L w=-—1 or 3 _z—40r3 | 5

k 722+ 62 =1 1522 4 22 = 56
o TE? 462 —1=0 1522 4+ 22 — 56 =0
(7T —1)(z+1)=0 .(x +2)(15z — 28) =0
x=% 1
or —1 L ax=-2or &8
Chapter 4 (Equations) Exercise 4D.2 127

(z+1)? =222~ 5z +11 (z4+2)(1—2)=-4


o2t 20+1 =222 b +11 Lr—at42-20=—4
Lo —Tr4+10=0 St —6=0
(z—2)(z—=5)=0 s (@ +3)(z—-2)=0
o x=2 or b ox=-3 or 2

5—4a? =32z +1) +2 z+z=3


T
o b—da? =6x4+3+2
2?4 2=3z
4?46z =0
a2t -3r42=0
o 2e(224+3)=0
(z—1)(z—2)=0
v 2x=0or 224+3=0
x=1or 2
z=0 or —3

2w Lto1 x 1—=2 x
2t —1=—z oz(z+3)=-9(1—2)
24 —1=0 2?4+ 3r=-9+9z
2z -1)(z+1)=0 22 —624+9=0
1
L x=g3 or —1 so(=3)?%=0
Ll izl

(z+3)2—2)=14 (x—4)(x+2)=16
L2 —a?46-3r=4 o2t —20—-8=16
ot —2=0 Ca? =20 -24=0
(z+2)(z—-1)=0 . (x+4)(z—6)=0
cx=-2or 1 . x=-4 or 6

(x—5)(x+3)=20 (42 — 5)(4a — 3) = 143


o2 —20—15=20 . 162% — 32z + 15 = 143
o2t =20 -35=0 1622 — 322 — 128 =0
(z+5)(x—T7)=0 16(z* — 22— 8) =0
o x=-5or 7 16(z +2)(z
—4) =0
o (@+2)@—4)=0
. x=-2or 4

224z +1=0 22 +62+2=0


oozl —dr=— x4 6r=—-2
oz —dz 4 (-2 = —14(-2)? +3% = —2+3?
2?46
s (x-22%2=3 s (z+38)2=7
. I+3=::\/7
‘. 56173::\/7
128 Chapter 4 (Equations) Exercise 4D.2

c 22 — 14z +46
=0
ozt — 1 = —46

(x—T77%=3
I—7—:

r="T

e 224+ 6x+7=0 f 22 =246


br =T
st z? -2 =
22 46r+3t=-7+32 22 =22+ (—=1)? =6 + (—1)?
s (@+3)?2=2 (x—1)2=
:E+3:::\/§ r—1=+V7
A 1':73::\/5 J‘Zl::\/?

g 22+ 62 =2 h 22 4+10 =8z


a4 6r+32=243% ozt —8x=-10
sL(E+3i=11 22 =8z + (—4)? = 10+ (—4)?
I+3:::\/_1 (x—4)7=6
. r=— ::\/fi xr — =::\/(_3

z=4::\/6

i 2?4+ 6x=—11
. xt46z+32=-11432
s (@+3)P2=-2
which has no real solutions, as
(¢ +3)? cannot be negative.

2 a 222 4z +1=0 b 222 — 10z +3=0


2?+20+1=0 2* —5r+2=0
12+2x:7% CL 1275x:7%
B x2+2x+12=—%+12 a? 5w+ (~9)? =~ + (-8
s+ 1)?=13 @-§F=-3+%52
x+1:::\/; (z-3)?=2
z+1—::% 1‘7%122 1T9

Lr=—-1=+
S

©
jot

£
Il
8

H
g =gb+ “sb 1 0=¢+5al
— “xd
%:m—l—s& 0:%-5—3:8—%:

H+E=F)+z+ il
2= +) )+ §-=S¢-)
+ok -
e S _ < .
A\, k=L pri—="E-% -
I _SE_yg) -
A\ I 0 < o

&__L_:m
TETeTE oc
e :::§7m
£
€ T¢
e L 8o
ave T ¢
o Tg
N Re TS
BN 4 &_
or g%

wd- =11 x
d p=S_e %

fpd—=1I—=n S
— <
I=x4"xd p_g_
TE=Fg— 0¢
%:&%—Q—S‘Cfi %:m%—sm

() +5="(Gp) +33+ % GF+E="G)rog—"


dor = oor T3 = (or +%)
e I I <L
T=sti="E-9
Ig I
oor /T =orT%® Fyr=§F-=u
ey I
or - —or % @:::%—m
Ic Lo
S Eoi— =28 Uv +E=y

<l C.Ab geiorexd (enoitsupd) » 193qsdD

b4 wd = %5 b 0=2a+uCl
+s
%——ifi§=sifi 0=%+m#+sm
b=ug— % E-=gb+% -
A=)+ E=(I-)+x8— 2 .. fe4 - =%Ctab+s
I=%U-w) . I=%¢+w)

T
N\ o4
=8
130 Chapter 4 (Equations) Exercise 4D.3

1 5
< 3+ —5=—-=
+CE2 xT

307 +1=—bu
2’ +1i=-3%a
2+ 2r=—1
2+ 50+ (3 =—3+(3)’
@+8)°=-35+% =135
5)2 1 25 _ 13

5 _
T+g==+4/3%13

_ 54 V13

4 ax? +br+c=0

Cat+ bt
a
S0
a

. 2 2
2x =b — &¢ fSapb )
fa a " (2(1) a (2a
( T+ =b )2 Zc, b
2a a 4a?
( T+ —
b )2 =
b2 — dac
2a 4a?
b b — dac
i 2 4a2

—b =+ /b2 — dac
=
2a

1 a 22-42-3=0 b 224+62+7=0
has a=1, b=—-4, ¢=-3 has a=1, b=6, c=7

_ (9 V(2 - 4)(-3) _ =6+ 1/62 —4(1)(7)


LT
2(1) eHereEn)
4::\/2_8 —6++/8
T = ) p=—"—Y"
2
4::2\/7 i —6::2\/5
T =
2 2
I=2::\/7
w=—3::\/§
Chapter 4 (Equations) Exercise 4D.3 131

2 +1=4x d 2 +4r =1
o2 —dr+1=0 a4 —1=0
whichhas a=1, b=—-4, ¢c=1 whichhas a=1, b=4, ¢c=—-1

_ (4 /R -4 \/42 —4(1)(-1)
o 200) A 201)
= 4+ /12 —4 ::m
xr =
2 2
4::2\/5 —4+ 2\/3
T = g —
2 2
c. 1=2::\/§
Lr=-2+% \/5

22 —4r+2=0 f 222 -20-3=0


has a=1, b=—-4, c=2 has a =2, b=-2, ¢= -3

_ (9 E V(24O = —(=2) £ /(=2)* — 4(2)(=3)


2(2)
4++/8 2::\/%
T = A
2 4
4+2V2 2427
T = s
2 4
212::\/5 l::fi
T =
2

322 -5z —1=0 h —22+42+6=0


has a=3, b=-5, ¢=—1 has a=-1, b=4, ¢c=6

Lo
_ =9 £ V(=5)2
G) —43)(-1)
_ —4x 42 — 4(—-1)(6)
.3 2(—1)
5::\/§
= —4 4+ /40
LT =Y
6 -2
—4+ Zm
p=_——2V 7
-2

xr = 2+ \/E

—222+
T2 —2=0
has a=-2, b=7, c=-2

) _—Tx 72 — 4(-2)(-2)
.= ey

-7+ \/fi
r=—2X"
—4
. 7++/33
.=
4
132 Chapter 4 (Equations) Exercise 4D.3

2 a (z4+2)(x—-1)=2—-3z b (2z+1)2=3-2
Lol —z422—-2=2-3z 4+ 4x4+1=3—-z
44 —4=0 oo 4a®+52-2=0
whichhas a=1, b=4, ¢=—4 whichhas a=4, b=5, ¢c= -2

oo At
a4 VE
Jaz ()_ . =5+ /52 —4(4)(—2)
2(1) 2(4)
— ::\/@ -5+ \/fi
r= —————— g=_2=Vol
2 8
—4::4\/5
Tr=
2

r=-2+2V2
c (z—22=1+z d (Br+1)%= -2z
Lo —dr+d=1+4z 92?46z 1= -2z
2> —524+3=0 922482 +1=0
whichhas a=1, b=-5, ¢=3 whichhas a=9, b=38, c=1

_ —(=5) £ V(=52 —41)B) =84 /82 —4(9)(1)


.z 20
A L3
)
5::\/fi —8::\/%
. = T =
2 8
—8::2fi
r=——"Y
8
3 —447
.=
9

e (@+3)2e+1)=9 f 22+3)2z—-3)==
2%+ +6r+3=9 o4z —9=z
222 + 72 —6=0 St —z—-9=0
whichhas a =2, b=7, ¢c=—6 whichhas a=4, b=-1, ¢=-9

T \/ 7% —4(2)(—6) _ (D= VED2 - 49)(-9)


L
r= 2(2) 2(4)
—7+£97
p=_tEVIl
1::\/@
4 8

z—1 1
g 27x:2z+1 h r—=-=1
x

Lr—1=Q2x+1)(2—-2) Lt —l=x
r—1=4z—-22°>+2—=z stz —1=0
L 222 -2 -3=0 whichhas a=1, b=-1, ¢= -1
whichhas a =2, b=-2, ¢=-3
R
_ (=D VEDE -4
xTr=
—(=2) £ /(=2)2 —4(2)(-3) 2(1)
2(2) o= l::\/g
2++/28 2
Tr=
4
e
2427
4
1—:\/7
T =
Chapter 4 (Equations) Exercise 4D.4 133

i 2m—L—3 xr

222 —1=3z
S22 —32-1=0
whichhas a =2, b= -3, ¢= -1

xT _ 2V(324D
_ 2(2)
3::\/fi
4

1 22-T7x+9=0 has a=1, b=-7, ¢c=9


a A=1b>—dac b Since A > 0, but 13 is not a square,
= (=7)2—4(1)(9) there are 2 distinct irrational roots.
=13

C r =
—b+ VA

2a
v —(=7)£/13
2
= 7413
2

So there are 2 distinct irrational roots as expected.

2 422 —4x+1=0 has a=4, b=—-4, c=1


a A=10—dac b Since A =0, there is one repeated root.
= (-4 —4(49)(1)
=0
—b+ \/Z
< T =
2a
_ —(=4)=*0
29
—4_ 1
T=35=3
So, there is one repeated root as expected.

3 a 22+5+#0 for any real value of z, since 2% # —5 for any real x.


22 +5 =0 has no real roots.
b 224+5=0 has a=1, b=0, ¢c=5
A =0
— dac
= 0% —4(1)(5)
= —20 whichis <0 V
134 Chapter 4 (Equations) Exercise 4D.4

4 224+ 7x—-3=0 22-32+2=0


has a=1, b=7, ¢c=-3 has a=1, b=-3, ¢=2
A = — dac A =0 —dac
=7 - 4(1)(-3) = (=3 —4(1)(2)
=61 =]\
Since A > 0, but 61 is not a square, Since A > 0, and 1 is a square, there
there are 2 distinct irrational roots. are 2 distinct rational roots.

322422 -1=0 522 +42—-3=0


has a=3, b=2, ¢=-1 has a=5, b=4, ¢c=-3
A = b% — dac A = b? — dac
=22 —4(3)(-1) =47 — 4(5)(—3)
=16 =76
Since A > 0, and 16 is a square, there Since A > 0, but 76 is not a square,
are 2 distinct rational roots. there are 2 distinct irrational roots.

2?4+ +5=0 1622 -8z +1=0


has a=1, b=1, c=5 has a=16, b=-8, c=1
A = — dac A= —dac
=12 —4(1)(5) = (=8)* —4(16)(1)
=-19 =0
Since A < 0, there are no real roots. Since A =0, there is one repeated root.

622 —5r —6=0 202 —Tx —5=0


has a=6, b= -5, ¢=—6 has a =2, b=-7, ¢= -5
A =b% — dac A =0 —dac
= (=5)* — 4(6)(-6) = (=7)* —4(2)(-5)
=169 =89
Since A > 0, and 169 is a square, there Since A > 0, but 89 is not a square,
are 2 distinct rational roots which can be there are 2 distinct irrational roots.
found by factorisation.

322 +4x+1=0 622 — 472 —8=0


has a=3, b=4, c=1 has a=6, b= —47, ¢c= -8
A =% — dac A =1 — dac
=42 —4(3)(1) = (—47)* — 4(6)(-8)
=4 = 2401
Since A > 0, and 4 is a square, there Since A > 0, and 2401 is a square, there
are 2 distinct rational roots which can be are 2 distinct rational roots which can be
found by factorisation. found by factorisation.
Chapter 4 (Equations) Exercise 4D.4 135

e 422 -32+2=0 f 822 +20-3=0


has a=4, b=-3, c=2 has a=8, b=2, ¢c=-3
A =b* —dac A =b* — dac
=(-3)* —4(4)(2) =22 4(8)(-3)
=-23 =100
Since A < 0, there are no real roots. Since A > 0, and 100 is a square, there
are 2 distinct rational roots which can be
found by factorisation.

a 22+4x+m=0 has a=1, b=4, c=m

A = — dac
= 4% —4(1)(m)
=16 —4m
i For a repeated root, For two distinct real roots, iii For no real roots,
A=0 A>0 A<O0
16 —4m =0 16 —4m >0 16 —4m < 0
—4m = —-16 —4m > —16 —4m < —16
m=4 m <4 m >4

b ma?+3x+2=0, m#0 has a=m, b=3, c=2


A = — dac
=3%—4(m)(2)
=9—-8m
i For a repeated root, For two distinct real roots, iii For no real roots,
A= A>0 A<O0
9—-8m=0 9-8m >0 . 9-8m<0
—8m = —9 —8m> -9 —8m < -9

m=3 m<3, m#0 m > %

¢ ma®—3z+1=0, m#0 has a=m, b=-3, ¢c=1

A= —dac
= (=3)> —4(m)(1)
=9—4m
i For a repeated root, For two distinct real roots, iii For no real roots,
A=0 A>0 A<O0
9—4m =10 9—4m >0 L 9—-4m <0
—4m = -9 —4m > -9 —4m < -9
m=3 m<%2, m#0 .om> 94
136 Chapter 4 (Equations) Exercise 4E

a 22-52+6=0 [Hath Deg Form1) [Hath Degl Form1)


aX2 +bX+c=0 aX2 +bX+c=
Using technology, a b c
r=3o0r2 c 1 -5 I oL

DELETE @RS [REPEAT

b 224+92+14=0 [Hath Deg Form1) [Hath Degl Form1)


aX2 +bX+c=0 aX2 +bX+c=
Using technology a b c
r=—-2o0r —7 1 9 ]

DELETE MRS [REPEAT

¢ 22 —8x+16=0
Using technology, = =4

[SOLVE)DEEIBCLEARICEDIT]

d 222 —62+5=0 R
aX2 +bX+c=0
Using technology, there
are no real solutions.
Press: [EXIT]

[SOLVE) (EERCLEARICEDIT] [SOLVE) RENE@(CLEARICEDIT]

e 8224102 -3=0 [HathiPegforn]) [d7c)Real [HthiDes)Fornd) [d7c)Real


aX2 +bX+c=0 aX2 +bX+c=0
Using technology,
z =0.250r —1.5
]

DELETE /MRS [REPEAT

f 422 4+2-8=0 [HatH) Deg Form1) [Hath Deg form1)


aX2 +bX+c=0 aX2 +bX+c=0
Using technology, a b c
r~1.29 or —1.54 c 4 1 §;|:—l.54§

-1+J129
8
DELETEWRIY [REPEAT

g —bu?4+z+7=0 CRfe R
aX2 +bX+c=0 aX2 +bX+c=0
Using technology, a b =
xx[mm]
L -5 1 - %2l -1.087.
z ~1.29 or —1.09

[SOLVE] [CLEARICEDIT]
Chapter 4 (Equations) Exercise 4E 137

h 1,2 9.-2x-5=0
22 3 _ B Heledforn] (FFcResd fornD) _(d7c)Reall
aX2 +bX+c=0 aX2 +bX+c=0
Using technology, a b Xl[_ ELEL
C 0.25 -2 X2l -0.368.
r ~ 8.36 or —0.359

REPEAT]

22+ 6 =7 B HetiDegdorn] Hath Deglforn]) [d7c])


aX2 +bX+c=0 aX2 +bX+c=0
224+ 6x—-7=0 a b
Using technology, L 1 o I o
r=1or -7
DELETE, REPEAT]

42% +42 =15 B Hethiledorn] (7cReal fornD) (d7c)Reall


aX2 +bX+c=0 aX2 +bX+c=0
4%+ 42 —15=0 a b c
Using technology,
A ol
r=150r—-25
(SOLVE)DEEMACLEAR)

102% + 63 = 53 a (dTe)Rea) HethlDedForn]) (d7c)Rea)


aX? +bX+c=S aX2 +bX+c=0
102? — 53z +63 =0 a c
Using technology,
C 10 -53 v
r=350rl8
[JNE0E REPEAT]

—32% 4+ 122 =10 a (GTe)Read (HethlDedForn]) (d7c)Rea)


aX2 +bX+c=0 aX2 +bX+c=0
—3224+ 122 —10=0 Xl[@fl}
x2l1.1835
Using technology,
r~282o0r1.18
DELETE, REPEAT]

r=8—2x2 B HetiDedorn]
aX2 +bX+c=0
222 +2-8=0 a b m[m@}
L 2 1 %2l -2. 265.
Using technology,
x A~ 1.77 or —2.27
DELETE, REPEAT]

6 = 2z
— 5x? B HatiDegorn] [Hath)DeglForn1)
aX2 +bX+c=0 a 2
522 =22 +6=0 a b
No Real Roots
Using technology, there are
Press: [EXIT]
no real solutions.
138 Chapter 4 (Equations) Exercise 4E

S 4=322 -2z [t Deg forn]) (7c)Rea) B EsthleglFornD) (dlcIReal


aX2 +bX+c=0 aX2 +bX+c=0
327 —22—-4=0
ol oo
a b c

Using technology, £ s - D
x ~ 1.54 or —0.869
EOVE)EERCLEAR
EDT)

h Tr —2 = 4a? B Fbedform] [dFe)Real


aX2 +bX+c=0 aX2 +bX+c=0
422 —Tx+2=0 a b e}
x1[m1
Using technology, Co4 o %2l 0.3596.

x ~ 1.39 or 0.360
[SOLVE)CEYERE(CLEARICEDIT]

i 3.8z +2.12% = 52.6 [Hath) Deg) Forn1) E HstDegForn]) (d7cIReal


aX2 +bx+c=£) aX2 +bX+c=0
2.12% + 3.8z — 52.6 = 0 a <
Using technology,
C 2.1 3.5 B i;[ »5.99}

x ~4.18 or —5.99
-52.6 4.181121809

z(x+5)+2(x+6)=0 [t Degforn]) (7c)Rea) B HsthleglFornd) (dlcIReal


aX2 +bx+c=9 aX2 +bX+c=0
22 +5r4+22+12=0 <
224+ Tr+12=0 7 I ot |
Using technology,
r=-3or—4 [SOLVE)REERECLEARIEDIT]

b z(l+z)+z=3 et Peslforn]) (d7c)Rea) a (dTe)Read


ax?2 +bX+c=‘9 aX2 +bX+c=0
z4+a’+a-3=0 a
2’ +22-3=0 oL
Using technology,
z=1or-3 PIENE)3

< (x—1)(z+9) =5z B Fbedform] [dFe)Rea)


aX2 +bX+c=0 aX2 +bX+c=0
2® +8x—9—5x=0 b < x1[m!1
3 I %2l -4.854.
2> +32-9=0
Using technology,
x ~ 1.85 or —4.85 SOLVE)(EERCLEAR
EDT)

d 3z(x+2) —5(x—3) =18 [Hath) Deg) Fornd) B [HathDedForn])


aX2 +bX+c=b aX2 +bX+c=0
322+ 62 —br+15-18=0 a
;;[ —1.18}
3224+2-3=0
Using technology, -1+J37
6
x ~0.847 or —1.18
Chapter 4 (Equations) Exercise 4E 139

de(z+1)= -1 Fornd) (d7c]Real


aX2 +bX+c=0
4 +4x+1=0 x10 R <2
Using technology,
xz=—-0.5
[REPEAT]

2x(x
— 6) =x — 25 [Hath)DeglForn])
a
[Hat Deg lorm1)
2
aX2 +bX+c=0
2% — 122 —2+25=0 a b c
-13 No Real Roots
227 — 13z +25=0
C 2

Press: [EXIT]
Using technology, there are
no real solutions.

22 =92 =0 [Hath)DeglForn]) [Hat Deg lorm1)


aX3 +bX2 +cX+d=0 aX3 +bX2 +cX+d=0
Using technology, a b c X1
C X2 0
x=3,0,or -3 X3 -3.

[REPEAT]

23 —2224+4=0 [t Deslorn) (d7e)es)


aX3 +bX2 +°l),(+d=0 aX3 +bX2 +cX+d=0
Using technology, 10
[
r~—1.13 5 1 -2

-1.130395435
[REPEAT]

2% —2? —14x +24=0 [Hath)DeglForn]) [Hat Deg lorm1)


aX3 +bX2 +cX+d=0 aX3 +bX2 +cX+d=0
Using technology, a b c d X1
x=3,2,or —4 C 1 -1 -1 X2 2
X3 -4

24
[REPEAT]

2
—2d42=2r—=x ' n) [Rea) Btibedforml) (IR
aX3 +bX +c‘)’(+d=0 aX3 +bX2 +cX+d=0
—2¥+2?—-22+2=0 X1C -
Using technology, = =1

SOLVE) EERCLEAR
EDIT) [REPEAT)

2 42?2 =3z —1 [Hath] DeglForn]) [Hathi Deg ormd)


aX3 +bX2 +cX+d=0 aX3 +bX2 +cX+d=0
228 422 —32+1=0 a b c X1
C 2 1 X2 0.5
Using technology, X3L-1.618

x=0.5,~ 0.618, or —1.62


0.6180339887
[REPEAT]
140 Chapter 4 (Equations) Exercise 4E

f 22°% + 8 = 5a? + 18z


2% — 522 — 182 +8=0
Using technology, [l (ot Degorml) El ([t Degfornl) (dFc)Res)
aX3 +bX2 +cX+d=0 aX3 +bX2 +cX+d=0
x =~ 4.36, 0.406, or —2.26 a b ¢ _d X1
C 2 -5 -18 IEE) X2| 0.408
X3L-2.261

4.355498448
[SOLVE]DEN3[3(CLEAR]

2t —a2?+2=0 B HatfiDedlflom]) (dFclReal B Fetfiledfom] (dFclReal


ao X4 +a1 X3+ - - +as=0
Using technology, there are a0 al a2 a3 o

no real solutions. *-
Press: [EXIT]

[STOIRYE|DELETE, (SOLVE)BEEA CLEARI(EDIT]

b 2?4223 32242 4=0 B HeatDegdforn] (TcIRes) B Fetibelforn] (dFcRea)


ao X2 +a1 X3+ - -+as=0 ao X4 +a1 X3+ - -+aa=0
Using technology, a0 al a2 a3 -

r~1.34 or —=3.17 C- 2 -3 1 i;[ —3.17r:|

1.340778787
[SOLVE]DENE[3(CLEAR]

¢ 2t —222+1=0 B Fibedfom]) [@FelRea) Bl Fefledforn] ([)Rea)


aoX4+a1 X3+ ++as=0 ao X4+a1 X3+ - -+as=0
Using technology, a0 al a2 a3 o x1[-11 x2
rz=1or—1 * - 0 X2 =-1J x2

(SOLVE] &3

d 2t -2 +322-2+6=0 B HathDegForn])
aoX4+a1 X3+ -+as=0
Using technology, there are a0 al a2

no real solutions. C- -1
Press: [EXIT]

[SOLVE]PENE[3(CLEAR] DELETE

6 a z(2? —1) =2z B FHbedfom] [@Felfea) B Fdifedforn] [dFclRea)


aXs +bX2 +c‘)’(+d=0 aX3 +bX2 +cX+d=0
P —x—20=0 X1
S
22 =32 =0 X2|
3L-1.732]
0

Using technology,
r=0,~1.73,or —1.73

b (x—2)(x+1) =2® gforn]) (d7c)Real


X2 +Ct)>(+d=0
22 —r—-2-2%=0 < I 0
x1

2t —2-2=0
Using technology,
-0.8105357138
z~ —0.811 (SOLVE) MR
CLEARIEDIT]
Chapter 4 (Equations) Exercise 4F 141

c (2% +1)(z® —2) =10 (GFe)Re) B Heiledforn] (FFcRead


ao X4+°a1 X3+: . -+as=0 aoX4+a1 X3+ -+as=0
=22 -2-10=0 a al a2 a3 > Xl[fi

2t —22-12=0
C- -1 o X2| =

Using technology, T = +2

[3[CLEAR]

a 22 —5=z+1 b Wegraph y =2%2—-5 and y =2 +1


22 —5—2—-1=0 on the same set of axes.
o2tz -6=0 [EXE]:Show coordinates
V1=x2-5
s (z+2)(z—-3)=0
r=—-2o0r3

The graphs intersect at (—2, —1) and


(3, 4).
the solutions are = —2 or 3.

a Wegraph y =2 and y=5—./T on We graph y = 2 and y =3z +4 on


x
the same set of axes.
the same set of axes.
[EXE]:Show coordinates
Yi=x 13y [EXE]:Show coordinates
V2=5-(yx) VI=2.x
V2=3x+4

T
11
INTSECT
¥=3.208712163 INTSECT
¥=-1.16227766

The graphs intersect at (3.21, 3.21).


The graphs intersect at (—1.72, —1.16)
the solution is = ~ 3.21. and (0.387, 5.16).
the solutions are = ~ —1.72 or
0.387.

¢ We graph y =3" and y =15 on the We graph y =5 x 27! and y = 90


same set of axes. on the same set of axes.
[EXE]:Show coordinates [EXE]:Show coordinates
Y1=3(x) 1y =6x2"(x~1)
V2=15-

INTSECT
X=2.464073621 ON=15 26001 ¥=90 x

The graphs intersect at (2.46, 15). The graphs intersect at (5.17, 90).
the solution is = ~ 2.46. the solution is = ~ 5.17.
142 Chapter 4 (Equations) Exercise 4F

e Wegraph y=22+45 and y=4z+/z We graph y = (z +1)(z — 3) and


on the same set of axes. y =+/x + 3 on the same set of axes.
[El [EXE]:Show coordinates B [EXE]:Show coordinates
Y1=x2+5
YI=(x+1) (r-3)13¥
Y2=4x+({x)
v2=J(x+3

11
INTSECT
X=1.6518184411 Oly=7.304883905 ° ¥=1.302775638

The graphs intersect at (1.52, 7.30) and The graphs intersect at (—1.30, 1.30) and
(2.83, 13.0). (3.56, 2.56).
the solutions are x ~ 1.52 or 2.83. the solutions are ~ —1.30 or 3.56.

a We graph y:12+%. We graph y = 2% — 2.


[El [EXE]:Show coordinates
B [EXE]:Show coordinates

3453323
X=-1.687401052 y°l¥=0

The z-intercepts are ~ —0.861, 1.24,


The z-intercept is ~ —1.59.
and 16.
the solution is = ~ —1.59. the solutions are
x ~ —0.861, 1.24, or 16.

¢ Wegraph y =z
+ Jx + 4. We graph y = 222 — o + 1.
[l [EXE]:Show coordinates [El [EXE]:Show coordinates
VI=ct (0 +E 13y V1=2x2-( (x+1)) 159

X=-0.57198505656

The x-intercept is ~ —2.62. The z-intercepts are ~ —0.572 and 0.821.


the solution is = ~ —2.62. the solutions are
z ~ —0.572 or 0.821.
4 1
e We graph y = 2% +27% — 30. We graph y =
2245 a241°
B [EXE]:Show coordinates
=x~(8)+2~(-x)-30Y B [EXE]:Show coordinates
Y1=(41(x2+5))-(1a(%2+1))

X=-2.242084731-348=
X=-0.5773502692 L’=D

The z-intercepts are ~ —2.24 and 2.34.


The z-intercepts are ~ —0.577 and 0.577.
the solutions are x ~ —2.24 or 2.34. the solutions are
x ~ —0.577 or 0.577.
Chapter 4 (Equations) Review set 4A 143

4 a i Wegraph y = 222 — 12z +11 and ii We graph y = 22> — 122 + 11 and


y =1 on the same set of axes. y = —7 on the same set of axes.
[EXE]:Show coordinates [EXE]:Show coordinates
V1=2x2-12x+11 Y1=2x2-12x+11
v2=1

INTSECT INTSECT

The graphs intersect at (1, 1) and The graphs intersect (touch) at (3, —7).
(5,1). . the solution is z = 3.
the solutions are = =1 or 5.

ili We graph y = 222 — 12z + 11 and


y = —10 on the same set of axes.

The graphs do not intersect.


there are no real solutions.

b 22 — 122+
11 =k
202 — 122+
(11 —k)=0 has a=2, b=—12, c=11—k
A =1 — dac
=(-12)? —4(2)(11 - k)
=144 — 88 + 8k
= 8k + 56
i For two real solutions, ii For exactly one real iii For no real solutions,
A>0 solution, A<0
8k + 56 > 0 A=0 o 8k+56 <0
- 8k > —56 s 8k +56=0 o 8k < —56
k>-7 .. 8k = =56 k<=7
o k=-T
which agrees with our results in a.

REVIEW SET 4A |-

1 a 227=38 b (z-2)?%=25
2?2 =19 {dividing both sides by 2} sz—2=4V25
x=::\/fi .o rx—2=45
144 Chapter 4 (Equations) Review set 4A

¢ 3x—V2)?2=6
(-2 =2 {dividing both sides by 3}
L r—V2=4V2
. I=\/_::\/§

i z=2\/§ or 0

2 a zi=-9 b x3=2_17 ¢ (z-1P°=2


has no real solutions ) e oz —1=1+2
asx“‘cannotbe ST =4/ 5= r=143
negative. z:%

3 a z(z+2)=0 b —(z+3)(2z-7)=0
L x=0o0r z+2=0 S (@+3)(22-7)=0
. x=0o0r
-2 SLrx+3=0o0 2x—7=0
o] el or%

¢ (z+5)(z+1)(z—6)=0
. ax+5=0o0or x4+1=0 or 2—-6=0
. x=-5 —1,0or6

5 a 322 52 =0 b 2 —dr—5=0
ox(3z—5)=0 S (+1)(x—-5)=0
wx=0 or 3x—-5=0 S ax=—-lorb
=0 or 3

¢ 22 +62+9=0
(x+3)*=0
Lo =-3

5 a (z+3)% =5z 429 b z(z—4)—(z—6)=0


a2’ +6r+9=>50+29 ot —dr—24+6=0
a4+ —20=0 a5 +6=0
oo (@+5)(z—4)=0 So(x=2)(z=3)=0
. x=-5or 4 . x=2o0r3

c (1—2x)(4—z) =39 d 4o — 3 = 2*
L d—2—8r+222 =139 s —4x+3=0
. 222 -9x—-35=0 S (@=1)(x-3)=0
2z +5)(x—7)=0 sox=1or3
Lwx=-2or7

e 322 =2 52 f 222 — 108


= 6z
s 322452 -2=0 o222 108 =0
— 62—
Lo Br—1DE+2)=0 oo 22?32 —54)=0
L oax=1%or =2 2@ +6)(z—9)=0
S (z+6)(x—-9)=0
. x=—6o0r9
Chapter 4 (Equations) Review set 4A 145

a 2?2 —6x+4=0 b 22 -2 —-1=0


L2’ —6r=—4 ot —2r=1
22— 62+ (—3)% = -4+ (-3)? ©oa? = 2r 4 (=) =1+ (—1)?
(x—-3)*=5 (x—1)%=
Tr — =::\/5 I—1=::\/§
‘L:?)::\/g "L:l::\/a

< 2%
4+ 8x =1
x2~4x:%
4z +22=1+42°
(z+2)3?=2
r+2==+
o S <
W
8

Il
lny
}
8

H
[\]

sk
Il
|
8

7 a 22 -Te+2=0
has a=1, b=-7, c=2

_ (0 VP a0®
s 2(1)
©op= 7::\/4_
2 -2
which has no real solutions as
v/ —12 is not real.
¢ 322-24+3=0
has a=-3, b=-1, ¢=3

e (=12 -4(=3)(3)
e 2(-3)
_lzzfi
et
[E=_ ::\/fi

6
9
a ==

co2t=9
S22 —9=0
(z+3)(x—3)=0
fl/':::3

c 3r—1=§
L322 —z=2
o3P —2-2=0
Bz +2)(x—1)=0
wax=-2orl
146 Chapter 4 (Equations) Review set 4A

9 622—2—-2=0 has a=6, b=—-1, c=-2

a A=0—4dac b T = _;:—A
= (1462
(12 _
s B
=49 e )
Since A > 0, and 49 is a square, there 14+
are 2 distinct rational roots. ) = =
M3
r=F5o0r —356
r=3zo0r—3
So there are 2 distinct rational roots as
expected.

10 222 —52+4=0 has a=2, b=-5 c=4


A = — dac
= (=5)* —4(2)(4)
=-7
Since A < 0, there are no real roots.

11 a’+br+c=0, a#0
—b =4 /b% — dac
Tr =
2a
X —b+4 /b2 -4 —b— /b2 —4
The sum of the solutions = Sl Vats =0dC AA
2a 2a
-2
T 2

. 73, a#0
a

12 a 22°-322-92+10=0 8 Ealelias] (@ofa) B Watibsion] (o)


. aX3 +bX2 +cX+d=0 aX3 +bX2 +cX+d=0
Using technology, a b < d X
s—3,1, 01 -2 T | [

b 32% = a(7x — 2) EetlOedleat Eeilsalion] G


. aX3 +bX2 +cX+d=0 aX3 +bX2 +cX+d=0
32® =72 — 2 a b —< X1
= 3 ;g{o.aaa:
3% — T2 +22=0
Using technology,
z=2 ) %3 or0

< 2%+ 60 = 23z + 227


2% —22%
— 23z +60=0
Using technology, Wt Dol eDelorn]) (@l
r=4,3, or -5 axs +1:x2 +°1),(+d=0c aX3 +bX2 +cX+d=0
d X1
C 1 -2 -23 X2 3
X3 -

60
Chapter 4 (Equations) Review set 4B 147

2(2? — 3) = 64 — 62° — 142


ot =322 — 644 62° + 142 =0
ot +62° — 302 + 142 —64=0
Using technology, B _Gislon) G B Eoibaten) GoEw
2+~ 1.84 or —6.92 ao X' ;I—omx:rutgA—O!a# aoX4+a1 X3+- - -+as=0

*- 8 -3 12 ;;[ —6.919}

1.835003664

13 We graph y = 10 x 27! and y = 35 b We graph y =+/z and y:éfl on


on the same set of axes. the same set of axes. 5
IZ<5]; SHonleadnd (hatad [EXE]:Show coordinates
Yi={x
¥2=(4.1x)-1

= 0

INTSEC'I;( Jfi INTSECT
X=1.728163201 =1.314596212

The graphs intersect at (2.81, 35). The graphs intersect at (1.73, 1.31).
the solution is x ~ 2.81. the solution is = ~ 1.73.
We graph y = 2® — ¢/x +5.
[EXE]:Show coordinates
V1=x"(3)-(x)+5 13y

The 2-intercept is ~ —1.84.


the solution is = ~ —1.84.

1 ~T2? = b —da? =12 ¢ (z-V3)?=16


22 = 13:7%5 ox—V3==+Vv16
r=0 125 r—V3==44
_ 3 28
L T8 Ta— \/_ +4

v /=125
V8
._ 5
T=-—3
148 Chapter 4 (Equations) Review set 4B

2 a =8 b 25=-18 ¢ (z-1)"?=4
oy el z= /18 @-12=1
- T=E\Te
481 r—1==+ %
.Z':::4
16 w71:::%

.Z':::% ' z:l::%


z=3or3

3 a Z-p b 22_ ¢ 5
q Y ab
. p=0, ¢#0 S 2x2=0, y#0 So=h=
Loxz=0, y#0 which is impossible
. x=0or 2z=0, y#0 - there are no solutions.

5 a 222 — 52 =0 b 322 -122=0


©oz(2z—-5)=0 o 3z(r—4)=0
wx=0 or 2r—5=0 co3r=0o0r z—4=0
ox=0or % s.ox=0 or 4

¢ 2 =Tz +6=0 d 2?4+ 4= —dx


L (z-1D)(@@-6)=0 Sl t4r+4=0
x=1or6 s (@+2%=0
Lox=-2

e 2 —12 =4z f 322 —2—10=0


coox?—4r—12=0 S Bz +5)(z-2)=0
(z+2)(x—6)=0 som=-2or2
L rx=-2or 6

5 a 2% — 11z = 60 b 22 +5z+5=0
L2t —llz+ (—4)2 =60+ (—4)? o245 =—5
(x—4)? =60+ 12 #* +52+ (5)°=-5+(5)°
(- = i r4+85\2 _ 5
)2=5
I—12—1=:_ % ( 3) 1
/361
. -y 11 _=4e x4 d=4,
2 /2
A£4
5_ 115
zf%:::Tg I+§———\/Z

w:%::% B %7Ӥ

z=30or & 5
) 2 r=-3+%
z=150r —4 5443
Chapter 4 (Equations) Review set 4B 149

c 4o —5r =6
2 5.,._3
x 71‘%75

gt
(-9 =4+ (B
(z-3)0°=35+%
52 _ 3 25

(r—3)7" =%
5)2
_ 121

5 _ 121
Ty =V e
5 _ 4 /121
I
TTET
5 _11
rT—3g==*%
oo 5 11
rT=5E%F
uL:%’orfg
xz?orfg

22 —dr=-5
2% —dx + (-2)? = =5+ (-2)?
(=22 =-1
which has no real solutions as (x —2)? cannot be negative.

a 224+52+3=0 b 322 +11lz-2=0


has a=1, b=5, ¢=3 has a=3, b=11, ¢=-2

—5 4 1/52 — 4(1)(3) —114 /112 — 4(3)(-2)


r=— S
2(1) 2(3)
o —5++/13 s —11+ /145
2 6
¢ 522 4+4x-2=0
has a=5, b=4, c=-2

_ —4x 42 — 4(5)(-2)

- 20)
—4+ \/%
r=——
0
s —44 214
o 10
—24+/14
s VE
5

a 22-8x+16=0 b 2:2—2-5=0
has a=1, b=-8, ¢c=16 has a=2, b=-1, ¢c=-5
A =b* — dac A =b*
— dac
= (=8)% —4(1)(16) = (=1)> - 4(2)(-5)
=0 =41
Since A =0, there is one repeated root. Since A > 0, but 41 is not a square,
there are 2 distinct irrational roots.
150 Chapter 4 (Equations) Review set 4B

¢ 322+52+3=0
has a=3, b=5, ¢=3
A =0 — dac
=57 —4(3)(3)
=-11
Since A < 0, there are no real roots.

9 222 -3x+m=0 has a=2, b=-3, c=m


A =b? — dac
= (=3)* —4(2)(m)
=9—-8m
a For a repeated root, b For two distinct real roots, ¢ For no real roots,
A=0 A>0 A <0
. 9-8m=0 S 9-8m >0 S 9-8m <0
—8m = -9 S —=8m > =9 coo=8m < =9
m=1 L m<d T
10 a i 2z(z+4)=8x+k) il 2z(z+4)=8(z+k)
When k=9, When k=09, 2z(x+4)=8x+9)
2z(z +4) =8(z+9) We graph y = 2z(z +4) and
22% + 82 = 8z + 72 y=38(z+9) on the same set of axes.
ZIQ =72 [B [EXE]:Show coordinates
woa? =36
L= ::\/%

r =16

The graphs intersect at (—6, 24) and


(6, 120).
the solutions are = = —6 or 6.
b 2e(x+4) =8(z
+ k)
2207 48z =8z
+ 8k
o222 =8k
Loa? =4k
< 2% =4k
.o r=xvV 4/6

L= ::2\/E

i The equation 2? =4k has two real solutions if % > 0.


i The equation 2% = 4k has one real solution if & = 0.
ili The equation x? =4k has no real solutions if k& < 0.
Chapter 4 (Equations) Review set 4B 151

1 a Wegraph y=222—-7 and y =3z on b We graph y =2(l—2) and y = —10


the same set of axes. on the same set of axes.
[EXE]:Show coordinates [EXE]:Show coordinates
Y1=2x2-7 oY Y1=x(1-x)
V2=3x Y2=-10

INTSECT
X=-1.265564437 -°¥=-3,7966983311 X=-2.70166

The graphs intersect at (—1.27, —3.80) The graphs intersectat (—2.70, —10) and
and (2.77, 8.30). (3.70, —10).
the solutions are 2 ~ —1.27 or 2.77. .. the solutions are = ~ —2.70 or 3.70.
¢ Wegraph y =2(z—2)+4(z—1) and
y =3 on the same set of axes.
[EXE]:Show coordinates
Y1=x(x-2)§4(x-1) o[y
V2=3

The graphs intersect at (—3.83, 3) and


(1.83, 3).
the solutions are = ~ —3.83 or 1.83.

12 a 2% — 152 = 227 Eeedlem) )Rl [t Oelforn) (7<) es)


23— 227 152 =0 e )] aX3 +bX2 +cX+d=0
q 3 2 =
d X1

Using technology, c 1 -2 -15 D X2 0


X3 -3

x=>5,0,o0r -3
PN REPEAT]

b 23422 -62+7=0 8 Eibsis) ok fornD) (d7c)Reall


. X3 +bX2 +cX+d=0 aX3 +bX2 +cX+d=0
Using technology, S c X1
X2| 0.8144
r~4.93,0.814, or —1.74 X3L-1.743.

4.929142304
(SOLVE]PHIEACLEAR) (REPEAT

¢ 4x'—
1122
42-82+6
3=0 Hetiealors] (G)Feal [Hatiegforn]) (d7c)Real
: aoX4+a1 X3+ - -+as=0 ao X4 +a1 X3+ - -+as=0
Using technology, o e e a2t Yasls Xl[mfifi

7~ 2.39 or 0.449 C IR L X2L 0.4485.

2.390541979
[SOLVE)PRYERCLEARICEDIT]
152 Chapter 4 (Equations) Review set 4B

13 a Wegraph y =2 and y =7 on the b We graph y=2% and y =9—2/ on


same set of axes. the same set of axes.
[El [EXE]:Show coordinates [EXE]:Show coordinates
Y1=x"(3)
¥2=9-2(4x)

g 5
INTSECT INTSECT
X=1.845270174/ “Y¥=6.2831855661

The graphs intersect at (2.81, 7). The graphs intersect at (1.85, 6.28).
the solution is x ~ 2.81. the solution is = ~ 1.85.

¢ We graph y=%— T+ 3.

B [EXE]:Show coordinates
Y I=((x2) 15)—(f (x+3)F

X=-2.148183918 S¥=0

The z-intercepts are ~ —2.15 and 3.58.


the solutions are z ~ —2.15 or 3.58.
Chapter 5
SEQUENCES AND SERIES

1 a 4,13, 22,31 b 45, 39, 33, 27 ¢ 2,6,18, 54 d 96, 48, 24, 12


A AA A A A A A A A A A
+9 +9 +9 —6 —6 —6 x3 x3 x3 fe2y==2 =2

2 23,5711, 13,17, 19, ....

a ux=3 b us =11 ¢ ujo =29 {the 10th prime number}

3 4,7, 10,13, 16, ....


a We start with
4 and add
3 each time. b u =4, U4:13

¢ ug=us+3+3+3
=16+3+3+3
=25

b u,=2n+5
up =2(1)+5 up =2(2)+5 ug =2(3)+5 ug =2(4)+5
= = =11 =13

5 u,=3n-2
a uy =3(1)—2 b us =3(5) -2 € uUgy = 3(27) -2

=1 =13 =179

6 —9,-6,—-1,6,15
a Using A, u, =n-—10
So, up=1-10 us =2
— 10
=-9 Vv =-8 x
Using B, u, =n?>—10
So, u; =1%2-10 uy =22
- 10
=-9 v =—6 Vv
ug =42 — 10 us =52
— 10
=6 Vv =15 v
Using €, u, =n3—10
So, u; =1%-10 uy =23
— 10
=-9 v =-2 x
So, B is the correct formula.

b U0 = 202 — 10

=390
154 Chapter 5 (Sequences and series) Exercise 5A

7 a 8,16, 24, 32, ...


The sequence starts at 8 and each term is 8 more than the previous term.
The next two terms are 40 and 48.
b 2,58 11, ...
The sequence starts at 2 and each term is 3 more than the previous term.
The next two terms are 14 and 17.
¢ 36, 31, 26, 21, ....
The sequence starts at 36 and each term is 5 less than the previous term.
The next two terms are 16 and 11.
d 96, 89, 82, 75, ....
The sequence starts at 96 and each term is 7 less than the previous term.
The next two terms are 68 and 61.
e 1,4, 16,64, ..
The sequence starts at 1 and each term is 4 times the previous term.
The next two terms are 256 and 1024.
f 2,6,18, 54, ...
The sequence starts at 2 and each term is 3 times the previous term.
The next two terms are 162 and 486.
g 480, 240, 120, 60, ....
The sequence starts at 480 and each term is half the previous term.
The next two terms are 30 and 15.
h 243,81, 27,09, ..
The sequence starts at 243 and each term is one third of the previous term.
The next two terms are 3 and 1.
i 50000, 10000, 2000, 400, ....
The sequence starts at 50 000 and each term is one fifth of the previous term.
The next two terms are 80 and 16.

a 1,4,916, ...
Each term is the square of the term number. The next three terms are 25, 36, and 49.
b 1,8, 27,64, ...
Each term is the cube of the term number. The next three terms are 125, 216, and 343.
¢ 2,6,12,20, ...
Each term is n(n + 1) where n is the term number. The next three terms are 30, 42, and 56.

a 95,91, 87, 83, ...


Each term is 4 less than the previous term, so the next two terms are 79 and 75.
b 5,20, 80, 320, ....
Each term is 4 times the previous term, so the next two terms are 1280 and 5120.
¢ 1,16, 81, 256, ....
Each term is the fourth power of the term number, so the next two terms are 5% = 625 and
64 = 1296.
d 2,3,5711,..
This is the sequence of prime numbers, so the next two terms are 13 and 17.
Chapter 5 (Sequences and series) Exercise 5B.1 155

e 2,4,7, 11, ...


A A S
243 ¥4
The difference between terms increases by 1 each time, so the next two terms are 11+5 = 16
and 16 + 6 = 22.
f 9,810, 7,11, ....
Each odd numbered term is 1 more than the previous odd numbered term, and each even
numbered term is 1 less than the previous even numbered term, so the next two terms are
7—1=6 and 11+1=12.

10 The sequence {2n} begins 2, 4, 6, 8, 10 (letting n =1, 2, 3, 4, 5, ....).


T 0o

The sequence {2n — 3} begins —1, 1, 3,5, 7 (letting n=1, 2, 3, 4, 5, ....).


The sequence {2n + 11} begins 13, 15, 17, 19, 21 (letting n =1, 2, 3, 4, 5, ....).
an

The sequence {3 —4n} begins —1, —5, —9, —13, —17 (letting n =1, 2, 3, 4, 5, ....).
O

The sequence {n? +2n} begins 3,8, 15, 24, 35 (letting n =1, 2, 3, 4, 5, ....).
-« ®

The sequence {2"} begins 2, 4, 8, 16, 32 (letting n =1, 2, 3, 4, 5, ....).


The sequence {6 x (%)”} begins 3, %, %, %, 1—36 (letting n=1, 2, 3,4, 5, ....).
W

The sequence {(—2)"} begins —2, 4, —8, 16, —32 (letting n =1, 2, 3, 4, 5, ....).
T

The sequence {15 — (—2)"} begins 17, 11, 23, —1, 47 (letting n =1, 2, 3, 4, 5, ....).

1 a 715,23 31,39, ...


15-7=8 The difference between successive terms is constant.
23-15=38 . the sequence is arithmetic with u; =7 and d = 8.
31-23=38
39-31=38

b 10, 14, 18, 20, 24, ....


14-10=4 The difference between successive terms is not constant.
18—-14=4 .. the sequence is not arithmetic.
20—-18=2
24-20=4

¢ 41, 35, 29, 23, 17, ...


35—-41= -6 The difference between successive terms is constant.
29 —35=—6 .. the sequence is arithmetic with u; =41 and d = —6.
23-29=-6
17—-23=—-6

d 6,1, -6, —11, —16, ....


1-6=-5 The difference between successive terms is not constant.
—-6—-1=-7 . the sequence is not arithmetic.
—11—(-6) = -5
16— (—11) = —5
156 Chapter 5 (Sequences and series) Exercise 5B.1

a 5,9,13,17, 21, ... b —4, 3,10, 17, 24, ... 23,18, 13,8, 3, ...
9-5=4 3—(-4)=7 18 -23=-5
13-9=4 10-3=7 13-18=-5
17-13=4 17-10=7 8§—13=-5
21-17=4 24-17=7 3-8=-5
(5% :5, d=4 U1:74, d=17 u =23, d=-5

d -6, —15, —24, -33, ....


—15— (—6) = —9
—24— (—15) = —9
33— (-24) = —9
uy = —6, d = —9

a 19,25, 31, 37, ...


i 25—-19=6 U, =up
+ (n—1)d il w5 = 6(15) + 13
31-25=6 Sy =1946(n—1) =103
37—-31=6 Uy =6n+13
up =19, d=6

b 101, 97, 93, 89, ....


i 97-101=—4 U =u1
+ (n—1)d il w15 = 105 — 4(15)
93 —-97=—4 Coou, =101 —4(n—1) =145
89 —-93=—4 . up =105 —4n
up =101, d=—4

¢ 8,91, 11,124, ..
up =ui
+ (n—1)d i w5 =13(15) + 63
ol

LU, =8+13(n—1)
Rl

ou, =130+ 6%
Nl
ol=
U
Il

Il
-<

d 31, 36, 41, 46, ....


i 36-31=5 U, =up + (n—1)d il w5 = 5(15) + 26
41-36=5 S U =31+5(n—1) =101
46 —41 =5 o Up = 5n 426
U1:31, d=5

e 5, -3, —-11, —19, ...


up =ur+ (n—1)d il w5 = 13 — 8(15)
cooup,=5-8(n—1) =—107
. U, =13 -8n
Chapter 5 (Sequences and series) Exercise 5B.1 157

f a, a+d, a+2d, a+3d, ..


i at+d—a=d i Uy, =up + (n—1)d iii ws =a-+14d
a+2d—(a+d)=d Soup=a+(n—1)d
a+3d—(a+2d)=d
w =a, d=d

4 6,17, 28, 39, 50, ....


a 17-6=11 The difference between successive terms is constant.
28— 17=11 .. the sequence is arithmetic with u; =6 and d = 11.
39—-28=11
50—39=11
b u,=u +(n—-1)d ¢ uso=11(50)—5
=6+11(n—1) =545
=1ln—5

d Let u,=325=1In—5 e Let uw,=T761=1In-5


. 330=11n . 766 =11n
n =30 Son=69%
So, 325 is the 30th member of the but n must be an integer, so 761 is not a
sequence. member of the sequence.
5 87,83,79, 75, 71, ...
a 83—-87=-4 The difference between successive terms is constant.
79—-83=—-4 .. the sequence is arithmetic with u; =87 and d = —4.
75 —T9=—-4
T1—T5=—4

b u,=u +(n-1)d ¢ ug =91 — 4(40) d Let wu,=-297=91—4n


=87—4(n—1) =91 - 160 . 4n = 388
=91 —4n =—69 coon=97
So, —297 is the 97th term of
the sequence.

6 a Up =3n — 2 b uy=31)-2=1, d=3


Uni1 =3(n+1) -2 ¢ usy=3(57) — 2 =169
=3n+1
Upp1 — Up =3n+1—(3n—2)
=3, a constant
Consecutive terms differ by 3.
the sequence is arithmetic.
d Let u,=450=3n—2
3n = 452
n=1503
We try the two values on either side of n = 1502, which are n =150 and n = 151:
50 = 3(150) — 2 =448 and w5 = 3(151) — 2 = 451
So, u1s0 = 448 is the largest term which is smaller than 450.
158 Chapter 5 (Sequences and series) Exercise 5B.1

7 a u"=71—7n u"+1=71—7(n+1)
2 2
:35%_%71 =T —17
- 2
_ 64—Tn

=32—1In
Uny1 — Up = (32— Zn) — (353 — In
=—I, aconstant
So, consecutive terms differ by —%.
the sequence is arithmetic.

b ulzn%m:32, d:,% < u75:71*T7(75):,227

d Let wu,=-200=_"""
—400="71—"Tn
n =471
. . n=07% 72

We try the two values on either side of n = 67%, which are n =67 and n = 68:
TL—7(67) _ 71— 7(68)
Ug7 = —199 and wugg = =—2021
So, the terms of the sequence are less than —200 for n > 68.

8 36, 353, 342, ...


a 351-36=-
i
22 342-3
2
51=-2
_ 2
Uy =uy + (n—1)d
uy = 36, d= —% —30=36—2%(n—1) {letting u, =—30}
—66=—2n+2
Zn =662
. n=100
So, —30 is the 100th term.

9 23, 36,49, 62, ...


36 —-23=13, 49-36=13, 62—49 13
u =23, d=13
Up =up + (n—1)d
=93+ 13(n—1)
=13n+10
Let w, =100000 = 13n + 10
99990 = 13n
" n=T691L
We try the two values on either side of n = 7691-%,137 which are n = 7691 and n = 7692:
uzge1 = 13(7691) + 10 = 99993 and wuzge2 = 13(7692) 4+ 10 = 100 006
So, the first term to exceed 100000 is wuzg92 = 100 006.
Chapter 5 (Sequences and series) Exercise 5B.1 159

10 a Upt1 = Up + 7 b U, =up
+ (n—1)d
Up41 — Up = 7 UL U200 = -12+ 199(7)

So, consecutive terms differ by 7. .. ugoo = 1381


the sequence is arithmetic.

¢ Let u,=1000=—-12+4+"7(n—1)
Tn —7=1012
n = 1019
n = 1453
but » must be an integer, so 1000 is not a member of the sequence.

1 a The difference between any two even numbers is even.


if all of the terms of an arithmetic sequence are even, then u; and d are also even.
b The difference between any two odd numbers is even.
if all of the terms of an arithmetic sequence are odd, then u; is odd and d is even.

12 a 32,k 3
Since the terms are consecutive, k —32 =3 —k {equating differences}
2k =35
k=173
b k7,10
Since the terms are consecutive, 7 —k =10—7 {equating differences}
7T—-k=3
k=4
¢ k, 2k—1, 13
Since the terms are consecutive, 2k —1—k =13 — (2k —1) {equating differences}
k—1=14-2k
3k =15
k=5
d k 2k+1, 8-k
Since the terms are consecutive, 2k +1—k=8—k— (2k+1) {equating differences}
k+1=7-3k
4k =6
b=
e 2k+7, 3k+5, 5k—4
Since the terms are consecutive,
3k+5—(2k+7) =5k —4— (3k+5) {equating differences}
k—2=2k—-9
k=17

f 2k+18, —2—k, 2k+2


Since the terms are consecutive,
2—k—(2k+18) =2k +2—(—2—k) {equating differences}
—3k—20=3k+4
—6k =24
k=-4
160 Chapter 5 (Sequences and series) Exercise 5B.1

g kK% Kk*+6
Since the terms are consecutive, k? —k = k>4 6 —k*> {equating differences}
B —k-6=0
(k+2)(k—=3)=0
. k=-2or 3
h 5,k k*-38
Since the terms are consecutive, k—5=%? —8—k {equating differences}
ook —26-3=0
(k+1)(k—=3)=0
w k=-1or 3

13 a ur=41 . u+6d=41 .. (1) {using u, =us + (n—1)d}


w3 =77 c.our+12d=77 ... (2)

We now solve (1) and (2) simultaneously:


—u; — 6d = —41 {multiplying both sides of (1) by —1}
uy +12d =77
. 6d =36 {adding the equations}
. d=6
So, in (1): uy + 6(6) = 41 Check:
oup +36=41 ur =6(7) —1
Sooup =5 =42-1
Now wu, =u;+ (n—1)d =41 v
B un:5+6(n71) U13:6(13)71

Uy, =6n—1 =78-1


=77 V
b us=-2 cooug+4d=-2 v () {using w, =uy + (n—1)d}
wy = —125 . ow+1ld=-125 .. (2)
We now solve (1) and (2) simultaneously:
—uy — 4d =2 {multiplying both sides of (1) by —1}
uy +11d = —12%

. 7d=-103 {adding the equations}

So, in (1): up +4(—3) = -2 Check:


up —6=-2 us =—3(5)+4
cooup =4 :7%+%

Now wu, =u;


+ (n—1)d 4 o g
LU, =4—-3(n—-1) .
) o un——5n+7
n_ 32 11 up =—-3(12)
- 36+11
+ 5
= B
25
_=-Z=_121 v
Chapter 5 (Sequences and series) Exercise 5B.1 161

¢ ur=1 cooup+6d=1 Sen(1) {using wu, = u; + (n — 1)d}


us =—-39 . owu+14d=-39 ... (2)
We now solve (1) and (2) simultaneously:
—u; — 6d=-1 {multiplying both sides of (1) by —1}
uy + 14d = —39
8d = —40 {adding the equations}
sod=-=5
So, in (1): up +6(=5) =1 Check:
u —30=1 ur = —5(7) + 36
up =31 =—-35+36
Now u, =u; + (n—1)d =1 v
L up=31-5(n-1) uys = —5(15)
+ 36
Uy =31—5n+5 = 75+ 36
Uy = —5n + 36 =-39 vV

d u=-16 . u;+10d=-16 ... (1) {using u, =u; + (n — 1)d}


ug=—11%2 - w+7d=-11}3 .. (2)
We now solve (1) and (2) simultaneously:
—uy — 10d = 16 {multiplying both sides of (1) by —1}
up+ Td=-113
—-3d =43 {adding the equations}
cod=-3
So, in (1): up +10(=2) = —16 Check:
- 15=-16 un = —3(11)
+1

Now
=t
wu, =u;+ (n—1)d
=2y
_ 32
_
. un=—1—%(n—1) ,732(8)+116 )
Uy =—-3n+1 " 24 15
=—3+t3
=-Z=-111 v
14 Suppose the common difference is d.
the numbers are 5, 5+d, 5+2d, 5+ 3d, and 10
5+4d =10
L 4d=5
d=5%=13
So, the sequence is 5, 63, 73, 82, 10.
15 Suppose the common difference is d.
the numbers are 1, -14+d, —1+2d, —1+3d, —1+4+4d, —1+5d, —1+6d, and 32.
—1+7d=32
7d =33
L d=% =42
So, the sequence is —1, 32, 82, 131, 17¢, 224, 272, 32.
162 Chapter 5 (Sequences and series) Exercise 5B.1

16 a Suppose the common difference is d.


the numbers are 50, 50 +d, 50+ 2d, 50+ 3d, and 44.
50 + 4d= 44
. 4d=—6
d=-8%=-11
So, the sequence is 50, 483, 47, 451, 44.
b up, =up
+ (n—1)d
LUy =50—13(n—1)
Uy = 51% . lén
Let u, =0
512 —1in=0
1y .Y - 1

n=34%
We try the two values on either side of n = 341 which are n =34 and n = 35.

ugg =513 —13(34) and g5 =513 — 13(35)


1
=3 2 =-1
So, the first negative term of the sequence is ugs = —1.

17 a Month 1. 5 cars Month 4: 31+ 13 = 44 cars


Month 2: 5+ 13 = 18 cars Month 5: 44 + 13 = 57 cars
Month 3: 18 + 13 = 31 cars Month 6: 57+ 13 =70 cars
b Every month after the first, the factory assembles 13 cars, so the difference between successive
months is always 13. Thus we have an arithmetic sequence with u; =5 and d = 13.
¢ u,=wu+(n—1)d d Let wu,=250=13n—-38
=5+13(n—1) . 258 =13n
=13n-38 Son=2~198
. up = 13(12) =8 {12 months = 1 year} So, the 250th car is made in the
— 148 20th month.

So, 148 cars are made in the first year.

a Week 1: 3000 —183 = 2817 L Week 3: 2634 — 183 = 2451 L


Week 2: 2817 — 183 = 2634 L Week 4: 2451 — 183 = 2268 L
b Every week Yafiah uses 183 L of water, so the difference between successive weeks is always
—183. Thus we have an arithmetic sequence with u; = 2817 and d = —183.
¢ u,=u+(n—1)d
=2817—183(n—1)
= 3000 — 183n
Let u, =0
. 3000 —183n =10
183n = 3000
.o n=~16.4
So, Yafiah’s tank will run out of water in the 17th week.
Chapter 5 (Sequences and series) Exercise 5B.2 163

total mass
1 a Average mass of oranges = b w, =140.75n
number of oranges
1126 kg
T8
=0.14075 kg
=140.75 g
2 Total mass of 12 eggs = mass of 12 eggs and carton — mass of carton
=743 - 32
=Tllg
total mass
a Average mass of eggs =
number of eggs
Tllg
D)
=59.25g
b wu, =32+59.25n
¢ The carton can only hold a maximum of 12 eggs.
the model is valid for 0 < n < 12.

3 a Dayl: 580 — (8 x2)=564 Day 3: 548 — (8 x 2) = 532


Day 2: 564 — (8 x 2) = 548 Day 4: 532 — (8 x 2) =516
u; =564 and d= —16
U, =uy + (n—1)d
=564 —16(n —1)
©u, = 580 — 16n
total mass
b Average mass of hay bales =
number of bales
9850 kg
580
__ 985
=% ke
Since the farmer uses 8 X 2 = 16 bales of hay each day,
985
the average mass of hay used each day = 5 x 16

= o5 kg

7880
So, the mass of hay remaining after n days can be approximated by wu,, = 9850 — 29

up =34, wug =80


Average number of friends made each week
total number of friends made from week 1 to week 9
number of weeks
80 — 34
8
= 5.75 online friends
164 Chapter 5 (Sequences and series) Exercise 5C

b u =34, d=575
up =ug
+ (n—1)d
=34+ 5.75(n—1)
. Up = H.75n + 28.25
¢ No, it is not a problem that the common difference is not an integer. The model is only intended
to estimate the number of online friends. We can simply round to the nearest whole number.
d uz =5.75(20) + 28.25
= 143.25
Valéria will have approximately 143 online friends after 20 weeks.

5 a wus=06950 .. wu;+49d=06950 ... (1) {using w, = ui(n —1)d}


uipo = 11950 . ug +99d =11950 ... (2)
We now solve (1) and (2) simultaneously:
—uy —49d = —6950 {multiplying both sides of (1) by —1}
uy +99d = 11950
50d = 5000 {adding the equations}
d =100
So, in (1): uy +49(100) = 6950 Check:
. up + 4900 = 6950 uso = 100(50) + 1950
uy = 2050 =6950 v
Now u, =u; + (n—1)d w100 = 100(100) + 1950
. u, = 2050 + 100(n — 1) =11950 v
U, = 2050 + 100n — 100
U, = 1950 4+ 100n
b i The common difference is 100. This means that the catering cost is €100 per guest.
ii The constant term is 1950 (when n = 0). This means that the venue hire is €1950 (with
0 guests).

¢ ugs = 100(85) + 1950


= 10450
The cost of venue hire and catering for a reception with 85 guests would be €10 450.

1 a 5,15,45,135, ... b 72,36, 18,9, ...

L _4 H_g4 1B _g4 L
5 15 45 72 2 36 2 18
up =5, 7=3 up =72, r:%

¢ 2,-8,32, ~128, ... d6,-2,2 -2, ..


S
2
2o
5
Eo
32
2_1
6 3
3_1
-2 3
Z3_1
2 A 3
Chapter 5 ( Sequences and series) Exercise 5C 165

2,6,0b,c, b 10,5, b, ¢, ....


5 1 A
S_3
2
. r=3 —=-
10 2
. r=3
2
b=6x3=18 _ 1_ 9l
b=5x35=23
and ¢=18x3=54
and c=23x1=12
12, =6, b, c, ....

6__1 —_1
12 2 r="3
b=—6x—-3=3 and c=3x 1_ _q1
3 =13
3,6, 12, 24, ...
i S-o Lor=2, uy=3 i Up = uy "t
3
Uy = 3 x 2n71

i wg =3 x28
=768
2, 10, 50, ...
E:5 coor=5 u =2
2
Coupy =2 x5
il ug =2 x5°
= 781250
512, 256, 128, ...
256_l . _1 -
m—Q . 7‘—2, U1—512 Up = ugr" Tt
_ 1\n—1
u, =512 x (5)

U, =512 x 217"
il wg=512x278

1,3,9,27,
i 323
1
- r=3 w=1 Up = ugr™ "t
Uy, =1x 3771
Up = 3n71

il wg =38
= 6561
12, 18, 27, ...
18 3
EZE 7‘:%, U1:12 Uy = ugr"

e 8
il ug =12 x ()
_ 22x3x38
T
_ 39
_ 19683
26 64
166 Chapter 5 (Sequences and series) Exercise 5C

f L16° 11
8° 42
1 22

8 u =151 ii
i —=-2 r=-2, Up = ugr™ -1
16 3 _ 1 n—1
SooUp =1 X (-2)
Pil =l
ug = 75 x (-2) 8

=16

4 a 5,10, 20, 40, ....

L_,
1
5
2_,
10
©_,
20
Consecutive terms have a common ratio of 2.
the sequence is geometric with u; =5 and r = 2.

b Up = ugr"!
Up =5 x 271
uys =5 x 24
= 81920

5 a 12,-6,3 -4, ..
—6 471 3 471 a2y W 471
12 2 -6 2 B =2
Consecutive terms have a common ratio of —%.
the sequence is geometric with uy =12 and r = —3.
b Uy = ur™ Tt

u, =12 x (—-3)" —1

12
U3 = 12 x (7%)

_ 22 %3
T ol2

3
0

3
w13 = —— =~ 0.00293
1024

6 8, —6,4.5, —3.375, ....


-6 _ 3 45 _ 3 —3.375
_ 3
8 4 —6 4 45 4
Consecutive terms have a common ratio of 7%.
the sequence is geometric with u; =8 and r = —3.
Up = ugr"!

u =8 (2) n—1

wo =8 (1) 9

u19 ~ —0.601
Chapter 5 (Sequences and series) Exercise 5C 167

7 8,42, 4,2V2, ...

8 2 V2 o2 V2 4 2 V2
Consecutive terms have a common ratio of —=
\/5

the sequence is geometric with u; =8 and r = %


Uy = ur"™

Up = ()
L
n—1

" (’%) 1
22
Up =
i1y
Uy = 22 2

8 a Upt1 = 3 X Up,
Un+1 _ 3
Un

So, consecutive terms have a common ratio of 3.


the sequence is geometric with 7 = 3.

4 9
um:fixi’)

¢ up =5 and ux = % are not integers.

Also, u3 =3 X uy =3 X % = % is not an integer.


U4:3><U3:3><%:4
and us =3 Xxug=3x4=12
So, all terms after ug are integers, since uy is an integer and any integer multiplied by 3 is an
integer.
3 terms are not integers.

9 a k 3k 54
3k 54
Since the terms are geometric, = b {equating the common ratio r}

9k? = 54k
9k = 54 {since k # 0}
k=6
Check: If k=6, the terms are: 6, 18, 54. v {r =3}
168 Chapter 5 (Sequences and series) Exercise 5C

b 1000, 4k, k
Since the terms are geometric, e {equating the common ratio r}
1000 4k
16k* = 1000k
16k = 1000 {since k # 0}
k=122
Check: If k=122, the terms are: 1000, 250, 222, v {r =1}
7, k, 28
Since the terms are geometric, ; = 2—: {equating the common ratio r}

k* =196
k= +£v196
k==+14
Check: If k=14, thetermsare: 7,14,28. v {r=2}
If k= —14, thetermsare: 7, —14,28. v {r=-2}

18, k, 2 )
Since the terms are geometric, — = % {equating the common ratio r}

k=4
= ::\/Z

k=42
Check: 1f k=2, thetermsare: 18,2, 2. v {r=4}
If k=-2, thetermsare: 18, -2, 2. v {r=-1}

k12, £
) k
Since the terms are geometric, — = % {equating the common ratio '}

44="59
2

k?* = 1296
= +v1296
= £36
Check: 1f k=36, the terms are: 36, 12, 4. v {r=1%}
If k=—36, thetermsare: —36,12, —4. v {r=-1}
25
k, 20, 2k 25,
Since the terms are geometric, — = ;—O {equating the common ratio r}

400 = 221
k* =64
= +v64
k=48
Check: 1f k=8, the terms are: 8, 20, 50. v {r=3}
If k= -8, thetermsare: —8, 20, —50. v {r= —g}
Chapter 5 (Sequences and series) Exercise 5C 169

g k, 3k 20—k
Since the terms are geometric, % = 203; k {equating the common ratio r}

9k = k(20 — k)
9k =20—k {since k #0}
10k = 20
k=2
Check: If k=2, the terms are: 2,6,18. v {r=3}

h k k+8 9%
) . k+8 9k . )
Since the terms are geometric, . = s {equating the common ratio r}

(k+8)* = 9k?
k?* 4+ 16k + 64 = 9k*
o 8k?— 16k —64=0
o 8(k*—2k—8)=0
. 8(k+2)(k—4)=0
. k=-2 or 4
Check: If k= —2, thetermsare: —2,6,-18. v {r=-3}
If k=4, the terms are: 4, 12, 36. v {r=3}

10 k—1,6, 3k
a Since the terms are geometric, % = % {equating the common ratio r}

36 =3k(k—1)
o3k —3k—36=0
3k -k —12)=0
o 3(k+3)(k—4)=0
. k=-3 or 4
Check: 1f k= —3, the terms are: —4,6, 9. v {r=-3}
If k=4, the terms are: 3,6, 12. v {r=2}
If k=-3, thenexttermis —9 x (—%) = 2—27
If k=4, thenexttermis 12 x 2 =24.

1 up=ur* =24 .. (1) b us=ur>2=8 .. (1)


and wy = uyr® =192 ... ?2) and wug =ur® =—1 .. 2)

Now HL-Z2 (@) Now M= (@)


6 5

ot =38 sordt==1
=18 o= 31
0T 8
cLr=2 )
Using (1), uy(2)% =24 =72
u =3 Using (1), wi(-3)" =8
Thus w, =3 x 2"} coouy =32
170 Chapter 5 (Sequences and series) Exercise 5C

c wy=uwr®=24 .. () d us=wur’ =5 .. (1)


and ups =wr'? =384 .. (2 and ur=wur®=% ..(2

wmrt
Now 2w = 88 o
@M}
14
g 5
Now U5 =1 {@+0)}
=16 Pl
1
r=x v 16 11
1 rT==T4/7
r=4 (24) 8 B
1 r==4 (272)Z
r =422 )
=42 Fr="E 202
T = ::L
Using (1), u;(v2)% =24 V2
=3 : (1),
Using w L)
(W) =
Thus w, =3 x (vV2)"~!
or u, =3x (—v2)"! net
Thus 1w, = 10 x (%) =10 x (V2)I="
n—1
or u, =10 x (—%) =10 x (—V/2)I"

a 2,6,18,54, ..
The sequence is geometric with u; =2 and r = 3.
Uy =2x 37t
We need to find n such that u,, > 10000.
Using a graphics calculator with Y; =2 x 3" (X — 1), we view a table of values:

Y1=2§3"(J%;1)

7 1468
8 4374
o EEEE
10 39366

({3 DELETE] EDIT

The first term to exceed 10000 is ug = 13122.

b 4,43, 12, 12V/3, ...


The sequence is geometric with u; =4 and 7 = /3.
Uy =4 x (\/g)n—l

We need to find n such that w,, > 4800.


Using a graphics calculator with Y, =4 x (v/3)"(X — 1), we view a table of values:
B HatiRadforn]
Y1=4§(‘f3 i ey
12 1683.6
13 2916
14 FEE
16 8748
5050.660155
(FORHULA VTR TSI (EDIT GPH-CON GPH-P

The first term to exceed 4800 is w14 = 2916v/3 &~ 5050.


Chapter 5 (Sequences and series) Exercise 5D 171

¢ 12,6, 3, 1.5, ..
The sequence is geometric with u; =12 and r = %
Up = 12 % (%)"71
U, =12 x 287"
We need to find n such that w,, < 0.0001.
Using a graphics calculator with Yy =12 x 2”(1 — X), we view a table of values:
FelFadlom]_/aRes)
YI=12x2"(1-x)
16 3.6e-4
‘ 17 1.8e-4
18 CHTEE
19 4.6e-5
9.155273438E-05
(FORMULA (VLT3 MO (EDI T ) GPHECON GPEEPIT)

The first term which is less than 0.0001 is w5 = fi ~ 0.0000916.

1 There is a fixed percentage increase each week, so the population forms a geometric sequence.
uy =500 and r =1.12
the population after n weeks is u,, = 500 x (1.12)".
a 0w =500x (1.12)1° il ugo = 500 x (1.12)%
~ 1552.92 ~ 4823.15
There will be approximately 1550 ants There will be approximately 4820 ants
after 10 weeks. after 20 weeks.
b We need to find when 500 x (1.12)™ = 2000.
[EXE]:Show coordinates

It will take approximately 12.2 weeks.

2 There is a fixed percentage decrease each year, so the population forms a geometric sequence.
ug = 555 and r = 0.955
the population after n years is u,, = 555 x (0.955)™.
a 2020 is 15 years after 2005.
w15 = 555 x (0.955)1°
~ 278.19
The population is approximately 278 animals in the year 2020.
172 Chapter 5 (Sequences and series) Exercise SD

b For the population to have declined to 50, we need to find when 555 x (0.955)" = 50.
Bl [EXE]:Show coordinates
Y1=5556x. BB5{(x.
¥2=50
200f

100}

X=62.274D6141 Y=60

So, in the 53rd year the population is 50. This is the year 2057.

3 There is a fixed percentage increase each year, so the herd size forms a geometric sequence.
ug =32 and r=1.18
the herd size after n years is u,, = 32 x (1.18)™.
a i us=32x(1.18)° i up =32 x (1.18)1°
~ 73.21 ~ 167.48
There will be approximately 73 deer after There will be approximately 167 deer
5 years. after 10 years.
b For the herd size to reach 5000, we need to find when 32 x (1.18)™ = 5000.
[El [EXE]:Show coordinates
Y1=32x1. 8"~ (x)
¥2=5000

So, it will take approximately 30.5 years.

L There is a fixed percentage increase each year, so the population forms a geometric sequence.
ug =178 and r =1.32
the population after n years is wu,, = 178 x (1.32)™.
a i u=178x(1.32)1 il ups = 178 x (1.32)%°
~ 2858.6 ~ 183979.0
There will be approximately There will be approximately
2860 marsupials after 10 years. 184 000 marsupials after 25 years.
b For the population to reach 10000, we need to find when 178 x (1.32)™ = 10000.
[l [EXE]:Show coordinates
Y1=178x¥.32"%(x)
¥2=10040

10 3|
.51043339 ¥=1000

So, it will take approximately 14.5 years.


Chapter 5 (Sequences and series) Exercise SE.1 173

5 There is a fixed percentage decrease each year, so the amount of radioactive material forms a
geometric sequence.
ug = 1.52 and r =0.82
a Ug = Ug X ’I"4

1.52 = ug x (0.82)*
= 1.52
(0.82)4

~ 3.36
The initial quantity of radioactive material was approximately 3.36 g.
b The amount of radioactive material after n years is u,, ~ 3.36 x (0.82)".
For the amount of radioactive material to reduce to 0.2 g, we need to find when
3.36 x (0.82)" =0.2.
B [EXE]:Show coordinates
=3.36%.82%(x)
2=,2

=14.21700853 ¥=0.2

So, it will take approximately 14.2 years, or 10.2 more years for the amount of radioactive
material to reduce to 0.2 g.

6 There is a fixed percentage increase each year, so Maria’s salary forms a geometric sequence.
u1p = 49852 and r = 1.023
a U110 = Ug X ’I"10

49852 = up x (1.023)*°
= 49852
(1.023)10
~ 3971241
So, Maria’s salary was €39 712.41 p.a. when she joined the company.
b Maria’s salary after n years is u,, ~ 39712.41 x (1.023)".
upy A~ 39712.41 x (1.023)™
~ 54 599.05
So, if Maria stays with the company for another 4 years, her salary will be €54 599.05 p.a.

1 The interest is calculated annually, so n =5 time periods.


us = up X (141)°
= 7000 x (1.06)> {6% = 0.06}
~ 9367.58
The investment will amount to £9367.58.
174 Chapter 5 (Sequences and series) Exercise SE.1

2 The interest is calculated annually, so n =3 time periods.


uz = up x (141)3
= 3000 x (1.048)3 {4.8% = 0.048}
~ 3453.07
The investment will amount to £3453.07.

3 a The interest is calculated annually, so n =4 time periods.


ug = ug x (1+14)*
= 2000 x (1.028)* {2.8% = 0.028}
~ 2233.58
The investment will amount to €2233.58.
b The interest earned = €2233.58 — €2000
= €233.58

L The interest is calculated annually, so n =4 time periods.


uy = up x (1+4)*
=20000 x (1.042)* {4.2% = 0.042}
~ 23577.67
The investment will amount to €23 577.67.
The interest earned = €23 577.67 — €20 000
= €3577.67

5 The interest is calculated annually, so n =3 time periods.


uz = up x (1+1)%
= 8000 x (1.029)3 {2.9% = 0.029}
~ 8716.38
The investment will amount to $8716.38.
The interest earned = $8716.38 — $8000
= $716.38

6 a The interest is calculated quarterly, so there are n =1 x4 =4 time periods.

Each time period the investment increases by i = do% < Mo


-, the amount after 1 year is uy = ug x (1 +i)*
=20000 x (1.012)* {1.2% = 0.012}
~ 20977.42
The investment will amount to $20977.42.

b The interest is calculated quarterly, so there are n =3 x 4 = 12 time periods.


", the amount after 3 years is wyp = ug x (1 +14)2
= 20000 x (1.012)*?
~ 23077.89
The investment will amount to $23077.89.
Chapter 5 (Sequences and series) Exercise SE.1 175

7 a The interest is calculated annually, so n =4 time periods.


ug = ug x (1+14)*
=30000 x (1.056)* {5.6% = 0.056}
~ 37305.85
The investment will amount to €37 305.85.
b The interest earned = €37 305.85 — €30 000
= €7305.85

8 The interest is calculated quarterly, so there are n =3 x 4 = 12 time periods.

Each time period the investment increases by i = £ et 1.1%.

", the amount after 3 years is u1a = ug x (1 + )2


=80000 x (1.011)** {1.1% = 0.011}
~ 91222.90
The investment will amount to $91 222.90.
The interest earned = $91 222.90 — $80 000
= $11222.90

9 Bank A:
The interest is calculated annually, so there are 10 time periods.
w10 = ug % (1 +14)%°
=92000 x (1.055)!° {53% = 0.055}
~ 157149.29
The investment will amount to $157 149.29 .
The interest earned = $157 149.29 — $92 000
= $65149.29
Bank B:
The interest is calculated quarterly, so there are 10 x 4 =40 time periods.
5.25%
Each time period the investment increases by i = = 1.3125%.

-, the amount after 10 years is w40 = ug x (1 4 7)%°


=92000 x (1.013125)* {1.3125% = 0.013125}
~ 154991.94
The investment will amount to $154991.94 .
The interest earned = $154 991.94 — $92 000
= $62991.94
Bank C:
The interest is calculated monthly, so there are 10 x 12 =120 time periods.

Each time period the investment increases by i = 51—? = 0.416%.

", the amount after 10 years is w20 = ug x (1 +14)12°


=92000 x (1.00416)'*° {0.416% = 0.004 16}
~ 151524.87
The investment will amount to $151 524.87.
176 Chapter 5 (Sequences and series) Exercise SE.2

The interest earned = $151 524.87 — $92 000


= $59524.87
So, Bank A offers Jai the greatest interest on his inheritance.
10 The initial investment ug is unknown.
There are n =4 time periods.
Now ug = ug x (14 1)*
20000 = ug x (1.075)* {7.5% = 0.075}

Uo = ST 20000
~ 1497601
Habib needs to invest £14977 now. {rounded up to the next pound}
11 The initial investment uq is unknown.
There are n = % =5 time periods.
Now us = ug x (1+1)°
15000 = ug x (1.055)° {5.5% = 0.055}

Ug = 15000 11477.02
(1.055)5
An initial investment of $11478 is required. {rounded up to the next dollar}
12 The initial investment wug is unknown.
There are n =3 x 4 =12 time periods.
4.2%
Each time period the investment increases by i = = 1.05%.
Now w32 = ug X (1 +i)12

25000 = up x (1.0105)'% {1.05% = 0.0105}


~ 25000
Ug ~~ 22054.85
"~ (1.0105)12
An investment of $22054.85 is required now.
13 The initial investment uq is unknown.
There are n =8 x 12 =96 time periods.
. ) i i i 3.6
Each time period the investment increases by i = 12% =0.3%.
Now ugs = ug x (1 +1)%
*. 4000000 = ug x (1.003)° {0.3% = 0.003}
Uo = Togams ~ 3000340
4000000

An initial investment of ¥3 000 340 is required.

1 a To index the amount of money for inflation, we increase it by 3% each year for 2 years.
indexed value = $8000 x (1.03)?
= $8487.20
b To index the amount of money for inflation, we increase it by 3% each year for 5 years.
indexed value = $14 000 x (1.03)°
= $16229.84
Chapter 5 (Sequences and series) Exercise SE.3 177

¢ To index the amount of money for inflation, we increase it by 3% each year for 7 years.
indexed value = $22 500 x (1.03)”
= $27672.16

2 a To index the amount of money for inflation, we increase it by 2% each year for 10 years.
indexed value = $1000 x (1.02)*°
= $1218.99
In 10 years’ time Hoang will require $1218.99 per week to maintain his current lifestyle.
b To index the amount of money for inflation, we increase it by 2% each year for 20 years.
indexed value = $1000 x (1.02)%
= $1485.95
In 20 years’ time Hoang will require $1485.95 per week to maintain his current lifestyle.
¢ To index the amount of money for inflation, we increase it by 2% each year for 30 years.
indexed value = $1000 x (1.02)*°
= $1811.36
In 30 years’ time Hoang will require $1811.36 per week to maintain his current lifestyle.

3 To index the value of the holiday package for inflation, we increase it by 2% each year for 4 years.
indexed value = $15000 x (1.02)*
= $16236.48

1 a Thereare n=3 x4 =12 time periods.


Each period, the investment increases by i = 32% =0.9%.

-, the amount after 3 years is ujo = ug x (1 +14)"2


= 5000 x (1.009)*?
= 5567.55
The investment will amount to $5567.55.

b real value x (1.02)* = $5567.55


$5567.55
.. real value =
(1.02)3
= $5246.43

2 a Thereare n =4 x 12 =48 time periods.


Each period, the investment increases by i =

*. the amount after 4 years is wugg = ug x (1 +14)*®


=20000 x (1.0035)*®
~ 23651.79
The investment will amount to €23651.79.
178 Chapter 5 (Sequences and series) Exercise SE.4

b real value x (1.034)* = €23651.79


€23651.79
. real value =
(1.034)4
= €20691.02

3 a Thereare n=6x 2 =12 time periods.


Each period, the investment increases by i = 37% =1.5%.

-, the amount after 6 years is uo = ug x (1 + )2


= 4000 x (1.015)"?
=~ 4782.47
The final value of the investment is $4782.47.

b The interest earned = $4782.47 — $4000


= $782.47
¢ real value x (1.032)° = $4782.47
real value = Siisadn
’ "~ (1.032)8
= $3958.90
d The investment has not been effective. The real value of the investment after 6 years is less
than what was originally invested.

1 uz=ugx (1—d)?
=2500 x (0.8)> {20% = 0.2}
= 1280
So, after 3 years the value of the lathe is €1280.

2 a us=upx(1-d)?° b The depreciation = €110 000 — €26 103.52


= 110000 x (0.75)° {25% = 0.25} = €83896.48
~ 26103.52
So, after 5 years the value of the tractor is
€26 103.52.

3 a ug=uox(1-d? b The depreciation = ¥87500 — ¥30013


= 87500 x (0.7)3 {30% = 0.3} = ¥57487
=30012.50
So, after 3 years the value of the laptop is
¥30013.
Chapter 5 (Sequences and series) Exercise SE.5 179

4 ’LL4=U0><(1—d)4

80000 = 250000 x (1 — d)*

U—@4=ififi
=0.32
1—d=+v0.32
d=1++v032
d =~ 1.75 or 0.248
d=~0.248 {as 0<d<1}
So, the printing press depreciated in value by 24.8% per year.

1 N=6, I%=37 PV =-60000, PMT =0, P/Y=1, C/Y=1


e Fornd)
Compound Interest
1% =3.7
PV =-60000
PM
3 EIS RGBT
c/Y=1
[ JCT% JCPV JCPMTICEV_
)i

FV ~ 74614.60
Enrique’s investment is worth 74 614.60 pesos after 6 years.

2 N=2x12=24, I%=5 PV =-6000, PMT =0, P/Y =12, C/Y =12


a
Compound Interest
n =24
1% =5

629.648013
v
n 1% J[ PV_J[PMT [ FV_]JEIN3N

FV ~ 6629.65
I will have $6629.65 in my account after 2 years.

3 N=18x2=36, I%=4, PV =-2000, PMT =0, P/Y=2 C/Y =2


a
Compound Interest
n =36
% =4

FV’ 4079.774687 -
n 1% JL PV _J[(PMTJ[FV

FV ~4079.77
My daughter will receive €4079.77 on her 18th birthday.
180 Chapter 5 (Sequences and series) Exercise SE.5

4 a N=3x4=12, I%=56, PV =-8000, PMT =0, P/Y =4, C/Y =4

Compound Interest

FV ~ 9452.47
Kenneth will have $9452.47 in his account after 3 years.

b N=8x4=232 I%=56 PV=-8000, PMT=0, P/Y=4, C/Y =4


Bornd)
Compound Interest
n =32
1% =5.6

FV ~12482.59
Kenneth will have $12482.59 in his account after 8 years.

5 a N=7x12=84, I%=42, PV =-5000, PMT =0, P/Y =12, C/Y =12

Compound Interest

PV PMTIFV

FV ~ 6705.48
There will be €6705.48 in the account after 7 years.
b Interest earned = €6705.48 — €5000
= €1705.48

6 N=4x4=16, I%=7 PV =-13000, PMT =0, P/Y =4, C/Y =4

Comgcl)gnd Interest
=7
=-13000

FV ~17159.08
Interest earned = £17 159.08 — £13 000
= £4159.08
Chapter 5 (Sequences and series) Exercise S5E.5 181

7 N=5x12=60, I%=4.5, PMT =0, FV =2500, P/Y =12, C/Y =12


a (orud) &0
Compound Interest

EI3N

PV ~ —1997.13
$1997.13 needs to be invested now.

8 N=4, I%=65 PMT=0 FV=10291731, P/Y =1, C/Y =1


a [orwd) =0
Compguna Interest

.5
PV 80000.00152
PM
FV =102917.31
P/Y¥=1 ¥
n_J[ 1% JC PV JCPMT

PV ~ —80000.00
You won $80 000 in the lottery.

9 N=5 I%=-25 PV=-458, PMT =0, P/Y=1 C/Y =1


B
Compguna Interest

=-25

v
EL3pd

FV ~ 108.69
The value of the stereo after 5 years is $108.69.

10 I% =45, PV =-40000, PMT =0, FV =45000, P/Y =4, C/Y =4


a

EL3e

They kept the money in the account for 11 quarters, or 2 years 9 months.
{rounded up to the next quarter}
182 Chapter 5 (Sequences and series) Exercise SE.5

11 I%=5.25, PV =-80000, PMT =0, FV =160000, P/Y =12, C/Y =12


e FornD &
Compound Interest
NP
.25
PV =-80000
PMT=0
FV =160000
P/Y=12 v
n_ ][ 1% PV R

~ 158.8
The money is doubled after 159 months, or 13 years 3 months.
{rounded up to the next month}

12 I%=-12, PV =-6800, PMT =0, FV =1000, P/Y =1, C/Y =1


[El d|
Compound Interest
n =14.9954788
1% =-12
PV =-6800
PMT=0
FV =1000
P/Y¥=1 ¥
n ) 1% JLPV AR

N =~ 15.0
It will take 15 years for the value to reduce to $1000.

13 N=3, PV=-10000,b PMT =0, FV =15000, P/Y =1, C/Y =1


e FornD &
Compguna Interest
n
¥ 4.47142426
PV =-10000
PMT=0
FV =15000
P/Y¥=1 v
n ][ 1% PV JAHORTZA]

1% ~ 14.5
An annual increase of 14.5% is required.

14 N =35x12=42, PV =-5000, PMT =0, FV =6165, P/Y =12, C/Y =12


B FornD &
Compound Interest

v
LN

1% =~ 6.00
The account paid 6.00% p.a. interest.
Chapter 5 (Sequences and series) Exercise SF 183

15 N=3x4=12, PV =-9000, PMT =0, FV =10493, P/Y =4, C/Y =4


a (o] &
Compound Interest
n =12
1% =5.148984738
PV =-9000
PMT=0
FV =10493
P/Y=4
n 1% J[ PV

I% ~5.15
The interest rate paid was 5.15% p.a.

16 N =4, PV =-68500, PMT =0, FV =26380, P/Y =1, C/Y =1


a8 [orn) &

v
[V [AHORTZ!

1% ~ —21.2
The annual rate of depreciation was 21.2%.

1 4,6,8,9, 10, 12, 14, 15, 16, ....


a S3=4+6+38 b S5=44+6+8+9+10
=18 =37
¢ So=44+6+8+9+ 10+
14+ 15+ 12+
16+ 18 +20+ 21
=153

2 55=S4+U5

v 20=13+us
U5=7

1
3 u,=-
n

a The first 5 terms are 1, . 3,1,


3 1, L.
b Sy=1+3+%+1%
12+6+4+3
12
25
12

¢ We need to find n such that Sn:1+%+§+.‘..+l>3‘


n

Using technology, we find that S1; =1+ % + % + .+ fi ~3.02.


So, S, >3 for n>11.
184 Chapter 5 (Sequences and series) Exercise 5F

& a 3,11,19,27, ...


i 11-3=8 19-11=8 27-19=38
The difference between successive terms is constant.
the sequence is arithmetic with u; =3 and d = 8.
u, =uy
+ (n—1)d
U =3+8(n—1)
. U, =8n—5
Now S, =3+11+19+27+
...+ (8n—5)

o8, =Y (8k—5)
k=1

i S5= 3(8k—5)
k=1

=3+11+19+27+35
=95

b 42,37, 32,27, ...


i 37—42 5 3237 5 27-32=-5
The difference between successive terms is constant.
the sequence is arithmetic with u; =42 and d = —5.
u, =uy
+ (n—1)d
U, =42-5(n—1)
. Uy =47 —5n
Now S, =42+37+32+27+ ...+ (47 — 5n)
n
S, =3 (47 — 5k)
k=1
5
iS5 = > (47 —5k)
k=1

=424
37+ 32+ 27+ 22
=160

¢ 12,6,3, 13, ...


6 1 3hrl 13 1 5
TPk Lol o 3 2 iS5 = 3,
k=1 (12x279)
T _= - —_— T - ii — 1-k

Consecutive terms have a common ratio of % =12+6+3+14+3


the sequence is geometric with u; = 12 — 931
and r= 1. =25
Up = upr™ !
L up =12 ()"
©oup =12x 2177
Now S, =12+46+3+11+. . +(12x2'"")
n n

Sp= > (12x 27k or 3 12(2)


k=1 k=1
Chapter 5 (Sequences and series) Exercise 5F 185

d 23,4463, ..
gl h3 43 3 6% 3 ..5523#1
'373 373 a7 i s= % (2x(3)7)
Consecutive terms have a common ratio of 2. =2+3+41+63 + 102
the sequence is geometric with u; = 2 = 26%
and r=3.
Up = ugr"!
wy = 2 x (%)7,,71

Now S, =2+3+4%+62+..+ (2 x (g)"*l)


Si=%k=1 (2x @) o k=1X2(3)
o k=1 B 3 k-1

11 1
e 1,5, 9,5 -
iy Ly Ly 5
i 2==2 4_Z 3_Z ji = 1-k
"173 012 173
1
i S5 ,;12
=
Consecutive terms have a common ratio of % =1+ % + % + % + 11—6
the sequence is geometric with u; =1 = 11_5
and r=3.
Up = ugr™!

un =1 (3)"
i =105™
Now S,=1+3+3+5+...+2'7"
S f: 1—-k i 1
= 2°7F or
= =y 2!
f 1,827, 64, ..
i S, =12 +2% 4334+ 4% 1 ... +n® {all terms are cubes}
n

Sp=> kK
k=1

il S5 =1+8+27+64+125
=225
3 6
a Y 4k=4+8+12 b > (k+1)=24+3+4+5+6+7
k=l —9q il =27
4 5
€ Y (Bk—5)=—-2+1+4+47 d > (11-2k)=9+7+5+3+1
i =10 k=t =25
7
e > k(k+1)=2+6+12+20+
30+ 42+ 56
k=1
=168
5
f Y10 x 2871 =10+ 20 + 40 4+ 80 + 160
o =310
186 Chapter 5 (Sequences and series) Exercise S5F

6 u,=3n-1
20
oup+us +ug+ ... +up = Z(?)k*l)
k=1

24+5+8+11+14+17+20+
23 +26 + 29 + 32 + 35
F38 +41 + 44 + 47 + 50 + 53 + 56 + 59
=610

7 a Y c=ctetet..tc=cn b > car =car +cas+ ...+ cay


. —/—/ —

. n times k=1 = C((ll +ag + +ap)

=cy a
k=1
[4 Z(ak | bk) ((Ll } bl) t ((LQ } b2) .=t (an } bn)

ik = (a1 4 a2+ oo+ an) + (b1 + by + . + by)


=Y ar+ Y, b
k=1 k=1

8 a k=14 2 + 3 4+..+(r-1+n
k=1
or n+n-1)+Mn—-2)+ 2 il

b 2 k=m+1)+n+1)+n+1)4 F(n+1)+(n+1)
k=1
=n(n+1)
ik _ n(n+1)
k=1 2
S — n(n+1)
" 2

¢ Y (ak+b)=>ak+ > b
k=1 k=1 k=1

=ay k+nb
k=1
= ‘m(n; b +nb {using b}

But Y (ak+b) =8n*+11n


k=1
“"("2+ D | nb=8n?+11n
2
w-o—nb:&zz—fi—lln

%n2+%n+nb=8n2 +11n

gnz + <% +b>n =8n? +11n

Comparing coefficients, we get g =8 and g +b=11

a=16 Lip=11
=3
Chapter 5 (Sequences and series) Exercise 5F 187

9 The sequence of positive odd integers is 1, 3, 5, 7, 9, ....


This is an arithmetic sequence with u; =1 and d = 2.
U, =u1 + (n—1)d
u, =14+2(n—1)
Uy, =2n —1
So, the sum of the first n positive odd integers can be represented by
n

Sp=>(2k—-1) 1 3 5 Fon+(2n—3)+(2n—1)
L or (2n—1)+(2n—3)+(2n—5)+.... 3 1
Zi(Zk 1) 2n 2n 2n 4+ ... 2n 2n

o =nx2n
= 2n?

i(?k— 1) =n?
k=1
n
So, S, =3 (2k—1) =n?
k=1

10 o k=n(n+ 1) and e k2=n(n+ 1)(2n+1)

Now S (k+1)(k+2)= 3 (k*+3k+2)


k=1 k=1

= SR+ Y 3k+ 32
k=1 k=1 k=1

N n(n+1)(2n+1) ——3§:k+2n
6 h=1
_ n(n+ )6(2n+1) +3x n(n+1) Lon

: n(n+1)(2n+1) 9n(n+1) +12_n


6 6 6
_nn+1)2n+1)4+9n(n+1)
R e +12n

_ n@2n? +3n4+1) +n(9n+9) +12n


=

_e n@n?+3n+1+9n+9+12)

_N
n(2n? 4+ 12n +22)

_ 2n(n? + 6n
+ 11)
6
n(n? + 6n 4 11)
3
188 Chapter 5 (Sequences and series) Exercise 5G

10
When n=10, 5 (k+1)(k+2) =6+ 12 + 20 + 30 + 42 + 56 + 72 + 90 + 110 + 132
k=1
=570
n 2
but > (k+1)(k+2)= w {from above}
k=1
20 10(102 4 6(10) + 11)
. when n =10, (k+1)(k+2) = e )
k=1
_10(171)
3
=570 v

1T a 2+6+10+14+18+22+26+30=128

b S,.= g (ur +u,) where uy =2, u, =30, and n=8

< Sn=g(2u1+(n—1)d) where u; =2, d=4, and n=28

587§(2x2+7><4)
=128

2 a The series is arithmetic with b The series is arithmetic with


uy =7, d=2, and n=10. u; =3, d=4, and n = 20.

Now S, = g(ml + (n—1)d) Now S, = g (2u1 + (n—1)d)


S1o=L2x7+9x%2) s Sp=22x3+19x4)
=5(14+ 18) =10(6 + 76)
=160 =820

¢ The series is arithmetic with d The series is arithmetic with


ulzé,d:%, and n = 50. uy; =100,
d = -7, and n = 40.

Now S, = % (2uy + (n—1)d) Now S, = g (2u1 + (n —1)d)

S Ss0=%(2xL+49x3) oo Sao =% (2 x 100+ 39 x (7))


=25(1 +1221) = 20(200 — 273)
= 3087% = —1460
Chapter 5 (Sequences and series) Exercise 5G 189

e The series is arithmetic with f The series is arithmetic with


uy = —31, d=3, and n = 15. uy; = 50, d:7§, and n = 80.

Now S, = g(Qul +(n—1)d) Now S,= g (2u1 + (n—1)d)


S5 =13(2x (—=31) + 14 x 3) Sso= 82(2 x 50+ 79 x (%))
L(-62+42) =40(100 — 27)
=150 = —740

a b+8+11+14+...4+101 37+334+29+25+....49
The series is arithmetic with The series is arithmetic with
uy =5, d=3, and u, = 101. uy =37, d=—4, and u, =9.
First we need to find n. First we need to find n.
Now u, =101 Now wu, =9
uy + (n—1)d =101 ur+(n—1)d=9
5+3(n—1)=101 L 37T—4n—-1)=9
3(n—1)=96 —4(n—1)=-28
. n—1=232 . n—1=7
n =33 n=2~8

Using S, = g(m +uy), Using S, = g(m + un),


S33 = 22(54101) Ss=§(37+9)
3% x 106 =4 x 46
=1749 =184

¢ 50+49% +49+481 + ...+ (—20) 84105


+ 13+ 154 + ...+ 83
The series is arithmetic with The series is arithmetic with
uy =50, d=—3, and u, = —20, u =8, d=2 3, and wu, = 83.
First we need to find n. First we need to find n.
Now u,, = —20 Now wu, =83
uy + (n— 1)d= —20 ©upt(n—1)d=83
8+3(n—1)=83
—in—-1)=-70 2n-1)=75
n—1=140 . n—1=30
n =141 n =231

Using S, = %(ul + up), Using S, = %(ul +uy),

Sia1 = (50 + (—20)) S31 = 31(8+83)


=11 %30 3L x 91
=2115 = 14103
190 Chapter 5 (Sequences and series) Exercise 5G

& 9,15,21, ..., 69, 75


a 15-9=6 21-15=6 75—-69=06 b The sequence is arithmetic with
the common difference d = 6. up =9, d=6, and u, =T75.
n Now w, =75
¢ Using S, = 5(“1 + un), ur+(n=1)d=175
Slzi%(9+75) 9**6(7l71):75
6(n—1) =66
=6x84
.n—1=11
=504
a n=12
There are 12 terms in the sequence.

10
5 a (2k+5)=7T+9+11+...+25 0 GeiRdion] GOkl
k=1 10
> (2K+5)
This series is arithmetic with w; =7, d =2, and n = 10. K=1

Using S, = %(ul +un) 5 2

Sio = 12(7 + 25) [logab]


[FMin[F2(Max|
=5x32 10
=160 ];1(2/{ +5)=160 v

b fj(k 50) = (—49)


+ (—48) + (—47) + ...
+ (=35)
k=1

This series is arithmetic with u; = —49, d =1,


and n = 15.
. n

Usipghn ss (Uil ), R
Si5 = 22(—49 + (—35)) -
=1 x(-84) > (k—50)=-630 Vv
k=1
=—630
20 k+3
< Z<%>=2+%+3+.‘..+§ 0 HtRedlom] @Rl
k=1
M i e N R )3, >k=1 (& K+3
20

This series is arithmetic with u; =2, d=


and n = 20. O
Using S, = %(ul +up),

Sn=2(2+3)
[FMin[FMax|
2( [logab]
20

—10x X2 Z(—k+3):135
=\ 2
v
=135
6 uy=5 n=7, u,=>53 7 uy =6, n=11, u, =—-27

Sn = g(ul +un) Sn — g(ul + Un)

S7=2(5+53) . S =26+ (-27))


=1 x58 =4 x(-21)
Chapter 5 (Sequences and series) Exercise 5G 191

8 The total number of bricks can be expressed as an arithmetic


series: 1+2+3+4+....4+n
We know that the total number of bricks is 171, so S, = 171.
Also, u; =1, d =1 and we need to find n, the number of .
terms (layers) of the series.

Sn=§(2u1+(n—1)d)=171
§(2x1+(n—1)x1)=171
on(24n—1) =342
n(n+1) =342
o 4n—342=0
(n+19)(n—18)
=0
. n=-19 or 18
But n >0, so n=18
So, the bricklayer built 18 layers.

9 a The number of laps Vicki swims each day can be expressed as an arithmetic sequence
20, 22, 24, 26, ...
So, uy =20 and d=2.

U, =up + (n—1)d
up =204+2(n—1)
U, =2n + 18

il ugzo =2(30)+ 18
=38 =78
Vicki swims 38 laps on the tenth day. Vicki swims 78 laps on the final day.

b Sp = g(ul + up)

S30 = 22(20 + 78)


=15x%x98
= 1470
Vicki swims 1470 laps in total.

10 a The amount of money the woman b S, = %(ul +up)


deposits each birthday can be expressed
as an arithmetic sequence S15 = £2(100 + 450)
100, 125, 150, .... =10 % 550
So, u; =100 and d = 25. = 4125
U, =up
+ (n—1)d The woman will have deposited $4125
Uy, = 100+ 25(n — 1) over the 15 years.
L Up =25+ 75
uys = 25(15) + 75
=450
The woman will deposit $450 into her
son’s account on his 15th birthday.
192 Chapter 5 (Sequences and series) Exercise 5G

11 The total number of seats in n rows can be expressed as an arithmetic series:


22+23+24+ ... 4u,
Row 1 has 22 seats, so u; = 22. Row 2 has 23 seats, so d = 1.

S, = g (2u1 + (n —1)d)
:g(2><22+1(n71))
= 3(44 +n—1)
Sy, = g(n +43) which is the total number of seats in n rows.

a Number of seats in row 44 of one section


= total number of seats in every row — number of seats in the first 43 rows
= Sas — Sa3
= %(44 +43) — 423 (43 +43)
= 1914 — 1849
= 65 seats

b Number of seats in each section = Sy = 1914 seats {from a}


¢ Number of seats in the whole stadium (25 sections) = Sy4 x 25
=1914 x 25
= 47850 seats

12 a The sum of the first 50 multiples of 11 can be expressed as an arithmetic series:


114+22+33+....+us9 where uy =11, d =11, n =50.

S, = g (2u1 + (n — 1)d)
Ss0 =22 (2 x 11449
x 11)
= 25(22 + 539)
= 14025
b The sum of the multiples of 7 between 0 and 1000 can be expressed as an arithmetic series:
T+14+21+28+....4u, where uy =7, d="1.
To find u,,, we need to find the largest multiple of 7 less than 1000.

Now &700 ~142.9, 50 wu, = 142 X 7 = 994


Also, n =142 {since u, =994 is the 142nd multiple of 7}

g n =1 2 (uq + up)

S1a2 = H2(7+994)
= 71071
Chapter 5 (Sequences and series) Exercise 5G 193

¢ The integers from 1 to 100 which are not divisible by 3 can be expressed as:
1,2,4,5,7,8, ..., 100 where u; =1, u, = 100.
Alternatively, these integers can be expressed as two separate arithmetic series A and B:
Sa=1+4+7+...+97+100 where uy =1, d=3, u, =100
and Sp=2+5+8+...+95+98 where u; =2, d=3, u, =98
Now for S4, u, =u; + (n—1)d and for Sp, wu, =u1 +(n—1)d
100=1+3(n—1) So98=2+3(n—1)
. 99=3(n-1) S 96=3(n—1)
L 33=n-1 co32=n-1
. n=34 S n=33

Using S, = Z(u1 +un), Sa=3%(14100) = 1717 and Sp = (24 98) = 1650


The total sum = S4 + Sp
= 1717 4 1650
= 3367

13 u6:21, 517:0

We need to find u; and us.

S, = g (2uy + (n —1)d)
. S17 =3 (2u1 +16d) =0
.our + 8d =0

Loup=-8d ... (1)


Also, u, =u; +(n—1)d
Lo U = Ul + 5d

oo 21=—8d+5d {using (1)}


co21=-3d
Lod=-T
So, w3 =—-8(—7)=56 and wuy;=56—7=49
The first two terms are 56 and 49.

14 The sequence is arithmetic with uy =4 and d = 6.

Now S, = 200, so g (2u1 + (n — 1)d) = 200


%(2><4——6(n—1))=200
g(s--fi(n—l))=2oo
. 4n+3n(n
—1) =200
o324 n—200=0
(Bn+25)(n—8)=0
~n=-%ors
. n=8 {as n>0}
there are 8 terms in the sequence.
194 Chapter 5 (Sequences and series) Exercise 5G

15 a The sequence is arithmetic with u; =7 and S, = 17.

Now S, =17, so §(2u1 +(n—1)d) =17


22x74+(2-1)d)
=17
14+d=17
d=3
b S, =242, so g(gul + (n —1)d) = 242
g(2><7——3(n—1))=242
3(14 +3n—3) =242
n(3n+11) = 484
oo 3n%+11n—484=0
o Bntdd)(n—11)=0
son=—%orll
. n=11 {as n >0}
there are 11 terms in the sequence.
16 13,21, 29, 37, .... is arithmetic with u; = 13 and d = 8.

Now S, = 1000, so §(2u1 + (n—1)d) = 1000


3(2 x 134 8(n — 1)) = 1000
3(26 +8n —8) = 1000
g(Sn +18) = 1000
4n? +9n = 1000
. 4n®+9n — 1000 =0
Using technology, n~ —16.98 or 14.73
. n~14.73 {as n >0}
But n must be an integer, and the sequence must exceed 1000, so we round up to 15.
n = 15 which means 15 terms are required.

17 The series of odd numbers can be expressed as an arithmetic series:


1+34+54+7+... where uy =1 and d =2.
a Now wu,=wu +(n—1)d
=1+2(n-1)
LUy =2n-1
b We need to show that S, is n>.
The sum of the first n odd integers can be expressed as an arithmetic series:
1+34+54+7+...+(2n—-1) {using u, =2n—1 froma}
n
So, Sn = 5(711 + un)

= §(1+(2n—1))
(2n)
S, =n? as required.
Chapter 5 (Sequences and series) Exercise 5G 195

18 The sum of the first n integers can be expressed as an arithmetic series:


1+2+3+4+...+n where uy =1, u, =n.

Sn:g(u1+un)

= 2(1+n)
n(n+1)
3 as required.

19 Let the three consecutive terms be z —d, z, and z + d.


Now, sum of terms = 12 Also, product of terms = —80
(z—d)+az+(x+d) =12 s (A=d)(4)(4+d) =—-80
3z =12 ooo4(4% —d?) =80
soz=4 o116 —d? =20
So, the terms are 4 —d, 4, 4+d s d>=36
=46

So, the three terms could be 4 — 6, 4, 4+6, whichare —2, 4, 10


or 4—(—6), 4, 4+ (—6), whichare 10, 4, —2.

S5 =480

g(ul + u,) = 480

%(ul + ’LL15) = 480

LU T U = 64

uy + (ug + 14d) = 64 {since wu, =uy + (n —1)d}


2uy + 14d = 64
uy +7d = 32
ug = 32

21 Let the five consecutive terms be x —2d, x* —d, z, x+d, and x + 2d.
The sum of the terms is 40.
(x —2d) + (x —d) + &+ (v +d) + (x + 2d) = 40
br =40
. z=8
So, the terms are 8 —2d, 8 —d, 8, 8+d, 8+ 2d.

The product of the first, middle, and last terms is 224.


(8 —2d)(8)(8+2d) =224
8(8% — (2d)?) = 224
64 — 4d* = 28
- 4d? =36
=9
= 223

So, the five terms could be 8 —2(3), 8 —3, 8, 8+3, 8+2(3), whichare 2,5, 8, 11, 14
or 8—2(—3), 8—(-3), 8, 8+ (-3), 8+2(—3), whichare 14,11, 8,5, 2
196 Chapter 5 (Sequences and series) Exercise 5G

22 a S,= w b The sequence is arithmetic with


+ up =7 and d=10—-7=3.
When n=1, S1ZM:7 41
) £ 2 Uy, =uy + (n—1)d
0, up = (.
Sty =T743(n—1)
When
=
’I’L—2,
_26@+11)
SQ—T—l?
oun=3n——4
So, uy =S5 — 5 ooouge = 3(20) + 4
=17-7 CLoug
= 64
=10
up =7 and wup =10

23 3-5+7-9+11—-13+15—.... can be expressed as two separate arithmetic series:


3+7+11+15+.... where uy =3, d=4, n=40
and -5—-9—-13—
... where u; = =5, d=—-4, n=40

Using S, = g (2u1 + (n—1)d),


the sum of the first series = 42 (2(3) + 39(4))
= 20(6 + 156)
= 3240
and the sum of the second series = 4 (2(—5) + 39(—4))
= 20(—10 — 156)
= —3320
-, the sum of both series = 3240 + (—3320)
= —80
So, 3—54+7—9+11—-13+15—.... to 80 terms is —80.

24 a u,=3+2n
up=3+2(1)=5, up=3+22)=7, u3=3+2(3)=9, u=3+2(4)=11
So, the graph is: A n

i @11),
(3,9)
8 @)
(1,5),
4

% 2 3 4 5 W
b S'VL=5+4+T7+9+ 11+ ... is an arithmetic series with u; =5 and d = 2.
Sy, is the sum of the areas of the first n rectangles.
¢ i The height of each rectangle increases by 2 units from the previous rectangle, so
Upt1 = Up + 2.
il The area of the (n+ 1) th rectangle is 1.
Sp+1 1is the sum of the areas of the first n rectangles and the (n + 1) th rectangle, so
Sn+1 = Sn T Up+1-
Chapter 5 (Sequences and series) Activity 1 Stadium seating 197

ACTIVITY 1

1 There are 13 tiers of concrete steps with total width


13 m.
. 13m .
each concrete step is E1a 1 m wide.

2 The spacing between rows is constant, so the arc


lengths of the rows form an arithmetic sequence.
the length of the arc at the back of each row
20.25 — 14.4
.increases by —— f frm
""" — ()45 m for each y / [ to centre
13 4 /[ of circular
row. stadium
We can summarise this in a table:

Arc length at back of row (m)

| Rowmmber [ 8] 9 [10] 1 [12] 13 |


Arc length at back of row (m) | 18.0 | 18.45 | 18.9 | 19.35 | 19.8 | 20.25

3 Number of seats in Row 1 = 25182


fengiiiafon
& 7O 07of 2OW
Row ©1
width of each seat
_ 144m
T 045m
= 32 seats
Since each row is 0.45 m longer than the previous row and each seat is 0.45 m wide, then there is
one more seat in each row than the previous row.
We can summarise this in a table:

Row number 1]2]3]a]s5]e6]7[8]o]o]n]i2]13


32 33 (34 [35(36 3738394041 ]42]43]44

L4 arc length = b X 2mr


360
6
144 = — x 27r
360
9= 14.4 x 360
27r

Le=22 nr

6
and 20.25 = 365 % 21 x (r +13)
o= 20.25 x 360
T 2n(r +13)
L= (r
_
+13)
198 Chapter 5 (Sequences and series) Exercise SH

i 2592 3645
Equating (1) and (2): — = T

2592 3645
r r+13
2592(r + 13) = 3645r
2592r + 33696 = 3645
1053r = 33696
. r=32

Substituting r = 32 into (1) gives: g =209


T X 32

=8

So, the angle of each sector of the circle is 8l ~ 25.8°.


s
o

Number of sectors in the circle = 322

B 360°

(%)

~ 13.96
So, the stadium has 13 sections as there is insufficient space for 14.

5 Total seating capacity = number of seats per section x number of sections


=(32+33+34+....+43+44) x 13
=4(32+44) x 13
=494 x 13
= 6422 seats

6 From 4, the radius 7 is 32 m.

1 a 3+6+12+24+48=93
b The series 3+ 6+ 12 + 24 + 48 is geometric with u; =3, =2, and n =>5.
Sn:ul(r"—l)
r—1

25 1 )
ARl
2—-1
3x31
1
=93
Chapter 5 (Sequences and series) Exercise 5SH 199

2 a The series is geometric with b The series is geometric with


up =2, r=3, n=28. up =5, r=2, n=10.
up_— (r™ — 1) up(r" —1)
-
Sn Sn

2(3% — 1) 5(210 — 1)
Sgi
SR S10 = 2-1
2(6561 — 1)
e 5(1024 — 1)
1
= 6560 =5x1023
= 5115

¢ The series is geometric with The series is geometric with


w =12, r=134, n=10. u1=\/7, r=\/7, n=12.

S, = Ul(l 1—pn
_: ) g —w@" =1
" r—1
P fi((fi)l? _ 1)
12(1 (%)10)
S0 =
1-3
T 2 ey
~ 189000
= 3089
128
240
e The series is geometric with The series is geometric with
uy = 6, r=—%, n = 15. u; =1, r=—%, n = 20.

_ur(l—r")
Sn = 1-—r 1—r

6(1 . (7%)15) 1 — (-2


S15 = T S = ( \/15 )
1-(-3) Haeer)
e 32769 ~ 4.00 ~ 0.585
8192

3 a V3+3+3v/3+9+.... 1246+3+13+....
The series is geometric with The series is geometric with
u1=\/§ and r = /3. up =12 and r= 3.
S =u1(r"—1) up(l—r")
& r—1 Sn =
1—7r

_ B -1
V-1
(Vi
V3+1 ) B 12(1 - (%)")
= i
2
_ B+ (3" -y
31 24(1
= ( ")
D=

_3+V3 <(\/§)n7 1)
2
200 Chapter 5 (Sequences and series) Exercise SH

¢ 0.940.09
4 0.009 + 0.0009 + .... d 20-10+5—-25 +....
The series is geometric with The series is geometric with
up =09 and r=0.1. up =20 and 7= —3.

Sp _wm@-r)
= — 1_pn
Sn:u1(17:)
~0.9(1—(0.1)™) -
- 1-0.1 e 20(1’(’5) )
=1-(0.1)" 1—(-%)
20 (1 - (7%)")
)

L S;=3 and S, =14


a 51=u1 b UQ=52—51

Loup = =4—-3

=1

r= Y2
uy
re1
_an
¢ u; =3 and r:% d Sn7u1(117: )
Uy = ugr"™ 3(1 (1),,>
u, =3 x (Hn7! = ?
us = 3 x ()* o
u5_2_17 : S”:%(lf(%)n)
S5 =3(1-(3))
_9 1
=5(1-3m)
_9(242
=3(33)
_ 1z27
__4fi413

10
5 a Y 3x281=3+4+6+12+...+384+
768+ 1536
k=1

This series is geometric with uy =3, r =2, and n = 10.


Sniul('rnfl)
r—1

S010 =
3(210
— 1)
-1
Chapter 5 (Sequences and series) Exercise 5SH 201

b U
k;(g)1\k=2 =24+14+3+...+55+
_ 1 1 1 1
5 12 + 1024

This series is geometric with uy =2, r = , and n=12.

o=
up(l—rm)

" 1-—r

25
€ 516X (—2)F = 12+ 24 4 (—48) + ... + 100663 296 + (—201 326 592)
k=1

This series is geometric with u; = —12, r = —2, and n = 25.


~u(l—r")
Sn = 1—r

-
_ —12(1 (-2)%)
525 = T+2
=—4(1-(-2)®)
= —134217732

Az = Az x 1.06 4+ 2000
= (A; x 1.06 + 2000) x 1.06 + 2000
= (2000 x 1.06 + 2000) x 1.06 + 2000
-, Az = 2000 + 2000 x 1.06 + 2000 x (1.06)? as required

Ay = Az x 1.06 + 2000
= [2000 + 2000 x 1.06 + 2000 x (1.06)*] x 1.06 + 2000 {from a}
" Ag =2000[1 + 1.06 + (1.06)* + (1.06)°] as required
Ajg = 2000[1 + 1.06 + (1.06)% + (1.06)® + (1.06)* + (1.06)® + (1.06)° + (1.06)"
+ (1.06)® + (1.06)°]
. Ajg =~ 26361.59
the total bank balance after 10 years is $26 361.59.
202 Chapter 5 (Sequences and series) Exercise SH

7 a The number of grains of wheat starts at 1, and each square


has double the number of grains of the previous square.
b The number of grains of wheat on each square can be
expressed as a geometric sequence 1, 2, 4, 8, ...
So, uy =1 and r=2.

Up = ugr™!
©ou,=1x27t
L Up = 2n—1

¢ There are 64 squares in total, so n = 64.


_ug(r™ —1)
Sn = r—1

_1e¥-y
L
=201
~ 1.84 x 10"
So, the king owed 2% — 1 =~ 1.84 x 10'° grains of wheat.

8 There is a fixed percentage increase each year, so Paula’s annual rent forms a geometric sequence.
u; = 5000 and r = 1.05
Paula’s annual rent after n years is u,, = 5000 x (1.05)"~L.
a wuy = 5000 x (1.05)°
= 5788.125
So, in the 4th year, Paula paid approximately $5790.
b 5 up(r™ —1)
" r—1
g _ B000((1.05)" —1)
L 1.05 — 1
~5000((1.05)" — 1)
Sn = 0.05
S, = 100000((1.05)" — 1)
¢ S7 =100000((1.05)" — 1)
~ 40710.04
So, Paula paid approximately $40 710 in rent during the first 7 years.

l1,1,1., 1 L
9 Sn 272783 16+""‘2n

aSi=5 S=3+3=1 Si=3ftits=s 1


1 1 1 3 1 1 1 7 1

1" 1A 1 15
Sy 2T2T38T716 16> n
1 1 1 1 1 il 2
Ss=stststutn=i5n L
1
2n —1 3
b S,= P
2n 3
M
Chapter 5 (Sequences and series) Exercise SH 203

- uy (1 —r™)
< S,Zf?, where u; _1
=5 and _1
r =3

1 1
5(1—(5)")
B 1
-3
1
Sp = *(E)n
1
S
2”

o
——

d As n— oo,
(l)"
2 — 0, and so S, — 1 (from below)
The diagram represents one whole unit divided into smaller and smaller fractions.
As n — oo, the area which the fraction represents becomes smaller and smaller, and the total
area approaches the area of a 1 x 1 unit square.

10 The series is geometric with us =6 and S5 = —14.


up =up Xr and uz =wug Xr
6 =ur = 6r
6
up = —
r

Now u1+u2+u3:§+6”6r:714
T

o 64 6r7 = —20r
624200 +6=0
2(3r2 +10r +3) =0
2(3r+1)(r+3)=0
or=-—3 1 or =3
Ug = U3 X T

=6rxr
= 62
Ifr=-1, w=6(-1)" If r=—3, us=06(-3)
=2 =54
3
11 The sequence is geometric with u; =6 and r=1.5.
5 uyp (r" — 1)
. r—1

_ (@) -1
TG
=12((L5)" — 1)
To find n such that S, = 79.125, we use a table of values with
Y =12 x (1.5/\X—1).
28.5
4 48.75
5 [EHFE
6 124.68

DELETE] ROW JHINN


S5 = 79.125, so n =>5.
204 Chapter 5 (Sequences and series) Exercise SH

12 Y 2x3" 1 =246+18+....
k=1

This series is geometric with u; =2 and r = 3.


5 — up (r'" — 1)
" r—1
2(3" — 1)
3-1

=3"-1
To find n such that S, = 177146, we use a table of values with
Y, = 3"X~-1.
9 19682
‘ 10 59048
11 RER
12 531440
177146
(FORHULA) DLEYEER
IR (ED T GPHCOR) GPEPLT
S11 =177146, so n =11

13 160, 80, 40, 20, ....


a The sequence is geometric with b S, = wlrt
P ; D
u; =160 and r = %
18
Uy = ug "t Sy = 160((2) 1)
- i 1
ug =160
x (%) 3§l
=160x (%)° 160(%4)
_ 160 -
= 128 -3
=1.25 _ 7320(7%)

= 318.75
¢ We need to find n such that S,, > 319.9.
5 — up(r™ —1)
B r—1

160((%)" - 1)
B % —

= —320((0.5)" — 1)
To find n such that S,, > 319.9, we use a table of values FeiRedlo] Gk
with Y; = —320 x (0.5"X~1). Y1=-320x
(.57
¥1 (x)-1)
‘ 10 319.68
11 319.84
12 EFERER
13 319.96
319.921875
DA IR (EDIT ) GPH-CON)GPH-PLT

S12 = 319.92, so n=12.


12 terms are needed for the sum of the terms to exceed 319.9.
Chapter 5 (Sequences and series) Exercise SH 205

14 a Option A: First year salary = $40 000


Second year salary = $40 000 + 5% x $40 000 = $42 000
Third year salary = $42000 + 5% x $42000 = $44 100
Total earned in three years = $40 000 + $42 000 + $44 100 = $126 100
Option B: First year salary = $60 000
Second year salary = $60 000 + $1000 = $61 000
Third year salary = $61 000 + $1000 = $62 000
Total earned in three years = $60 000 + $61 000 + $62 000 = $183 000
So, over three years Felicity would earn more under Option B.
b i Let A, be the amount of money earned under Option A in the nth year.
A, forms a geometric sequence with A; = 40000 and r = 1.05.
A, = 40000 x (1.05)""1
i Let B,, be the amount of money earned under Option B in the nth year.
B,, forms an arithmetic sequence with B; = 60000 and d = 1000.
B,, = 60000 + 1000(n — 1)
= 59000 + 1000n
If A, = B,, 40000 x (1.05)""! = 59000 + 10007, [EXE]:Show coordinates
1=40000X )~( (Z=T
We graph A, = 40000 x (1.05)"~! and
B, = 59000 + 1000n on the same set of axes and find their
points of intersection.
Since n >0, n~ 13.07.
the money earned under Option A will exceed that of
Option B after approximately 13.1 years.
n_ 1 : g g ;(zul + (n—1)d)
Sn = £l for a geometric i for an arithmetic
r—1 )
series series

Ty=—
40000 x ((1.05)™
)
— 1) Tp = %(2(60 000) + 1000(n — 1))
1.05 — 1
= 600001 + 500n(n — 1)
40000 x ((1.05)" — 1)
= 0.05 = 60000n + 50012 — 500n,
= 800000((1.05)" — 1) dollars
= 50012 + 59 500 dollars

i Initially Option B is better than Option A, total graph 1


so Tgp > T4 for small values of n. income ($) graph 2
graph 1 represents 74, graph 2
represents 1.

n (years)
206 Chapter 5 (Sequences and series) Exercise 51

ii The point P is where T4 meets Tz, which is when


800000((1.05)™ — 1) = 50012 + 59 500n.
We graph T4 = 800000(1.05™ — 1) and
[EXE]:Show coordinates
Ty _= 500n* 2 + 59 500n on the same set of axes and find =
D O e
their points of intersection.

Since n >0, P A (22.3, 1580000). 2288661

iii Option B provides the greater total income for 0 < n < 22 years.

2 _ 3 3 3
1 a 03=55+10+1
™05 + -
3 3
uy _ (100) = (1500) |
3 3 Nl |
U1 (15) (1o5) 10
the series is geometric with u; = 753 and r = 3.1
Since we are adding all the terms, it is an infinite geometric series.

b We need to show that 0.3 = 1.


Now
3_23 3 3
0.3—E--m+m+...

So,let 25+ 25+


Sp==5+

Since n — oo, then S = !


1—r
3
__ T
1-(1p)
-1
i
0.3=1 as required

2 a 04=0.444444.... b 0.16 =0.161616....


_ 4 4 4
=10+ 100 + 1000 T _ 16 | 16% piloil
102 104 106
is an infinite geometric series with . e . ) i
is an infinite geometric series with
uy =+ 4 and r ==.1
10 10 up 16
=_ 155 and 1=2,35.
g=1 u
i4 o 8= bcn i

S
= T
16
__To0
-1 i1
4 100
=4 drat
— — 99
04=2 s
o 016 =45
Chapter 5 (Sequences and series) Exercise 51 207

¢ 0.312=0.312312312....
312 312 312
=1t ie T T
is an infinite geometric series with u; = 22
1000

3
0312
= 1%

3 Checking Exercise SH question 9 d: S = B =1 v

4L a 18+ 12+ 8+ % + ... is an infinite b 189—-6.3+2.1—0.7+.... isan infinite


geometric series with u; = 18 and geometric series with u; = 18.9 and
r= % T = —%.

_u 8= u1l
§= 1—r o 1—r
18 189
= 1— 32
= T
1- (*3)

=54 =14.175

= 3
5 a ) = 2+ Z+ & +... isan infinite geometric series with u; =3 and r= 1.
k=1
5=11—7r
3
_ I
s 1
-1
=1

. k
b > 6(—2)"=6-6x(2)+6x(£)?—... isan infinite geometric series with
k=0
up =6 and r=—%

§=11-7r
or 6
I SRV
1-(-3)
=230 2
(: 4?)
208 Chapter 5 (Sequences and series) Exercise 51

6 Let the terms of the geometric series be w1, uyr, wuir?, ...
Ul
uy + urr 4+ ugr? =19 and S= =27
1-r
(1 4r ) =19
" =271 —71) ... (2)
W=y e (1)
Equating (1) and (2), % =27(1—r)
B=(1-n1+r+r?)
B=ttrtri—r—r2—1®
Vo143
P=t
-3
Substituting 7 = 3 into (2) gives uw; =27(1 - ) =9
the first term is 9 and the common ratio is %

7 Let the terms of the geometric series be uy, uir, uir?, ..


2t
uyr =8 and S:liT:IO

Loy = % - (D o =10-10r .. (2)


Equating (1) and (2), 53 =10-10r
T

— 5012
8=50ro
. 50r? —50r +8=0
2(25r% — 25r +4) =0
2(5r —1)(5r —4) =0
L r=1% or %
Using 2), if r=2%, uw;=10—10(%)=38
if r=2, u=10—10(3)=2
either u; =8, r:% or wu =2, r:%.

8 z, x—2, 217, ...

a The sequence is geometric, =2 LIk


x r—2

so(r—2%=x22-T)
oa?—dr+4=22%—-Tzx
o2t —3z—-4=0
(z+1)(z—4)=0
. x=—1lor4d

b When z = —1, the sequenceis —1, —3, —9, .... with » =3, and since |3|> 1, the series
is divergent so it does not converge.
When o = 4, the sequence is 4,2, 1, ... with r =1, andsince |$| <1, the series
4
converges. The limiting sum in this case is S = 1 =8.
1—3
Chapter 5 (Sequences and series) Exercise 51 209

9 a Total time of motion


=14 (90% x 1)+ (90% x 1) + (90% x 90% x 1)
+ (90% x 90% x 1) + (90% x 90% x 90% x 1) + ...
1+0.9+0.9+ (0.9)2 + (0.9)* + (0.9)* + ....
=1+2(0.9) +2(0.9)2 +2(0.9)> + ... as required

ground

b The total time of motion can be written as [2 + 2(0.9) + 2(0.9)2 +2(0.9)3 +...] — 1
So, S, = M —1, where w3 =2, r=0.9
-

_201-09")
Sn= 1-09 1
_2(1-09")
Sp = = 1

S, =20(1-0.9") -1
Sp,=20—20x%x09"—1
Sn =19 —20(0.9)"
¢ For the ball to come to rest, n must approach infinity.
As n—o00, 0.9" -0 and so 20 x 0.9" — 0 also.
Sp — 19 (from below)
So, it takes 19 seconds for the ball to come to rest.

10 Total distance travelled

=t 2(Dhr2(HEnt
=h2(af+§+ @)+ @)+
e

=h——%h< 13) up }
{as |r| = %‘<1 and S =
e

1—r
-7
=h+32h(4)
=Th
T

But 7h =490, so h = 70.


ST

The ball was dropped from a height of 70 cm.

0-—_ 9 9 9
1 a 0.971—0+m+m+.... which is geometric with u; = 10

9
09=5=-"1 1
=1
-1
210 Chapter 5 (Sequences and series) Activity 3 Von Koch’s snowflake curve

9
b Uy = — <
T 10m
Sn_ul(lff")
1—r

(@)
9 1

- 1
1-15
9 1"
m(l—lon)
- 9
0
1
Sn=1- 1o

12 ST -®) @) () ()
mi B2 (B2 (Y2 2 3

B 1U1 {as it converges to 4 and is geometric}


—T

1 2

2
=4 andso 2-3x=
2—3x
xr

AT
ACTIVITY 3

1 a () has perimeter 3 units.


each side of triangle C has length 1 unit.
Now, Cy has been formed by dividing each side of C) into thirds, then making another
equilateral triangle along the middle third of each side.
So each side of Cy has length % units.
-, the perimeter of Co = § x4 x 3 X
= 4 units % units
Chapter 5 (Sequences and series) Activity 3 Von Koch’s snowflake curve 211

Similarly, C's has been formed by dividing each side of C5 into thirds, then making another
equilateral triangle along the middle third of each side.
So each side of C has length % x 3 = & units.
the perimeter of C5 = = x4 x4 x 3

Also, each side of Cy has length 1 x 1 = 51 units.


the perimeter
of Cy = 2= x 4 x 4 x4x 3

Also, each side of C5 has length % X 2—17 = 8% units.


the perimeter of C5 = g7 x 4 x 4 x4 x4 x 3
= 26 units
256 64 16
bCS_W_é C4_?_é C3_T_é Cz_é
Cy % 3 C3 ? 3 Co 4 3 Cq 3

Consecutive terms have a common ratio of %.


the sequence is geometric with u; =3 and r = %.
Up = ur™!
Up =3 X (%)"71
the perimeter of C,, = 3 x (%)n_1 units
As n — o0, (%)nflfloo
So, von Koch’s curve has infinite perimeter.

2 ()} has area 1 unit?.


a (4 canbe divided into 9 equilateral triangles as shown,
each with area § units?.

Cy is formed by adding 3 equilateral triangles to C',


each with area § units?.
So, Cy has area Ay =1+3x §
=1+ 1 units®

Cy is formed by adding 4 x 3 = 12 equilateral


triangles to C, each with area § x § = g7 units?,
So, C3 has area Az =1+
% +12x g7
=1+ 3[1+ 2] units®
212 Chapter 5 (Sequences and series) Review set SA

Cy is formed by adding 4 x 12 = 48 equilateral


triangles to C3, each with area L
9 si = %9 units?.

So, Cy has area Ay =1+ %[l-fi—%] 48><fi

el gl
1+ 31+ 2+ (3)? units®
C5 is formed by adding 4 x 48 = 192 equilateral triangles to Cy, each with area
$1 X 7351 = gag7 units®.
So, C5 has area A5 =1+ 3[1+ 2+ (3)?] + 192 x 2=
L3 d+ (32 + 5]
=1+2[1+2+(3)?+(3)°] units®
b A; =1 unit’
Ay =1+ 1 =1.333333333.... units®
Ay =1+ 1[1+ 3] = 1481481481 ... units®
Ay =1+ 21+ %+ (%)%~ 1.547325103 units®
As =1+ 3143+ (3)*+(3)’] = 1.576 588935 units®
As=1+314+2+ 3+ (3’ +(3)"] » 1.589595 082 units®
Ar=1+11+3+ 3+ @)+ (3)* + (3)°] = 1.595 375 592 units®
¢ Area within von Koch’s snowflake curve
=1+21+3+3)°+@)1°+..]
:1——%><1 1 - {as r=73,4 |r[ <1 so converges}
-9
1,9
1+3x %
=§=16umts

3 Yes, the perimeter of von Koch’s curve is infinite whereas the area of von Koch’s curve is finite.

1 5,9,11, 12, 15, 19


a upy=9 b ug=19 ¢ S;=5+9+11+12=37

2 a 7-1,-9 -17, ...


= =18 The difference between successive terms is constant.
-9—(-1)=-8 .. the sequence is arithmetic with u; =7 and d = —8.
—17—-(-9) = -8

b 4,-2,1,-3, ..
2__1
4 412
1__1 ) T3__1
12
Consecutive terms have a common ratio of 7%.

the sequence is geometric with u; =4 and r = —%.


Chapter 5 (Sequences and series) Review set 5SA 213

¢ 1,1,2,3,5,8, ..
1-1=0 The difference between successive terms is not constant.
2-1=1 .. the sequence is not arithmetic.
3—-2=1
5-3=2
8—-5=3

17
1 2
Z_oki’exiy
3 3
SF25
5 5
5_8 3
1 1 22 3 3 5 5
Consecutive terms do not have a common ratio.
the sequence is not geometric.
So the sequence is neither arithmetic nor geometric.

3 Since the terms are consecutive, (k—2)—3k=k+7— (k—2) {equating differences}


k—2-3k=k+7—-k+2
—2-2k=9
2k = —11
e 1)
k=—-%
1\n—1
b un =06(3)
6(Lynt+1-1
“"+1:¥7§ for all n b u=6 r=3
6(5)7"71
Un
15
{u,} is a geometric sequence. € u = 6(%)
~ 0.000183

Substituting r =2 into (1) gives Substituting r = —2 into (1) gives


U1X25:% U1X(72)5:¥

B u1><32:% cooug x (—32) =48


up =% up=—¢

Uy =2 x 2" or —3x(=2)"7!

6 Let the numbers be 23, 23 +d, 23+ 2d, 23+ 3d, 23+ 4d, 23+ 5d, 23+ 6d, 9
Then 23+7d=9
7d=—14
L d=-2
So, the numbers are 23, 21, 19, 17, 15, 13, 11, 9.
214 Chapter 5 (Sequences and series) Review set 5SA

7 u1=-2, n=9, u, =54


n
Sn o 5(”1 +un)

So = §(—2+54)
=234
total amount of juice collected
a Average amount of juice collected
number of lemons
274.3 mL
6
~ 45.7T mL
Uy, & 45.7n

w1z ~ 45.7 x 13
=~ 594
So, approximately 594 mL of juice would be collected from squeezing 13 lemons.

18 —12+8—
... b 8+4v2+4+...
is an infinite geometric series with is an infinite geometric series with
u; =8 and r:%.

5= 5=
1—r 1—r
18 1+
:le_\f
-
1-(-2)2 -7 (0+5)
_ 54
=% or 1034
st 8
N 2

_8+4V2
e 1
2

=16+8v2
T+11+154+19+....+99 b 354334 +32+305 +....+20
The series is arithmetic with The series is arithmetic with
w =7, d=4, and u, = 99. up =35, d=—%, and u, = 20.
First we need to find n. First we need to find n.
Now u, =99 Now u,, =20
L ur+(n—1)d=99
up + (n—1)d =20
T+4n—-1)=99
©35—3(n—-1)=20
4(n—1)=92
. n—1=23 o 3n-1)=15
on=24 n—1=10
on=11
Using S, = %(ul + uy),

Saq = Z(7+99)
=12 x 106
= 1272
Chapter 5 (Sequences and series) Review set SA 215

1 Year 2011: 700000 x 0.9 = 630 000 sheets of paper


Year 2012: 630000 x 0.9 = 567 000 sheets of paper
There is a fixed percentage decrease each year, so the amount of paper used each year forms
a geometric sequence.
In 2008, the school used 700000 =- 0.9 + 0.9 ~ 864198 sheets of paper.
uy ~ 864198 and r =0.9
For the decade from 2008 to 2017, n = 10.
_u(1—7r")
Sn = 1—r

g~ 364 198(1 — (0.9)19)


i 1-09
~ 5628705
The school used approximately 5630000 sheets of paper in total in the decade from 2008 to
2017.

12 86, 83, 80, 77, ....


83 —-86=-3 The difference between successive terms is constant.
80 —-83=-3 .. the sequence is arithmetic with u; =86 and d = —3.
77T—-80=-3
up =ur
+ (n—1)d
=86—3(n—1)
Uy =89 — 3n

81,12 . or 8512 .
The numerators form an arithmetic sequence with u; =3 and d = 2.
U, =up + (n—1)d
Un, =3+2(n—1)
=2n+1
The denominators form an arithmetic sequence with w; =4 and d = 1.
Uy, =up + (n—1)d
Up, =44+ 1(n—1)
=n+3
So, the sequence is wu,, = 21 1.
n+3
100, 90, 81, 72.9, ....
90 9 81 9 729 9
0 10 9% 10 8 10
Consecutive terms have a common ratio of %.
the sequence is geometric with u; = 100 and r = 1%
Uy = ur™
un =100 x ()"~ =100(0.9)"!
7
13 YR =1"+22
432+ 47+ 52+ 67+ 7
=l 1444 9416+25+36+49
=140
216 Chapter 5 (Sequences and series) Review set SA

b Lk+3 45,61
Lz stitits
N 80 75 72 70
Tt to
_ 207
)
— 99
20

S, = 3n? + 5n
2
. Un = Sn _ Snfl
302 +5n _ 3(n— 1)2 +5(n—1)
L 2 2
3245 —3n%—2n+1) —5(n—1)
h 2
_ 32 4+5n—3n2+6n—3—5n+5
- 2
_ bn+2
T2
ou, =3n+1

Using part a, u, —up—1 = 3n+1)—(3(n—1)+1)


=3n+1-3n+3-1
=3
The difference between consecutive terms is constant for all n, so the sequence is arithmetic.

15 The interest is calculated half-yearly, so there are n =5 x 2 =10 time periods.


4.25%
Each time period the investment increases by i = = 2.125%.

-, the amount after 5 years is w19 = ug x (1 414)™°


=12500 x (1.02125)1° {2.125% = 0.02125}
~ 15425.20
The investment will amount to £15425.20.

The interest is calculated monthly, so there are n =5 x 12 =60 time periods.


4.25%
Each time period the investment increases by i = ~ 0.354%.

-, the amount after 5 years is ugo = ug x (1 +1)%°


~ 12500 x (1.00354)% {0.354% = 0.003 54}
~ 15453.77
The investment will amount to £15453.77.
Chapter 5 (Sequences and series) Review set SA 217

16 a N=06x4=24, I%=52, PV =-60000b PMT =0, P/Y=4, C/Y =4


a (o]
Compound Interest
n =24
1% =5.2

FV =8180.464024
P/Y=4 v
o JCT% JCPY JCPMTICFV (R

FV ~ 8180.46
The future value of the investment is €8180.46.
b Interest earned = €8180.46 — €6000
= €2180.46

17 The initial investment uq is unknown.


There are n =4 x 4 =16 time periods.
6.5%
Each time period the investment increases by i = = 1.625%.
Now, uig = up x (141)*
6212.27 = ug x (1.01625)'6 {1.625% = 0.01625}
U = Torga ~ 480000
6212.27

Chelsea originally invested $4800.

18 I% =8, PV =-4000, PMT =0, FV =10000, P/Y =12, C/Y =12

1%
PV =-4000
PMT=0
FV =10000
P/Y=12
n_ ][ 1% JCPV

N ~ 1379
The amount of money needs to be invested for 138 months, or 11 years 6 months.

19 a To index the amount of money for inflation, we increase it by 2.5% each year for 4 years.
indexed value = €6000 x (1.025)*
= €6622.88
b To index the amount of money for inflation, we increase it by 2.5% each year for 7 years.
indexed value = €11 200 x (1.025)7
=€13313.28

20 a Thereare n=3x 12 =36 time periods.


6.2%
Each period, the investment increases by i = ~ 0.517%.

", the amount after 3 years is usg = ug x (1 +14)3°


=20000 x (1.00517)*% {0.517% = 0.00517}
~ 24076.91
The future value of the investment is $24 076.91 .
218 Chapter 5 (Sequences and series) Review set 5A

b real value x (1.018)* = $24076.91


$24076.91
-, real value = ———
(1.018)3
= $22822.20

21 28,23,18, 13, ...


23 —28= -5 The difference between successive terms is constant.
18—-23=-5 .. the sequence is arithmetic with u; = 28 and d = —5.
13-18=-5
Up =u1
+ (n—1)d
cooup =28 —=5(n—1)
. U, =33 —5n
E
Sy == (2u1 + (n —1)d)
2
n
:5(2><2875(n71))

2(56 —5n+5)
= 2(61 — bn)

22 12,19, 26, 33, ....


a 19-12=7 26-19=7 33-26=7 S, = %(2u1 + (n—1)d)
the common difference d = 7.
" Sm:l—QU(QX 12+9><7)
up =ug
+ (n—1)d
= 5(24 + 63)
LU, =1247(n—1)
=435
S U, =Tn+5
L ug=T(8)+5
=61

¢ S, =915 so Z(2us+ (n—1)d) =915


2

g(g x 12+ 7(n—1)) =915


2(24+ n —7) =915
. n(Tn+17) = 1830
n? 4+ 17n — 1830 = 0
Using technology, n = 7&72 or 15
. n=15 {as n >0}

23 a wus=ugx (1—d)°
=135000 x (0.85)° {15% = 0.15}
~ 59900.22
So, after 5 years the value of the truck is $59900.22.
b The depreciation = $135000 — $59 900.22
= $75099.78
Chapter 5 (Sequences and series) Review set SA 219

24 a 128,64, 32,16, ..., 515


1
The sequence is geometric with u; = 128, r = %, Uy = 512

Up = ugr™Tt

~ 125(4)""
=27 x 2!
1 o7y ol—
gz =2 x27"
279 — 28771

—-9=8-n
n=17
So, there are 17 terms in the sequence.

b S, = w1 17
—r")
n

1—7r

128(1 = (%)17)
Sip=——"7
1-3
131071
T Ub12
~ 256

25 Let the terms of the geometric series be w1, uyr, uir?, ...

uy +urr = 90 and wuyr? =24


ur(l+r) = =90 Lwm=224
w = 90 ) B
1= Tir -

90 24
Equating (1) and (2) gives — ==
147
90r? = 24 + 24r
90r2 — 24r — 24 =0
6(1572 —4r —4) =0
6(57+2)(3r—2)=0
r=-2or 2

Using 2), if r=—2, u = —2 — = 2150


(=3) b
) _ 2 _ 24 24
if r=3, ul——(g)z—?—54
3 7
either uy =150, r=—2 or w3 =54, r=32.
Since || <1 in each case, both series converge.

26 a Every week after the first, Tim smokes 5 less cigarettes, so the difference between successive
weeks is always —5. Thus we have an arithmetic sequence with u; = 120 —5 = 115 and
d=-5.
220 Chapter 5 (Sequences and series) Review set SA

n
b u,=u; +(n—-1)d < Sn=§(u1+un)
=115-5(n—1) =
.%(11
=120 om & =12 122(x 151; )
s Tim will smoke 1380 cigarettes before
. bn =120 .
he successfully quits.
on=24
So, it will take 24 weeks before Tim has
smoked his last cigarette.

27 a Total prize value for Option 1 = $8000 x 24


= $192000
b i Amount won in each of the first three months:
Month 1: $1000
Month 2: $1000 + $600 = $1600
Month 3: $1600 + $600 = $2200
il The amount (in dollars) won in month n forms an arithmetic sequence with u; = 1000
and d = 600.

Sy = g(2u1 +(n—1)d)

Saq = 2(2 x 1000 + 23 x 600)


= 12(2000 + 13800)
= 189600
So, the total amount won over the 24 month period was $189 600.
< i Amount won in each of the first three months:
Month 1: $500
Month 2: $500 x 1.2 = $600
Month 3: $600 x 1.2 = $§720
ii The amount (in dollars) won in month n forms a geometric sequence with u; = 500 and
7 =F172k
S — up(r'™ —1)
i r—1

Gos — 500((1.2)24 — 1)
24 121
~ 196242.12
So, the total amount won over the 24 month period was $196242.12.

d Option 3 is worth the greatest amount of money overall.


e Syy =250000, r=1.2 but u; is unknown
up((1.2)%% - 1)
250000 =
121
e — 250000 x 0.2
LT o) -1
~ 636.97
So, the new initial amount is $636.97.
Chapter 5 (Sequences and series) Review set SB 221

28 a 160, 80v/2, 80, 40v/2, ....


The sequence is geometric with u; = 160 and r = %
Up = uyr”!
1 n—1
=160><<E>
1 11
U127160><<75)

1
=1 60 x o

160
T 32v2
-
V2
=5v2

b i Snful(l_rn) i §=-4
1—r 1—r

(- (%)) 1oL
(\" _
x5
160

V2
V2

: 10
160v2 _ V241
= X
60(1—(%) ) VZ-1' V2+1
" Sl():l—l _160v2(v2+ 1)
-2 - 21
i 60(1- -1 45w =32 320 + 1 160v/22
- L
V2

_10(1-5)
- 1
-7
_ 160% 33
= 1
'
155 V2
S *%
V2
_155V/2 o V2+1
T VZ-1 T V2+1
_ 155V2(V2+1)
B 2-1
=310+ 155v2

REVIEW SET 5B

1 a Thesequence {(3)"} begins 1. %, 5=, 57, 53 (letting n=1,2,3,4,5, ..).


b The sequence {12+ 5n} begins 17, 22, 27, 32, 37 (letting n =1, 2, 3, 4, 5, ....).
222 Chapter 5 (Sequences and series) Review set 5B

C The sequence {nLH} begins %, 1, 2,2, 2 (letting n=1,2,3,4,5,..).

u, = 68 — 5n, Upy1 =68 —5(n+1)


=63 —5n
Unt1 — tn = (63 — 5n) — (68 — 5n)
= —b, a constant
Consecutive terms differ by —5.
the sequence is arithmetic.
uy = 68 —5(1) d=—5 ¢ ugr = 68 —5(37)
=63 = —117

Let wu, = —200 =68 —5n


. bn =268
. n=>53%
We try the two values on either side of n = 532, which are n =53 and n = 54:
usz =68 —5(53) and w54 =68 — 5(54)
= —-197 = -202
So, usg = —202 is the first term which is less than —200.

3,12, 48, 192, ...


12 e L
3 12 48
Consecutive terms have a common ratio of 4.
the sequence is geometric with u; =3 and r =4.

up = ugr”!
L oup =3 x4t
©oug =3 x 48
=196 608

uy = 31 cooup +6d=31 (1) {using u, =u; + (n —1)d}


us = —17 . ouy+14d =17 ... (2)
We now solve (1) and (2) simultaneously:
—u; — 6d=-31 {multiplying both sides of (1) by —1}
up + 14d = —17
. 8d=—48 {adding the equations}
s d=-6
So in (1), ug +6(—6) =31 Check:
. uy —36=31 ur =7—3
6(7)
oup =67 =31 v
Now wu, =u; + (n—1)d w5 = 73 — 6(15)
up =67 —6(n—1) =-17 v
Uy, =T73—6n

Uzq = 73— 6(34)

=-131
Chapter 5 (Sequences and series) Review set 5B 223

5 a 24,231 221, ..
23%—24=-3 221 -231=-3
the sequence is arithmetic with u; =24 and d = 7%.
Now u, =u; + (n—1)d
—36=24—3(n—1)
—60=—3n+3
3 243
n= g
n =381
So, —36 is the 81st term of the sequence.

b ougs =24+34x
(—2) < Sn_§(2u1+(n—1))
102
S - Sio=40(2x 24439 x (-2))
__3
=72 =20(48 - 4F)
=-13 =375
a The series is arithmetic with b The series is geometric with
u =3, d=6, n=12. u1=24,r=%,n=124
Now S, =22 (2us + (n —1)d) g - md=r)
i iF—n
S1p=2(2x3+11x6
12 2 ( ) 24(17(%)12)
S12 = 6(6 + 66) - Sip = _—
=432 -3
=43(1—-(3)")
12285
o256
~ 48.0

24, a, 6, ....
a If the sequence is arithmetic, then a —24 =6 —a {equating differences}
2a = 30
a=15

b If the sequence is geometric, then % = — {equating common ratios}


a

a® =144
a==+12

a up =60 .. ou+24d=60 .. (1) {using u, = u1 + (n —1)d}


ug3 =135 . ug +42d =135 ... (2)
We now solve (1) and (2) simultaneously:
—u; — 24d = —60 {multiplying both sides of (1) by —1}
uy +42d = 135
18d =75
d= 75 _ 25
18 6
224 Chapter 5 (Sequences and series) Review set 5B

So in (1), uy +24(2) =60 Check:


" up +100 = 60 ugs = 32(25) — 28
cooup = —40 =60 v
Now u, =u;+ (n—1)d u43:%(43)7%
o = =40+ 2(n—1)
6 =135 v
Up = —40+ Bn -
up = 8n — 28
b The common difference is 27)5 ~ 4.17. This means that Stacy makes approximately £4.17
profit per customer.
The constant term is —22%5 ~ —44.17. This means that the setup fee for the hot dog stand is
approximately £44.17.

< u, = £n — 28
LoUuUze = %(36) - %
~ 105.83
So, Stacy made an estimated profit of £105.83 on the third day.

9 5,10, 20, 40, ....


The sequence is geometric with u; =5 and r = 2.
Soup x 2t
=5
We need to find n such that w, > 10000.
Using a graphics calculator with Y; =5 x 2 (X—1), we view a table of values:
B HetiRedflom] Gh/dReal
Y1=5§2"(3€{;1)
‘ 10 2560
11 5120
ey 10240
13 20480
10240
(FORHULA) CIENET3
WSO (EDI T 6PH-CON) GPHEPT

The first term to exceed 10000 is u12 = 10240.

10 o Total distance travelled


=3+2x3x08+2x3x(0.8)%+2x3x(0.8%+....
=3+2x3x0.8[1+0.8+(0.8)*+ (0.8)° +....]
3m
=3+48x 71 {as r=20.8, |r| <1 soconverges to lu—lr}

—34 48
702
=3+24
= 27 metres

11 The interest is calculated annually, so n =7 time periods.


uy = up x (1414)7
=8000 x (1.03)" {3% = 0.03}
~ 9838.99
The final value of the investment is €9838.99.
Chapter 5 (Sequences and series) Review set 5B 225

12 a The interest is calculated annually, so n = 3 time periods.


uz = up x (141)*
=7000 x (1.06)* {6% = 0.06}
~ 8337.11
The investment will amount to $8337.11.
b The interest is calculated quarterly, so there are n =3 x 4 =12 time periods.
" . . . .6
Each time period the investment increases by i = % =1.5%.

", the amount after 3 years is u1o = ug x (1 +14)'2


= 7000 x (1.015)' {1.5% = 0.015}
~ 8369.33
The investment will amount to $8369.33.
¢ The interest is calculated monthly, so there are n =3 x 12 =36 time periods.
. . ) . .6
Each time period the investment increases by i = %’ = 0.5%.

", the amount after 3 years is usg = ug x (1 +14)3°


= 7000 x (1.005)%*¢ {0.5% = 0.005}
= 8376.76
The investment will amount to $8376.76.
2v3
13 a Since the terms are geometric, b When k= 2—?, r= % = %
kE_ k-1 . Y
i {equating the'
common ratio r} o 6
2 2 7%

FoaE-n When k=-283 ,=k_"73


4 4
K2 =4k%—4
3k% =4
2 _ 4 SS op— 3 B
k=3
2 2v/3
k=*m=+5"
14 Since Seve walks an additional 500 m = 0.5 km each week, we have an arithmetic sequence with
uy; = 10 and constant difference d = 0.5.
U, =ug + (n—1)d
up, =104 0.5(n — 1)

a us2=10+0.5(52—1) {52 weeks in a year}


=355
Seve walks 35.5 km in the last week.
b In total, Seve walks 10 + 10.5 + 11 + .... + 35.5, which is an arithmetic series with
u; =10, u, = 35.5, n=52.

Sn = g(ul + un)

Sso = %(10 +35.5)


=1183
Seve walks 1183 km in total.
226 Chapter 5 (Sequences and series) Review set SB

15 a 121 — 1.1 +1 — ... is an infinite b %+%+%+.... is an infinite geometric


geometric series with u; = 1.21 and series with wy — X and r = 2
3 7
r= _%‘ . S _up
g _u o I
1—7r 14
3
= i
121
(7%
=— 1— 7

1331
=—=0.634
=13~ 653
2100

16 24,8, 3,8 5,8 .


The sequence is geometric with uy =24 and r =

wl=
un =24 x ()"
up =24 x 3177
We need to find n such that w,, < 0.001.
Using a graphics calculator with Y; = 24 x 3/ (1-X), we view a table of values:
B HetiRadNorn] [b/dRes
Y1=2§X3Ag}—x)
‘ 9 3.6e-3
10 1.2e-3
11 En
12 1.3e-4
819683
[FORAULA AT ICEDIT ) GFH-CON GPHPLT

8
The first term which is less than 0.001 is uq; = ~ 0.000406.
19683

17 N=6x2=12, PV =-200000, PMT =0, FV =250680, P/Y =2, C/Y =2

Compound Interest
n =12
1% =3.799979755
PV =-200000
PMT=0
FV =250680
P/Y=2
n 1% PV

I% =~ 3.80
The interest rate is 3.80% p.a.

18 I%=5.8, PV =-5000, PMT =0, FV =12000, P/Y =12, C/Y =12

und Interest

PV

~ 181.6
It will take 182 months, or 15 years 2 months for the investment to amount to €12 000.
Chapter 5 (Sequences and series) Review set 5B 227

19 a Thereare n =38 x4 =32 time periods.


Each period, the investment increases by i =

. the amount after 8 years is gy = ug x (1 +1)%?


= 7500 x (1.00925)32 {0.925% = 0.00925}
~ 10069.82
The future value of Richard’s investment is $10069.82.

b real value x (1.031)® = $10069.82


. real value =
$10069.82
(1.031)8
= $7887.74
20 us =up x (1 —d)°
=9800 x (0.74)° {26% = 0.26}
=2174.63
So, after 5 years the value of the photocopier is $2174.63.

8. /31—3k
21 ak;(2 ) 144128 411493
+8+64 +5+31
This series is arithmetic with u; = 14, d= —2,5» and n=38.
Using S, = g(ul + uy),

Sy = (14 +31)
=4x17%
=170
15
b >750(0.8)F1 ~ 50 + 40 + 32 + .... + 3.436 + 2.749 + 2.199
k=1
This series is geometric with u; = 50, r = 0.8, and n = 15.
_u(1—7r")
S" I} 1—7r

50(1 — (0.8)1°
s = 20
=250(1—(0.8)")
~ 241

¢ S50 =53 +5(3) +5(3)" + .


This is an infinite geometric series with uy =5(%)" and r =
o
228 Chapter 5 (Sequences and series) Review set 5B

22 a wg=24 - wxr® =24 .. (1) b S"ZM(LID


o
upp —=768 .. ouyp x 010 _=768 ... (2) $@n 1)
10 - —_—
Now —=e =
wur @ {@®+ . M} iF
2—-1

P =39 = 2(2 - 1)

r=2 o Sis=32(2"° —1) = 245751


Using (1), w3 x 2° =24
24 3
Ur=135 =1
Uy = ugr™!

un = (32!
23 a Uy, =4n
— T
oupg=4x10-7
=33
b The difference between consecutive terms is constant for all n, so the sequence is arithmetic.

¢ S, = g(2u1 + (n—1)d) where u; =4(1) —7=-3 and d=4.


U5 + U + U7 + ... + U0 (Ul tug +ug + ... +ug +urs + ure + ury + ...+ U30)
—(u1 +UQ+’U,3+..‘.+7_L14)

:S30_Sl4

B(2x(=3)+29x4) — L2x (-3)+13x4)


= 1328

24 1% =48, PV =-7800, PMT =0, FV =9000, P/Y =4, C/Y =4


a [ornd)
Compound Interest
T 99647848

N =120
Ena will be able to buy the car in 12 quarters, or 3 years.

25 There is a fixed percentage increase each year, so the population forms a geometric sequence.
ug = 3000 and r =1.05
the population after n years is w,, = 3000 x (1.05)™.
a 2007 is 3 years after 2004.
uz = 3000 x (1.05)*
= 3472.875
There were approximately 3470 iguanas on the island in 2007.
Chapter 5 (Sequences and series) Review set SB 229

We need to find when 3000 x (1.05)™ = 10000.


[l [EXE]:Show coordinates
=3000x1.05%(x;
2=10000

INTSECT
=24.67654751 ¥=10000 =

So, it will take approximately 24.7 years for the population to reach 10 000.
the population will exceed 10000 in the 25th year, which is 2029.

0O

> 50(2z — 1)k~ is a geometric series with r =2z — 1 and converges if —1 <r <1
k=1
—1<2z-1<1
L 0<22<2
L 0<zr<l
When =03, 20 —1=06—-1=-04
o0

> 50(22 — 1)1 = 50(—0.4)° 4 50(—0.4)" + 50(—0.4)2 + ...


k=1

which is geometric with u; =50 and r = —04.

Now as 0 < 0.3 <1, the series converges and S = 1u1


-

50
1404
50
CaT
5
_ar5
=353

The initial investment uq is unknown.


There are n =3 x 2 =06 time periods.
6.5%
Each time period the investment increases by i = = 3.25%.
Now ug = ug x (1+1)°
100000 = ug x (1.0325)° {3.25% = 0.0325)
100000
U ~ 82539.08
"~ (1.0325)6
Michael invested $82539.08 three years ago.

112360 x 1.06 | 119101.60 x 1.06


=119101.60 ~ 126 247.70

V,, = 100000 x (1.06)" dollars


Each year, the amount of money in the safe increases by a constant amount of $6000, with an
initial amount of $6000.
So, we have an arithmetic sequence with u; = 6000 and d = 6000.
Sy, = 6000 + (n — 1)6000
S, = 6000n dollars
230 Chapter 5 (Sequences and series) Review set 5B

e Tn=Vo+ Sy
. T3:V3”S3 and T4:‘/4+S4

= 119101.60 + 6000(3) ~ 126 247.70 + 6000(4)


= 137101.60 dollars ~ 150247.70 dollars
So, the table is:

e] 0 | 1]
f We need to find when T, =V, + S, = 400000
100000 x (1.06)™ + 6000n = 400 000

[EXE]:Show coordinates
=100000x1 . 06~(x)+6000x
2=400000

So, it will take approximately 18.3 years, or 19 whole years, for Michael to have the $400 000
needed to buy his house.
Chapter 6
MEASUREMENT

a Arc length = L X 27r


360
—= 60
555 X 2m X 5 cm
~ 5.24 cm

Arc length = L X 2mr


360
_=555
142 X 2m X 5r cm

~ 12.4 cm

Perimeter = 271
18.2
=2X7TX =1 mm
~ 57.2 mm

Perimeter = 21 4 71
=2x34+4+7x34cm
~ 17.5 cm

34cm

Arc length = i X 2mr ', perimeter = 2r + arc length


360
~2x8+17.5cm
_ 125
X 2m
x 8
360 ~ 33.5 cm
~ 17.5 cm 8cm

Arc length = = X 2mr ', perimeter = 2r + arc length


360
~2x442.23 cm
—=555
32 X 2m x4
~ 10.2 cm
~~ 2.23 cm
4cm

Arc length = . X 27r . perimeter = 2r + arc length


360
~2x62+281m
— 260
=355 X 27 X 6.2 6.2m ,
~ 40.5m
~28.1m
232 Chapter 6 (Measurement) Exercise 6A

0
f Length of shorter arc = 360 < 27r

_ 150
360
X 2w
x 14 cm

~ 36.7 cm
T1dem
Length of longer arc = . X 27
360
= 138 % 27 x (14+14) cm
~ 73.3 cm
perimeter = length of shorter arc + length of longer arc + length of two ends
~36.7+73.3+2x 14 cm
~ 138 cm

3 Arc length = L X 27r


360

26=£X27r><1” 26 cm
360
26 % 360
T 36 x2m
rcm
~41.4

The radius of the circle is approximately 41.4 cm.

L Arc length = b X 27r


360

36=£><27rxr g
360
= 36 x 360
127 x 2w
~ 16.2 r mm

Perimeter = 2r + arc length


~ 2 x 16.2 + 36 mm
~ 68.5 mm

5 a Area b Area
=’ = % x 72
:flx(%)Z :%X’IFX(S‘Q)Z <|
~ 133 cm? ~ 106 mm? 8.2mm

< Area d Area


=1xm? S O g
= % X T X (3.5)2 3(20 ] 10cm
~ 9.62 m? = 355 X x 10
~ 58.5 cm?
3.5m
Chapter 6 (Measurement) Exercise 6A 233

o
8m <>
100°

0 0 2
Area = — x 772 Area = — x 71
360 360
s
=350 XT™x38 2 360 — 100
x % (9.2)?
360
~ 120 m?
~ 192 cm?

Area = —0 X TT 2
360
67
16.2 = X T X 12
360
2 16.2 x 360
67 X ™
r =~ 5.26 {as >0} rem

The radius of the sector is approximately 5.26 cm.

Area = =0 x 72
360

28.8 = 130
360
1
7 y2
5 28.8 x 360
136 x 7
r~ 4.93 {as >0} Tem

0
Now, arc length = — x 277
360
~ 136
~ 360 X 21
222 X 4.93 cm
~ 11.7 cm

Perimeter = 2r + arc length


~2x493+11.7 cm
~ 21.5 cm

a Total perimeter of track


= length of two straight segments q running track
+ length of two semi-circular ends
=2 x 500 + 27r
l¢—— 500 m —
= 1000
+ 7d
1000 + wd = 1600
wd = 600
600
d=
T
~ 191
The diameter of the semi-circular ends is approximately 191 m.
234 Chapter 6 (Measurement) Exercise 6A

b 4 minutes 25 seconds speed = dlts,t ance


1me
=4 x 60 + 25 seconds 1600 m
= 265 seconds T 265 s
~6.04 ms™?

Jason’s average speed is approximately 6.04 ms~!.

9 a In AABY, (AB)?=16%+16° {Pythagoras}


. AB=1/162+162 {as AB >0} ™. 16 mm
. AB = 16v2 mm
The circle with centre X has diameter AB, and radius ry = AX. <> Y
. AX=1xAB i
. AX =1 x16v2 mm
. AX =8V2~11.3 mm
b Perimeter of shaded crescent
= arc length of circle with centre X + arc length of circle with centre Y
=mrx+ 3 xmry
=7 x8/2+ 3 xmx16
= 8r(14+2)
~ 60.7 mm
¢ Area of shaded crescent
= area of semi-circle with centre X + area of AABY — area of quarter-circle with centre Y
=4 xmr 4+ § x base x height — 1 x 7y
=1lxmx(8V2)P
+4x16x 16— 1 x 7 x 16
= 128 mm®

Im 28 cm

a Area of lampshade
= area of sector with radius R — area of sector with radius r

360 (
9
2360
5 T ((r +0.28)? —r?)
9
=3—7T(T‘ +056r+00784—r)

= 2 7(0.56r + 0.0784)
Chapter 6 (Measurement) Exercise 6B.1 235

b Arc length = . X 27r 1m


360 /\

1=« 27r
360 -

g =350 *, Fr
27r X /

gilis0
r

0.2860
¢ Area of lampshade = ik m(2r 4 0.28) {from a}
0.28 180 .
S 032 = oo X — X 7 (2r +0.28) {using b}

= Oé—is(w +0.28)
— 028+ (0.28)2
) 2r
- 0.04— (0.28)2
T 2r
o — (0:28) 2

0.04
r=0.98 m

Now ¢ =220
r

180
T 7 %098
0 ~ 58.5

d Arc length = L 2R
360
58.5
N o X 2T X (0.98 +0.28) m
~1.29 m

EXERCISE 6B.1

8.3cm

8.3cm

The net of the cube includes six squares with side length 8.3 cm.
", the surface area = 6 x (8.3)% cm?
=413.34 em®
236 Chapter 6 (Measurement) Exercise 6B.1

1.43m 143m

067m %M G
0.92m

The net of the rectangular prism includes six rectangles: two with length 1.43 m and width
0.92 m, two with length 1.43 m and width 0.67 m, and two with length 0.92 m and width
0.67 m.
. the surface area = 2 x (1.43 x 0.92) + 2 x (1.43 x 0.67) + 2 x (0.92 x 0.67) m?
=5.7802 m*

3mm

The net of the rectangular prism includes two squares with side length 3 mm, and four rectangles
with length 5 mm and width 3 mm.
fac
", the surarea = 2e x 3) mm?
x 3% 4 4 x (5
= 78 mm?

2 a

xem
zcm

o 6cm
5cm
5 6cm
4dcm 5cm

The net of the triangular prism includes two triangles with base 4 cm and height 5 cm, a
rectangle with length 6 cm and width 4 cm, a rectangle with length 6 cm and width 5 cm, and
a rectangle with side lengths 6 cm and = cm.
Let the hypotenuse of the triangular end be = cm.
z? =5%+ 42 {Pythagoras} 5om zem
Lo =1/52442 {as z > 0}
— V41 4em

-, the surface area =2 x (3 x4 x 5) + (6x4) + (6 x5) + (V41


x 6) cm?
~ 112 cm?
Chapter 6 (Measurement) Exercise 6B.1 237

b 3.8cm

1.2cm
_

3.8 cm
1.2cm

The net of the triangular prism includes two equilateral triangles with side lengths 1.2 cm, and
three rectangles with length 3.8 cm and width 1.2 cm.
Let the height of the triangular end be & cm. hem
R? +(0.6)% = (1.2)? {Pythagoras} 12cm
h=y/122-(06)2 {as h>0}
= Vv1.08 0.6 cm
. the surface area =2 x (3 x 1.2 x VI.08) + 3x (3.8 x1.2) cm?
~ 14.9 cm?
<
10m 6m «
6m
! 12m

rm 12m

The net of the triangular prism includes two triangles with base 10 m and height 6 m, and three
rectangles: one with length 12 m and width 6 m, one with length 12 m and width 10 m, and
one with side lengths 12 m and 2 m.
Let the hypotenuse of the triangular end be x m. 10m
2% = 6% + 10 {Pythagoras}
Low=1/62+10? {as x> 0}
=136
. the surface area =2 x (1 x 10 x 6) + (12 x 6) + (12 x 10) + (12 x V136) m
~ 392 m?

3 a

2.5cm >
2.5cm

Surface area = 6 x (2.5)2 cm?


=37.5 cm?
238 Chapter 6 (Measurement) Exercise6B1

ldem
@ l4cm

Surface area = 2 x (8 x 3) + 2 x (14 x 8) + 2 x (14 x 3) cm?


= 356 cm?
<

2.8cm
10.6 cm

Let the hypotenuse of the triangular end be x cm.


2% = (2.8)% + (10.6)* {Pythagoras} .
=/(2.8)2 + (10.6)? {as x> 0} 10.6 om
=/120.2

Surface area = (6.2 x 2.8) +2 x (3 x 10.6 x 2.8) + (10.6 x 6.2) + (v/120.2 x 6.2)
~ 181 em?

4 a

20cm

The net of the pyramid includes one square with side length 20 cm, and four isosceles triangles
with base 20 cm and height 26 cm.
. the surface area = 20% + 4 x (4 x 20 x 26) cm?
= 1440 cm®
Chapter 6 (Measurement) Exercise 6B.1 239

5cm
5cm

—_—

6cm T
hcm

/i
The net of the pyramid includes one equilateral triangle with side length 6 cm, and three
isosceles triangles with base 6 cm and slant height 5 cm.
Let the height of the triangular base be x cm.
2% + 32 = 6 {Pythagoras} e
xr =162 —32 {as z > 0}
=27 .
=3V3 3cm

Let the height of the triangular sides be i cm. 3cm


h? + 3% =57 {Pythagoras} - T
h=+/5%-32
_ \/E {as h>0} 5cm hem

=4

. the surface area = (1 x 6 x 3v/3) + 3 x (3 x 6 x 4) cm?


~ 51.6 cm?

height 5.6 m <—h mAb‘

8.2m

The net of the pyramid includes one square with side length 8.2 m, and four isosceles triangles
with base 8.2 m.
Let the height of the triangles be h m.
h? = (5.6)% + (4.1)* {Pythagoras} i
h=+/(62+@A12 {as h>0} 5-6m;
= V48.17
4.1m

Surface area = (8.2)% + 4 x (3 x 8.2 x V48.17) m?


~ 181 m?
240 Chapter 6 (Measurement) Exercise 6B.1

5
; «—219cm——>

81cm

34cm
v
<«—75cm—>

a 219¢m The area of the top surface is the same as the


bottom surface. This area is a trapezium, so
total area of top and bottom surfaces
81lcm
—2x (75“19) x 81 cm?
og = 23814 cm?
b (219 —75) =144 cm x? =812 + 144> {Pythagoras}
-+
2 = 27297
= V27297 {as = > 0}
81lcm

75cm

So, the four sides are:

uen [N
8lcm 219cm 75cm
[
V27297 cm

area area area area

=81 x 34 cm? =219 x 34 cm? =75 x 34 cm? = /27297 x 34 cm?


= 2754 cm? = 7446 cm? = 2550 cm? ~ 5617 cm?
¢ The surface area = area of top and bottom surfaces + area of four sides
A2 23814 + 2550 + 2754 + 7446 + 5617 cm® {using a and b}
~ 42181 em?
~ 42181 < 10000 m?
~ 4.2181 m?
-, timber cost &~ 4.2181 m? x €128/m>
~ €539.92
~ €540
Chapter 6 (Measurement) Exercise 6B.1 241

. wall 2.5
24m $20.80
wood stain $23.60
T (for doors) 5m $15.40

The four walls are:


side back side front

2A4m - m

3.5m 4.2m 3.5m 4.2m


area area area area
=3.5x 2.4 m? =4.2x24 =3.5x 2.4 m? =4.2x24
=84 m? — 1.83 x 0.91 m? =84 m? —2.2x08m?
= 8.4147 m? =8.32 m?
The ceiling is:
area = 4.2 x 3.5 m>
35m _=14.7Tm 2

4.2m

So, total area to be painted = 8.4 + 8.4147 + 8.4 + 8.32 4 14.7 m?


= 48.2347 m?
Two coats of paint are needed, so 2 x 48.2347 ~ 96.5 m®> must be covered.
. . 16 96.5 .
One litre of paint covers i 4m?, so = -~ 24% L of paint are needed.

So, six 4 L tins and one 2 L tin of paint are needed.


. the cost of paint = 6 x $32.45 + $20.80
= $215.50
Now, the door is: 0.8m area = 2.2 X 0.8
=1.76 m?
29m Two coats of stain on both sides are needed, so
4 x 1.76 = 7.04 m? of stain are needed.
One litre of stain covers 5 m?, so % =1.408 L
of stain are needed.
So, one 2 L tin of stain is needed.
the cost of stain is $23.60.
So, total cost = $215.50 + $23.60
= $239.10
242 Chapter 6 (Measurement) Exercise 6B.1

7 a 25cm

16cm

90 cm

The surfaces of the steps are:


side back

25cm 48 cm front of step


16cm 16cm
75cm 90 cm 90 cm

area area area

=25%16 +50 x 16 =90 x 48 cm? =90 x 16 cm?


+ 75 x 16 cm? = 4320 cm? = 1440 cm?
= 2400 cm?
base

top of step
75cm

90cm 90 cm

area area

=90 x 25 cm? =90 x 75 cm?


= 2250 cm? = 6750 cm?
. the surface area = area of two sides + area of back + area of three front of steps
+ area of three top of steps + area of base
=2 x 2400 + 4320 4 3 x 1440 4 3 x 2250 4 6750 cm?
= 26940 cm?
Chapter 6 (Measurement) Exercise 6B.1 243

25m

1.8 m

The sides of the pool are:


side deep end side shallow end side
25 m 12m 12m

1.8m 1.8m

area area area


8+1.
=(18+11>><25m2 =12x1.8m?
X 2m =12x1.1m?
X 2m
=216 m =132 m
= 36.25 m*
The base of the pool is:
2% =252+ (0.7)* {Pythagoras}
12m 1.8 oo =1/2524(0.7)% {as x>0}
= v625.49
Tm

area = v/625.49 x 12 m?
~ 300 m>
-, the surface area = area of two sides + area of deep end side
+ area of shallow end side + area of base
~ 2 x 36.25 4 21.6 + 13.2 4 300 m?
~ 407 m®

The sides are:

38cm

8.5cm

area = 8.5 x 38 cm?


=323 cm?
244 Chapter 6 (Measurement) Exercise 6B.2

The ends are: 7\ area of triangle = % X 8.5 x 7.35 cm?


&vfiv, 7.35
o = 31.2375 om?
g . area of end = 6 x 31.2375 cm?
8.5cm = 187.425 cm?
-, total surface area = 6 X sides + 2 x ends
=6 x 323 + 2 x 187.425 cm?
= 2312.85 cm®
~ 2310 cm?
9 a Surface area of prism e
=2xa(x+2)+2x 2(2r) + 2 x 2z(zr+ 2) cm?
= 22(x 4 2) + 22(2z) + 4x(z + 2) cm? zem ‘
=222 + 4 + 422 + 42% 4 8z cm? 2 om
= (102° + 12z) cm? (z+2)cm

b 2 om triangular side Let the height of the triangular sides be i cm.


2
\ h* + (%) =z? {Pythagoras}
rcm
22
oo h= IZ_T {as h >0}

Fom _[
Ll <

_VE
I'el2
Surface area of pyramid = area of base + area of four triangular sides

=z xr+4x (%xzx@) cm?

— 224 290(\2/590) cm?

= 2% + /322 cm?
= (14 v3)2? cm?

34N XD
1aT °
12cm

A = 2nrh
=
+ 2mr?
-
A = 4nr?
=2x7Tx8Xx12+2x7 x 8
=4
x 7 x 3.42
~1005.3 cm? ~ 145.3 cm?
Chapter 6 (Measurement) Exercise 6B.2 245

N\
< hollow 6cm d

/"
The cone is hollow at the top, so we only | 14m |
N
have the curved surface. The diameter d = 14 m.
A=nmrs . 14
6 x 10 so the radius r—?_7m.

~ 188.5 cm? A =7mrs+mr?


=7 xTx18+7x 7°
~ 549.8 m?
e hollow top only f

O
12cm

L— 20 cm —>| 45km

The diameter d = 20 cm, Rt ii km,


e % — 10 em. so the radius r = B 2.25 km.

The cylinder is hollow at the top, so we A=dmr? 3


only have the curved surface and one =4 xmx(2:25)
circular end. ~ 63.6 km?
A =2nrh 4 mr?
=2x7x1 12+0x
7 x 102
~ 1068.1 cm?
2 A = area of curved surface + area of flat end 3cm
= % x 4ar? + 7r?
=2xm7x34+71x3%
~ 84.8 cm?
—5—

8cm 4.6m

A = 2rrh + 2712 A = 4nr?


=2Xx7TXx8x36+2x7x 8 =4 xwx(23)?
~ 2210 cm? ~ 66.5 m?
246 Chapter 6 (Measurement) Exercise 6B.2

< d

A=7rs+ar? Let the slant height of the cone be s cm.


=7 x 38 x 86 + 7 x 382 Now s? = (1.6)% + (1.2)? {Pythagoras}
~ 14800 mm? o s=+/(16)2+ (122 {as s> 0}
=Vi
=12

A = 7rs + mr?
=7 x1.2x2+7x
(1.2)?
~12.1 em?

L a The diameter d = 0.6 m,

so the radius r = % =03 m. T

A = 2nrh + 2mr? 10m


=2x7x03x10+2x7
x (0.3)?
~19.4 m?
fa—>]
0.6 m

b Cost of coating one pylon = surface area of one pylon x cost of coating per m?
~19.4 m? x $45.50/m>
~ $883.38
¢ Total cost of coating 24 pylons ~ $883.38 x 24
~ $21201

5 a s2=22452 {Pythagoras}
s=1/22 452 {as s> 0}
=v29 5m

~ 5.39

b Area of canvas = 7rs + w2 -2m


2me
=7 X2 X V2947
x 22
~ 46.4 m?
approximately 46.4 m? of canvas is required to make the tent.

¢ Cost of canvas = area of canvas X cost per m>


~ 46.4 m? x $18/m>
~ $835.24
Chapter 6 (Measurement) Exercise 6B.2 247

6 a The diameter d =8 m, _T,

so the radius r = % =4 m.
6m
Area of base = 772 l
=7 x 4% i
~ 50.3 m? l4m-»]

b Cost of lining the base = area of base x cost per m?


~ 50.3 m* x $23.20/m?
~ $1166.16
¢ Area of curved wall = 27rh
=2X7TXxX4X%X6
~150.8 m?
d Cost of lining the curved wall = area of wall x cost per m?
~ 150.8 m? x $18.50/m?
~ $2789.73
e Total cost of the lining = cost of lining the base + cost of lining the curved wall
~ $1166.16 + $2789.73 {using b and d}
~ $3960

7 a The radius is r, so the diameter is 2r. hollow top


Since the height is the same as the diameter, c and bottom
the height is also 27.
Surface area = 27rh
=2XTXTX2r
= 47r?

b The surface area is 91.6 m?


4mr? =916
2 96 4T

91.6
=\ {as >0}
~ 2.70
ha~2x270 {h=2r}
~ 5.40
So, the height of the cylinder is approximately 5.40 m.

8 a Surface area = 7rs hollow r


=T X7rX3r
= 371r? /

/
3r
248 Chapter 6 (Measurement) Exercise 6B.2

b i The surface area is 21.2 cm?


- 3mr? =21.2
5
re ==
212
3

21.2
r= {as >0}
Br
oo ra1.50
. 3r~3x1.50
s~ 4.50
So, the slant height of the cone is approximately 4.50 cm.
ii Let the height of the cone be h cm. r
h? 4% = (3r)2 {Pythagoras}
h? 4 r? =92
h? =8r? hem 3r
h= 3?2 {as h >0}
~ /8% (1.50)2 {from b i}
~ 4.24
So, the height of the cone is approximately 4.24 cm.

9 a Surface area = 277h + 277° T


=2XTXEX2+2XT
X2
= dna? + 2ma? 2zxm
= 67z? cm? l

~em]
b Surface area = $ x 477 + 7r? rem
= 2772 + 7r?
= 37r? cm?

¢ Let the slant height of the cone be s cm.


s =22 + (22)° {Pythagoras}
= 5p?
s=+hz {as s> 0}
Surface area = 7rs 4 712
=7 xaxVhr+7xa?
= Vbra? + wa?
= 72?(V5+ 1) cm?
Chapter 6 (Measurement) Exercise 6B.2 249

10 a Surface area of a sphere = 4772


The surface area is 647 cm?
4mr? = 64w
r? =16
.r=4 {as r >0}
The radius of the sphere is 4 cm.

Surface area of a solid cylinder = 27rh + 2772


The radius is 6.3 cm and the surface area is 1243 cm?
2xmx63xh+2x7mx6.3=1243
12.6mh + 79.387 = 1243
12.67h = 1243 — 79.387
1243 —79.387
h
12.67
h~25.1
The height of the cylinder is approximately 25.1 cm.

< Surface area of a cone = 7rs + 772


The slant height is 143 mm and the surface area is 60 000 mm?
T X1 x 143 4+ 7 x 2 = 60000
1437r 4 7 = 60000
7r? 4 1437r — 60000 = 0
Using technology, r ~ 84.1 or —227 but r >0 r~84.1.
The radius of the cone is approximately 84.1 mm.

1 Surface area
= surface area of hemispherical top
+ surface area of cylindrical base
=% x4m? + (2nrh+mr?)
=2xmx(12)? + 2x7x1.2x28+7x
(1.2)
~ 34.7 m?

b Surface area
= surface area of half cylindrical top
+ surface area of rectangular prism base
‘ 2m
=% x (27rh + 2mr?)
+ (2x6x24+2x12x24+12x6)
12m 6m
=1 x3x12+7x3% + (24+48+72)
~ 285.4 m?
Surface area
= surface area of hemispherical top
+ surface area of conical base
6cm = % x dnr? + 7rs
=2x7x22 + Tx2x%x6
~ 62.8 cm?
250 Chapter 6 (Measurement) Exercise 6C.1

12 Surface area of a sphere = 4772


The surface area is ~ 7.618 x 10° km?
4mr? & 7.618 x 10°
r > 7.618 x 107
47

7.618 x 107
TR {as >0}
47

~ 24600
The radius of Neptune is approximately 24 600 km.

13 a Arclength AB = 3%0 x 2 x m x radius of sector

= o X 27s
360
_ Oms
~ 180

b Arc length AB = circumference of base circle

bms _ 2nr
180
0— 3607
S

¢ Surface area of cone = area of sector + area of base circle

x ws? + mr? {using a and b}


~ 360
3607
2
= xxs® + 77
360
= 7rs + 712

4m C A 2.3cm

3m
8m 4.2cm

V = length x width x height V' = length x width x height


=8x3x4m’ =4.2x26x2.3cm®
=96 m* =25.116 cm®
Chapter 6 (Measurement) Exercise 6C.1 251

32m—>{ 1.73m?

V' = area of cross-section x length V = area of cross-section x length


=1.73x32m? =74.6 x 5 cm®
= 5.536 m® =373 cm®
e f )
138cm T
g 13.9cm
e—129em—1 >0cm i
V' = area of cross-section x length |8.2 m‘l
= (45 Xx base
base xx height)
height) x leng length V = arh
=_ (4(L x 8.6 x ?8) x 12.9 cm 3 — 7 x (8.2)% x 13.9 cm®
= 765.486 cm’ ~ 2940 cm®

l<—2.7m—>| 3m

The diameter d = 2.7 m, The diameter d = 80 cm = 0.8 m,

so the radius r = % =1.35 m. so the radius r = % =0.4m.


V =xr’h V =ar?h
=7 x (1.35)?% x 3.4 m® =7 x (0.4)* x 3 m®
~19.5 m? ~ 1.5l m*
i Let the prism have height 7 cm.
h* + 5% =132 {Pythagoras}
13cm - K2 495 =169 13cm
. 2 _
24 cm Sooht =144
10cm s h=12 {as h >0} O
5cm
Volume = area of cross-section x length
= (3 x10x 12) x 24 em®
= 1440 cm?®
a Uncooked cake Cooked cake
252 Chapter 6 (Measurement) Exercise 6C.1

b i Volume of cake mix = area of bottom of tin x height of cake mix


=7ar? xh
=7 x20%x2cm?
~ 2510 cm?
il Volume of cooked cake = area of bottom of tin x height of cooked cake
=mr’ xh
=7 x20® x (7T—1.5) cm®
~ 6910 cm?
volume increase
¢ Percentage increase = ——— x 100%
original volume
__ 6910 — 2510
x 100%
=T 2510
=175%

3 a Let the equal length sides be 2 mm.


. 3z =900
x =300
So, the end is made up of six 300 mm by
300 mm squares.
T mm
V = area of end x length 1200 mm
= (6 % 300 x 300) x 1200 mm® S00mm
= 648000000 mm?

b V = area of end X length


= ( (area rg circle
of large r=0.8m
4.6m — area of small circle) x length
=— (mR* 2 —_ 7r?).2 x length R—12m
3 = (mx 1.22 =7 x 0.8%) x 4.6 m®
L&l.(jm# ~11.6 m®
2.4m

<
‘me 2cm
5
6cm
10cm

7, 3cm
7cm
V = area of end X length
= (area of large rectangle — area of small rectangle) x length
=(7x10—3x6) x 3 cm®
=156 cm®
Chapter 6 (Measurement) Exercise 6C.1 253

L a The diameter d=1m 0. 05


.
so the radius
1
r = ke 0.5 m.
- -

j
So, the external radius of a pipe is 0.5 m.

b internal radius = external radius — width of concrete 2.5m

r=05—-0.05m
=0.45m
So, the internal radius of a pipe is 0.45 m.
< Volume of concrete necessary to make one pipe
= volume of whole cylinder — volume of hollow section
=7x(05)? x25 — 7 x (0.45)% x 2.5 m*
~0.373 m*

5 a Depth of floor = 120 mm 0.12mi—


=120+ 10 =100 m 3~
—012m 6.5m
Volume of concrete = length x width x depth 9.2m
=9.2x6.5x0.12m?
=7.176 m*®
b 7.176 m? of concrete is needed. Since concrete is only supplied in multiples of 0.2 m?,
7.2 m® of concrete will need to be ordered.
Cost of concrete = volume of concrete x cost per m*
=7.2m® x $135/m*
= $972
So, it will cost $972 to concrete the floor.

6 a b The diameter of the small circle d = 12, so the


. 12
radius r = oy 6 m.

So the radius of the large circle R=7r+1=7m.


1m
Surface area of concrete
= area of large circle — area of small circle
=aR? —mr?
=7 x 7 —7x 6
~ 40.8 m?
¢ Volume = surface area of concrete x depth
~408x 0.1 m* {as 10 cm=0.1 m}
~ 4.08 m*
approximately 4.08 m® of concrete is needed for the path.

7 a V=x0.06xg>xl
~0.06 x (3.8)% x 9.9 m*
~ 8.58 m®
254 Chapter 6 (Measurement) Exercise 6C.1

b Circumference = g
2mr =g
r= 9
27

Now, volume of cylinder = area of end x length


V=nr?xl
2
=7 X (i> x 1
27
circumference = g
g2
=7 X = x 1

=2+ xl
¢ Using the formula from b, V = Lg% x1
=4 x(3.8)?x9.9 m?
~11.4m?
. difference 0
So, the percentage difference = —— x 100%
original

~ 32.6%
& ~0.0796, so 0.08 would be a better approximation of 4= than 0.06.
The treefellers’ formula gives a slightly lower volume, indicating that not all of the timber is
usable.

Length of piping required = 1000 km


= 1000 x 1000 m
= 1000000 m
Internal diameter of piping = 13 mm
= (13 +10+ 100) m
=0.013m
internal radius = 0.013 =2 m
= 0.0065 m
External radius of piping = internal radius + wall
= 0.0065 m + 2 mm
=0.0065 m + (2 =+ 10 < 100) m
= 0.0065 m + 0.002 m
= 0.0085 m
Total volume of piping = volume of whole cylinder — volume of hollow section
=7 x (0.0085)% x 1000000 — 7 x (0.0065)* x 1000000 m*
A2 94.248 m®
Weight of plastic required = volume of plastic required x weight of plastic per cubic metre
~ 94.248 m® x 0.86 tonnes/m*
~ 81.1 tonnes
Chapter 6 (Measurement) Exercise 6C.1 255

9 a Volume of garden = length x width x depth


=86x24x0.15m* {15 cm=0.15m}
= 3.096 m*
Volume of trailer = length x width x height
=22x18x(0.6-02)m* {60 cm=0.6m, 20 cm=0.2 m}
= 1.584 m®
. . 1 f soil ired
Number of trailer loads required = e T
volume of soil per trailer load
~3.096 m3
T 1584 m3
~1.95
So, I will need 2 trailer loads of soil.
b Cost of soil = number of loads x cost per load
=2 x $87.30
= $174.60
¢ 1 Area of garden = length x width Number of loads of bark needed
=8.6 x 2.4 m? I area of garden
= 920.64 m? " area covered by one load
~20.64m?
T 11m?
~ 1.88
So, I will need 2 trailer loads of bark.
ii Cost of bark = number of loads x cost per load
=2 x $47.95
= $95.90
d Total cost of establishing garden = cost of soil + cost of bark
= $174.60 + $95.90
= $270.50

10 a Let the triangular prism have height 2 cm.


h? +75% =125 {Pythagoras}
h? + 5625 = 15625
h? = 10000
h =100 {as h >0}
Each vertical support post is 100 cm in height.

b Volume of tent = area of triangular end x length


= £ x 150 x 100 x 200 cm®
— 1500000 cm® iy
= 1500000 =+ 100° m®
— 3
piaL 75cm
¢ Total area of canvas
= area of two triangular ends + area of two rectangular sides + area of rectangular base
=2 x (4 %150 x 100) + 2 x (200 x 125) + (200 x 150) cm?
= 95000 cm?
256 Chapter 6 (Measurement) Exercise 6C.1

"ah L= e
cm rcm

5cm xcm
3cm xem

Let the height of the rectangular prism be Let the sides of the cube be & cm.
h em. B V =34.01 cm®
s ©oxxaxx=34.01
e 2= 3401
g =l = v/34.01
h=58 267 e
The side length is approximately 3.24 cm.
The height is approximately 2.67 cm.

¢ Let the radius be r cm.


V =43.75 cm®
o xr?x4.6=43.75
2 _ 4375 4.6cm
T X 4.6

r= :i'fG {as r >0}

~ 1.74
The radius is approximately 1.74 cm.

6cm

8cm

Let the height of the trapezoidal cross-section be i cm.


h?+1.5% =62 {Pythagoras}
h=4/62—1.52
=1/33.75 {as h >0}
Volume of gold bar = area of cross-section x length

480 = (#) x \/33.75 x length e


", length = 4= =
B335
~12.7
The length of the gold bar is approximately 12.7 cm.
Chapter 6 (Measurement) Investigation The volume of tapered solids 257

a V), = length x width x height


=10 x 10 x 15 cm®
= 1500 cm®

i There are n prisms with equal thickness and


total height 15 cm.
each prism has height LB cm.
n

ii From the diagram alongside, the distance from


.. 15k
the apex to the base of the kth prism is 2. cm. Bk
=2
n

(E%E> i, 15cm
Using similar triangles, =& =
10 15

o _ k
T
10k
T =
n

iii The volume of the kth prism = area of base x height


15
=T
X T X —
n
10k _ 10k _ 15
n n n

~==
1500k>

2
Yok”
3 o {using a}

i From the 6th row of the spreadsheet, we see that Volume of solid (V) is 1500 cm?®, as
calculated in a.
258 Chapter 6 (Measurement) Investigation The volume of tapered solids

il Inspecting cell B6, we see that the formula for V), is B3 x B4 x B5, which represents
base length x base width x height.
Inspecting columns F and H, and checking what each cell reference represents, we see the
following logic:
If k<n,
Vpk?
calculate the volume of the kth prism n3
, and enter it in the cell.
Otherwise,
enter 0 in the cell.
Inspecting cell B13, we see that the volume of the pyramid is approximated by summing
the volumes of the prisms calculated in columns F and H.
iii Setting n =1 in cell B10, we see in row 13 that the approximate volume of the pyramid
is now 1500 cm® = V.
iv. The volume of the kth prism = Vp,’j = % X k? = 12k% cm®.
n

The approximate volume of the pyramid when n =5 is the sum of the volumes of the
1st, 2nd, 3rd, 4th, and 5th prisms.
Vo~ 12(1%) +12(2%) +12(32) + 12(4%) + 12(5?)
~ 660 cm®
We set n =25 in cell B10 to check our answer.

v Approximate volume (V cm?)


10 577.500
100 507.525
1000 500.750
10000 500.075
100000 500.008

vi The approximate volume appears to approach 500 cm?, so we expect this to be the actual
volume of the pyramid. This is % of the volume of the corresponding solid with uniform
cross-section.

3 a Volume V_ = area of base x height


=7 x5 %10
= 2507 cm®
~ 785.398 cm®

b | There are n cylinders with equal thickness and


total height 10 cm.
each cylinder has height 1 cm.
n

10cm

|<—50m->‘
Chapter 6 (Measurement) Investigation The volume of tapered solids

From the diagram alongside, the distance from


. . 10k
the apex to the base of the cylinder is 10k cm.
n

Using g similar triangles,


gles, —&5 =~ 10 ()
r, k
5 n
5k
T = —
n

The volume of the kth cylinder = area of base x height

—rxr?x
ke n

(5k>2 10
=X |(—] x—
n n

2
=7 X 250 x -
n

Vek? 3
=——cm
3
From the 5th row of the spreadsheet, we see that Volume of solid (V) is approximately
785.398 cm?, as calculated in a.
Setting m =1 in cell B9, we see in row 12 that the approximate volume of the cone is
now 785.398 cm® ~ V.

Approximate volume (V cm?®)


10 302.378
100 265.739
1000 262.192
10000 261.839
100000 261.803
2507
iv. The approximate volume appears to approach == ~ 261.799, so we expect this to be

the actual volume of the cylinder. This is % of the volume of the corresponding solid with
uniform cross-section.
260 Chapter 6 (Measurement) Exercise 6C.2

2.8cm

V= dmd V = % (area of base x height)


— 4 %7 x (48)° em?® = %(length x width x height)
~ 463 cm® = 2(2.8 x 1.7 x 2.9) cm®
~ 4.60 em®
< d
21.6m

3.7cm

el
V= 3mrd V = £ (area of base x height)
3 12
:%xwx(%) cm?® = 3(mr°h)
3 1 18.2) 2 3
~ 26.5 cm* =gz|\mx (5 x 21.6 ) m

~ 1870 m*
e V=

f Let the diagonal of the square base


be x cm.
12cm 2?2 =82 48?2 {Pythagoras} 8cm
ooa? =128
8cm Sz =128 {as x>0}
=8v2
Let the height of the pyramid be & cm.
B + (4V/2)? = 122 {Pythagoras} .
h? 432 =144 i
B? =112 B
h=+112 {as h >0} 4y/2 cm
Now V = L(area of base x height)
=18
3
x8xV112) ecm®
~ 226 cm®
Chapter 6 (Measurement) Exercise 6C.2 261

2 a V = volume of rectangular prism 4+ volume of hemisphere 24m


= length x width x height + 1 x 477?
=46x46x12 + 2 x 1 x (%)’
~ 29000 m*

bV = volume of rectangular prism + volume of pyramid


= length x width x height + %(area of base x height)
=6x6x12 + 2(6x6x4)m’
12m
=480 m*

6m

¢V = volume of hemisphere + volume of cone


=1 x #mr® + L(area of base x height)
=2x7x5 + & xmx5 x9cm’
~ 497 cm®

3 a Volume of cylinder = mr?h

Volume of each conical end = % (area of base x height)


W=

() 1
Wl

=

i

2
W=

X
=

~0.848 m®
Total volume of tanker = volume of cylinder + volume of 2 conical ends
~10.179 + 2 x 0.848 m*
~11.875 m*
~11.9 m*
So, about 11.9 m® of concrete can be held in the tanker.

b If the ends were hemispheres, the end sections would be as long as the radius of the hemisphere.
. total length=4m+2x0.9m
=58m
262 Chapter 6 (Measurement) Exercise 6C.2

¢ The two hemispherical ends combine to make one sphere with radius 0.9 m.
Volume of sphere = 477
=3 x7x(09)°m’
~ 3.054 m*
Volume of 2 conical ends ~ 2 x 0.848 m*
~ 1.696 m®
Difference in volume of ends = volume of sphere — volume of 2 conical ends
~ 3.054 — 1.696 m®
~1.36 m®
So, the tanker could fit about 1.36 m® more concrete if the ends were hemispheres instead of
cones.
d Surface area of cylindrical part of tanker = 27rh
=2x7mx09x4m?
~ 22.62 m?
i Let the slant height of the cone be s m.
s2=124(0.9)2 {Pythagoras}
s=1/1240.9? {as s> 0}
0.9m =+v1.81m

Surface area of 2 conical ends = 27rs


=2x7x0.9x 1.8l m?
~ 7.61 m?
Total surface area of tanker
= surface area of cylindrical part + surface area of 2 conical ends
A 22.62 + 7.61 m?
~ 30.2 m?
So, the surface area of the tanker with conical ends is about 30 m?.

ii The two hemispherical ends combine to make one sphere with radius 0.9 m.
Surface area of sphere = 477>
=4 x 7 x (0.9)? m?
~10.18 m?
Total surface area of tanker
= surface area of cylindrical part + surface area of sphere
~ 22.62 + 10.18 m?
~ 32.8 m?
So, the surface area of the tanker with hemispherical ends is about 33 m?.
e The hemispherical ends allow a greater volume to be carried by the tanker. They also allow
the length of the vehicle to be shorter. However they have a greater surface area which means
they require more steel to manufacture, so they would cost more to produce. This would be a
one-off cost however, so for the permanent advantages, the hemispherical design is better.
Chapter 6 (Measurement) Exercise 6C.2 263

Let the height of the cone be h cm.


V =706 cm®
1 x 7% (12.3)% x h =706
Let the radius be » m.
50.43 x 7 x h = 706 V =73.62m?
h
706 3 xmxr®=73.62
50.43 x
~ 4.46 3 73.62
The height is approximately 4.46 cm.

The radius is approximately 2.60 m.

¢ Let the radius be r cm.


V =203.9 cm®
I xmxr?x6.2=2039
b o d . 203.9
3 XX 6.2

203.9
o A {as r >0}
3 XX 6.2

~ 5.60
The radius is approximately 5.60 cm.
<

2r

l<—2r—>l 14—27“—4

Let the height and diameter of the cylinder V (area of base x height)
ol

be 2r.
7 x 12X 2r)
wl=

V =nr’h 3
Wt

=mx7r?x2r
3
=

= 2773
Remaining volume = volume of cylinder — volume of cone
=2mr® — Zmp®
4 3
4.
_=377

which is the volume of a sphere with the same diameter, 2.


264 Chapter 6 (Measurement) Activity 1 Density

6 a i V=T-@r-h wh?
2cm > f
2 6cm
=’T>;2 (3% 6—2) cm® _{
=4z
x 16 cm®
~ 67.0 cm®

wh?
ii V=T(3r—h)
T x 32
(3x5-3)md
3

=37 x12m?
~ 113 m®
[N}

h)
|=|
w|

|
wE
o
|3
=

<

w
Il

Il

|
a
B

=
<
3

n
33

S
Il

=
W~
g
|
e

><371'T
4.3
W=

% volume of sphere
N=

This volume is half the volume of a sphere because when h = r, the cap is a hemisphere.

ACTIVITY 1
mass
1 a Density = b Volume = length x width x height
e
10g
—2x2x2cm?
B
T 2cemd =8 cm
-3
=5gem Density = ——o>
volume
_106¢g
" 8cm?
=1325gcem™
¢ Volume = %mfi
_=3 4 3 3
X7
x (4.5)° mm
~ 382 mm®

Density = mass
volume
. 103g
™ 382 mm®
~0.00270 g mm™?
Chapter 6 (Measurement) Activity 1 Density 265

. mass
2 Density =
volume
. mass
a Mass = density x volume b Volume =
density

Volume of salt = —mass of salt


3 density of salt
B 80 g
2.16 g cm—3
~ 37.0 cm®

4 1mm=0.1cm, 250 m= 25000 cm


-, volume of copper wire = 7r-2h
= x 0.12 x 25000 cm®
~ 785 cm®
mass of copper wire = density of copper x volume of copper wire
~ 8.96 g cm3 x 785 cm®
~T7040 g

5 Volume of gold bar = area of cross-section x length


N (6 +10 )><4L><22cm3

=704 cm® 4ch 22cm


10cm
density of gold = _mass of gold
volume of gold
_ 13.60 kg
"~ 704 cm3
~0.0193 kg cm™® or 19.3 gcm™3

6 Volume of steel ball = 477

-, mass of steel ball = density of steel x volume of steel ball


~8.05 gcm™3 x 11.5 cm®
~925¢g

Volume of lead sphere = %mfi


=4n(1.2)°
~7.24 cm®
', mass of lead sphere = density of lead x volume of lead sphere
~11.34 gcm™ x 7.24 cm®
~821lg

mass of steel ball __ 92.5g 1127


mass of lead sphere 82.1¢g

the steel ball weighs ~ 12.7% more than the lead sphere.
266 Chapter 6 (Measurement) Activity 1 Density

7 Density of water =1 g cm™3


Density of oil = 0.92 g cm®
Oil has a lower density than water, so oil will float on water.

. mass . . . B h .
8 a Density = e SO if a heated substance expands, its volume will increase, resulting in a
volume

decrease in density.
b Water in its solid state is ice, which floats in water.

9 Let the height of the pyramid be h m.


In AABD, BD? = 2007 + 2002 {Pythagoras}
-, BD? = 80000
*. BD = /80000 {as BD >0}
=200v/2 m
. BO=1BD=100v2m
In AEBO, 72 + (100v/2)? = 200° {Pythagoras} E
h% 420000 = 40000
h? = 20000 200m hm
h=+/20000 {as h >0}
=100v2 B 100/2m O

-, volume of pyramid = %(area of base x height)


= 2 (length x width x height)
= 1(200 x 200 x 100v/2) m®
40000002 e
3
-, mass of pyramid = density of stone X volume of pyramid
3 4 000 000v/2 2 m?
=225tm~ 3

~ 4240000 t

10 a Volume of Uranus ~ $77°


~ 4 x 7 x (2536 x 107)° m?
~ 6.83 x 10*2 m*
. fU
b Density of Uranus = I
volume of Uranus
__ 8.681 x 10%° kg
™ 6.83 x 1022 m?
~ 1270 kg m?
Chapter 6 (Measurement) Exercise 6D 267

1 a 800 mL = 800 cm® b 12 L = (12 x 1000) cm®


= 12000 cm®
¢ 46kL =4.6m? d 3200 mL = 3200 cm?

2 a 84cm®=84mL b 1800 cm® = (1800 = 1000) L


=18L
¢ 1.8m?=18kL d 7154 m® = 7154 kL

3 3.85x 10" L = 38500 L


=38.5 kL
=385 m?

L a V =length x width x height _Da


=34x1.8x21m? 21m
=12.852 m?
The tank’s capacity is 12.852 kL. 3dm 1.8m

b V=mh — =
_ 5.7\ 3 _f
=7 X ? X 2.4 m 24m

~61.2 m® i
The tank’s capacity is approximately 61.2 kL. ‘ 57m ’

¢ V = 1(area of base x heigh) [ T6m——>


= %(wr2 x h) ‘T
2
—%xwx(m) x 4.5 m® 4.5m
2
w0
o=

=
e

>
X

@=
@
@
<

8
B
o
w
X

~ 12200 cm®
Approximately 12200 cm® of soup fits in the pot.
b Capacity ~ 12200 mL
~ (12200 +1000) L
~12.2L

Approximately 12.2 L of soup fits in the pot.


268 Chapter 6 (Measurement) Exercise 6D

6 dam wall area of trapezium A = (a ;_ b)

end of . 25 4 160
catchment [25m 160m ;%r0m 5 ( ) x 170 m?
area ' —L
i =15725 m?
. a+b
170m—>Lf250m area of trapezium B = ( )

5 (80 + 160) % 9250 m?

= 30000 m?
Total surface area of the reservoir = 15 725 + 30000 m?
= 45725 m?
V = area of cross-section x depth
= 45725 x 13 m*
= 594425 m®
The capacity of the reservoir is 594 425 kL.

T b T

15em 4.1m
4.5cm l
& area of base = 1.2 m?

V =nr?h .
=7 x (45)2 x 15 om® V = area of base x height
b s =12x41m
~ s 3
The capacity of each tin is = . - .
approximately 954 mL. The capacity of the mixing vat is
4.92 kL.

capacity of vat
¢ Number of tins to be filled from one vat =
capacity of one tin
4.92KL
~ 954 mL
(4.92 x 1000 x 1000) mL
Q

954 mL
4920000
~
954
=~ 5155.8
So, 5155 tins could be filled from one vat.
d Value of one vat of jam = number of tins x cost per tin
= 5155 x $3.50
= $18042.50
Chapter 6 (Measurement) Exercise 6D 269

120 mm

160 mm

160 mm
120 mm

External surface area = 2 x (120 x 88) + 2 x (160 x 88) + 2 x (160 x 120) mm?
= 87680 mm?
b It is useful to specify the “external” surface area when talking about a container as the external
surface area may be different from the internal surface area.
¢ i The walls of the box are 4 mm thick.
the internal length, width, and height of the box are 120 — 2 x 4 = 112 mm,
160 — 2 x 4 =152 mm, and 88 —2 x 4 =80 mm respectively.
internal volume of box = internal length x internal width x internal height
=112 x 152 x 80 mm®
= 1361920 mm®
The box can hold 1361920 mm?® of jewellery.
i Capacity of box = (1361920 =+ 10°) mL {1 cm® = 10° mm®}
=1361.92 mL

iii Volume of wood used to make the box


= total volume of box — internal volume of box
= external length x external width x external height — 1361920 mm?® {from ¢ i}
=120 x 160 x 88 — 1361920 mm?
= 1689600 — 1361920 mm®
= 327680 mm®

9 10kL = 10m?
Volume of pond = area of base x depth
=ar?h
10 =7 x (24)? x h
10
™ x (2.4)2
~ 0.553
The pond is approximately 0.553 m (or =~ 55.3 cm) deep.
270 Chapter 6 (Measurement) Exercise 6D

10 a The area of the roof is in m?, so we convert 12 mm to metres.


12 mm = (12 +1000) m = 0.012 m
The volume of water which fell on the roof = area of roof x depth
=110 x 0.012 m*
=1.32m?
b 1.32m® =1.32 kL, so 1.32 kL of water entered the tank.
¢ The volume added to the tank = area of base x height
=7x22x hm®
=47 x hm?®
The volume added to the tank must equal the volume
which falls on the roof, so
47 x h=1.32
1.32 =" .
h= o {dividing both sides by 47}
7T

h~0.105 m
The water level rises by about 10.5 cm.

11 Original tin: V = 7r?h original tin


7.2\ 2 3
=7 X (7> x 15 cm
_9m
5 15cm
New tin: = % cm®
2
e (1_0> xh =22
2 5
972
= 5 x 52
=7.776 cm
The height of the new tin must be about 7.8 cm.

12 a V = (arca of base x height)


8.6 2 :
=%X7r><(;) x 13 cm® T
2
i 24037 cm® 13cm
300
~ 252 cm® l
The capacity of the glass is about 252 mL.
b i When 75% full, the glass holds approximately
252 x 0.75 ~ 189 mL of wine.
o 24037

g —
ii V= 7 % 0.75 cm® 8.6 cm
2
Lo (fi)
2
xh=2207 300
075
24037 x 0.75
bR (4.3)2 Ih
=3.25 cm
The wine will rise 3.25 cm.
Chapter 6 (Measurement) Activity 2 Minimising material 271

13 hollow «—3m—
5m
3m T

/ 2m 1.9m
2.9m

I
mark
10cm
below
top

Subtracting the 10 cm line from the top, the end of the container up to the level which can be filled
looks like the diagram shown.
Area of end = 2 X area of trapezium

—2x ((1'9“2“2'9) x 1.5) m?


=7.2m?
Volume of wheat in a container = area of end x length
=72x5m?
=36 m®
Volume of cylindrical silo = wr2h C D T
=rx42x25m?
~ 1256.64 m>
volume of silo l
Number of truck loads = ——————
volume of container
1256.64
~ 736 l«~4m
~ 34.9
So, 35 truck loads are needed to fill the silo.

ACTIVITY 2

1 a Volume = length x width x height


=2rXxT XYy yem
s V=21
b The container must hold exactly 1 litre of fluid. s
1L = 1000 cm® 2zem
222y = 1000

¢ 22%y=1000
z?y = 500
e 500

2 Surface area = 2 X (area of longer rectangular ends) + 2 x (area of shorter rectangular ends)
-+ area of bottom
=2x 2z xy)+2x (zxy)+2zxw
= day + 2xy + 22°
. A=22"+6xy
272 Chapter 6 (Measurement) Review set 6A

3 A B C
1 xvalues yvalues A values
2 1 500 3002
3 2 125 1508
4 3 55.555556 1018
5 4 31.25 782
6 5 20 650
7 6 13.888889 572
8 7 10.204082 526.5714286
9 8 7.8125 503
10 9 6.1728395 495.3333333
11 10 5 500

L4 The smallest value of A is ~ 495.33, when z = 9.


500 6.17cm
When z =9, y=9—2’\'6.17

The dimensions of the box that your boss desires are 9cem
shown alongside. 18cm

1 a Arclength= L X 27r
360 10 cm
105
360
X 21
x 10 cm
~ 18.3 cm

b Perimeter = 2r + arc length


~2x10+18.3 cm
~ 38.3 cm
0
¢ Area = — X 77°
360
_= 105
555 X ™ x 10 2

~ 91.6 cm?

0
2 Area = — x 712
360

2Ur = 20wy
360
2 24rw
™ =30 rem
360 X
r =108 {as r >0}
~ 10.4
The radius of the sector is approximately 10.4 cm.
Chapter 6 (Measurement) Review set 6A 273

a hollow top b
- and bottom .

"
10cm

l Surface area = 4772


=4 x 7 x (5.2)? cm?
Surface area = 27rh ~ 339.8 cm?
=2x7x6x10 cm?
~ 377.0 cm?

¢ Surface area = 7rs + 772


=7 x4x12+7
x 4% cm?
~201.1 cm?

mplN
2.5m 5m
3m

Surface area of shed


= area of 2 rectangular ends + area of 4 rectangular sides + area of 2 triangular ends
=2x(3x25)+4x (5x25)+2x (3 x3x2)m?
=7lm?
b Since the shed is to be painted with two coats of zinc-alum, we need enough zinc-alum to cover
an area of 2 x 71 = 142 m%.
The zinc-alum covers 5 m? per litre.
total area to be painted
Number of litres of zinc-alum needed = -
area covered per litre
142 m?
5m2 L1
=284L
Since the zinc-alum must be purchased in whole litres, we need to purchase 29 L of zinc-alum.
Total cost of the zinc-alum = number of litres to be purchased x cost per litre
=29 L x $8.25/L
= $239.25
274 Chapter 6 (Measurement) Review set 6A

5 Total area of netting


=6 x (27rh + 7r?)
2
:6><(2><7r><%><17+7r><(%) ) m? 17m

~ 2668.4 m?
So, the total area of netting in the cages is approximately
2670 m2. «—T75m—>

3.2m

V = area of end X length = area of end X height


=1x24x13x32m f85.3x100m
= 4.992 m® = 853 om?
~ 4.99 m®
¢V = volume of external cylinder — volume of internal cylinder
=7 R*h — 7r2h
2 2
=X (E) x1.2 — 7mx (%> x 1.2 m?
2 2
~ 0.452 m®

7 Volume of cone = % (area of base x height)


=1 3 x7x(1.6)*x1.2m? 1.2m
~3.22 m® ‘
Tom has had approximately 3.22 m* of sand delivered.

8 245 L = (245 x 1000) mL = 245000 mL


c total amount of molten iron
Number of spikes made = ——8M——————
amount of iron in each spike
245000 mL
~ 15mL
~ 16333.3
So, 16 333 spikes can be made.

9 Volume of sphere= 477°


=3 x7x27
cm®
~ 82400 cm®
27cm
So, the beach ball has volume of approximately 82 400 cm?.
Chapter 6 (Measurement) Review set 6A 275

10 a 65L = (65 x 1000) mL b 65 L = (65+ 1000) kL


= 65000 mL = 0.065 kL
65000 cm? of petrol is required to . 0.065 m* of petrol is required to
fill the tank. fill the tank.

11 a V = length x width x height


=122x86 x 7 cm® 7 cm
=734.44 cm®
The capacity is 734.44 mL.
8.6 cm
12.2 cm

b V = area of end X length 60 cm


1 2
= 5 x mr” x length 1.4m

=1xmx ()% 140 em® {14 m = 140 em}


~ 198000 cm®
The capacity is approximately 198 000 mL or 198 L.

12 The dimensions of the roof are in m, so we convert 15.4 mm to metres.


15.4 mm = (15.4 +1000) m = 0.0154 m
The volume of water collected by the roof = area of roof x depth
=12 x 5.5 x 0.0154 m*
=1.0164 m*
The volume added to the tank = area of base x height
2 .
=7 X (4—35) x h m?
= 4.62257 x h m®
The volume added to the tank must equal the volume
which falls on the roof, so
4.6225m x h = 1.0164
10164
T 4.62257
h =~ 0.0684 m
So, the level in the tank rises by about 68.4 mm.

13 a Height of cone
= total height of silo — height of cylinder
— height of hemisphere _T

—33-18-1%n 33m 1.8m


2
=0.7m l
=70 cm

Ll.BmJ
276 Chapter 6 (Measurement) Review set 6A

b Let the slant height be s m. 0.8m


52 =(0.7)% + (0.8)* {Pythagoras}
s =1/(07)2+(0.8)2 sm
~ 1.06 {as s> 0}
So, the slant height of the cone is approximately 1.06 m.
¢ Surface area of hemisphere = %(surface area of sphere)
= $(4nr?)
=3 x7x(0.8)%) m?
~4.02 m?
Surface area of cylinder = 27rh
=2x7x0.8x
1.8 m?
~ 9.05 m?
Surface area of cone = 7rs
~7 x 0.8 x 1.06
~ 2.67 m?
.. total surface area ~ 4.02 + 9.05 + 2.67 m>
~15.7 m?
So, approximately 15.7 m? of steel is used to make the feed silo.

d Volume of hemisphere = %(%777‘3)


=4x4x7x(08)°m?
~1.07 m?
Volume of cylinder = 772h
=7x(08)?%*x1.8m3
~3.62 m*
Volume of cone = & (7r°h)
=1 x7x(08)?x0.7m?
~ 0.469 m*
volume of silo = volume of hemisphere + volume of cylinder 4 volume of cone
~ 1.07+ 3.62 + 0.47 m®
~5.16 m*
~52m
So, the silo can hold about 5.2 m® of grain.
e 52m®=52kL
So, the capacity of the silo is about 5.2 kL.
Chapter 6 (Measurement) Review set 6B 277

G40 )

1 a Arc length = e x 27
360
0
6.18 = = x 27 X 4.62 6.18 cm 4.62 cm
0= 6.18 x 360
27 X 4.62
0° ~ 76.6°

b 0
Area:3—><7rr 2

0
a Length of shorter arc = =0 X 27r
_=
1203 x5 cm
2m 0
X5
~~ 10.47 cm

Length of longer arc = % X 2mr e


T 1200 N
120 5cm
=555 X 27 x (5+2)
cm
~ 14.66 cm

Perimeter = length of shorter arc + length of longer arc + length of two ends
~ 10.47 +14.66 + 2 x 2 cm
~29.1 cm

b Area of larger sector = 3;2.0 x 7r?

=10
x 7 x (5+2)% em?
~ 51.31 cm?
0
Area of smaller sector = — X 772
360
—=350
120 X ™ X5 2 em?2
cm
~ 26.18 cm®
Area of figure = area of larger sector — area of smaller sector
~ 51.31 — 26.18 cm®
~ 25.1 cm?
278 Chapter 6 (Measurement) Review set 6B

3 a

3.6cm

Let the height of the triangular end be h cm. f T


h?+1.8% = 3.6 {Pythagoras} 3.60m hem
h=1361-18 {as h>0} N
~ 3.12 1.8cm

Surface area = area of two triangular ends + area of three rectangular sides
~2x
5 x3.6x312 + 3x6.8x 3.6 cm’
~ 84.7 cm?
b

74mm @

Surface area = 27rh + 277>


=27x (%) x 74 + 27 x (2£)” mm?
~ 7110 mm?
<
2.0m

2.2m 1.3m

Let the height of the triangular side with base 2.2 m be d m. T


d?+1.12 =2.0 {Pythagoras} 2.0m dm
L d=+v202-112 {as d>0} l
[1
~ 1.67 11m

Let the height of the triangular side with base 1.3 m be h m. A


h* 2 +0.65 2 _=2.0 902 {Pythagoras} S0 .
h=1+/2.02-0.652 {as h >0} l
~ 1.89 0
0.65m

Surface area of pyramid = area of base + area of two triangular sides with base 2.2 m
+ area of two triangular sides with base 1.3 m
~22x 13 + 2x4x22x167 + 2x1x1.3x1.8)
m?
~ 8.99 m?
Chapter 6 (Measurement) Review set 6B 279

The panelling for the gazebo includes 6 interior and


6 exterior triangular panels for the roof, 5 interior and
5 exterior rectangular panels for the walls, and one
hexagonal panel for the floor.
Total surface area
= roof area + wall area + floor area
=2x6x(:x1.2x15)+2x5x (1.2x0.75)
+6 % (& x 1.2 x 1.04) m? 0751“1
=23.544 m?
~ 23.5 m?

Length of cylinder is 325 mm = 32.5 cm,


40
and radius of cylinder is 4—20 =20 mm = 2 cm. _I mm
fe——— 325 mm ———»|
Surface area of cylinder
= 2nrh + 2712
= (2x 7 x2x325)+
(2 x 7 x 2%) cm?
~ 434 cm?
So, approximately 434 cm? of bubble wrap is needed to line the cylinder walls.

6 a V = 13 (area of base x height)


=1x7x(44)? x81em’ ‘
~ 164 cm® 8.1cm

L—/
4.4cm

b 3m V = area of end X length


=(Bx5-Lix2x1)x5m®
4m B—24><5'm 3

=120 m*
5m
5m

¢ V=%71'7"3
4 X T X (27) mm3

~ 10 300 mm® ‘ 7
280 Chapter 6 (Measurement) Review set 6B

7 a The front face of the letter F is made up of 3 rectangles. 0.5m


Volume of letter F \‘/'\
= area of front face x length 0.4m
= (1.7x0.4+0.4x0.4+0.5x 0.4) x 0.5 m?
=1.04 x 0.5 m® .
=0.52 m? 1.7m 0.4m
Frank will need 0.52 m® of plastic. 1
b Surface area of letter F
= 2 x area of front face + 2 X area of side face
+ area of top horizontal faces le—
04m 0.4m0.1m
+ area of bottom horizontal faces
2% 1.04) 4+ (2 x 1.7 x 0.5) + [(0.9 x 0.5) + (0.5 x 0.4)]

F (0.5 % 0.5) + (0.5 x 0.4) + (0.5 x 0.4)] m*


2.08+1.7+0.45+0.2+0.25+ 0.2+ 0.2 m?
=5.08 m?
Frank will need 5.08 m? of fibreglass.

8 Volume of igloo T
;
_ 11 (volume of cylinder) + $(volume of sphere)
25) 4 R3 l
= 5(mr*h) + 3(57R?) O.SnT
=3x7x(05)%x11 + & x4 x7x(13)?>m’
1.1m
~5.03 m®
So, the volume of the igloo is approximately 5.03 m®.

9 Available volume of bench = total volume of bench — volume of sink


= 3845 x 1260 x 1190 — 750 x 550 x 195 mm?
= 5684755500 mm?
= (5684755500 = 10%) cm?®
= 5684 755.5 cm®
-, capacity = (5684 755.5 + 1000) L
= 5684.7555 L
~ 5680 L
The bench has a storage capacity of approximately 5680 L.

10 a V = volume of cone + volume of hemisphere 51 mm


L Fy it 4.3
3t + 3 (3w )
Nl(}\

1 xmx ( 512 )3 mm?


ol

ol
Cfl

X
I
X

X

5

~ 133464 mm?*
145 mm
~ (133464 + 10%) cm?®
~ 133.464 cm®
~ 133 cm®
So, approximately 133 cm? of gelato is sold with each cone.
Chapter 6 (Measurement) Review set 6B 281

b 10 L = 10000 cm?
Number of cones which can be filled = fotal volume of gelato
volume of gelato per cone
10000 cm?*
™ 133.464 cm?
~ 74.9
So, 74 full cones can be sold from 10 L of gelato.

1 10 kL =10 m?
Volume of cylindrical drum = 10 m?
ar’h =10
mx 1 x3=10
r? = E
37

r=,2 {as >0}


3
~ 1.03
So, the radius of the drum is approximately 1.03 m.

12 a Surface area of the Sun ~ 4772


~ 47 x (6.955 x 10%)% m?
~ 6.08 x 10*® m?
The Sun’s surface area is approximately 6.08 x 10'® m?.
b Volume of the Sun ~ 7m7°
~ 47 x (6.955 x 10°)* m®
~ 1.41 x 10" m3
The Sun’s volume is approximately 1.41 x 1027 m®.

13 a Volume of hemispherical top = % X %71’7'3


]
=3 7 x 3% em®
=187 cm®
~ 56.5 cm®
b Volume of cone-shaped base = 51 x area of base x height
=3 xmr’h
_1 17 x 3% x h em?
= 37h cm®
Now, volume of cone = 3 x volume of hemisphere
3rh = % x 187
3h=9
h=3
So, the cone-shaped base has height 3 cm.
282 Chapter 6 (Measurement) Review set 6B

< 3cm Let the slant height of the cone-shaped base be s cm.
s =3%+3> {Pythagoras}
3cm
o s=V18 {as s> 0}
scm s 3\/§

Outer surface area of spinning top


= surface area of hemispherical top + surface area of cone-shaped base
=i xdam? + ars
=27 x3% + Tx3x3V2
~ 96.5 cm?
Chapter 7
RIGHT ANGLED TRIANGLE TRIGONOMETRY

| Y,

O
.

1 Each triangle has a right angle, an angle of 37°, and hence a remaining angle of
180° — 90° — 37° = 53°. So, the triangles are all equiangular and hence similar.

HYP | HYP | 4DJ


2

30
ailw

als

el

B
wlw

cls

C
alw

gl

D
clwe

als

OPP ADJ OPP


3 All of the triangles are similar, so the ratios between corresponding side lengths —, —, —
HYP” HYP’ ADJ
are constant.
284 Chapter 7 (Right angled triangle trigonometry) Exercise 7A

1 a b
o 5cm 12cm
3m
4m 13cm

i sinf=—=—
OPP 4 T
i sinf=—"—2
OPP
=2212
HYP 5 HYP 13
. ADJ 3 i ADJ 5
il cos=—== il cos=—=—
HYP 5 HYP 13

=n
fii ta X~g
ADJ
2 3
=n
i ta 2P
ADJ
g— 12
5

< O
15cm . Sem

17cm

i sinezfl:E ii Cogg:fl,E jii tanezflf—


HYP 17 HYP 17 ADJ s T
d Let the unknown side be = cm.
z°+5°=8
2 2 _ Q2
{Pythagoras} 8cm 5em
2% +25 =64
2
z” =39 zcm
x =39 {as x>0}

i sing=2HYP
2P_5 8 il cosg = ADL
HYP
_ ¥39
8
= 2T
fii ta n
ADJ

g 5
e Let the unknown side be x cm. 7em
22 +42 =172 {Pythagoras}
2 +16 =49 zcm 4cm
2% =33
r =33 {as x>0}

i sinQ:%:@ ii cost9:fl:é jii tanezflf—


HYP 7 HYP 7 ADJ

f Let the unknown side be = cm. Yo


2% =7+ 47 {Pythagoras}
a? =49 416 7em e
2% =65 i
x =65 {as z > 0}
1B
i sinf=—-=—
OPP 7 .
ii
ADJ
50 = — = —
4
jii tanf=—=
OPP
i
- HYP /65 €08 HYP /65 - ADJ a
2 a sin20° = 0.342 b sin76° ~ 0.970 ¢ cos27° =~ 0.891
d cos43° ~ 0.731 e tan32° ~ 0.625 f tan70° ~ 2.747
Chapter 7 (Right angled triangle trigonometry) Exercise 7A 285

a X 49cm b i sinb6°=—-—~_—~0.83

ii cosh6° = — ~ == =~ 0.56

3.3cm jii tan56° = — ~ — ~ 1.48


5.9cm ADJ 3.3

zZ

< i sin56° =~ 0.83 il cosH6° = 0.56 iii tan56° ~ 1.48

a [PQ] is longer than [QR]. R

b c0s23° = ADI 0
HYP PR
OPP QR
and sin23° =
HYP PR
If PQ>QR, then PQ
=< > QR P 0 Q
c0s23° > sin 23° Check: 0.921 > 0.391 v

c tan23°:%:%
ADJ PQ

But QR<PQ, -. QR3 <1


. tan23° <1 Check: 0424 <1 v

a Let ABC = 6.
Base angles of an isosceles triangle are equal. A
BAC
=0
ABC + BAC +90° = 180° {angles in a triangle}
0+6=90° im
.20 =90°
L 0=45°
So, ABC = 45° b
b AB%? = AC?+BC? {Pythagoras}
— 12412
=2
. AB=+2 {as AB
> 0}
~ 141m

c
¢ owoako _ OPP
1 Sil ko ADI1 ~ 0.707
i sin ——4h HYP—fi~0.7O7 cos4b “ mr s

OPP l
jii tan45°=—=-=1
ADJ 1

d sin45° = 0.707, cos45° ~0.707, tan45° =1


286 Chapter 7 (Right angled triangle trigonometry) Exercise 7B

6 The hypotenuse of a right angled triangle is always the longest side of the triangle.
the opposite and adjacent sides will always be shorter than the hypotenuse.

So, sinf = e and cosf = ADL i always be less than or equal to 1.


HYP HYP

A PP ADJ b
7 a i s1nA:O—:2 il cosA=—=-
HYP c HYP c

iii tanA= St iv sinB = opp _ b


ADJ b HYP c
AD. . PP
\lcosB:—J=2 vi tanB:O—zE
HYP c ADJ a

b A+ B+90° = 180° {angles in a triangle}


A+ B =90°
. A=90°-B
¢ Let §=B.
i sinB=2=cosA {using a} i cosB=2=sinA {using a}
c c

. sinB = cos(90° — B) {using b} oo cos B=sin(90° — B) {using b}


sinf = cos(90° — ) . cost = sin(90° — 6)
b 1 1 .
iii tanB = == ® ey {using a}

1 .
. tanB = n(90° —B) {using b}
1
e tan(90° — 0)

1 a opp A 2
x ADJ ADJ H,ZP

xr

k 1
HYP OPP

The relevant sides ; are HYP and OPP, so The relevant sides are ADJ and HYP, so
we use the sine ratio. we use the cosine ratio.
OPP ADJ
8

sin 21° = { sinf = —


HYP} cos 50 o2
i {cos s @ = —HYP}
>
Chapter 7 (Right angled triangle trigonometry) Exercise 7B 287

d ADJ A
a

‘P
xT

HYP
ADIJ
The relevant sides are ADJ and HYP, so
The relevant sides are ADJ and OPP, so we use the cosine ratio.
we use the tangent ratio. ADJ
PP c0s56° =2 {cosh = ==}
tan38° = Z {tand = o x HYP
t ADJ

ADJ HYP
x n
O]

OPP ADJ
HYP B xT

> opp
The relevant sides are ADJ and OPP, so The relevant sides are HYP and OPP, so
we use the tangent ratio. we use the sine ratio.
. . PP
sin36° =< {sinf = orp
n HYP

xcm 0 HYP
ADJ
gnad 11m
8cm
HYP
OPP
The relevant sides are HYP and OPP, so
we use the sine ratio. The relevant sides are ADJ and HYP, so
we use the cosine ratio.
sin61°= 2 {sin@zfl} ADJ
8 HYP cosd7° = = {cosf = ==}
8 X sin61° = 11 HYP
11 X cos47° =
x =~ 7.00
x &~ 7.50
So, the side is about 7.00 cm long.
So, the side is about 7.50 m long.

ADJ A OPP ADJ a


3cm zcm
OPP
2

4cm

HYP zem
HYP
The relevant sides are ADJ and OPP, so
we use the fangent ratio. The relevant sides are HYP and OPP, so
we use the sine ratio.
tan69° =< {tanf = fl}
3 ADJ
3 x tan69° = x sin56° = 2 {sinf = fl}
T HYP
T~ 7.82 T X sinb6° =4
So, the side is about 7.82 c¢m long. . 4
" sin56°
T~ 4.82
So, the side is about 4.82 c¢cm long.
288 Chapter 7 (Right angled triangle trigonometry) Exercise 7B

f HYP
48° HYP zcm

5m
ADJ OPP
0 Tm 7cm
OPP & ADJ

The relevant sides are ADJ and OPP, so The relevant sides are ADJ and HYP, so
we use the fangent ratio. we use the cosine ratio.
OPP
tan48° =2 {tanf = —} cosT1® =7 {cosf = fl}
5 ADJ T HYP
5 x tan48° = x T XcosTl® =T
x ~ 5.55 Tr=
7
cos 71°
So, the side is about 5.55 m long.
x~21.5
So, the side is about 21.5 cm long.

9 OPP h HYP
rcm 8.8m
[
ADJ zm
8cm OPP
HYP 0 ADJ

The relevant sides are HYP and OPP, so


The relevant sides are ADJ and OPP, so we use the sine ratio.
we use the tangent ratio. T OPP
OPP sin36° = {sinf = —}
8.8 HYP
tan67° =< {tanf = —}
8 ADIJ 8.8 x sin36° =z
8 x tan67° = x r~5.17
z~ 188
So, the side is about 5.17 m long.
So, the side is about 18.8 cm long.

HYP ADJ OPP


rm 3.29cm
ADJ
[

7.6m Tem
OPP HYP

The relevant sides are ADJ and OPP, so The relevant sides are ADJ and HYP, so
we use the tangent ratio. we use the cosine ratio.
tan50° = -8 {tanf = %} 3.29 ADJ
T ADJ cos47® = —— {cos = —1}
x HYP
z X tan50° = 7.6 T X cos47° = 3.29
7.6 3.29
" tan50°
Tr=

cos 47°
T~ 6.38 T~ 4.82
So, the side is about 6.38 m long. So, the side is about 4.82 cm long.
Chapter 7 (Right angled triangle trigonometry) Exercise 7B 289

k HYP HYP
Trm

7.22cm
ADJ Zem
OPP
© OPP 9.01m
AD] ©
The relevant sides are ADJ and OPP, so
we use the tangent ratio. The relevant sides are ADJ and HYP, so
T OPP we use the cosine ratio.
tan45° = — {tanf = —} .01
7.22 ADIJ cos78° = 2.0t {cosf = %}
7.22 x tan45° = x €T

=722 . x X cosT8 =9.01


So, the side is 7.22 cm long. 9.01
LT =
cos 78°
Loz ~43.3
So, the side is about 43.3 m long.

a tan53° =2 {tanf = @} HYP


3 ADJ ym
3 x tanb3° =x
Loz~ 3.98 3m
ADIJ
rm
0 OPP

. AD
b i y? =32 +22 {Pythagoras} i cos53° = {cosb = m}J

|w
y? ~ 9+ 3.98°

<@
.y xcoshd® =3
Yy~ V24.85 {as y > 0} 3
y ~ 4.98 cos 53°
y ~ 4.98

a sin35° =% 5 {sinf
= 2r}
HYP HYP
5 xsin3h® =z 5m
rm

r =~ 2.87 OPP
]
c0s35° =2 fcosh = fl} ym
5 HYP ADJ
5xcos35° =y
y~4.10

b tan64° =2 {tanf = %} OPP a


8 ADJ Tm ADJ
8 x tan64° = x 8m
o~ 16.40
ym
cos64° = 2 {cosf = fl} HYP
v HYP
Yy X cos64° =8
i 8
" cos64°
y ~ 18.25
290 Chapter 7 (Right angled triangle trigonometry) Exercise 7B

<
o 97
tan42° = —
_
{tan = i opP
[T 9.7
. x X tand2® = 9.7 ADJ e
9.7 T cm
T = >
tan 42 yom
r =~ 10.77 HYP

sind2° = 27 {sinf = fl}
y HYP
oy Xxsind2° =9.7
97
sin42°
oy~ 14.50

AB
5 a tan50° =
.
o {tan@ = orp
o
. 6.2 x tan50° = AB
6.2cm
. AB~7.39 cm
6.2 ADJ
cosb0® = — {cosf =—1} C
BC HYP
BC X cos50° = 6.2
6.2
BC =
cos 50°
BC ~ 9.65 cm
Perimeter of triangle ABC = AB + AC + BC
~ 7.39+ 6.2+ 9.65 cm
~ 23.2 cm
Area of triangle ABC = x base x height
Nl

x AC x AB
W=

~ x 6.2 x 7.39 cm?


W=

~ 22.9 cm?
b In AABD, BAD = 180° —90° — 62° {angles in a triangle}
" BAD = 28°
In AABC, tanBAC = e {tanf = @}
AB ADJ

" tan28° = % {BAC = BAD}


AB x tan28° = 3.4
34
" tan28°
. AB=6.39 cm

Also, sin28° = 34 {sinf = %}


AC HYP
. AC x sin28° =34

Al sin 28°
AC =~ 7.24 cm
Chapter 7 (Right angled triangle trigonometry) Exercise 7C 291

Perimeter of triangle ABC = AB + AC + BC


~6.39+ 724434 cm
~ 17.0 cm
Area of triangle ABC = % x base x height
=1 xABxBC
1%
2
6.39 x 3.4 cm?

Q
~ 10.9 cm?

In AACD, cos31° = 5 {cosf = fl}


AC HYP

AC x cos31° =5

AC= 2
cos 31°
. AC~5.83 cm

In AABC, AB? +BC? = AC? {Pythagoras}


AB? + 4% ~ 5.83%
. AB~1/5.832 42 {as AB >0}
AB =~ 4.25 cm

In AACD, tan31° =L5 {tang = Xy


ADJ
5 x tan31° = CD
. CD =~ 3.00 cm
Perimeter of quadrilateral ABCD = AB + BC 4+ CD + AD
~4.254+443.00+5 cm
~ 16.2 cm

Area of quadrilateral ABCD = area of triangle ABC + area of triangle ACD


=3 XABxBC + x AD x CD
~ 5 X425 x4 + £ x5x3.0cm?
0
~16.0 cm?

b n
HYP
fe 3m opp
ADJ 5cm
7cm
HYP

cosf =2 {cos&:fl} sing = 2 {siné’:%}


5 HYP 7 HYP
L . 0 p—=cos ena—l(3
(3) L. p_wn—l(B
B =sin (7)
. 0 ~=53.1° oo 0= 45.6°
292 Chapter 7 (Right angled triangle trigonometry) Exercise 7C

<

2.3m
6m
PP
OPP o 3.4m
HYP
Tm
ADJ sing = 23 {sinf = fl}
3.4 HYP
tanf = B {tanf = %} 0 =sin~! (%)
i
6 = tan (£) 0 ~ 42.6°

0 ~ 40.6°

e OPP
1m ADJ HYP
2.5cm 4.1cm
Q

4.1m &
ADJ

tan@zfi {tan@zfl} cosf = 22 {cosezfl}


4.1 HYP
0 =tan"! (fi) 0 = cos* (%)
0~ 13.7° 0 ~ 52.4°

S
itHYP h ADJ
2cm

9.9m
OPP 4cm
HYP
9.9
sinf
= — {sinf
= —}
10.2 HYP
cosd = 2 {COS@ZE}
0= sinfl(%) 4 HYP

0~ 76.1° 0 =cos™! (%)


0 = 60°

1.1 m =110 cm}


Chapter 7 (Right angled triangle trigonometry) Exercise 7C 293

ADJ
a cosf=2 {cosf
= —}
9 HYP 0
. 0=cosT'(2) sm Im
. 6 =56.3°
b ¢

b i 90°+60+¢=180° {angles in a triangle} il sing = g {sin¢g = %


o ¢~ 180° —90° — 56.3° ing a s
. ’ ~ 33.7° e ¢=sin”'(5)
3. ~ 33.7°
a b 7.2cm
¢ 13m .

0 9 5.8cm
10m

10 ADJ
cosf=— {cosf=—1} 5.8 OPP
13 HYP tana = — {tana=-—}
o - (m) 7.2 ADJ
= cos 13 a =tan~! (fi)
0~ 39.7° e
sing=—
10. opPP
{sing=——1}
et7.2 OPP
13 HYP tanf =-—= {tanf =—}
6= sin-1 (fl) 5.8 ADJ

) . B=tan"'(Z3)
¢~ 503 . Br511°
¢ cosf=2L {c s€=fl}
HYP

6 =cos! (%)
0~ 61.5°
) 2.1 . OPP
sm¢7fl {smqfiffi}

6= s~ (34)
¢ =~ 28.5°

a sinf=2 {sin@:fl}
5 HYP
6 =sin"'(g) which is undefined OPP }?r(np
This triangle cannot be drawn with the given dimensions. il
(In any right angled triangle, the hypotenuse is the longest
side.) b

b cosf=22 {cosézfl}
7.4 HYP

. §=cos™!(8}) which is undefined


This triangle cannot be drawn with the given dimensions.
(In any right angled triangle, the hypotenuse is the longest
side.)
294 Chapter 7 (Right angled triangle trigonometry) Exercise 7D

< cosé’:E {Cosezfl}


9.3 HYP Y

. 9=cos_1(%) 9.3m
. 60=0°
. . i . 9.3m
The resultant figure is not a triangle, but a straight line of HYP
length 9.3 m.

5 a 2°+32=47 {Pythagoras}
L a?+9=16
27 4cm

z=V7 {as x>0} L


r~2.65 3cm 2cm

tanf = 2 {tanfi:m}
x ADIJ
2
n—1( <fi)
0 =—iatan
0=~ 37.1°

b m3
sin 38° == £T (smo= O
{sinf = HYP}
10 x sin38° = 10m 8m
. ~6.16

sin@:% {sin@:%} A
) 10 x
Ny
sin 38
o —
©osinf) = —————
8
0 = sin—! (10><sin38°>
f 3
0 ~ 50.3°

y =102 — a2 + /8 — 22 {Pythagoras}
~ /100 — 6.162 + /64 — 6.162
~ 13.0

1.5cm pe

2cm
In the shaded right angled triangle, In the shaded right angled triangle,
1.5
cos61° = 2 cosqx = —
T 6
. 2
a == cos—
cos (L8
(6
~—
o

" cos61°
o~ 75.5°
Chapter 7 (Right angled triangle trigonometry) Exercise 7D 295

< 2.43cm d

6.94cm

In the shaded right angled triangle, (3)


coshl® = —
5

- 5xcoshl®=2
2
x=2x5xcosb51°
T~ 6.29

In the shaded right angled triangle,

) 3.5
sin21° = 28T
Slng -
8 L gl A4S
_=sin"o—1(35(22 N sin 21°
—O
2 ( 5 ) oA~ 126
6 _= 2sin9ain—l(35
(T)
0~ 51.9°

Let the angle between the diagonal and the longer side
be 6.
3.8m

92m ‘ 0~ 2240
So, the angle between the diagonal and the longer side
is about 22.4°.

Let the shorter side have length x cm.

tan43.2° =
2om 12.6
12.6 x tan43.2° = x
r =~ 11.8
So, the length of the shorter side is about 11.8 cm.
296 Chapter 7 (Right angled triangle trigonometry) Exercise 7D

4 a tan23° =32 x

= 3.4 3.4cm
tan 23°
~ 8.01
Area of rectangle = length x width -
~ 8.01 x 3.4 cm?
~ 27.2 em?
b c0s68° = —
21
21 X cos68° = x
x &~ 7.87

sin68° = L
21
21 x sin68° =y
y~19.5

Area of rectangle = length x width


=yxx
~ 19.5 x 7.87 m?
~ 153 m?

5 The diagonals bisect each other at right angles, so


BM =3.5cm and CM = 6 cm.
In ABCM, « will be the larger non-right angle as it
is opposite the longer side that is not the hypotenuse.

tana = =
3.5
a=tan™! (%)
a = 59.74°
The required angle is 2« as the diagonals bisect the angles at each vertex.
So, the angle is about 2 x 59.74° ~ 119°.

6 The diagonals bisect each other at right angles, so


BM = 6.9 cm. The diagonals also bisect the angles
at the vertices, so BAM = 10.9°.

In AABM, sin10.9° = &2 k4

69
" sin10.9°
T~ 36.5
So, the lengths of the sides of the rhombus are
about 36.5 cm.
Chapter 7 (Right angled triangle trigonometry) Exercise 7D 297

The diagonals divide the kite into four right angled


triangles.
Let BM = z cm.

In AABM, sin40°

z 6 Xsin40°
In ABCM, sin
10 10

N
i _1(6><sin40°)

NI
S R
10
) ,1<6><sin400)
X sin e

<
10

>
We draw perpendiculars [AM] and [BN] We draw perpendiculars [AM] and [BN]
to [DC], creating two right angled to [DC], creating two right angled
triangles and the rectangle ABNM. triangles and the rectangle ABNM.

In AADM, sin70° = % In AADM, sin70° = %


oy =3xsin70° r =5 X sin70°
= o
y _ 3 xsin70°
In ABCN, sin55° In ABCN, sina— &6 — 5xsin70®
6
T x

3 x sin70° o = sin~! <5 X sin70°)


=220
sin 55° - 6
o344 a ~ 51.5°

We draw perpendicular [AM] to [DC], creating a B


right angled triangle AMD.

In AAMD, sin50° = %
. AM = 4sin50°
Area of rhombus = base x height
=DC x AM
=4 x 4sin50°
~ 12.3 cm?
298 Chapter 7 (Right angled triangle trigonometry) Exercise 7D

b We draw perpendiculars [BM] and [CN] to [AD], creating


two right angled triangles and the rectangle BCNM.
Now, AD =AM+ MN+DN =6
AM+3+DN=6 {MN=BC}
. AM+DN=3
DN=3-AM .. (I)
T
In AABM, tan70° = —
AM

. z=AMxtan70° .. (2)
x
In ACDN, tan60° = —
DN
.z = DN x tan 60°
.= (3—AM) x tan60° {using (1)}
= 3tan60° — AM X tan60°
So, AM X tan70° = 3tan60° — AM x tan60° {equating =}
. AM x tan70° + AM X tan60° = 3 tan 60°
. AM(tan 70° + tan 60°) = 3 tan 60°
e 3tan60°
" tan70° ~+ tan 60°
. AM~ 1.16 cm
2 ~ 1.160 x tan 70° {using (2)}
~ 3.187

b The diagonals bisect each other at right angles, so


AM = 6.5 m.

In AAMD, 6.5% + 2% = 8 {Pythagoras}


4225+ 2% =64
.2t =21.75
x=+v21.75 {as x>0}
v ~ 4.663
The required length is 2z as the diagonals bisect each other.
So, the length of the shorter diagonal of the rhombus is approximately 2 x 4.663 ~ 9.33 m.
Chapter 7 (Right angled triangle trigonometry) Exercise 7D 299

— C In AAMD, € will be the smallest non-right angle as it is


opposite the shortest side.

cosf = £
8

0 =cos™! (%)
0 ~ 35.659°
The required angle is 26 as the diagonals bisect the angles
at each vertex.
So, the angle is about 2 x 35.659° ~ 71.3°.

1 ACB = 90° {angle in a semi-circle}


AABC is right angled at C.
4.
cos34° = 43
AB

- ! AB=_2%3
" cos34°
4.3
2r =
cos 34°
4.3
Lr=—
2 X cos 34°
~ 2.59
So, the radius is approximately 2.59 cm.

BAC = 90° {angle in a semi-circle}


AABC is right angled at A.

sin72° = 181
BC

. BC=-18
sin
16.1
72°
16.1
2r = —
sin 72°
16.1
r = —
2 X sin 72°
~ 8.46
So, the radius is approximately 8.46 cm.

12 a We complete the isosceles triangle.


For the shaded triangle,

cosf = 4
5

TN
6 =cos™t (%)
0 ~ 36.9°

N
300 Chapter 7 (Right angled triangle trigonometry) Exercise 7D

b <
S
10cm

We construct the right angled triangle as


shown.
We construct the altitude as shown. For the shaded triangle,
For the shaded ti)angle, tan g = 1%

sin62° = — = e
TT 5 = tan (S

- "7 Saee coa=2tan


! ()
Lor~11.3 c.oa~61.9°

13 Zem We complete an isosceles triangle and add the perpendicular


mN bisector of the base.
Ix1l4cem=57cm, 3 x89°=445°
For the shaded triangle, sin44.5° = =
5.7 X sin44.5° = x
x ~ 3.995
2z ~ 7.99
the chord is about 7.99 cm long.

14 We complete an isosceles triangle and add the perpendicular


bisector of the base.
6.6cin 13 x 13.2cm == 6.6 cm

g For the shaded triangle, sinf = o


9.4
I
= sin (fi)
20 = 2sin™' (§5)
26 = 89.2°
So, the angle subtended by the chord is about 89.2°.

15 We draw the line from C to P.


For the shaded triangle, sinf = %

So, the angle between the tangents is about 47.2°.


Chapter 7 (Right angled triangle trigonometry) Exercise 7E 301

1 Let the flagpole’s height be h m.


For the 63° angle, OPP = h m, ADJ = 9.32 m
h
hm o, tan63° =
9.32
9.32 x tan63° = h
B . h=183
9.32m
So, the flagpole is about 18.3 m high.

2 a Let the height above sea level be h m.


For the 18° angle, OPP = h m, /
HYP = 150 m. * m

. sin18° = e Am 80m
150 /
150 x sin18° = h [l

h~46.4
So, you are about 46.4 m above sea level.

b Let the distance walked be z m.


For the 18° angle, OPP = 80 m, HYP = 2 m.

sin1ge =& xT

80
Lox=
sin 18°
o~ 259
So, you have walked about 259 m up the hill.

3 Let w m be the width of the river.


For the 37° angle, OPP = 120 m, ADJ = w m. B 120m C
120 H
. tan37° = —
w
wm
120 .
w = river
tan 37°
w ~ 159 A
So, the river is about 159 m wide.

L Let the angle of incline be 6. 200m


For the angle §, OPP = 5.5 m, HYP = 200 m. ‘A 5.5m
5.5
sinf = —
200

p=sin (3)
0~ 1.58°
So, the angle of incline is about 1.58°.
302 Chapter 7 (Right angled triangle trigonometry) Exercise 7E

5 a Let the angle of elevation at A be 6.


For the angle 6, OPP = 56 m,
ADJ = 113 m.
tan = 15—163

6= tan~1 (£%) Sepnp LR


0 ~ 26.4°
So, the angle of elevation from A to the 9
top of the building is about 26.4°.
56m
b The angle of depression from the top of
the building to A is an alternate angle
to 6, so it is also about 26.4°. A

6 By alternate angles, the angle of elevation of the cliff


from B is also 8°.
So, if the distance of the boat from the base of the cliff
is x m, then

tang® = 20
x

20 8°
s tan 8°
20m
Lor A 142
The boat is about 142 m from the base of the cliff. H
rm B

7 By constructing an altitude of the isosceles triangle, 8.3m


we form two right angled triangles.

For the shaded triangle, cosf = 81 / \


8.3 0.6m 15m 0.6m
0 =cos™t (%)
0~ 12.6°
So, the pitch of the roof is approximately 12.6°. 8.3m

BIm o]
8 By constructing an altitude of the isosceles triangle, B
we form two right angled triangles. 23° .

For the shaded triangle, cos23° = 88 A


AB / A
0.8m 16 m 0.8m
N 8.8
cos 23°
. AB =~ 9.56 B
So, the timber beam AB is about 9.56 m long. 232 fi
A [1
Chapter 7 (Right angled triangle trigonometry) Exercise 7E 303

Let the width of the field be = m.


the length of the field is (x4 20) m.
z + 20
. tanbhh® =
T

. o x tanb5° =z + 20
x X tan55° — x = 20
(@+20)m oo x(tanbh® — 1) = 20
2
"~ tanb5° — 1
Lo R46.7
So, the shorter side of the field is about 46.7 m.

10 We draw perpendiculars [DM] and [CN] to [AB].

In ABCN, tan10° = g
. x=3tan10°
So, AM=5—-2—(3tan10°) m
=3 —3tan10° m
in AAMD,
tan(8 — 90°) = %
_ 3—3tan10°
- 3
. f—90° =tan~! (— 30
— 3?“1 )

B —90° ~ 39.48°
s B A129°

1 A Let the height of the flagpole be f m.

5/ In AOAC, tan38® = 2200


200 x tan 38° = AC

In AOBC, tan36° = eC
200
36° 38° . 200 x tan36° = BC
0 200 m C Now f=AC-BC
= 200 tan 38° — 200 tan 36°
f~10.9
So, the flagpole is about 10.9 m high.
304 Chapter 7 (Right angled triangle trigonometry) Exercise 7E

12 E A Let the distance the boat has to move be m.


40 ACB =CAE =4° {equal alternate angles}
b and ADB = DAE = 2.7° {equal alternate angles}

In AABD, tan2.7° = %
A A | BD 15

D smlGC B - ~ tan2.7°
B 15
In AABC, tan4® = —
BC
BC — 15
tan 4°
Now z=BD-BC
_ 1515
" tan2.7° tan 4°

o ~104
So, the boat must move about 104 m closer to the cliff.

13 The helicopter flies horizontally at 100 kmh~1. 555.6m


Distance travelled = speed x time H
=100 kmh™! x 20 s
=100 kmh™! x 60
—22x 60 hm
~ 0.5556 km
~ 555.6 m
Now 6 =60° {equal alternate angles}
. tan60° ~ Slia | il
555.6
555.6 x tan 60° ~ h
h =~ 962
So, the helicopter is about 962 m above the ground.

14 By constructing an altitude from B to [AC], we form two


right angled triangles.
Let the distance of B from the shore be & km.
XBC = 45° and ABX =30° {angles in a triangle}
So, AXBC is isosceles.
XC=BX =xkm and AX = (5 —z)
km
5—x
In AABX, tan30° =
xr

. 30°
owXtan=5 —x
. x(tan30° +1) =5
5
~ tans0° +1
r~3.17
So, the shortest distance from the boat to the shore is about 3.17 km.
Chapter 7 (Right angled triangle trigonometry) Exercise 7E 305

15 The angle at the centre of the pentagon is 360°. {angles at a point}


LY o
DAB = 28 _ 790
AABD is isosceles with equal sides AD and AB.
[AC] perpendicularly bisects [BD].
DAC = 1 x 72° = 36°
Consider the shaded triangle shown. A
. P 362
Let the altitude of this triangle be & m
and the hypotenuse be = m. L
10
tan36° = — [
D B
h 10m C
10
~ tan36°

and sin36° = 22 T

10
"~ sin36°
Now d=xz+h
10 10
= Sn36° ' tan36°
d =~ 30.8
the width of land required for the plot is about 30.8 m.

16 B 12 1 The angle at the centre of the circle is 360°. {angles at a point}


the angle between any two consecutive numbers around the
10 2 °
clock is 292 — 300,
9 3

8 4

7 5
6

We complete the isosceles triangle and add the perpendicular bisector of the base.
20 =4 x30° {as 26 is the angle between 10 and 2}
=120°
0 = 60°
For the shaded triangle,
. 12
sin60° = —-
12
r=
sin 60°
ora13.9
So, the radius of the clock is about 13.9 cm.
306 Chapter 7 (Right angled triangle trigonometry) Exercise 7E

17 H G

E —

!4 C
B B
a Consider the base of the cube, letting BD be x cm.
Using Pythagoras, 22 = 13% 4 132 D C
oot =338
oa=v338 {as x>0}
~ 18.4
So, BD is about 18.4 cm long. A" 13em
b ADBEF is right angled at B. F
13
tanf = ——
an V338
. _ —1(_13 13cm
. 0 =tan (\/fi)
. 0=~353°
o £ 20 D LB
So, FDB is about 35.3°. 338 om

18 Consider the base of the


prism, letting VX be cm. v X
Using Pythagoras,
22 =62 +92 6cm
oot =117 i
o =+V117 {as z >0} v 9cm i
~10.8
So, VX is about 10.8 cm long.

ARVX is right angled R


at V.
tand = Vii7
8 8cm

0 —tan—1(_8
= tan (m) v <

0 ~ 36.5° i et
So, RXV is about 36.5°.
Considering again the base of the prism, let YZ be y cm.
Using Pythagoras, 3 = 9% 4 32
y> =90
. y=+9 {as y>0}
~9.49
So, YZ is about 9.49 cm long.
Chapter 7 (Right angled triangle trigonometry) Exercise 7E 307

d U T AUYZ is right angled U


at Y.
8
tan
¢ = — 8cm

4
8cm V90
¢ =tan™! (%
H X V90 cm
40.1
. ~ o
S~
&
v 9om W3cm
So, YZU is about 40.1°.

19 a We draw the isosceles triangle cross-section 30cm


of the cone as shown.
Let the height of the cone be i cm.

. tan20° = 30
h 20°
_ 30
" tan20°
h~82.4
So, the cone is about 82.4 cm high.

~ L xmx30%
x 82.4 cm?®
~ 77700 em®
~ 77700 mL
~T77.7TL
The cone has a capacity of about 77.7 L.

20 D

B 1% 1.2m

a i Consider the base of the prism, letting CE be


T m.
Using Pythagoras, 22 = 1.2% + 1.6°
1.2m
oat=4
L x=2 {as x>0}
So, CE is 2 m long.
1.6m
il ADEC is right angled at E.
Let CD be y m.
Using Pythagoras, ? = 0.242 + 22
y? = 4.0576 m

oy = V4.0576 {as y > 0}


0.24
E
" &
C
=~ 2.01 2m
So, CD is about 2.01 m long.
308 Chapter 7 (Right angled triangle trigonometry) Exercise 7E

b Let DCE be 6. D

B _ —1/(0.24 ul
. 6 =tan (T) E o C
0 ~6.84°
So, DCE is about 6.84°.

8.5m

a Consider the side of the room containing


. A F
point F and not containing S. * 0.7m
Let the distance from F to the top of 2.3m I%.Sm
Elizabeth’s head E, be = m.
) 8.5
Using Pythagoras, 22 = 0.7% 4 8.5 o
st =T274
. x=+V7274 {as x>0}
ASFE is right angled at F. S
LetUsingSE Pythagoras,
be y m. pa e ~ym
|5.6

y? = 5.6% + (V/72.74)* ey

. y=v1041 {as y >0}


~10.2 V7274 m

So, the spider is about 10.2 m from Elizabeth’s head.

b Let FSE be 6. S
VT72.74
tanf =
5.6

s f=tan™! <@) 5.6m


s 0~ 56.7°
No, the spider cannot see Elizabeth because the angle rh E
FSE is about 56.7°, and the spider can only see up V72.74 m
to an angle of 42°.
Chapter 7 (Right angled triangle trigonometry) Exercise 7E 309

e, A
f
B}

a AERK is right angled at R. K


Using Pythagoras, EK? = 652 + 342
- EK? = 5381 34m
. EK=+v5381 {as EK > 0}
~73.4m E R
65m
So, Edward is approximately 73.4 m from the kite.
Let the point directly below the kite at a height
level with Edward be X.
AKXR is right angled at X.
For the 37° angle, OPP = KX, HYP = 34 m

sin37° = X34
. KX =34 x sin37°
AKXE is right angled at X. K
For angle §, OPP = KX, HYP = /5381 m 34m
Ginf — 34sin37°
/5381 R Ox
9 —sin-! 34sin 37°

. . - o o381 V5381 m )
. 0~16.2
34sin37° m
So, the angle of elevation from
Edward to the kite is about 16.2°. E X

58 mm

32mm B C

Consider the base of the pyramid, letting AC be z mm.


Using Pythagoras, 2 = 32% + 322
= 2048
. x=v2048 {as z >0} A D
310 Chapter 7 (Right angled triangle trigonometry) Exercise 7E

AACX is isosceles as AX = CX.


We draw the perpendicular bisector [XY] of [AC].
12048
In AAXY, cosf =
58
1
5v2048
0 =cos™! ( 2 )
58

0 ~ 67.0°
So, the angle between [AX] and [AC] is about 67.0°.

24

. 3.14m
A
1.26m

Consider the triangular end of the prism.


Let the height of the triangular end be & m.
tan50° = el
0.63
0.63 x tan50° = h
h =~ 0.751 0.63m
Volume of solid = area of triangular end x length
= £ x base x height x length
~ £ x1.26 x0.751 x 3.14 m®
~ 149 m®

Consider the triangular end of the prism.


Let the base of the triangular end be = cm.
17
tan4° =
3 T cm
N 7
"~ tan4°
~ 243.1
So, the base of the triangular end is about 243.1 cm long, or about 2.431 m long.
Volume of solid = area of triangular end x length
= % x base x height x length
~1x2431%0.17x 1.6 {17 cm=0.17 m}
~0.331 m*
Chapter 7 (Right angled triangle trigonometry) Exercise 7E 311

Consider the base of the pyramid, which is an equilateral


triangle with sides 8 cm.
Let the height of this triangle be d cm. 8em
Using Pythagoras, d* 4 4% = §°
d*=64—16 O
4cm
d? =48
d=18
We can now find the area of the base of the pyramid.
Area = 1 x base x height
:%><8><\/Ecm2
= 4V/48 cm?
Now we need to find the height of the pyramid.
We need to find the distance from each corner of the base to
the centre of the base. Let this distance be = cm.
We divide the base into 3 equal isosceles triangles, each with
equal base angles of % = 30°.
4cm

cos30° = & e
xr

© 7T cos30° r
Consider the side faces of the pyramid, which are isosceles triangles with base 8 cm.
Let the equal side lengths of the isosceles triangle be y cm.
4
cos68° = —
Yy
4 yem
cos 68°

We can now find the height of the pyramid.


Let the height of the pyramid be & cm. 4em
Using Pythagoras,
2 4 2 (4 >2
o (cos30°) - <C0568°
4
_ 4\ 4 \? cos6se /| hem
Lo (cos68°) - (cos30°) fas >0
h ~ 9.627
So, the pyramid’s height is about 9.627 cm.
cos 30°
312 Chapter 7 (Right angled triangle trigonometry) Exercise 7E

Volume of solid = % (area of base x height)


~ 4 x 418 x 9.627 cm®
~ 88.9 cm?

25

stars

a We need to find the distance from the


Earth (in orbit) to the star 61 Cygni. dm (0314 )"
1.49 x 10" m 3600

The radius of the Earth’s orbit is ~ 1.49 x 10*! m.


0.314
The parallax of 61 Cygni is about 0.314 arc seconds, or about degrees.

Let the distance from Earth to 61 Cygni be d m.


. (0.314)O 1.49 x 101
sm{ — | = ———
3600 d
1.49 x 101
d=
. (0.314\°
sin m

d~9.7877 x 10'® m
9.7877 x 1016
d~n — light-years {1 light-year
~ 9.461 x 10'°> m}
9.461 x 1015
d =~ 10.3 light-years

b 11.4 light-years ~ 11.4 x 9.461 x 10*® m


1.078554 x 101" m
~1.078554 x 10'" m
1.49 x 10 m
Let the parallax of 61 Cygni be 6.
1.49 x 1011
sinf =
1.078 554 x 1017
0~ sin-1 1.49 x 101
i 1.078 554 x 1017

0~ T7.9153 x 107° degrees


~ 7.9153 x 107° x 3600 arc seconds {1 arc second = 53 degree}
~ 0.285 arc seconds
So, the parallax of 61 Cygni is about 0.285 arc seconds.
Chapter 7 (Right angled triangle trigonometry) Exercise 7F 313

The bearing of Q from P The bearing of Q from P


=126° = 180° + 65°
= 245°

< d

v v
The bearing of Q from P The bearing of Q from P
=90° — 30° =270° +41°
= 060° =311°
e N f
A

v o
The bearing of Q from P The bearing of Q from P
=180° — 28° = 360° — 52°
=152° = 308°

3 a The bearing of B from A is 136°.


the bearing of A from B is 136° + 180° = 316°.
b The bearing of B from A is 018°.
the bearing of A from B is 018° 4 180° = 198°.
¢ The bearing of B from A is 291°.
the bearing of A from B is 291° — 180° = 111°.
314 Chapter 7 (Right angled triangle trigonometry) Exercise 7F

d The bearing of B from A is 206°.


the bearing of A from B is 206° — 180° = 026°.

4 a The bearing of B from A is 072°. N (o-interior


b The bearing of B from C —= 3490
360° — 18° co-interior angles
sum to 180°
7 angles sum
to 180°

¢ The bearing of A from B = 360° — 108°


= 252°

d The bearing of A from C = 360° — 49° — 18°


= 293°

5 Suppose Walter starts at S and finishes at F. T;T

tanf =
10
0 =tan™" (&) ~ 35.0°

So, the bearing ~ 90° + 35°


~ 125°

6 a Suppose Julia starts at S and finishes at F.


Now % =200% +100> {Pythagoras} B
= 50000
L xR 224 {as >0} 200 m
Julia is about 224 m from her starting point.

b tanf = %g =2 S Lir}
100 m :
6 = tan™1(2) ~ 63.4° v
So, the bearing ~ 270° + 63.4°
~ 333°

7 a Suppose Kenneth starts at S and finishes at F.


Now 2% =24%+30% {Pythagoras}
= 1476
. ox A~ 384 {as >0}
Kenneth is about 38.4 km from his starting point.

b tanf
= —
30km
0 =tan""(39) ~ 51.3°
So, the bearing of S from F = 6 {equal alternate angles}
~ 051.3°
So the bearing of the starting point from where Kenneth is now is about 051.3°
Chapter 7 (Right angled triangle trigonometry) Exercise 7F 315

8 a Suppose Paul starts at S and finishes at P. N

b Let the distance Paul has travelled east be x km.

sin53° = =1.5
1.5 x sin53° =z
r~1.20
Paul is about 1.20 km east from his starting point.

¢ Let the distance Paul has travelled south be y km.

cos53° = L
1.5
1.5 x cos53° =y
.y ~0.903
Paul is about 0.903 km south of his starting point.

9 Suppose Tiffany starts at S and finishes at F.


Let the distance Tiffany has travelled west be x km.
6 = 323° —270° = 53°

- coshb3° =2
4
. 4 xcosh3® =z
T~ 241
Tiffany is about 2.41 km west from her starting point.

10 Suppose the train starts at S and finishes at F.


Let the distance travelled by the train be x km.
6 =90°—72° =18°

cos18° = 12
x

LT = - ~ 12.6
cos 18°
the train travelled about 12.6 km on the bearing 072°.
316 Chapter 7 (Right angled triangle trigonometry) Exercise 7F

Suppose the orienteer starts at P, then travels to Q, and


finishes at R.
NPQ = 360° — 236° {angles at a point}
= 124°
NQP = 180° — 124° {co-interior angles}
= 56°
PQR = 146° — 56°
=90°
APQR is right angled at Q.
a PR? =720 +460° {Pythagoras}
PR = 4/720% + 460> {as PR > 0}
~~ 854
So, the finishing point is about 854 km from the
starting point.

b tanf = 460
720
0 = tan 1(4—28)
0 ~ 32.6°
So, the bearing of the finishing point from the
starting point is about 236° — 32.6° ~ 203°.

Suppose the cruise ship first sails from P to Q, then from


Qto S.
NQP = 180° — 112° {co-interior angles}
= 68°
PQS = 360° — 68° — 202° {angles at a point}
=90°
APQS is right angled at Q.

tanf = —=
13.6

6 =tan™" (1%5)
0~ 79.3°
-, the bearing of the cruise ship from P & 112° + 79.3°
~ 191°
Now, PS?=13.6% + 722 {Pythagoras}
PS=+/13.62+722 {as PS>0}
~ 73.3
So, the cruise ship is about 73.3 km on a bearing of 191° from P.
Chapter 7 (Right angled triangle trigonometry) Exercise 7G 317

13 Suppose the yachts both depart from O.


OAN = 180° — 34° = 146° {co-interior angles}
AOB = 124° — 34° = 90°
AAOB is right angled.

tanf = —
1

0 =tan""(1)1 ~51.8°
I=
|
-, the bearing ofB from A ~ 360° — 146° — 51.8°
~ 162°
Now, AB? =112+ 142 {Pythagoras}
. AB=+/112
+ 142 {as AB >0}
~ 17.8
So, yacht B is about 17.8 km from yacht A on the bearing of about 162°.

a i The projection of [AH] onto the base plane is [EH].

ii The projection of [BE] onto the base plane is [EF].

iii The projection of [AG] onto the base plane is [EG].

iv. The projection of [BH] onto the base plane is [FH].


318 Chapter 7 (Right angled triangle trigonometry) Exercise 7G

b i The projection of [RX] onto the base plane is [MR].

ii The projection of [NX] onto the base plane is [MN].

2 a | The projection of [AF] onto the base plane is [EF].


the required angle is AFE.

il The projection of [BM] onto the base plane is [FM]. A B


the required angle is BMF.

iii The projection of [AD] onto the base plane is [DE].


the required angle is ADE.

iv The projection of [BN] onto the base plane is [FN]. A B


the required angle is BNF.
Chapter 7 (Right angled triangle trigonometry) Exercise 7G 319

b i The projection of [AB] onto the base plane is [AM]. E

the required angle is BAM.


B

D‘ c
A M
ii The projection of [BN] onto the base plane is [MN]. E

the required angle is BNM.


B

4
D‘ C

/
A M

W
iii The projection of [AE] onto the base plane is [AN]. E

the required angle is EAN.


B
D


AT M

i The projection of [CF] onto the base plane is [FG].


the required angle is CFG.

tanf = ¢
8 6cm

s f=tan"! (g) H 10cm G


<0 =36.9°
The angle is about 36.9°.
ii The projection of [AG] onto the base plane is [EG].
.. the required angle is AGE.
Let EG be = cm.
Using Pythagoras in AEFG,
2? =10° + 8
oox? =164
oz =164 {as z > 0}
Let AGE be a.
6
tana = —
V164
tan— 1 6 )
« —= tan (\/m

o=~ 25.1°
The angle is about 25.1°. V164 cm
320 Chapter 7 (Right angled triangle trigonometry) Exercise 7G

The projection of [BX] onto the base plane is [FX].


The required angle is BXF.

tan 3 = B
4

o B=tan™! (%)
. 3~ 56.3°
The angle is about 56.3°.
iv The projection of [DX] onto the base plane is [HX]. A B
. the required angle is DXH. 8
Let HX be x cm. D F
Using Pythagoras in AHGX, 6cm m, X
a? =107 4 47 H 10em G
z? =116
z=+v116 {as x>0}
Let DXH be ¢.
6
e =
- tand V116
_ —1 6
¢ = tan (Tm)
¢~ 29.1°
The angle is about 29.1°.

The projection of [PR] onto the base plane is [RS].


the required angle is PRS.

tanf = kil
12

0 =tan~! (%)
0 ~ 33.7°
The angle is about 33.7°.
The projection of [QU] onto the base plane is [RU].
the required angle is QGR.

tana = E
12
a=tan"! (%)
cooam33.7°
The angle is about 33.7°.
The projection of [PU] onto the base plane is [SU].
.. the required angle is PUS.
Let SU be x cm.
Using Pythagoras in ASTU,
2% =122 + 122
2% =288
x =288 {as x> 0}
Chapter 7 (Right angled triangle trigonometry) Exercise 7G 321

Let PUS be 5.

t .8
anf =
= —1 8
B = tan (m)
B~ 25.2°
The angle is about 25.2°.

iv The projection of [QM] onto the base plane is [MR].


the required angle is QI\7[R4
Let MR be cm.
Using Pythagoras in AMUR,
2 =62 + 122
oo 2?2 =180
. z=+180 {as x>0}
Let QMR be 6.
8
tan¢ = %

- -1 8
(bftan (m)

6~ 30.8°
The angle is about 30.8°.

The projection of [QR] onto the base plane is [MR].


the required angle is MfiQA

tanf = L
0.6

6 =tan™' (%)
0 ~ 59.0°
The angle is about 59.0°.
The projection of [QU] onto the base plane is [MU].
the required angle is QIAJM.
Let MU be z m.
Using Pythagoras in AMRU,
2% =0.6% +2.47
2% =612
o =+v6.12 {as >0}
Let QGM be a.
o tana = E=1F,
6.12
=
a = tan
-1 (m)
1

a = 22.0°
The angle is about 22.0°.
322 Chapter 7 (Right angled triangle trigonometry) Exercise 7G

iii The projection of [QN] onto the base plane is [MN].


the required angle is QI?IMA
MN | RU, so MN=RU =24m
1
tan8 = 1

o= tan_l(rz)
3 = 22.6°
The angle is about 22.6°.

d i The projection of [AX] onto the base plane is [AM].


the required angle is XAM.
Let AM = DM be z cm.
(The base of the figure is a square, so its diagonals
[AC] and [BD] perpendicularly bisect each other.)
Using Pythagoras in AAMD,
2% +2? =6?
22% = 36
22 =18
r=+18 {as x>0}
Let XAM be 6.
V 1 8
. cosl = —
0 1

@ =cos™! <3§>
0 =~64.9°
The angle is about 64.9°.

il The projection of [XY] onto the base plane is [MY].


the required angle is XYM.
Let XY be z cm.
Using Pythagoras in AXYD,
2 +3% =107
22 =100-9
22 =091
=01 {as >0}
Let XYM be a.
- cosa:%

A . a=cos
WAk (\/fi)
.
LoaTLT
The angle is about 71.7°.
Chapter 7 (Right angled triangle trigonometry) Review set 7A 323

1 a AC?=AB?+BC? {Pythagoras} B
_=8 Q2 +6 @2 ADJ OPP
=100 8cm 6em
. AC =100 {as AC > 0}
. AC=10 A C
The hypotenuse is 10 cm long. HYP
bsingzfl:fizé ccosazfl:i:é dtang:flzfizg
HYP 10 5 HYP 10 5 ADJ 8

2 a cosh9° ~0.515 b sin8° ~0.139 ¢ tan76° ~ 4.011

3 a b OPP

HYP - 7m
8m ADJ
Trm

ADIJ om

[ HYP
OPP
oz _ADJ isina° 75 . OPP
cos64° = = {cosf = fi} = {sinf = fi}
8 xcosb4’ = o2® =sin~ ()
. x =351 o raBll

< OPP
9cm
O

ADIJ
rcm
HYP

tanb8° = = {tanf = %}
ADJ
9
xr=
tan 58°
T =~ 5.62

4 a tan2s° = 22 {tan6 = e R
PR ADJ
PR — 40
" tan28° Q
PR~ 75.2 cm
. o _ 40 . OPP 40 cm
sin 28° = & {sinf = _HYP} ]
40
" sin28°
QR ~ 85.2 cm
Perimeter of triangle PQR = PQ + PR + QR
~ 40+ 75.2+85.2 cm
=~ 200 cm
324 Chapter 7 (Right angled triangle trigonometry) Review set 7A

b Area of triangle PQR = 12 x base x height


=1 2 xPQxPR
~ L2 x40 x 75.2 cm?
~ 1500 cm?

5 6 =180° — 90° — 57° {angles in a triangle} D 6cm E


. 0=33°

tan57° = & {tanf = O xecm


T ADJ yem
. 6
" tan57° C
r ~ 3.90
Now y* =a?+6% {Pythagoras}
.y~ V/3902+62 {as y >0}
Loy~ T7.15

6 By constructing the altitude of the isosceles triangle,


we form two right angled triangles.

For angle 0, cosf = 91


9.8
. f=cos! (%)
0 =8.19°
So, the pitch of the roof is approximately 8.19°.

65°

We construct the altitude to form two right We construct perpendiculars to form two
angled triangles. right angled triangles and a rectangle.
sin65° = ¥
€T

s11r116°—g 3
5
y = 3 X sin65°
2 — 5 xsin16°
2
sin40° = ¥
s x=2xD5xsin16° x

Loz 2.8 . o X sin40° = 3 x sin 65°


3sin 65°
LoT=
sin40°
Lorr4.2
Chapter 7 (Right angled triangle trigonometry) Review set 7A 325

¢ We construct the altitude to form two right


angled triangles.
4.3
sin56.5° = —= o
x

43
" sin56.5°
TR H.2

8 Let the height of the lighthouse be 2 m.

tan 34° = 2
20
20 X tan34° = h
h~13.5
the height of the lighthouse is about 13.5 m.

9 a b

The bearing of Q from P The bearing of Q from P


=90° + 28° =270° — 39°
=118° =231°

< The bearing of Q from P


= 360° — 31°
=329°

v
10 Suppose the helicopter starts at S and finishes at F. IRI

tan = ol i
12
0 =tant (%) ~~ 22.6°

.. the bearing ofF from S ~ 180° + 22.6°


~~ 203°
Now, z?=12*+5% {Pythagoras}
a2t =169
x =13 {as = > 0}
So, the helicopter is 13 km on the bearing of about 203° from the helipad.
326 Chapter 7 (Right angled triangle trigonometry) Review set 7A

AC? = 15% + 432 {Pythagoras}


. AC = /152 4432 {as AC >0}
=v2074 m

"
2074
ch - A
B

7
tanf =
. V2074

6 = tan 17
( m)
~ 8.74°
So, the angle of depression from A to B is approximately 8.74°.

12 We draw the isosceles triangle cross-section rem


of the cone as shown.
Let the radius of the cone be r cm.
1Sy tan17.5° = —
. 17.5°
18 x tan17.5° =r
. r~568

V= 1mr’h
~ 3 X x 5.68% x 18 cm®
~ 607 cm®
~ 607 mL
~ 0.607 L
The cone has a capacity of about 0.607 L.

13 a The projection of [AC] onto the base plane is [BC]. m A


the required angle is ACB. -

tanf = s
6 8cm
A =tan"! (%)
0 ~ 53.1° c fi B
The angle is about 53.1°.
Chapter 7 (Right angled triangle trigonometry) Review set 7B 327

b The projection of [AD] onto the base plane is [BD].


the required angle is ADB.
Let DB be & cm, and the centre of the circular base be O.
OB =OD =3 cm {both radii of circle} 8cm

Using Pythagoras in ABOD,


z? =32+ 32 5B
a2 =18
L x=+18 {as z>0}
Let ADB be a.
A
——8
©otana na = NG

-. —
a=tan 18
(\/fi) 8cm

a = 62.1°
The angle is about 62.1°. C fi /‘ B

D V18cm

1 Let the unknown side be x cm. HYP


122 _=5"+11
r2 2 {Pythagoras} 5om xcm
z° =146 OPP
z -=146 {as = >0} Ll
Tom
ADJ

sinf = OpP _ 5 , cosf = 2D e , tanf = Oileks 5


HYP 146 HYP 146 ADJ 11

) OPP 2.2
2 a 2(;111:11 b i sin26 A
26° = —~ =~ 0 044
A C
- il cos26° = ADI 45 5 0.90
HYP 5.0

il tan26° = OFF ¢ 22 0.49


ADJ 4.5
ADJ
4.5cm 5.0cm ¢ sin26° ~ 0.44, cos26° ~ 0.90, tan26° ~ 0.49
HYP
328 Chapter 7 (Right angled triangle trigonometry) Review set 7B

3 a HYP b ADJ .
OPP 7m 10cm OPP
4m

13cm
ADJ
HYP
sinf = 2 {sin@zfl}
7 HYP cosg = 20 {co 9=fl}
13 HYP
. O =sin"t (%) - —1(10
0 =cos™'(19)
~ 34.8°
= 39.7°

¢ tang =22 {tané’:fi} ADJ


6.2 ADJ
6.2m
0 = tan~! (4—3)
~ 36.0°
4.5m HYP
OPP

4 sin23° =LAB fsinf= 2t}


HYP A B

. AB= sin2 23° 47 mm


~ 120 mm g
C
tan23° = 2 {tanf = %}
AC ADJ

. AC= &
tan 23°
~ 111 mm

5 2% +19? = 322 {Pythagoras} M


z? =32° - 19°
32
r? = 663 o 19cm

x =663 {as = >0} [

~ 25.7 K Trcm L

cosf =2 {cos@zfl}
32 HYP
i,
0 =cos™'(33)
0 =~ 53.6°

sina = 32 {sinezfl}
32 HYP

. a=sin! (%)
oo 36.4°
Chapter 7 (Right angled triangle trigonometry) Review set 7B 329

6 g The diagonals of a rhombus bisect each other at right angles.

W So, tang 013


= 1 {tan = ADJ}_o
‘:‘ 7.5cm g =tan"! (%)

~2 124°
So the larger angle of the rhombus is about 124°.

7 ABC = 90° {angle in a semi-circle}


AABC is right angled at B.

cos32° = 43
AC
AC — 4.3
" cos32°

or = 243
" cos32°
4.3
Lr=—
2 X cos 32°
~ 2.54
The radius is approximately 2.54 cm.

Let the height of the building be h m.

tan20° = — "
x 4+ 80
h = (z + 80) tan 20°
Also tan23° = h
xr

(x + 80) tan 20°


tan23° =
x

2 tan 23° = x tan 20° + 80 tan 20°


. z(tan23° — tan 20°) = 80 tan 20°
s 80 tan 20°
" tan23° — tan 20°
~ 481.25
h ~ (481.25 4 80) tan 20° m
~ 204 m
The building is about 204 m tall.

9 a A B AAHG is right angled at H.


D AHG = 90°
6 cm

F
/7 4 cm
H 8 cm G
330 Chapter 7 (Right angled triangle trigonometry) Review set 7B

b E F Consider the base of the prism.


Let FH be z cm.
4cm
Using Pythagoras, z% =42 + &°

N
Scm o2 =80
=80 {as x>0}
ADFH is right angled at H.

tanf an = T
— D

6 =— tan tan—1 ( (-680) 6em

9i33'9 - v

So, DFH ~ 33.9°. V80 cm

10 Suppose Aaron starts at S, travels to O, and finishes at F.


FON; = 360° — 303° =57° {angles at a point}
OSN, = 360° — 213° = 147° {angles at a point}
. N;OS = 180° — 147° = 33° {co-interior angles}
. FOS = 57° +33° = 90°
AFOS is right angled at O.

tanf = 25
3
6 =tan™"' (%2) ~ 39.8°
-, the bearing of F from S ~ 213° 4 39.8°
~ 253°
Now, 2 =2.5%4 32 {Pythagoras}
r=1/2524+32 {as x>0}
~ 3.91
So, Aaron is about 3.91 km on a bearing of about 253° from his starting point.

1 Let the height of the pyramid be /& m,


and the diagonal of the base of the
pyramid be 2z m.

Consider the base of the pyramid.


2%+ 2% =122 {Pythagoras}
2% =144
L2t =T2
L:\/fi {as z > 0}
12m
Chapter 7 (Right angled triangle trigonometry) Review set 7B 331

tan40° = V7
h

T VT2
tan 40°
~ 10.1

Volume of pyramid = % x (area of base) x height


~ 3 x12x12x101m’
~ 485 m®

12 a The projection of [BH] onto the base plane is [FH].


the required angle is BHF.
Let FH be x cm.
Using Pythagoras in AFEH,
2% = 5%+67
2t =61
x=\/6_1 {as = >0}
Let BHF be 0
2
tanf = NG

0 = tan 12
(fi)
0 ~ 14.4°
The angle is about 14.4°.

b The projection of [CM] onto the base plane is [GM].


the required angle is CMG.
Let GM be « cm.
Using Pythagoras in AGHM,
2? =52+3?
Loa?=34
Cx=+34 {as z>0}
Let CMG be o

tancy—i
V34
a == tan I (N)

a =~18.9°
The angle is about 18.9°.
332 Chapter 7 (Right angled triangle trigonometry) Review set 7B

¢ The projection of [XM] onto the base plane is [MY]. B X


.. the required angle is XMY. l"
2
tanff =2 A ’ G
o. Ba2L8°
B=tn(f) L
o
o M H
6cm
The angle is about 21.8°.

25 OPP
13 a i In AAMH, sin65° =-— {sinf = —}
AH HYP
AH= 2
sin 65°
AH =~ 27.6 cm

i In AAMH, tan65° = =2
AM
{tang = 2Py
ADJ
25
"~ tan65°
. AM ~ 11.66 cm
CM ~ 11.66 cm
. {altitude of isosceles triangle bisects the base}
. AC~2x11.66 cm
. AC=23.3 cm

b CH=AH= —2 {from a i} -
B —
c
sin 65°
25 e A <
and AC=2x {from a ii}
tan 65°
50
" tan65°
. i hcm
Let the height of the prism, AE, be h cm and the
length of the prism, EH, be x cm. JdG

In AADC, AD? +CD? = AC? {Pythagoras} L


E T cm H
9 2 _ 50 )2
& rEes (tan 65°
2
222 = ( N )
tan 65°

(D)
Chapter 7 (Right angled triangle trigonometry) Review set 7B 333

In AAEH, AE?+EH? = AH? {Pythagoras}


5, o ([ 25 )2
Rt (sin65°

Volume of prism = length x width x height


~ 16.49 x 16.49 x 22.11 em®
~ 6010 cm®
Chapter 8
NON-RIGHT ANGLED TRIANGLE
TRIGONOMETRY

1 Using a calculator:

0 cosf | sinf | cos(180° — ) | sin(180° — 6) | cos® @ + sin? —cos


12° 0.9781 | 0.2079 —0.9781 0.2079 1 v -0.9781 v
25° | 0.9063 | 0.4226 —0.9063 0.4226 1 v —0.9063 v
381 0.7880 | 0.6157 —0.7880 0.6157 1 v —0.7880 v
56° | 0.5592 | 0.8290 —0.5592 0.8290 1 v —0.5592 v
70° | 0.3420 | 0.9397 —0.3420 0.9397 1 v —0.3420 v
85° | 0.0872 | 0.9962 —0.0872 0.9962 1 v —0.0872 v

From the table we can see that:


o cos?0 +sin’f =1
e cos(180° — 0) = —cosd
e sin(180° — ) =sinf for each value of 6.

2 P has coordinates (cos6, sin6).


For point P, 6 = 52°.
*. P has coordinates (cos 52°, sin52°)
~ (0.616, 0.788).
Q has coordinates (cos 6, sin6).
For point Q, 6 = 140°.
*. Q has coordinates (cos 140°, sin140°)
~ (—0.766, 0.643).
3 a sin(180° — @) = sinf b sin(180° — 6) = sinf
s, sin(180° — 45°) = sin45° . sin(180° — 50°) = sin50°
. sin135° = sin45° . sin130° = sin50°
So, the obtuse angle is 135°. So, the obtuse angle is 130°.

< sin(180° — 6) = sin6 d sin(180° — 6) = sinf


s, sin(180° — 18°) = sin 18° o.osin(180° — 71°) =sin71°
. sin162° = sin 18° . sin109° = sin71°
So, the obtuse angle is 162°. So, the obtuse angle is 109°.

4 a sin(180° — 6) = sin6 b sin(180° — #) = sin6


-, sin(180° — 103°) = sin 103° . sin(180° — 119°) = sin 119°
. sin77° = sin103° oo osin61° =sin119°
So, the acute angle is 77°. So, the acute angle is 61°.
Chapter 8 (Non-right angled triangle trigonometry) Exercise 8B 335

< sin(180° — ) = sin6 d sin(180° — #) = sin6


sin(180° — 126°) = sin 126° s, sin(180° — 155°) = sin 155°
s, 8in54° = sin 126° sin 25° = sin 155°
So, the acute angle is 54°. So, the acute angle is 25°.

a cos(180° — 0) = —cos6 b cos(180° — ) = —cosb


cos(180° — 3°) = —cos3° cos(180° — 22°) = — cos 22°
cos 177° = —cos 3° . cos158° = — cos22°
So, the obtuse angle is 177°. So, the obtuse angle is 158°.
< cos(180° — 0) = — cosf d cos(180° — 0) = — cos6
cos(180° — 47°) = — cos47° s, cos(180° — 63°) = — cos 63°
. c0s133° = — cos47° . cos117° = —cos 63°
So, the obtuse angle is 133°. So, the obtuse angle is 117°.

6 a cos(180° — 0) = —cos b cos(180° — @) = —cos0


cos(180° — 95°) = — cos 95° s, cos(180° — 102°) = — cos 102°
cos85° = — cos 95° cos 78° = — cos 102°
So, the acute angle is 85°. So, the acute angle is 78°.

< cos(180° — 6) = —cos@ d cos(180° — 6) = —cos@


cos(180° — 146°) = — cos 146° . cos(180° — 162°) = — cos 162°
. c0s34° = — cos 146° cos 18° = — cos 162°
So, the acute angle is 34°. So, the acute angle is 18°.

EXERCISE 8B

1 a A b A 31km B
9cm

B
25km

10cm
C
C
Area = Lacsin B
1 ) Area = 1bcsin A
=3 x 9x 10 x sin40° 1 T Lt
98.9 om? =5 X 25 x 31 x sin82
~ 28.9 cm
~ 384 km®
c A d
9.7cm B
A

5m 6cm

c 8m B ¢
Area = 1bcsin A
The triangle is right angled at C. rea f L
— 21 o
Area = 1 (base x height) = RMOSC im0
1
=3 X8x5H ~ 18.7 em?
=20 m?
336 Chapter 8 (Non-right angled triangle trigonometry) Exercise 8B

e A f A_8m g

10.2cm 13m 15m

¢ 64cm B
C
Area = LabsinC
Using the sides adjacent to the included
=3 % 6.4x10.2 x sin120° angle,
~ 28.3 cm? area = %ac sin B

=3 x 15 x 8 x sin 60°
~ 52.0 m?

2 Area = 150 cm? A


1acsinB =150 17cm
1 x 2 % 17 x sin68° = 150 p68°
2 x 150 zem C
r= ———
17 X sin 68°
x~19.0

3 a A b A

60°
57cm
2lcm

c 60 60! B
C B
An equilateral triangle has all sides and
Area = 1besinA angles equal.
=1 %21 x 21 x sin49° Area = %absinC
~ 166 cm? = $ X 57x 57 x sin60°
~ 1410 cm?

L Area = 2 x area of AABC


=2X %bcsinA
=2x 3 x4x06xsin52°
~ 18.9 cm?

5 Area= 2 x area of AABC


=2x 2besinA
12cm
=2x 12 x12x12xsin72°
~ 137 cm?
Chapter 8 (Non-right angled triangle trigonometry) Exercise 8B 337

6 a Areaof APQR = $pgsin R Q


=1 x14 x 17 x sin37°
~ 71.616 m? 14m
b Let the length from Q to [RP] be h m. 0
Area of APQR = 1 x base x height 17m
71.616 ~ 3 x 17 x h
71.616
h 1
7 X 17

h~8.43
So the length of the altitude from Q to [RP] is about 8.43 m.

7 Area =6 x area of one triangle 12 cm 60°


=6x 4 x 12 x 12 x sin 60° 12 cm
~ 374 cm?

8 Let the side length be x cm.


Area = 2 x area of one triangle
=2x % x2x2xsin63°
2%sin63° = 50
2 50
sin 63°

50
€T = {z >0}
sin 63°
T~ 749
So, the sides are approximately 7.49 cm long.

9 Area of one triangle = % m?

Let the side length of the triangle be = m.


fi rm
1 xaxzxsinT2®=

. 2 x 338 o
’ " 5 xsin72° 360°+5="72°
2 x 338
TSm0
o119
So, OA~11.9m
338 Chapter 8 (Non-right angled triangle trigonometry) Exercise 8B

10 a If the included angle is 6, then % X5 x8xsinf =15


20sinf = 15
. sinf = % =3 3

Now sin~! (%) ~ 48.6°


0~ 48.6° or (180 — 48.6)°
0~ 48.6° or 131.4°

5cm
5cm

A \
8cm 8cm

b If the included angle is 6, then % X 45 x 53 x sinf = 800

Now sin™'(829x2)
~ 42.1°
0 ~42.1° or (180 —42.1)°
0~ 42.1° or 137.9°

45km
45km

53 km 53 km

11 Each coin is made up of 12 triangles.


Let half the length of a diagonal of a coin be 7.
Total area of8 coins = 8 x 12 x % X r X r X sin30°
_4R02(1
=48r°(3) 360°
+ 12 = 30°
= 2472 T

Area of $5 note = (4 x 2r) x (2 x 2r)


=8r x 4r
= 32r?
. 2472
Fraction covered = ——
32r2
3
~1
1 is not covered.

12 a Shaded area
= area of sector — area of triangle
=28 xmx12° — & x12x 12 x sin85° 12em
~ 35.1 em®
Chapter 8 (Non-right angled triangle trigonometry) Exercise 8C 339

b Shaded area
= area of triangle — area of sector
=1x12x (12+18) xsin37° — 2L x 7 x 122 A
~ 61.8 cm?
18 cm

¢ Shaded area
= area of sector — area of triangle ‘
135 x7x T — §X7x7xsin135°
=3B V
~ 40.4 mm?

13 Area of segment AXBD


= area of sector ACBD — area of AACB
=200 x 7 x 7.3%
— 1 x 7.3 x 7.3 x sin 100°
~ 20.264 cm?
Area of segment AXBE
= area of sector AFBE — area of AAFB
=80 x 7w x8.7% — 3 x 8.7 x 8.7 x sin80°
~ 15.572 cm?
Shaded area = area of segment AXBD — area of segment AXBE
~ 20.264 — 15.572
~ 4.69 cm?

1 a Let the remaining side have length = cm. e


By the cosine rule:
3cm
2?2 =32 452 -2 x3 x5 x cosb0°
2=+/32+52—2x3x5xcosb0° {as x>0} e
T~ 3.84
The remaining side is about 3.84 cm in length.

b Let the remaining side have length m.


By the cosine rule: 8m
2?2 =824+ 7% -2 x 8 x 7 X cos64°
r=\8
172 2x8xT7xcos64° {as x>0} e
T~ 799 L
The remaining side is about 7.99 m in length.
340 Chapter 8 (Non-right angled triangle trigonometry) Exercise 8C

¢ Let the remaining side have length = cm.


By the cosine rule:
22 =102 + 172 — 2 x 10 x 17 x c0s27° s
Trcm
-2 x 10 X 17 x cos27°
. x=1/1024172 {as z > 0}
10cm
. x~9.28
The remaining side is about 9.28 cm in length.

d Let the remaining side have length = cm.


By the cosine rule: 15cm
22 =212 +15% — 2 x 21 x 15 x cos 105°
oo =1/212 4152 — 2 x 21 x 15 x cos105° {as x>0} 2lem
xrcm
xR 288
The remaining side is about 28.8 cm in length.
e Let the remaining side have length = km.
By the cosine rule: 4.8km
22 =6.3%2 4+ 4.82 — 2 x 6.3 x 4.8 x cos 32° x km
6.32 +4.82 —2 x 6.3 x 4.8 x cos32° {as z >0}
6.3km
or R 3.38
The remaining side is about 3.38 km in length.

f Let the remaining side have length m.


By the cosine rule:
22 =6.22 4 14.82 — 2 x 6.2 x 14.8 x cos72° 6.2m zm
L =1/622+14.82 -2 x 6.2 x 14.8 x cos72° {as x>0}

- 14.8m
The remaining side is about 14.2 m in length.

2452 _ g2
2 a By the cosine rule: cosf = £t
2X4x5 5m

G (u) 2
2x4x5
2 _g2 4m

6 =cos™t (%) 6m
0 ~ 82.8°
2 2 _ g2
b By the cosine rule: cosf = SeidLE o
2x6x11

(62 +112-92
0 = cos <—2><6><11 9cm
6cm
0 =cos™! (£)
0 ~ 54.8° 1lem
Chapter 8 (Non-right angled triangle trigonometry) Exercise 8C 341

X 72 +10% — 132
¢ By the cosine rule: cosf = el e on
2x7x10

72 13’-’
— 132
+10% 13m
. 0 =cos”
T2x7x10
0= COSil( 20 )
140
. 0 =~98.2°
3 By the cosine rule: By the cosine rule:
122 4132 — 112 4112
132 — 122
cosBAC = cos ABC=
2x12x13 Tox13x11
12cm 11cm
A
BéC -1
=cos™!($33) ). ABC=- cos —1(146
(3%)
BAC =~ 52.0° - ABC~ 59.3°

Also, ACB = 180° — BAC — ABC A T3em ‘


~ 180° — 52.0° — 59.3°
- ACB ~ 68.7°
o 2, =2 _ 102
4 a cosPOR =27 —10° P
2x5x7
~ 10cm
.i —ema—l(_26
PgR = cos (—%) 5om
- PQR ~ 112° .
b Area of APQR~£ x 5 x 7 x sin112° 7cm
~16.2 cm?
5 a
13cm 7cm
11cm 4cm

17cm 9em

The smallest angle is opposite the shortest The largest angle is opposite the longest
side. side.
By the cosine rule: By the cosine rule:
132+ 172 — 112 4247292
cosf = coSp = ———
2x 13 x17 2x4x7

6 =cos™! (%37) —
¢ = cos o=l (_16
(7%)
~ 40.3° ~ 106.6°
The smallest angle measures about 40.3°. The largest angle measures about 107°.
2 £2 42
6 a By the cosine rule: oSO =t 4
2x2x5
5cm 2cm
cosf = 18 0.65
20
b By the cosine rule:
22 =524+3%-2x5x3xcosf 1 =
T cm
z=1/52+32-2x5x3x065 {as x>0}
T~ 3.81
342 Chapter 8 (Non-right angled triangle trigonometry) Exercise 8C

7 a Area = 11.6 m?
1x6x4xsind =116
sinf = 116
12

6 _= sin
ain—1 (11.6
(1—)
0~ 75.2°
b Let the third side have length = m.
By the cosine rule: 2% =62 +42 —2 x 6 x 4 x cosf
/62 +42 —2x6x4xcos7H.2°
r ~6.30
The third side is about 6.30 m in length.

8 E C

8.4m

s
L\, 81m
a In AABD, by the cosine rule:
DB? = 7.6% + 8.1 — 2 x 7.6 x 8.1 x cos 30°
. DB =+4/7.62+8.12 -2 x 7.6 x 8.1 X cos30° {as DB > 0}
l ..
Now
DB~4.09
AC=AD
m
+DC=76+84=16m
In AABC, by the cosine rule:
BC? =8.1% + 16 — 2 x 8.1 x 16 x cos 30°
. BC=+/8124+162
-2 x 8.1 x 16 x cos30° {as BC
> 0}
. BC~9.86 m
2 2 _ 2
b In AABD, cosABD = &1 2 +D
~
B" —7.
x 8.1 x DB
67
ABD ~ cos—1 8.12 +4.09% — 7.62
) o 2% 8.1 x 4.09
- ABD =~ 68.2°
. ABE ~ 68.2° {base angles in an isosceles triangle}
2 25 2
In ADBC, cosDBC = 2B~2 +B
~
C” —8
x DB x BC
47
- DBC ~ cos 4.092 4 9.862 — 8.42
j ' 2 % 4.09 x 9.86
. DBC ~ 57.5°
¢ Area of ABCD = 1 x DB x BC x sinDBC
~ § % 4.00 x 9.86 x sin 57.5°
~17.0 m?
Chapter 8 (Non-right angled triangle trigonometry) Exercise 8C 343

9 a By the cosine rule: 7 =22+ 6% -2 x 2 x 6 x cos60°


436 — 122 % (3)
s 49=2"
coat—6r—13=0 6em 7em
b Lo + 6EV6 2 — 2(1)(
4 -1 3)
_6::\/@ rem

e, -2
=3+V22
But >0, so x =3+ V22

10 a By the cosine rule:


2 2, Q2
11" =2*+8" —2x x x 8 X cosT0 0 vem 11em
o121 = 2% + 64 — 162 X cos 70°
oo 2?2 = (16c0s70°%)z
— 57 =0
Using technology, = ~ —5.29 or 10.8. 8cm
But x>0, so x~10.8.

b By the cosine rule:


52 =32 4 2% — 2 x 3 x x x cos 120° 5em
s — 6z x (—1)
25=9+2% 3em
a2 +3r—-16=0
Trcm
—3+4/3%2 —4(1)(-16)
2

But >0, so v =— + VB3 o7


¢ By the cosine rule:
52 = 2% + (22)% — 2 x & X 27 X cos60° 5cm
ZCI m
25 = 2% + 42® — 42’ x (3)
= 5% — 222
= 322 2x cm
2 _ 25
2 3
r=4/8 {as x>0}
z = 2.89

11 By the cosine rule: 52 =22 462 — 2 x & x 6 x cos40°


5cm
o
25 =22 + 36 — 122 cos 40°
. 22— (12c0s40°)z +11=0 xcm
Using technology, = ~ 1.41 or 7.78.
6cm
344 Chapter 8 (Non-right angled triangle trigonometry) Exercise 8C

12 Let CAB be a and DAC be .


. 72 4122 — 92
In AABC, by the cosine rule: cosa = ————
2x7x12

Q= COs
—1(112
(m)

2 2 _ 12
In ADAC, by the cosine rule: cos3 = D
2x8x12

. B=cosT(EE)
Now in ADAB, DAB =« + 3
A ges B = Cos —1(112
(1—68) —+ cos —1(87
(1—92)
~111.2°
8em .
By the cosine rule: 2 Q 2 2
BD” ~ 8% 4 7% —2 x 8 x 7 x cos 111.2 o

BD~ /82 +72—2x8x 7 xcos111.2°


D
BD~ 124
[BD] is about 12.4 cm long.

13 In ABCD, by the cosine rule:


BC? =52 462 —2 x5 x 6 x cos 130°
. BC=+/52+62—-2x5x6xcos130° {as BC> 0}
. BC~9.98
In AABC, by the cosine rule:
BC2=821+92 -2 x8x9xcosf
©. 9.98% ~ 64 + 81 — 144 cos6
145 — 9.982
. cosf R —04 —
144

L drxalo (145 L 9.98 2 )


144
O =T1.6°

14 Let the distance from P to C be = cm. A


~o 524102 62 ‘
In AABP, cosPAB = o X't i e

__ 89 ' 10cm
= 7100 rlem

- PAC =60° —cos™! (&) {since AABC is equilateral} 5 h


- PAC ~ 32.87°
Now, in AAPC, by the cosine rule:
22 =1+0%
5% — 2 x 10 x 5 x cos PAC
. za /10
2%2 x 55
104 2
x cos3 2.87° {as = > 0}
T ~6.40
So, P is about 6.40 cm from C.
Chapter 8 (Non-right angled triangle trigonometry) Exercise 8D.1 345

1,2,3

7cm

sinA _ sinB _ sinC a


5 We notice that or equivalently,
a b sin A sin B sin C'

EXERCISE 8D.1

1 a Using the sine rule,


x _ 23
sin48° sin37°
23 x sin48°
r = ———
sin 37°
x~28.4

b Using the sine rule, 1l1cm


x 11 115°
sin 115° sin 48°

xTr=
11 xsin115° rem
sin 48° 482
r~13.4
346 Chapter 8 (Non-right angled triangle trigonometry) Exercise 8D.1

¢ Using the sine rule,


T 48
sin 51° sin 80°
4.8 X sin51°
r=
sin 80° x km
z~3.79

d The unknown angle is 180° — 100° — 45° {angles in a triangle}


= 35°
Using the sine rule,
@ . 6
sin100° sin35°
6 x sin 100°
g oL BIC D
sin 35°
x~10.3
k6m
e The unknown angle is 180° — 58° — 55° {angles in a triangle}
=67°
Using the sine rule,
58° xcm
T o 4
sin67° sin55° P 55°
4 x sin67°
"~ sinB5°
r~4.49

f The unknown angle is 180° — 108° — 31° {angles in a triangle}


=41°
Using the sine rule,
108°
T _ 9
sin31° sin41°
9 x sin 31°
T sin4l°
r =~ 7.07

£
2 a Using the sine rule,
a 18
sin63° sin49°
= 18 Lt
x sin
O} 63°
sin 49°
a~21.3 cm
18cm

b The unknown angle is 180° — 25° — 82° {angles in a triangle}


=73°

i
Using the sine rule,
b x4
sin73° sin25°
Pl 34 x sin 73°
"sin25°
b~ 76.9 cm 34 cm
Chapter 8 (Non-right angled triangle trigonometry) Exercise 8D.1 347

¢ The unknown angle is ~ 180° — 48° — 21° {angles in a triangle}


=111°
Using the sine rule,
c 6.4 A
sin 48° sin 111°
c— 6.4 X sin48°
T sint11°
48° 21°
c~5.09 cm c 6.4om B

3 a BAC=180° — 58° — 48° {angles in a triangle}


A
=T74°
Using the sine rule,
7cm
AB 7 BC i
- = — and — = —
sin 58° sin 48° sin 74° sin 48°
. . o 48°
AB — 7 ><'81n58° . BC= 7 ><'s1n74° C 58 8 B
sin 48° sin 48°
AB ~ 7.99 . BC~9.05
So, BAC =74°, AB~7.99cm, and BC =~ 9.05cm.
b XZY =180° —43° — 29° {angles in a triangle}
_ o

Using the sine rule, 505


XZ 19 and XY 19
sin29° sin43° Sin108° sin43° X< 15cm
X7 — 19 >< sin 29° XY = 19 ><. sin 108° 2
sin 43° sin 43°
XZ~135 o XY =265
So, XZY =108°, XZ~13.5cm, and XY ~ 26.5cm.

4 First we find the length of the diagonal, d m. D


In AABC, using the sine rule,
d 22
sin 118° sin 30°
d— 22 X sin 118°
s sin 30°
d ~ 38.85
Now 6 =180° —30° — 118° = 32°
ACD = 90° — 32°
= 58°
DAC = 180° — 95° — 58° = 27°
In AACD, using the sine rule,
T 38.85 and Y 38.85
sin 27° sin 95° sin 58° sin 95°
— 38.85 x sin27° Ly 38.85 x sin 58°
sin 95° o sin 95°
z~17.7 Sy~ 33.1
348 Chapter 8 (Non-right angled triangle trigonometry) Exercise 8D.2

5 Using the sine rule, ——— = Zo— 1l


sin 30° sin 45°
z 2x—11
T T 1
2 V2
e =73(2x—11)
z =2 (2x —11)

=2 112
— 11v2
2

(1- v = -1V 112

P 11v2
2(1-v2)
(112 o (2t 2v/2
2-2V2 2422
—22v/2-22(2)
22 _ (2\/5)2

_-22v2-44
= o=y
—22V/2 44
o —4
=4v2+11
v=11+4v2

EXERCISE 8D.2

1 a Using the sine rule,


60 cm sinz® _ sin20°
30em 30 60
. o 30xsin20°
sing® = —/———
60
.1 (30 X sin20°>
T = SIn R —
60
x ~ 9.85

b Using the sine rule,


7° 58° sinz® _ sin58°
7 mm i 9
o O
9mm SO 7 X sin 58
9
e (7 x sin58°)
T = sIn T

r~41.3
Chapter 8 (Non-right angled triangle trigonometry) Exercise 8D.2 349

< Using the sine rule,


sinz® sin41°
17 14
17km '14km
. o 17 xsin4l®
sinz® = ————
14
L (17 x sin41°)
T = sin —_
14
r~ 52.8

a Let BAC be 2°.


. . B o 5 4 (o}

C Using the sine rule, LR k)


8 11
8cm . 1lcm o 8 X sin45°
11

5 =y (8 % sin 45°
B A T = s e Y
11
r ~30.9
BAC is approximately 30.9°.

Let ABC be z°.


B A . . in z° in 42°
Using the sine rule, oo
23 32
23cm o 23 X sin 42°
sing® = —————
32
G
" (23 X sin42°
T = sin .
32cm 32
B
T~ 28.7
ABC is approximately 28.7°.

< Let ACB be 2°. - .


B 30m Using the sine rule, sine? _ sin37
A 30 36
. o 30xsin37°
sinzg® = —/————
36
36m
Loea (30 X sin 37°
T = sin —_
36
r~30.1
C
ACB is approximately 30.1°.

. 1 o in 48°
.
a Using the sine rule, Sm; s

ging© _ T X sin48° Sm 7m
’ ° 48°
sinz® ~ 1.04
But sinz® is always between —1 and 1 (inclusive), so
we cannot solve for = and the question cannot be solved.
b This means that it is impossible to draw a real diagram with the dimensions Mr Whiffen has
given.
350 Chapter 8 (Non-right angled triangle trigonometry) Exercise 8E

4 a1 Using the sine rule, A


sinACB _ sin30°
7 : 9 9cm 7cm

oB 7 % sin 30°
. sinACB = o C YT B
o~ 1 o

. ACB =sin™! (%) or its supplement

- ACB ~22.9° or 180° — 22.9°


- ACB~229° or 157.1°
But 157.1+ 30 > 180, so this case is impossible.
. ACB ~ 22.9°
1] BAC ~ 180° — 30° — 22.9° {angles in a triangle}
. BAC =~ 127°

b Area of triangle ABC = 1bcsinA


A2 X9 x7xsinl27°
~ 25.1 em?

1 Using the exterior angle of a triangle theorem,


6° +28° = 53°
. 0=25
Let the flagpole be & m high.

Using the sine rule, — T - - 20 and sinb3° = b


sin 28° sin 25° T
__ 20 X sin 28° oo h=xsin53°
sin 25° ~ 22.22 x sin 53°
T R 22.22 ~17.7m
the pole is approximately 17.7 m high.

2 Q By the cosine rule:


m PR? = 63% + 1752 — 2 x 63 x 175 x cos 112°
P R - PR=+/632+ 1752 — 2 x 63 x 175 x cos 112°
PR =~ 207
So the distance from P to R is approximately 207 m.
Chapter 8 (Non-right angled triangle trigonometry) Exercise 8E 351

3 By the cosine rule:


2202 + 3402 — 1652
cosT =
2 x 220 x 340
T — cos—! <136775)
149600
T ~239
the tee shot was about 23.9° off line.

4 ) By the cosine rule:


x=1/62+4.52
-2 x 6 x 4.5 x cos 148°
o~ 10.1
the orienteer is about 10.1 km from her starting
point.

By the cosine rule:


22 = 23.8% +31.92 — 2 x 23.8 x 31.9 x cos 83.6°
o =1/23.8%2 +31.92 — 2 x 23.8 x 31.9 X cos 83.6°
o~ 37.6
B and C are about 37.6 km apart.

a =180° —40° {co-interior angles}


= 140°
PXC = 360° — 155° — 140° {angles at a point}
= 65°
Using the cosine rule in APXC:
PC? =42 + 6> — 2 x 4 x 6 x cos65°
PC = /42 +62 — 2 x 4 X 6 X cos 65°
{as PC > 0}
PC ~ 5.6315 km
So, Esko hikes about 5.63 km.
campsite

b By the cosine rule:


42 4 5.6315% — 62
cosf ~
2% 4 x 5.6315
_1( 4% +5.6315% — 62
©. 0= cos —_—
2 x 4 % 5.6315
. 0~ T74.9°
So, Esko hikes on a bearing of 40° + 74.9° ~ 115°.
352 Chapter 8 (Non-right angled triangle trigonometry) Exercise 8E

¢ i Ritva travels a total distance of 4 + 6 = 10 km.


distance
Ritva’s time taken to reach the campsite =
speed
_ 10km
" 5kmh—!
= 2 hours
distance
Esko’s time taken to reach the campsite =
speed
__ 5.6315 km
~ Bkmh-1
~ 1.88 hours
So, Esko will arrive at the campsite first.
ii Difference in time taken to reach campsite ~ 2 hours — 1.88 hours
~ 0.123 hours
~ 7.37 minutes
~ 7 minutes 22 seconds
So, Esko will need to wait for about 7 minutes and 22 seconds before Ritva arrives.

d Now [3=~180°—40° —74.9° {co-interior angles}


~ 65.1°
So, the hikers need to walk on a bearing of 360° — 65.1° ~ 295° to return directly to P.

By the cosine rule:


232 + 262 — 52
cosf =
2 x 23 x26

0 =cos™t (%)
-
goal posts 0~ 9.38°
~

the angle of view is about 9.38°.

0 =13.2°—-8.3°
=4.9°
DBC = 90° — 8.3° {angles in a triangle}
=81.7°
ABD = 180° — 81.7° {angles on a line}
81.7°
=98.3°

In AABD, by the sine rule, — D . 42


sin 98.3° sin 4.9°
- AD= 42 ><' sin 98.3°
sin 4.9°
AD =~ 486.56 m
Chapter 8 (Non-right angled triangle trigonometry) Exercise 8E 353

h+ 42
In AADC, sin13.2° =

h+ 42 ~ 486.56 x sin 13.2°


h+42~111.1
h ~ 69.1
the hill is about 69.1 m high.

9 By the sine rule, — LB ,150


sin40° sin 72°
g 180 x sin40°
. (180 — 40 — 68)° =T72° o sin 72°
’ a ~101.38

a sin22° & —2 b cos22° ~ 2


101.38 101.38
. 2~ 101.38 x sin22° oy~ 101.38 x cos 22°
.z~ 38.0 Sy =94.0
the tree is about 38.0 m high. .. the tree is about 94.0 m from the
building.

10 a N

b Using the cosine rule in ALPQ: 242 =212 4+ PQ? — 2 x 2.1 x PQ x cos 53°
5.76 = 4.41 + PQ? — 4.2 x PQ x c0s53°
o,
. PQ? — (4.2c0s53°)PQ — 1.35 = 0
Using technology, PQ ~2.98 or —0.453
But PQ >0, so PQ~2.98.
So, the yachts are about 2.98 km apart.
354 Chapter 8 (Non-right angled triangle trigonometry) Exercise 8E

sin LPQ ~ sin53°


¢ Using the sine rule in ALPQ,
2.1 2.4
- sinlPQ
o~
= 2.1 X sin53
1 O

2.4
= .—1(21 in53°
LPQ =sin~! (&)
2.4

LPQ ~ 44.3°
The bearing of the Queen Maria from the Porpoise is 223° — 44.3° ~ 179°.

N . in AC B _e sin 67°
1 Using the sine rule, =
C 12 15
=~ 12 X sin67°
sin ACB = ——————
15

.
S
ACB =sin
1 (1( 2 Xs=in67°
15km ykm

- ACB ~ 47.4°
Now, CAB = 180° — 67° — 47.4°
A m B . CAB & 65.6°

12 Distance = speed X time


\ So, after 45 min = 0.75 h,
AT =6 x 0.75 = 4.5 km
and AP =8 x0.75
=6 km
By the cosine rule:
A 6km =P PT? = 4.5% + 6% —2 x 4.5 x 6 X cos 120°
PT = 1/4.52 + 62 — 2 x 4.5 x 6 X cos 120°
PT ~ 9.12
So, after 45 minutes they are about 9.12 km apart.

13 a By the cosine rule:


QS? =82+ 122 —2 x 8 X 12 x cos 70°
. QS=+4/82+122—-2x8x
12 x cos70°
~11.93
total area = area of APQS + area of AQRS 4 12 km =8
area~ § x 8 x 12 x sin70° + 1 x 10 x 11.93 x sin 30°
~ 74.9 km?
b 1hais 100 m x 100 m = 0.1 km x 0.1 km
=0.01 km?
1 km? = 100 ha
area ~ (74.9 x 100) ha
~ 7490 ha
Chapter 8 (Non-right angled triangle trigonometry) Exercise 8E 355

14 Using the sine rule in AABC,


AC 200
sin 110° sin 38°
200 x sin 110°
. AC= -
sin 38°
~ 305.26

and in AACD, z_ 30526


sin 8° sin 30°
e 305.26 X sin 8°
: sin 30°
=~ 84.969
the metal strip is about 85.0 mm wide.
15 By the cosine rule:
872 + 1432 — 1762
cosf =
2 x 87 x 143

0 =cos™! <_2958)
: 24882
. 0~ 96.8°
Also by the cosine rule:
1022 + 1362 — 1762
cosa =
2 x 102 x 136
4 (—2076)
. Q= Cos
27744
©oar94.3°
. i in6
Now, by the sine rule, = py _ sin
143 176
143 x sin 96.8°
sin 3, =~
176
.1 (143 x sin 96.8°
. By~ sinT! (—)
176
- By ~ 53778
¢, =180° — B, — 6 {angles in a triangle}
~ 180° — 53.778° — 96.8°
b, ~ 29.394°
Also by the sine rule, > Py _ sina
136 176
136 x sin 94.3°
osinfy &
176
. —1( 136 x sin94.3°
Py AsinTt <—>
176
" By~ 50.404°
¢y =180° — B, — {angles in a triangle}
~ 180° — 50.404° — 94.3°
by ~ 35.304°
So, (4 + B, = 53.778° + 50.404° ~ 104.2°
and ¢, + ¢y ~ 29.394° + 35.304° =~ 64.7°
356 Chapter 8 (Non-right angled triangle trigonometry) Exercise 8E

the angles at each corner of the park are: ~104.2°


~94.3°

~96.8°
R 64.7°

Area of the park = area of two triangles


=3 x87x
143 x sinf + & x 102 x 136 x sina
~ 3 X 87 x 143 x sin96.8° + § x 102 x 136 x sin94.3°
~ 13100 m?

16 A In ABCD, using the cosine rule:


97.403°
BD? = 50% 4 100% — 2 x 50 x 100 x cos 83°
B . BD = 4/50% + 1002 — 2 x 50 x 100 x cos 83°
~ 106.213 m
106.213m In ABCD, using the sine rule:

< 97.855° sinBDC _ sin83°


100 m
50 106.213
50 X sin83°
sinBDC ~
106213
50 x sin 83°
BDC ~ sin™* (
106.213 )

BDC ~ 27.855°
BDE ~ 95° — 27.855°
~ 67.145°
In ABED, using the cosine rule:
BE? ~ 602 + 106.213% — 2 x 60 x 106.213 x cos 67.145°
. BE & /602 +106.213%2 — 2 x 60 x 106.213 x cos 67.145°
~ 99.654 m
In AABE, using the cosine rule:
502 + 802 — 99.6542
cos BAE ~
2 x50 x 80

502 + 802 — 99.6542>


. BAE ~cos™! (
8000

~ 97.403°
area of the property
= area ABCD + area ABED + area AABE
~ 4 % 50 x 100 x sin83° + 4 x 60 x 106.213 x sin67.145° + 4 x 50 x 80 x sin 97.403°
~ 7400 m?
Chapter 8 (Non-right angled triangle trigonometry) Exercise 8F 357

17 Suppose Sam and Markus are m and y m from the tree respectively, and the tree is & m high.

N In APST, tan25° =2 x

- h
" tan25°
~ 2.145h

In APMT, tan15° =2
y
! o h
Tl tan 15°
~ 3.732h

But STM = 65° {equal alternate angles}


and 100% = 2% + 42 — 2xy cos 65° {cosine rule}
10000 ~ (2.145h)% + (3.732h)% — 2 x (2.145)(3.732)h? cos 65°
10000 ~ 11.762 A?
h? ~ 850.17
h ~ 29.2
So, the tree is about 29.2 m high.

1 Using the sine rule, A


sinC _ sin40°
T 8 8cm 1lcm

§inC — :
11 x sin40 )

8 ¢ B
* o

C =sin? (%) or its supplement

C ~62.1° or (180 — 62.1)°


C ~62.1° or 117.9°

2% o sinBAC _ sin ABC (sine rule} A


a - b
sinBAC _ sin 63° 10.3em
8.4 10.3

sin BAC = Sexsngs


><1§1;1 &

™ / 8.4 X sin 63° r . g


. 03
BAC =sin™! (— ) or its supplement

. BAC ~46.6° or 180° — 46.6°


. BAC ~46.6° or 133.4°
We reject BAC = 133.4°, since 133.4° + 63° > 180° which is impossible in a triangle.
BAC ~ 46.6°
358 Chapter 8 (Non-right angled triangle trigonometry) Exercise 8F

sin ABC ~ sin38°


{sine rule}
22.1 16.5
. N 22.1 in 38° A
sin ABC = SEn
' 16.5
B pplemeni 22.1cm 165 cm
- ABC =sin™! (w)
16.5

. ABC
i ~55.5° or 180° — 55.5° ¢ B
. ABC =~ 55.5° or 124.5°
both of which are possible as 124.5° + 38° = 162.5° which is < 180°.

sinACB _ sin 18°


%S o {sine rule} A
sin ACB — 4.3 X sin 18°
’ N 3.1
g . o
o, ACB =sin"*! (%) or its supplement A

. ACB~25.4° or 180° — 25.4° h


~ ¢ 3.1km
. ACB = 25.4° or 154.6°
both of which are possible as 154.6° + 18° = 172.6° which is < 180°.

3 a Using the sine rule,


sinPQR _ sin50° P
11 9
- sinPOR— 11X sgin50° . o

PQR =sin~?! (%) or its supplement

. PQR ~69.4° or 180° — 69.4° R Q


PQR~ 69.4° or 110.6°
b P

11m 9m

. Q1 Q2

¢ For the case in which PQR ~ 69.4°:


i QPR ~ 180° — 50° — 69.4° {angles in a triangle}
. QPR ~ 60.6°
il Area of triangle PQR = 4 x 9 x 11 x sin QPR
~ 1%
2
9x 11 x sin 60.6°
~43.1 m?
Chapter 8 (Non-right angled triangle trigonometry) Review set 8A 359

iii QR — = PQ < {sine rule}


sin QPR sin PRQ
QR 9
sin60.6° sin50°
9 X sin 60.6°
QRN —
QR ~ 10.2 m
So, perimeter of APQR ~ (11 +9+10.2) m
~ 30.2 m

For the case in which PQR ~ 110.6° : P


i QPR ~ 180° — 50° — 110.6° {angles in a triangle} 19.4°
. QPR ~19.4°
~ 11m
i Area of triangle PQR = 4 x 9 x 11 x sin QPR 9m
~ 1 x9x 11 xsin19.4° 110.6°
~16.5 m? R
iii QRA = PQA {sine rule}
sin QPR sin PRQ
QR 9
sin19.4° ~ sin50°
9 X sin 19.4°
R~ —rs00
QR ~3.91 m
So, perimeter of APQR ~ (11 4+ 9+ 3.91) m
~ 23.9 m

REVIEW SET 8A

1 a sin(180° — ) = sin6 b sin(180° — #) = sin6


sin(180° — 120°) = sin 120° . sin(180° — 165°) = sin 165°
-, sin60° = sin 120° o, sin15° = sin 165°
So, the acute angle is 60°. So, the acute angle is 15°.

2 a cos(180° — ) = — cos¢ b cos(180° — ) = — cosf


cos(180° — 19°) = — cos 19° . cos(180° — 84°) = — cos 84°
. cos161° = —cos19° . c0896° = — cos 84°
So, the obtuse angle is 161°. So, the obtuse angle is 96°.
360 Chapter 8 (Non-right angled triangle trigonometry) Review set 8A

3 a b
7km
6cm

9cm 8 km

Area = § X 6 x 9 x sin83° Area = £ X 7 x 8 X sin30°


~ 26.8 cm? =28x 1
=14 km?
B 7.6m

9.3m

Area =1 x 7.6 x 9.3 x sin111°


~ 33.0 m?

Area = 2 x area of AABC


=2x £ x5x5 xsin65°
~ 22.7 cm?

5 a Let the remaining side have length z cm.


By the cosine rule: zem
7cm
2° =97+ 7 —2x9x
7 xcos81°
L =192 +T2-2x9xTxcos81° {as x>0}
9cm
. x~10.5
The remaining side is about 10.5 cm in length.

b Let the remaining side have length m.


6m 6.8m
By the cosine rule:
2% = 6% +6.8% — 2 x 6 x 6.8 x cos 130°
Trm

62 +6.82 —2x6x6.8xcos130° {as x>0}


. x=~11.6
The remaining side is about 11.6 m in length.

6 a Using the sine rule,


Tl 8
sin52° sin43° 8cm L
_ 8x5inb2°
T sin43°
T~ 9.24
Chapter 8 (Non-right angled triangle trigonometry) Review set 8A 361

. . 102 + 122 — 112


b Using the cosine rule, cosf = 107
12- 117
2 x10x 12

0 — cos— (102122 112 10m


- 2x 10 x 12 i
0 =cos™'(333)
0 ~ 59.2° 12m
¢ Using the cosine rule,
22 =4249% —2x4x 9 xcos37°
4
c=V2+92_2x4x9xcos37° {as z>0} o 2o
T~ 6.28
zem

If the unknown is an angle, you should use the cosine rule in order to avoid the ambiguous case.

By the cosine rule:


DB2 =72+ 112 — 2 x 7 x 11 x cos 110° A L L
. DB=+/T2+112—2x7x
11 x cos110° {as DB
> 0}
~ 14.922 cm

Total area = area AABD + area ABCD


~ 2 X Tx11xsin110° 4 4 x 16 x 14.922 x sin40°
~ 113 cm?

2 2 172
By the cosine rule: cosg = L H12 — 177
2x9x12

0 = cos—1 2 22217 172


(1
2x9x12
=64)
0 _= cos el (Tfi
0~ 107.2°
So, area of triangle ~ % x 9 x 12 x sin 107.2°
~ 51.6 cm?

10 a By the cosine rule: 72 =22 4+ 82 — 2 x x x 8 X cos 60°


o 49 = 2% + 64— 167 x % 8cm 7cm
o2 —8x+15=0
(z—=3)(z—5)=0
. x=3 or b LT

b There are two possible values for x, so Kady can draw


two triangles: 8cm

7cm

So, Kady’s response should be that she needs more cm\


information to know which triangle to draw.
362 Chapter 8 (Non-right angled triangle trigonometry) Review set 8A

11 a By the cosine rule: M


212 = 2% + 3% — 2 x & x 3 X cos 120° e
o4 1 6z x (-1)
=2 49— P
.
. 2?4312
2 439
-432=0
=
= e 0

¥ 34
-3+ /32 2 —_ 4(1)(—432)
i

2(1)
_ —=3E£V1737
- 2
3 3193
2 =0
But >0, so v =-5+34%~193
¢ Perimeter of triangle LMO ~ 3 + 21 4+ 19.3 cm
~ 43.3 cm

12 D ADB = 60° — 42° {exterior angle of triangle}


18° —18°

In AABD, —% _ = _ 30 {sine rule}


i sin42° sin 18°

'. r =
30

X sin42°
sin 18°

A 30m B A 64.96
Let the height of the tree be h m.
Now sin60° = 2
x

h =~ 64.96 x sin 60°


h ~ 56.3
So the tree is about 56.3 m tall.

13 Using the sine rule in AACD, — At = - 10


sin 102° sin 32°
. AC = 10xsin102°
sin 32°
. ACx185cm
ACD = 180° — 32° — 102° {angles in a triangle}
= 46°
. BCA =90° — 46°
=44°
- BAC = 180° — 44° — 68° {angles in a triangle}
= 68°
Now, BAC = ABC = 68°
. AABC is isosceles with AC = BC
. BCx=18.5 cm
r~18.5
Using the cosine rule in AABC, y2 =22+ 22— 2%z Xz x cos44°

.y~ /185241852 — 2 x 1852 x cos44° {as y > 0}


Loy~ 138
Chapter 8 (Non-right angled triangle trigonometry) Review set 8B 363

14 Let AC be
x km.
Now AC+CB=2x+10
and AB=xz+10—4
=+6 km

ACB = 180° —60° {angles on a line}


=120°
By the cosine rule:
(z+6)? = 2% +10% — 2 x 2 x 10 x cos 120°
2?4+ 12z + 36 = 2 + 100 — 20z x (—1)
12z + 36 = 100 + 10z
2z =64
T =32
So, the boat travelled 32+ 10 = 42 km.

15 a Using the sine rule, A


sin ABC _ sin42°
7 = 5 7cm 5cm

sin ABC
~
= 7 X sin42
1 o

C B
7 X sin 42°
- ABC =sin~! ( ) or its supplement
5

- ABC ~69.52° or 180° — 69.52°


- ABC~69.5° or 110.5°
b For ABC ~ 69.5°, CAB ~ 180° —42° —69.5° {angles in a triangle}
~ 68.5°
area of AABC &~ 1 x 7 x 5 x sin 68.5°
~ 16.3 cm?
For ABC ~ 110.5°, CAB = 180° —42° —110.5° {angles in a triangle}
~ 27.5°
area of AABC ~ § X 7 x 5 X sin27.5°
~ 8.09 cm?

AT/ 147 Y ] ]

1 a sin(180° — #) = sinf b sin(180° — #) = sin6


sin(180° — 31°) = sin31° . sin(180° — 62°) = sin 62°
*. sin149° = sin 31° . sin118° = sin 62°
So, the obtuse angle is 149°. So, the obtuse angle is 118°.

2 a cos(180° — @) = — cos@ b cos(180° — §) = —cos@


cos(180° — 122°) = — cos 122° . cos(180° — 175°) = — cos 175°
-, cos58° = —cos122° . cosb® = —cos175°
So, the acute angle is 58°. So, the acute angle is 5°.
364 Chapter 8 (Non-right angled triangle trigonometry) Review set 8B

3 a By the cosine rule:


1lcm
13em o 132 +192 — 112
2x13x19

o -1 132+192-112
19cm Lo 2% 13 % 19
2° =cos ! (%)
x~34.1

b By the cosine rule:


e o 2% =152 + 172 — 2 x 15 x 17 x cos 72°
2 =1152+172
—2 x 15x 17 X cos 72°
2 km coox~18.9

L Let the included angle be 6.


area = 80 cm?
11.3cm
1 x11.3x19.2 x sinf = 80
160 19.2cm
sinf = —————
11.3 x 19.2
o 160 .
0 =sin~! <—) or its supplement
11.3 x 19.2
0~ 47.5° or 180° —47.5°
0~ 47.5° or 132.5°
So, the included angle is either =~ 47.5° or 132.5°.

5 a Using the cosine rule:


52 + 62
— 32
cos)
= ———
6cm 3cm 2x5x%x6

G =cos—t A6 =3
o 2x5x6
5cm . 52
0 =cos ™t (E)
0 ~29.9°
Using the cosine rule:
b 1lcm
82 4+ 112
— 152
cosf) = ——
2x8x11
8cm
0= cos—1 82 +11%2
— 152
15cm s 2% 8x 11
. —40
0 =cos™ ( 176 )
0 ~ 103°
Chapter 8 (Non-right angled triangle trigonometry) Review set 8B 365

C Using the cosine rule:


A AC? =117 +9.8% — 2 x 11 x 9.8 x cos T4°

11cm
9.8cm L AC=1/112 1 9.82 — 2 x 11 x 9.8 x cos 74°
. AC =~ 12.554 cm
B - AC~12.6 cm
. rule,
. the sine
Using 22inACB ___ ssin74°
11 AC
. = 11 in 74°
sin ACB B
~o 235072
12.554
11 X sin 74°
- ACB ~sin~! ( ) or its supplement
12.554

- ACB ~57.4° or 180° — 57.4°


- ACB ~57.4° or 122.6°
!
impossible as 122.6° + 74° > 180°
ACB measures about 57.4°
and BAC measures 180° — 74° — 57.4° ~ 48.6°.
Q PRQ = 180° — 46° —83° {angles in a triangle}
83° =51°
R Using the sine rule,
PQ _ 10 and QR 10
460~ 10cm sin51° sin83° sin46° sin83°

- Pe= B R
) o

v
) o

. PQ~T7.83 cm . QR=~=7.25cm
X

14cm 16 cm

Y 19cm z

Using the cosine rule:


= B 2 _ 1 9 2 2 2 2 _ 162
cos YXZ = et 167 19° cos XYZ = 147 +19° —16%
2x 14 x 16 9% 14 x 19

- szzco
% s fl
2 ( M
2 _ ) 192 . XY
. Z: co s* (M 2 +19% 2 _ 16 162 )
2x 14 x 16 2><14><_19

L YRZ = cos°”! () -- XYXTZZm~ co55.sm5° ()


" YXZ ~783
XZY = 180° — YXZ — XYZ {angles in a triangle}
~ 180° — 78.3° — 55.5°
. XZY =~ 46.2°
366 Chapter 8 (Non-right angled triangle trigonometry) Review set 8B

7 a Using the sine rule,


T 6
sin 39° sin 33°
6cm
_ 6 x5sin39°
sin 33°
T cm
o ~6.93

b Using the cosine rule,


7cm 2?2 =824+ 7% -2 x 8 x 7 x cos99°
8cm
Lo =14/82+T72—-T 2x8
xcos99

o114
rcm B

< 43m The unknown angle is


- 180° — 32° —115° {angles in a triangle}
= 33°
. 5 4.3
m h sine rule,1
Using the z_—
e Saas

.T 43xsin115°
YT sin33e
T~ T7.16

Using the sine rule,


sinz® sin 100°
5.2 9
5.2 x sin 100°
. sinz® =
9
.1 (5.2 X sin 1000)
. X = SsIn T

or A 34.7

2.135
The throwing circle has radius = 1.0675 m.
OS = 1.0675 4 17.64 S
= 18.7075 m T

gl
17.64m

1.0675m
20°

Using the cosine rule,


PS? = 1.0675% + 18.7075% — 2 x 1.0675 x 18.7075 x cos 20°
PS = \/1.0675% + 18.70752 — 2 x 1.0675 x 18.7075 x cos20° {as PS> 0}
PS~17.7m
So, Paul actually put the shot approximately 17.7 m.
Chapter 8 (Non-right angled triangle trigonometry) Review set 8B 367

9 Let the height of the tree be 2 m.


180° —90° — 18° =72°
Using the sine rule, —
h 50
sin 8° sin 720
50 x sin 8°
= Tem720
h ~ 7.32
So, the tree is about 7.32 m high.
10 The unknown angle is 180° — 68° — 71° {angles in a triangle}
=41°
. . AB 150
Using the sine rule, — ==
sin 71° sin41°
-~ AB = 150 >< sin 71° 68°
sin 41° 150 m

. AB~216.18 m
71°
. . BC 150
Also using the sine rule, — = —
sin 68° sin41°
BC — 150 >< sin 68°
sin 41°
BC ~ 211.99 m
So, the perimeter of the triangle ~ 150 + 216.18 4 211.99 m
~ b78m
Area of the triangle = bcsinA
~ 1 x 150 x 216.18 x sin 68°
~ 15000 m?

1 ASP = 210° — 113° = 97°


By the cosine rule:
AP? = 310% 4 430% — 2 x 310 x 430 x cos97°
. AP = /3102 4 4302 — 2 x 310 x 430 X cos97°
{as AP >0}
. AP ~ 559.90 m
Using the sine rule, Sng xrn
430 AP
sinf ~ 430 x sin 97°
559.90
- §resin=t (430 X sin97°>
559.90
0~ 49.664°
Now ASN = 360° — 210° {angles at a point}
= 150°
So, Peter is about 560 m from Alix on a bearing of (180° — 150°) + 49.664°
~ 079.7°
368 Chapter 8 (Non-right angled triangle trigonometry) Review set 8B

12 Using the cosine rule in AABC:


AC? =72 4+9% -2 x 7x9 x cos 105°
AC=1/724+92-2xT7x9xcosl05° {as AC> 0}
. AC~12.75 m
Using the cosine rule in AACD:
=~
cos ADC s 12122 107
4102
—— 12757
12.752
2x12x 10
T B 122 2 i 2
. ADC
~ cos! e a7
2x12x10

ADC ~ 70.2°
Using the sine rule in AABC: Using the sine rule in AACD:
sinBAC _ sin 105° sinCAD __~ sin70.2°
9 AC 10 AC
St 9 in 105° AR 10 in 70.2°
- sinBAC & 22300 - sinCAD ~ 29002
12.75 12.75
. BAC ~ sin—1 (9 X sin105°> CAD ~ sin—! (10 X sin 70.2")
12.75 12.75

.. BAC ~ 43.0° - CAD ~47.5°


Now BAD = BAC + CAD
. BAD ~ 43.0° +47.5°
- BAD =~ 90.5°
Also, BCD = 360° — 105° — BAD — ADC
{angles in a quadrilateral}
A~ 255° — 90.5° — 70.2°
~ 94.3°

13 Q Using the sine rule,


sin@Q sin47°
9.6 11
m - sinQ = 9.6 X sin
3
47 0

11
iy ATY 9.6><sin47°>
e < 11
P 96m R o Q307
14 a By the cosine rule:
BD? = 120% + 1252 — 2 x 120 x 125 X cos 75°
. BD= \/1202 + 1252 — 2 x 120 x 125 X cos 75°
{as BD >0}
. BD~149.2 m
The area of the block
= area of AABD + area of ABCD
~ 4 % 120 x 125 x sin 75° + 3 x 149.2 x 90 x sin 30°
~ 10600 m*
Chapter 8 (Non-right angled triangle trigonometry) Review set 8B 369

b Area~ 10600 m>


~ (10600 = 10000) ha {10000 m? = 1 ha}
~ 1.06 ha

15 a B By the cosine rule, 62 = 22482


—2 x x x 8 x cos44°
36
= 22 + 64 — 16z x cos44°
rm 6m 2?1151+
28~ 0
L5144 /11.512 — 4(1)(28)
TR
2
A 8m c 11.51 + 4.524
x%%

r~8.02 or 3.49
Frank needs additional information as
there are two possible cases:
(1) when AB~ 8.02m and 8.02m B
6 m
(2) when AB =~ 3.49 m 3.49m 6m
44° 44°
8m
Case (1) Case (2)
b The area of the plot is a maximum when z ~ 8.02 m.
Volume = area x depth
=1 x8xzxsindd®x01 {10 cm=0.1m}
~ 4 x 8.02 x sin44° x 0.1
~2.23 m®

16 a i The sum of the interior angles of a regular hexagon is


M o omm (6—2) x 180° = 720°.
720°
A (R B - AMB= =120°

By the cosine rule in AAMB:


AB? =122 +12% — 2 x 12 x 12 X c0s120°
. AB= /122 +122 -2 x 12 x 12 x cos120° {as > 0}
AB
. AB ~20.8 mm
il Area of the hexagon = 2 x area of AAMB + area of rectangle M
~2x 3 x12x12xsin120° x 812
+ 20. I.@mmm
~ 374 mm> A B

b Area of circular hole in nut = 7 x (%)2


= 16w mm?
Volume of nut 8 mm
= area of base x height
= (area of hexagon — area of circular hole) x height
~ (374 — 167) x 7 mm® -
12
~ 2270 mm®
Chapter 9
POINTS IN SPACE

2 a i AB=+/(6-0)2+(-4-0)2+(2-0)? il The midpoint is


= /62 + (—4)2
4 22 (0+6 0+ -4 0+2)
2’ 2 7 2 )
=v36+16+4 hichis (3, -2, 1)
which 1s , —2, 1),
=56
= 2v/14 units
Chapter 9 (Points in space) Exercise 9A 371

i AB=1/(0-4)2+(1-1)2+(-2-0)2 ii The midpoint is


I TE TR <4+0
Sl
1+1
0
O+—2)
a2
— V65071 which is (2, 1, —1).
=20
= 2V/5 units

| AB= E-DP T (3= IPF 02 ii The midpoint is

— VTP R <1+5
27
-1+ -3
2 7
2+0)
2 )
=V16+4+4
which is (3, —2, 1).
=24
= 2v/6 units

i AB=/(=6— 22+ (T— 02+ (3—5)? ii The midpoint is


= (42 + 72+ (—2)2 (—2+—6 047 5+3)
2 027 2 )
—V16+4914
which is (=4, %, 4).
69 units

| AB= A= P T =BP F (1-2P ii The midpoint is

— FI AP aR (—l+4
2 7
5+1
27
2+—1)
2 )
=V25+16+9
which is (%
=50
= 5v/2 units

i AB=/(-5-2)2+(3—-6)2+(2——3)? ii The midpoint is


= (-7)?%+(-3)2 +52 (2+—5 6+3 —3+2)
oW oL 2 ?
=Vv49+9+25
which is (-3, 5
ol

D=
83 units

AB=/(-5—-2)2+(5—1)2+
(3 —3)2 AC=+/(-2-2)2+(3—-1)2+
(6 — —3)2
= ,/(,7)2+42 + 62
/T2 e
= V49
+ 16 + 36 =+v16+4+81

= /101 units = 1/101 units

BC=+/(-2- 52+ (3-52+(6-3)? A

= /324 (-2)? 4 32
=V0+4+9 V101 V101

22 units

AB = AC = /101 units and BC # AB,


8

so AABC is isosceles.
372 Chapter 9 (Points in space) Exercise 9A

b AB=/(-1-3)2+(-4—-1)>4+(0—5)2

= VO + (37 + (-5
— VIETOT )
=50
= 5v/2 units 53

AC=+/(2=32+(T——1)2+(-3-5)
= /12 + &+ (-8)2 B
=V1+64+064
= /129 units c

BC=\/2— 12+ (7— 42+ (300


= /32 +112 4+ (-3)2
=v9+121+9
= 1/139 units

AB # AC # BC, so AABC is scalene.

4 AB= /(-6 -3+ (712


+ (13— —2)2 AC= /(532 +(9—1)2+
(—4— —2)?
= /(=9 + 62 + 152 =22+ 82 + (-2
=B +36+225 =VI+61+4
= 1/342 units = /72 units

BC—E~ 6P T E 77T (A 17 B
_ BT 2T (IR
+4 + 289
=121
= V414 units V312 Vial4

AB? + AC? = (v/342)2 + (V72)? = 414


and BC? = (V414)% = 414
triangle ABC is right angled at A. A V72 C

5 a The midpoint of [PQ] is (# 4+2‘8, ‘12+ 7) which is (3, ~2, 3).

The midpoint of [QR] is (6+2‘5, Sz ”2‘9) which is (1, =5, —1).


So,Mis (%,-2,3) andNis (3, -5, —1).
Chapter 9 (Points in space) Exercise 9B 373

b PR=/(-5-1)2+(-2—4)2
+ (-9
— VO T (oP+ (57
= /36 + 36 + 64
=136
= 2v/34 units

MN = /(3= 9)" + (=5~ —2 + (-1 - 3)2


= VT (B2 1 (4P
—VITOTTS
= v/34 units
=3 1 xPR

So, [MN] is half the length of [PR].

6 PQ (k—2)2+(-1—4)2+(-2—--32=7
k2 —4k+4+(-5)2+12=7
Sk —4k+4+25+1=7
k2 —4k+30=7
o k?—4k+30 =49 {squaring both sides}
ook =4k =19
ok —4k+(—2)? =19+ (-2)* {completing the square}
(k—2)?=23
L k—-2= ::\/fi

k=24 \/fi

Volume = length x width x height Volume = length x width x height


=3x3x3 =5X2x6
= 27 units® = 60 units®
374 Chapter 9 (Points in space) Exercise 9B

Volume = length x width x height


=b5x4x2
= 40 units®

2 a Dis (-7,0,3) andEis (—T7,4,0).


b Volume = area of end x length
= 1 x base x height x length
=1x4x3x7
= 42 units®

¢ AB=+/(0-02+(4-0)2+(0-3)?
= /T2 (32
=V0+16+9
=25
= 5 units

d Surface area of prism


= area of base + area of 2 triangular faces + area of 2 rectangular faces
=7x4 + 2 x area of AOAB + area of quadrilateral OADC + area of quadrilateral ABED
=28 +2x3x4x3 + 7x3 + 7x5
= 96 units®

3 a To find the centre of the base, we locate the


midpoints of the diagonals.

The midpoint of [OB] is (% 0+62 M)


2
which is (3, 3, 0).
The midpoint of [AC] is (% 0+62 M)
2
which is (3, 3, 0).
the centre of the base is (3, 3, 0).
the apex (3, 3,9) lies directly above the
centre of the base.

b Volume = % (area of base x height)


=1x6x6x9
= 108 units®
Chapter 9 (Points in space) Exercise 9B 375

¢ i Mis (% % %) which is (6, 3, 0).


i MD=\/3—62+(3-3)72+(9-0)
_T
=v9+0+81
=90
= 3/10 units
i Area of triangle ABD = 1 x 6 x 3V/10
= 9v/10 units?
Surface area of pyramid
= area of base + area of 4 triangular faces
—6x6
+ 4x9VI0
=36+ 36V'10 Al o i
M
= 36(1 + V/10) units? ==y

& The midpoint of [OB] is ((”210, ks 0”°>


which is (5, 9, 0).
1040 0 1800)
The midpoint of [AC] is ( 0 nlegle® O PR O
which is (5, 9, 0).
the centre of the base is (5, 9, 0) which
lies directly below the apex (5, 9, 12).
. ) M B(10,18,0)
Volume = 3 (area of base x height)
= % x 18 x 10 x 12
= 720 units®
Let the midpoint of [AB] be M.

M is (10;105 0;“,%) which is (10, 9, 0).

MD = /(5 —10)2 4+ (9 —9)? + (12— 0)? D


AR
=25+ 0+ 144
=169
= 13 units
Area of triangle ABD = % x 18 x 13 B 2
= 117 units? e 18
376 Chapter 9 (Points in space) Exercise 9B

Let the midpoint of [BC] be N.

Nis (1‘”0,
2
e2
M)
2
which is (5, 18, 0). D

ND =+/(5- 5)2+
(9— 18)% + (12— 0)2
— P (R 12
=0+ 81+ 144
=225
[1
= 15 units B N c
-— 1) —>
Area of triangle BCD = £ x 10 x 15
= 75 units®
Surface area of pyramid = area of base + area of 4 triangular faces
=18 x 10 + 2 x area of AABD + 2 X area of ABCD
=180 + 2x 117 + 2x 75
= 564 units?

5 a Base radius of cone


= distance from centre (0, 0, 0) to point (4, 5, 0)
=/A=0?+(5-02+(0-0)2
= V42152402
=vV16+25+0
= /41 units

b Volume of cone = (area of base x height)


=1x7mx(V41)*
x6
= 827 units®
¢ Let the slant height be s units.
52 =62 + (v/41)> {Pythagoras}
s =36+41 6 units S units
§° =177
s =717 {as s >0}
So, the slant height of the cone is v/ 77 units. 41 units

d Surface area of cone = 7rs + 772


=7 x VAL x V7T + 7 x (V41)?
~ 305 units?
Chapter 9 (Points in space) Exercise 9B

Radius of sphere
= distance from centre (2, 3, —1) to point (—4, —6, 10)
(—4—-2)2+(-6—3)24 (10 — —1)2
= V(O (9 112
+ 121
+ 816
= /3
= 1/238 units
Volume of sphere = $77°
=4 57 x (V238)3

Centre of sphere = midpoint of [PQ]


3 (71+75 147 2+—8)
B ol wle L 2
=(-3,4, -3)

Radius of sphere = distance from centre (—3, 4, —3) to point P(—1, 1, 2)


=/(CI-32+(1—-42+@2--32
=21 (=32 +5

=V4+9+25
= /38 units

Volume of sphere = 3777


=2 x 7 x (V38)?
~ 981 units®
Surface area of sphere = 477>
=4 x7TX (\/@)2
~ 478 units?

i A has coordinates (10, 40) on the Ay(m B


2-dimensional plane. 150 1
A is (10, 40, 0) on the 3-dimensional .
plane. C

B has coordinates (50, 160) on the


2-dimensional plane. |
Bis (50, 160, 0) on the 3-dimensional i
plane.
C has coordinates (110, 140) on the
2-dimensional plane. 50
Cis (110, 140, 0) on the 3-dimensional .
plane. A
D has coordinates (70, 20) on the °D
2-dimensional plane. e & (m)
Dis (70, 20, 0) on the 3-dimensional 5 100
plane. Y
378 Chapter 9 (Points in space) Exercise 9B

il The apex is 15 m above the centre of the base.


To find the centre of the base, we locate the midpoints of the diagonals.

The midpoint of [AC] is (10 2, 10 e, %) which is (60, 90, 0).


The midpoint of [BD] is (50 - o 2z, %) which is (60, 90, 0).
the centre of the base is (60, 90, 0).
the apex is (60, 90, 15).

b We need to find the side lengths of the base of the tent. T(60,90,15) B(50,160,0)

AD = /(70 — 10)2 + (20 — 40)? + (0 — 0)2 C(110,140,0)


- TP 3
= /4000 m
CD = /(70 — 110)2 + (20 — 140)2 + (0 — 0)2 N
=

/(—40)2
V16000m
+ (—120) + 02 [
A(10,40,0)
s (70,20,0)
Volume of air inside tent = % (area of base x height)
= 1 x /16000 x v/4000 x 15
= 40000 m*
¢ Let the apex (60, 90, 15) be T.
Let the midpoint of side [AD] be M.

M is (10 10, 2% M) which is (40, 30, 0).


MT = /(60 — 40)2 + (90 — 30)% + (15 — 0)?
=1/202 + 602 4 152
= V4225
=65m
Area of triangle ADT = % x /4000 x 65
= £/1000 m*
Let the midpoint of side [CD] be N.

Nis
) (110+7O
2
)
140+20
2
040
o ) which is (90, 80, 0).
NT = /(60 — 90)2 + (90 — 80)2 + (15 — 0)2
=4/(—30)? + 102 + 152
=1225
=35m
Area of triangle CDT = % x /16000 x 35
- V16000 m >
= 31/16000 m*
Chapter 9 (Points in space) Exercise 9C 379

Area of material needed for the tent = area of 4 triangular faces


=2 x area of AADT + 2 x area of ACDT
=2 x £4/4000 + 2 x 32+/16000
~ 8540 m?

1 a The required angle is ABE.


Now AE = 6 units
and AB =5 units

. tanf = g
5
. f=tan"! (g)
~ 50.2°
The angle is about 50.2°.

b The required angle is CAG.


Now CG = 6 units
and AC=+/(0—2)2+ (5—0)2+ (0—0)2
=/(-2)? +52 402
=V4+25+0
= V29 units

. tanf = 0
V29
.
(_6
0 =tan ian—1(E)
~ 48.1°
The angle is about 48.1°.

2 a Ais (3,0,0) andBis (3,6, 0).

The midpoint
M of [AB] is (% 0—;6 0—;°) which is (3, 3, 0).
380 Chapter 9 (Points in space) Exercise 9C

b Dis (0,6, 2), and AADO is right angled at O.


Now OA = 3 units

and OD = /(0 —0)2+


(6 —0)2+ (2 — 0)2
=1/0% 462 + 22
=v0+36+4
= /40 units
3
. tan
an = ——
ADO\/AE

- ADO = tan™! (
N——
F o

- ADO ~ 25.4°
¢ Now BM
= 3 units
and BD = ./(0—-3)2+
(6 —6)2+ (2 —0)?
T
—VF07d
= /13 units

B tan BMD = g

= BMD = tan~! (@)

- BMD ~ 50.2°

3 a Qis (86,0).
The midpoint M of [QR] is (% 6;—6 %) which is (4, 6, 0).
b Tis (80,7).
Now QM = 4 units

and QT =+/(8—8)2+ (0—6)2+ (7 —0)2


=02+ (-6)2 + 72
=0 +36+49
= /85 units

; tanQIQ[T = @

. QMT = tan™! (@)

. QMT =~ 66.5°
Chapter 9 (Points in space) Exercise 9C 381

¢ 1 The required angle is OQS.


Now OS = 7 units
and OQ=+/(8—0)2+(6—0)%2+(0—-0)?
=1/82+62+02
= /64 + 36
= /100
= 10 units

tan an 0QS
0QS == o— S
0QS =tan™" () 7
. 0QS ~ 35.0°
The angle is about 35.0°. 0 10
ii The required angle is TMP.
Now PT = 7 units
and MP=./(8—4)2+(0—-6)%2+(0-0)2
= /42 + (=6)? + 02
=V16+36+0
= V52 units
< 7
an
. tanTMP 7
= —

C. TP —= tan tan—1( <¢—5_2)


TMP T

. TMP ~ 44.1°
The angle is about 44.1°.

4 a The midpoint M of [BC] is (# 4;—4 0;—0 ) which is (2, 4, 0).


b i The required angle is DMT, where T is the
centre of the base.
To find the centre of the base, we locate the
midpoints of the diagonals.
The midpoint of [AC] is

SN
(4+0
2
O 2
2 _)
0+4
2
040
which is (2, 2, 0).
The midpoint of [BO] is

(fl,
2
&2
M)
2
which is (2, 2, 0).
the centre of the base is T(2, 2, 0).
Now DT =5 units and MT = 2 units
. tanDMT = 3
- DMT = tanfl( )
nojot

. DMT =~ 68.2°
The angle is about 68.2°.
382 Chapter 9 (Points in space) Exercise 9C

il The required angle is DAT.


Now DT =5 units
and AT =\/2— 22+ (2- 02+ (0—0)2
= 1/(,2)24,22 + 02

=VI+1+0
= /8 units

i tanDKT:%

.
. DAT
AT
=—
tan —-1( (fi)
5

- DAT ~60.5
The angle is about 60.5°.

5 a Dis (-8, 3,5).


The midpoint
M of [AD] is (‘”2‘8,3_;3, 5_;5) which is (—4, 3, 5).
b 1 The required angle is AEN, where N is the
midpoint of [OB].

Nis (28
2
28
2
2E0)
2
which is
(0,3,0), and Eis (-8, 6, 0).

Now AN = 5 units
and EN=./(0——8)2+(3—6)2+(0—-0)?
= /B 1 (-3 + 02
=v64+9+0
= /73 units

- tan AEN = \/%

. EN =— tan tan—' (\/fi)


AEN (==

- AEN~30.3°
The angle is about 30.3°.
Chapter 9 (Points in space) Exercise 9C 383

ii The required angle is MBT, where T is the


centre of the base.
To find the centre of the base, we locate the
midpoints of the diagonals.
The midpoint of [OE] is

(°+ 2 = 2
w)
2
whichis (4, 3,0).
The midpoint of [BC] is

(“‘8
2
6+0
727
O—;O) whichis (—4,3,0). XA
the centre of the base is T(—4, 3, 0).
Now MT =5 units M
and BT = \/m

=vV16+9+0
=V25
= 5 units i 5 B

- tanMBT= g -1
MBT = tan~'(1)
MBT = 45°
The angle is 45°.

< AB=/(0(6 —0 )2
32+ +
(057
= /021321 (-5)
=V0+9+25
= V34 units
4
. tan
an = ——
ABM m

ABM = tan~! (
N—
&kg

ABM =~ 34.4°

6 a To find the centre of the face ABCD, we locate the


midpoints of the diagonals.
Ais (5,0,5), Bis (5, 6,5), Cis (0,6,5), and
Dis (0,0,5).
The midpoint of [AC] is (5—;° 0—;6 %)
which is (%, 3, 5).
The midpoint of [BD] is (5—;4), 6—;4), ?)
which is (3,3, 5).
the centre of the face ABCD is M(3, 3, 5).
384 Chapter 9 (Points in space) Exercise 9C

b i Eis (5,0,0) and Fis (5,6, 0).


Now EF = 6 units
and DE=+/(5—0)2+ (0—0)2+ (0—5)?
= /52102 + (-5)
=v25+0+25
= V50 units

-, tanEDF = S
V50
. Ef)F:tan_1< 6 )
-, EDF ~ 40.3° r}l
S

ii Now AE =5 units

and AM = (%75)2+(370)2+(575)2

= /(=82 +32 402


=\Z2+9+0
= ,/8L
=\V71
=%units

ME —(@
tanAME — 10
2 /fi—
= — >—\ 1
2

.. AME =~ 52.0°

7 a Aylais located at (30, 20). Ay (m)


b The bird is sitting in a tree, 10 m directly above i i i
Ayla. We describe the location of the bird using 20 =
3-dimensional coordinates. fi'fi i -
the bird is located at (30, 20, 10). 40 LA
c i The worm is located at %, which is at i/
>
(10, 10, 0). 30
The distance from the worm to the bird
20,
= /(30— 10)2 + (20 — 10)2 + (10 — 0)?
= /202 £ 102 + 102 10 o %
= 1/400 + 100 + 100

= V600 0 20 30 4 ~*m
=10v6m
~ 24.5 m
Chapter 9 (Points in space) Exercise 9C 385

i Let the angle that the bird flies to the ground be 6. bird

sin 0 T i£ V6
106 10v6
= 10m
0 _= sinwn—1(_10
(10\/6)

0~ 24.1° worm -
So the bird flies at an angle of about 24.1° to the ground.

8 a The plane is 500 m, or 3 km above (-3, 4). L y (k) N


. the plane is at (73, 4, %) W"I"E
b Distance of plane from control centre // S
/ A\
=4/(0—=3)2+(0—4)2+(0—3)? x (km)
=5 5
=B (e \ N L
/
=4/9+16+% \ ~__|
—5

=\
AT
_ Jio1
=72
~ 5.02 km
¢ The aeroplane is at (73, 4, %), so it is 3 km west N
and 4 km north of the control centre.
The runway is 1 km east of the control centre, so it
plane P¢
is 34+ 1=4km east and 4 km south of the plane.
APQR is right angled isosceles, with :
QP = QR = 4 km. dkm+
QPR = QRP = 45° {equal base angles} E
the runway is at a bearing of 90° + 45° = 135° Q
from the plane. 4km R

d Pisat (—3,4,%) andRisat (1,0,0).

PR=/(1—--3)2+(0—4)2+(0—
1)
|
— e (0 (-
16+ 16+ 1

S
(o
rof=
)
o
=
3
386 Chapter 9 (Points in space) Review set 9A

0.5
Now cosf = —
PR

©. cosf = ( 012 ) =fi

§
2

=11
(\/fi)
. 6 =cos

0~ 84.95°
So, the angle of the plane’s descent is 90° — 84.95° ~ 5.05°.

1 a b <

°(3.1.0)
X

2 a i PQ=+(-3-12+(-6—--2)24(2-0)2 i The midpoint is


4+ (-4)2 +22
=/(—4)2 (1+73 -2+ -6 0+2>
P 2 72
=16+ 16 +4 -
which is (-1, —4, 1).
=36
= 6 units

b i PQ=+/(-2--3)2+(-7—-1)2+(1-6)? il The midpoint is


= 4 (-5)2
/12 + (=8)2 <73+72 1+ -7 6+1)
2 72 7 2
=V1+64+25 e s .
— /90 which is (—5, -3, 5).

= 3+/10 units

3 AB=+/(3—-2)2+(5-5)2+(-3-1)2 AC=+/(0——-2)24(-1-5)2+(2—-1)2
= /52 4+ 02 + (—4)? =4/22+(-6)2
412
=+v25+0+16 =V4+36+1
= V41 units = V41 units

BC=/(0—3)2+(—1-52%+(2—
—3)2 N
— VT (CoP + 5
=v9+36+25
= V70 units

AB = AC = /41 units and BC # AB, V70


so AABC is isosceles.
Chapter 9 (Points in space) Review set 9A 387

Radius of hemisphere = distance from centre (0, 0, 0) to point (2, —5, 0)


VB P50 0P
CVETPI®
=V1+25+0
= V29 units

Volume of hemisphere = 1 x 477 3

=2 x 7 x (v29)?
~ 327 units®
Surface area of hemisphere = % x 4mr? 4 72
= 3mr?

=3 xmx (V29)?
~ 273 units’

The midpoint
of [OB] is (0_;8 % %)
which is (4, 4, 0).

The midpoint
of [AC] is (8—;0 % %)
which is (4, 4, 0).
the centre of the base is (4, 4, 0) which
lies directly below the apex D(4, 4, 6).
Volume of pyramid
= 1 (area of base x height)
=1 x8x8x6
=128 units®
The midpoint
M of [AB] is (% % %) which is (8, 4, 0).
MD=/E-8P
T G—47+ 60y
_ O
=+16+0+36
=52
= 2v/13 units

Area of triangle ABD = 1 x 8 x 213 D


= 8V/13 units?
Surface area of pyramid
= area of base + area of 4 triangular faces
=8x8 + 4x8/13
=64+ 3213
= 32(2 + /13) units®
~ 179 units?
388 Chapter 9 (Points in space) Review set 9A

e Now MB = 4 units and MD = 2,/13 units

-, tanMDB = MB
MD
4
BENE
.. 4
"R =— tan -1 (—2\/fi)
MDB

-, MDB = 29.0°

6 The point (—3, —4, 8) lies directly above the point


(=3, =4, 0) on the X-Y plane.
Radius of cylinder
= distance from centre (0, 0, 0) to point (—3, —4, 0)
=/(-3-0)2+(-4-0)2+(0-0)?

~ VP (APt
=v9+16+0
=25
=5 units
The point (—3, —4, 8) is 8 units above the X-Y plane, so the height of the cylinder is 8 units.
Volume of cylinder = mr-2h
=7x52x8
~ 628 units®
Surface area of cylinder = 27rh 4 2772
=2xmTx5x8 + 2xmwx5H
= 807 + 507
= 1307
~ 408 units®
7 a Qisat (5,4,0) and Uisat (5,4, 6).
The midpoint
M of [UQ] is (5_;5 % %) which is (5, 4, 3).
b Now UM = 3 units
and SU=/(5—-0)2+(4—-0)2+
(6 —6)
= /52 +42
+ 02
=v25+16+0
= V41 units

- tanUMS = @ S Va1 U
3

- UMS = tan™" (3@) : j?’

- UMS ~ 64.9° N
Chapter 9 (Points in space) Review set 9A 389

¢ 1 The required angle is RPV.


Now RV = 6 units
and PR=./(0—5)2+ (4—0)2 + (0 — 0)2
= /(=52 + 42 102
=+v25+16+4+0
= V41 units
= 6
. an = —
tanRPV\/H

- . RPV
RPV ==tan
tan— [ (JH)
—5=

. RPV ~43.1°
The angle is about 43.1°.

ii The required angle is MOQ.


Now MQ = 3 units
and OQ = PR = v/41 units
{diagonals of rectangle are equal in length}
M = e S3 M;
. tanMOQ T

-,- MOQ
MOO == tan tan—1( (m)
-3 OA\/H 9
- MOQ ~ 25.1°
The angle is about 25.1°.

8 a The fossil at P has coordinates (2, 7, —2.5). {O‘y m


The fossil at Q has coordinates (8, 3, —2.9).

b PQ=+/(8-22+(3-7)2+(—29—-25)2 %
= /624 (—4)2 + (—0.4)? »
=36+ 16+ 0.16
= V52.16 "Q
~7.22m
The distance between the fossils is about 7.22 m. - 5 z1m>
v

¢ FossilQis 2.9—2.5=0.4 m deeper underground than fossil P.


.
sinf = 0.4
P
Y= Usaie -
i. 9*sin71<
— . > 04.4m
V52.16 V/52.16 m
O~ 3.17°
The angle of depression from P to Q is about 3.17°. Q
390 Chapter 9 (Points in space) Review set 9B

REVIEW SET 9B 3

1 a 1 AB=/(-1—-3)2+(6-0)2+(4—5)? il The midpoint is


=/22 462+ (—1)2 (—3+—1 0+6 5+4)
O i o2 iy WO
=V4+36+1
which is (-2, 3, §).
= /41 units

b i AB=4/(—2—-7)2+(1—-4)2+(-1-6)? il The midpoint is


— /52+(,3)2+(,7)2 (—7+72 441 6+71>
A== =0y =D
=v25+9+49
which is (7

olo
vl
ol
> 9 ) [
= /83 units

2 a PQ=+/(—2— 52+ (—2-02+(2—1)?


=3+ (=22 +12
=V0+4+1
= v/14 units P

PR=+/(-1—-5)2+(5-0)2+(-1-1)2
52 + (-2)2(-2)
= /42 ++52+ e T
=V16+25+4
= V45 units

QR=+/(-1— =22+(65— —22+ (-1 —2)2


= /12472
+ (—-3)2

=v1+49+9
= V59 units

PQ? + PR? = (V14)? + (V/45)? = 59


and QR? = (v/59)? = 59
triangle PQR is right angled at P.

An _ Vi5
b tanPQR = fi P V/45 units R

. P@R: tan71< )
=k

* PQR ~60.8° /14 units


/59 units
Chapter 9 (Points in space) Review set 9B 391

3 The distance from (4, =2, 1) to (1, 3, k)


is VI-42+(B-—22+(k-12=8
(32152 +k2—2k+1=38
9+25+k2—-2k+1=8
Vk?—2k+35=8
. k?—2k+35=064 {squaring both sides}
ook —2k=29
kP =2k + (1) =29+ (1) {completing the square}
(k—1)2 =30
. k—1=+V30
. k=1++30
L a Volume of prism
= area of end x length
= 1 x base x height x length
=1x6x4x8
= 96 units®

b AB=+/(0—0)2+(6—-0)%2+(0—4)?

- AP
=v0+36+16
=52
= 2v/13 units
¢ Dis (—8,0,4) andEis (-8, 6,0).
Surface area of prism
= area of base + area of 2 triangular faces + area of 2 rectangular faces
=8 X6 + 2 x area of AOAB + area of quadrilateral OADC + area of quadrilateral ABED
=48 + 2x 3 x6x4 + 8x4 + 8x2V13
=104+ 16V13
~ 162 units®
5 a Centre of sphere = midpoint of [PQ]
(4+76 —242 3+75)
2 2 7 2
=(-1,0,-1)
b Radius of sphere = distance from centre (—1, 0, —1) to P(4, —2, 3)
(4—-1)24(-2-0)2+(3—-1)2
_ FT (O E
=V25+4+16
=15
= 3v/5 units
392 Chapter 9 (Points in space) Review set 9B

¢ Volume of sphere = %7’1’7’3 Surface area of sphere = 4772


=47 x (3V5) =4 x7x (3V5)?
~ 1260 units® ~ 565 units’

6 a The midpoint
M of [BD] is (% 12;6, °+21°) which is (6, 9, 5).
b | The required angle is DAT, where T is the centre
of the base.
To find the centre of the base, we locate the
midpoints of the diagonals.
The midpoint of [AC] is

(8 0 7 0+12 %) which is (4, 6, 0). C(0,12,0)


2 2 7
The midpoint of [BO] is X

(8”027 1240
2 7
%) which is (4, 6, 0).
the centre of the base is T(4, 6, 0).
Now DT = 10 units
and AT = \/(4—8)2+ (6—0)2 + (0 — 0)?
e ®
=V16+36+0
= V52 units
10
- tanDAT
an = ——
\/fi

.. DAT == tan—!
DAT tan (10
(\/5_2>

.. DAT ~ 54.2°
The angle is about 54.2°.

il The required angle is MCP, where P is the point


on the base plane which is directly below M.
Mis (6,9,5), soPis (6,9,0).
Now MP = 5 units
and PC=+/(0—6)2+(12—-9)2+ (0 — 0)?

= /(C6Z 32+ 02
=V361940 Kb 0
VB 2 o
1

- . tanMCP
tanMC = ——
/T
- 5

_ MCP~p _= tan -1 <\/E)


5

. MCP ~ 36.7°
The angle is about 36.7°.
Chapter 9 (Points in space) Review set 9B 393

7 a 200m=02km=$km
the hiker is located at (2, —4, £).

b Distance of hiker from base camp (0, 0, 0)

= J0-22+0- -2+ (0- 12


=2+ -y
TW
=4/4+16+ 5

_
T%
50
~ 4.48 km

¢ i 500m=0.5km=3km
the mountain top is located at (—4, 1, %)
ii Distance between the hiker and the mountain top

= P+
(1 -4+
(A-2 (G- 3)
—Jcor 5+ (&)
)

ol
@
>

S
Il

2ol

iii The mountain top is 0.5 — 0.2 = 0.3 km above the hiker.
inh 0.3
=23 5 M
e (m) 6109
10 MGlOQk

6 =—sinsin (=2510 0.3km


) By
6~ 2.20° L m )

The angle of elevation from the hiker to the mountain top is about 2.20°.
394 Chapter 9 (Points in space) Review set 9B

8 a Radius of cone
= distance from centre C(2, 3, 0) to point B(1, —1, 0)
=+/(1-2)2+(-1-3)2+(0—-0)?
= VDR T (A + 07
— VITT670
= /17 units

AB=/(1-2)2+(-1-3)2+(0-15)?

= VDR T (AP + (3P


=VI+16+25
= V42 units
the slant height of the cone is /42 units.

Surface area of cone = 7rs + 712


=7 x V1T x V42 + 7 x (V17)?
~ 137 units?
b Now AC =5 units
and BC = /17 units

- tan ABC = —— = 5
an Ji7
)
_ ABC
R
= tan -1
(—fi)
5

- ABC ~ 50.5°
Chapter 10
PROBABILITY

1 The possible outcomes for the uppermost face when the die is rolled are 1, 2, 3, 4, 5, and 6.

2 Each outcome is equally likely to occur, and there are 6 possible outcomes. We expect the relative
frequency of rolling a 2 to be %.

3 Note: The answers below are only one of many possible results from the experiment. Your results
will differ.

relative frequency of rolling a 2 = 2—30 =0.15

relative frequency of rolling a 2 = 4—70 =0.175

5 | Number of rolls | Frequency of rolling a 2 | Relative frequency of rolling a 2


396 Chapter 10 (Probability) Investigation 2 Tossing drawing pins

6 04 relative frequency

0.3

0.2
° 0 .
° °
°

0.1

0 >
0 200 400 600 800 1000 1200
number of rolls

The relative frequencies become more consistent and approach the value +6 ~ 0.167 as the number
of rolls increases.

7 As the number of rolls increases, the relative frequency of rolling a 2 will approach %.

Note: These are example results only, your results will differ.

Two backs

Back and side

Two sides

; Rl egirc
Two backs

Back and side

Two sides

4 The whole group’s data has a larger sample size and hence will provide more reliable probability
estimates.
Chapter 10 (Probability) Exercise 10A 397

L 113
1 a P(inside a square) = i
~0.78 on%

b P(on a line) = e 6cm L


145 inside
~0.22
-
6cm

Total frequency = 17 +38 + 1944 =78

a P(20 to 39 seconds) = % b P(at least one minute) = P(> 60 seconds)


~ 0.487 )
78
~ 0.051

¢ P(between 20 and 59 seconds inclusive) = 38;;19

~ 0.731

3 a Thesurvey lasted 2+7+114+8+74+4+34+0+1


= 43 days

b 1 P(0calls) ~ 433 1o} frequency

~ 0.0465 1‘;
hil P(=5
> calls) N
4434041
— o
6\

~ 0.186 2
0
2T+ 01 23456738
W P(<3 calls) ~ 43 number of calls per day
~ (0.465
398 Chapter 10 (Probability) Exercise 10A

Total frequency = 37 +81 +48 + 17 +6+ 1


=190

a P(4 days gap) = o0 ~ b P(at least 4 days gap) =~ L L7401


o0

~ 0.0895 ~ 0.126

) 13— b f femal ki t school €


a P(female 15 year old at school € is a smoker) ~ — e I - W
49 <— total number of females aged 15 at school €

~ 0.265
b At school E, there are 40+ 39 =79 15 year olds.
744 =11 of these students are smokers, so 79 — 11 = 68 are non-smokers.

P(15 year old from school E is not a smoker) = % ~ 0.861

¢ There are 48 + 44 =92 smokers in total,


and 201+ 214 =415 15 year olds.
92
-, P(15 year old from any of the five schools is a smoker) ~ o

~0.222
Chapter 10 (Probability) Exercise 10A 399

2014/15 2015/16 | 2016/17 | 2017/18


Access 1612
Billing 3582
Contracts 836
Credit control 136
Customer Service 1940
Disconnection 248
Faults 384
Privacy 60
Provision 311

9109
a P(complaint received in 2016/17 was about customer service)
__ 1181 <—— number of customer service complaints in 2016/17
"™ 8085 <— total number of complaints in 2016/17
~ 0.146
b There were 1822+ 2102 + 3136 + 3582 = 10642 billing complaints in total,
and 3015+ 4282 + 8085 4+ 9109 = 24491 complaints in total.
10642
P(complaint received at any time was related to billing) ~ ~ 0.435
491

¢ In 2017/18, 3582 complaints were related to billing and 384 complaints were related to faults.
So, 9109 — 3582 — 384 = 5143 complaints in 2017/18 did not relate to either billing or faults.

P(complaint received in 2017/18 did not relate to either billing or faults) ~ %

~ 0.565

7 Summer Temperatures

Vi days x> 10°C


Mean days max. > 30° .

1 o 5.3 =— number of August days> 35°C


a . P(AugUSt day N C)™ 31 <— number of days in August
~0.171

ii 12.0 days in August are > 30°C, so 31 — 12.0 = 19 days in August are < 30°C.
19
P(August day < 30°C) ~ = ~ 0.613
b There are 9.4+ 12.3+12.0 =33.7 days in total during summer which are > 30°C,
and 30+ 31+31 =092 days in total during summer.

P(any summer day will be > 30°C)~ % ~ 0.366


400 Chapter 10 (Probability) Exercise 10B

¢ Thereare 0.3+ 1.240.7 =2.2 days in total during summer which are > 40°C, and 1.2 days
in July which are > 40°C.
1.2
P(a summer day > 40°C is in July) ~ 35 & 0.545

EXERCISE 10B
1 We extend the table to include totals for each row and column.

2603
4907
7510
a The total attendance for the match was 7510 people.
b 1 2439 out of the 7510 people at the match were children.

P(a child is selected) ~ 2439 0.325


7510
il 4907 out of the 7510 people at the match were not season ticket holders.

P(a non-season ticket holder is selected) ~ % ~ 0.653

iii 1824 out of the 7510 people at the match were adult season ticket holders.

P(an adult season ticket holder is selected) ~ % ~0.243

2 We extend the table to include totals for each row and column.

a 1 232 out of the 447 adults surveyed attended university.

P(attended university) ~ % ~ 0.519


ii 197 out of the 447 adults surveyed did not attend university and are currently employed.

P(did not attend university and is employed) ~ 41179: ~ 0.441


iii 25 out of the 447 adults surveyed are unemployed.

P(is unemployed) ~ % ~ 0.0559

iv 7418+ 225 =250 adults of the 447 surveyed are unemployed or attended university.

P(is unemployed or attended university) ~ % ~ 0.559

b 7 out of the 25 unemployed adults surveyed attended university.

P(an unemployed adult attended university) ~ % ~0.28


Chapter 10 (Probability) Exercise 10B 401

3 a Mi

b i 436 out of the 721 students surveyed play sport.

P(plays sport)~ % ~ 0.605

ii 131 out of the 721 students surveyed play sport and are in the junior school.

P(plays sport and is in junior school)


~ % ~0.182

iii 81+ 176 = 257 students out of the 721 surveyed do not play sport and are in the middle
or senior school.
P(does not play sport and is in middle school or higher)
~ e 0.356

k& We extend the table to include totals for each row and column.

m
|
ek s [| 15| 202 | 2 | on
373 [ m2 [ 150 [1235
a i 743 out of the 1235 bookings made were in the peak season.
743
P(in the peak season)~ —— ~ 0.602
1235
ii 148 out of the 1235 bookings made were for a single room in the off-peak season.

P(a single room in the off-peak season) ~ 112;4385~ 0.120

iii 3734 712 = 1085 bookings out of the 1235 were for a single or a double room.
1085
P(a single or a double room)=~ ol o 0.879

iv 2254420+ 98 4+ 52 = 795 bookings out of the 1235 were made during the peak season
or were for a family room.
P(during the peak season or a family room) ~ % ~ 0.644

b 52 out of the 492 bookings made during the off-peak season were for a family room.

P(booking made in off-peak season is for family room) ~ % ~ 0.106

¢ 420+ 98 = 518 bookings out of 712+ 150 = 862 were made in the peak season for double
or family rooms.
P(booking made for a double or family room was in peak season) ~ % ~ 0.601
402 Chapter 10 (Probability) Exercise 10C

1 a {AB,CD} b {1,2,3,4,56,7,8} ¢ {MM, MF, FM, FF}


2 a F’isthe event that the captain’s name will not contain the letter “e”.
b i U = {Peter, Quentin, Ronan, Sam, Thomas} il E = {Peter, Quentin}
iii E’ = {Ronan, Sam, Thomas}
<
Ronan
Peter
Sam
Quentin
Thomas
U

3 aU=1{1,23,45,6,7,8,9, 10,11, 12, 13, 14, 15, 16}


b i A={4,8,12, 16} il B=1{1,4,9, 16}
<
A B
235
6 7
10 11
U 13 14 15

A4 a coin b die2
A
T 6
H
ot

die
123456
W e
=N

die1
123456

€ spinner d spinner2
A
D 4
C 3
B 2
A 1
i spinner1
123456 % ABCD 7
5 a 5-cent 10-cent b coin spinner

it
A\ e}

<T<H
T
Chapter 10 (Probability) Investigation 3 Coin tossing experiments 403

< spinner 1 spinner 2 d draw 1 draw 2

Swo
XN
<

Swv
fi\
NN

Swov
w

6 Bill’s number
A
Ot
ke
W
N

Adam’s number
1234567

7 coinl coin2 coin3

1 a There are four possible outcomes in this experiment:

H H

b i 1 outcome corresponds to no heads. ii 2 outcomes correspond to 1 head.


iii 1 outcome corresponds to 2 heads.

I
TH T
I
404 Chapter 10 (Probability) Investigation 3 Coin tossing experiments

d The results show a symmetric distribution. There is only 1 outcome corresponding to each of
2 heads and no heads, while there are 2 outcomes corresponding to 1 head. The probabilities
of 2 heads and no heads are similar, and the probability of 1 head is about twice as large.

2 heads | 2494 2494


10000
—0.2494
1 head 5032 5032
10000
_ 5032

no heads | 2474 274


10000
_ 9474

These results agree with our conclusion in d. We expect 2 heads about 25% of the time,
1 head about 50% of the time, and no heads about 25% of the time.

2 a There are eight possible outcomes in this experiment:

H H H

b i 1 outcome corresponds to no heads. ii 3 outcomes correspond to 1 head.


iii 3 outcomes correspond to 2 heads. iv 1 outcome corresponds to 3 heads.

=
3 heads | Mt It %—0125

2 heads | It T Mt Mt MM | 81 2 =0.3875
1head | JH I JHE JHE JHE 11 27 %:0.3375
no heads | Mt Mt | 12 ; —0.15

d The results show a symmetric distribution. The probabilities of 3 heads and no heads are
similar, while the probabilities of 2 heads and 1 head are similar, and about three times as
large.
Chapter 10 (Probability) Exercise 10D 405

3 heads

2 heads

1 head

no heads

These results agree with our conclusion in d. We expect 3 heads about 12.5% of the time,
2 heads about 37.5% of the time, 1 head about 37.5% of the time, and no heads about 12.5%
of the time.

EXERCISE 10D g

1 Total number of marbles =5+3+4+7 =15

a P(red) =_3_1
=5 b P(green) =e LM
53

¢ P(blue) = 1—75 d P(not red) = P(green or blue)


547
15
_n
T 15
_4 5
. 543
e P(neither green nor blue) = P(red) f P(green or red) = -
=1
5 =5

2 a 8are brown and so 4 are white.


b ii P(brown) ==t 208
St ke iil P(white)
i) g
= Tk 1

3 a P(multiple of 4)
= P(4, 8, 12, 16, 20, 24, 28, 32, or 36)
%|‘9
=

b P(between 6 and 9 inclusive)


=P(6, 7,8, 0r9)
=
36
1l
9
406 Chapter 10 (Probability) Exercise 10D

< P(> 20) d P9 =5


= P(21, 22, 23, 24, ...., 35, or 36)
3620
36
1636
_4 9
e P(multiple of 13) f P(odd multiple of 3)
= P(13 or 26) =P(3, 9, 15, 21, 27, or 33)
-2 _ 6
"~ 36 T 36
=L _1
18 =%
g P(multiple of both 4 and 6) h P(multiple of 4 or 6, or both)
= P(multiple of 12) =P(4, 6, 8, 12, 16, 18, 20, 24, 28, 30,
=P(12, 24, or 36) 32, or 36)

T
336 _B36
—=5 1 =1 3

4 a P(ona Tuesday) = % b P(on a weekend) = 2


. 4 x 31 s
¢ P(in July) = S P GEE Fo {over a 4 year period}
— 124
T 1461
4x314+3x28+4+1x29
d P(in January or February) = {over a 4 year period}
3 X 365+
1 x 366
BT (= 19

5 a Let A denote Antti, K denote Kai, and N denote Neda.


Possible orders are: {AKN, ANK, KAN, KNA, NAK, NKA}

b i P(A in middle) = % i P(A at left end) = 2


=1 =1 3
3
iii P(A does not sit at right end) iv. P(K and N are together) =
|

=1—P(A at right end)


wiv

2
6
|k
who
Chapter 10 (Probability) Exercise 10D 407

6 Let G denote “a girl” and B denote “a boy”.


a Possible orders are: {GGG, GGB, GBG, BGG, GBB, BGB, BBG, BBB}
b i P(all boys) = P(BBB) = § i P(all girls) = P(GGG) = §
iii P(boy, then girl, then girl) iv P(2 girls and a boy)
= P(BGG) = P(GGB or GBG or BGG)
=1 =

v P(girl is eldest) vi P(at least one boy)


= P(GGG or GBG or GBB or GGB) B % {all except GGG}
s
T8
!2

7 a {ABCD, ABDC, ACBD, ACDB, ADBC, ADCB, BACD, BADC, BCAD, BCDA, BDAC,
BDCA, CABD, CADB, CBAD, CBDA, CDAB, CDBA, DABC, DACB, DBAC, DBCA,
DCAB, DCBA}

b i P(A sits on one end) = g S


Nl=

. g ) 12
ii P(B sits on one of the two middle seats) = o %

iii P(A and B are together) = % =1

iv. P(A, B, and C are together) = ;—i =1

8 5-cent a P(2 heads) = 1 b P(2 tails) = *


A
T < P(exactly one tail) d P(at most one tail)
H = P(HT or TH) = P(HT or TH or HH)
20-cent _2 = %
H T 1

N2
9 a coin

T
H
spinner
1 2 3 4 5 )

b i P(Hand5)= % ii P(T and prime number)


=P(T2, T3, or T5)
_310

fii P(an even number) iv P(H or 4) coin


= P(H2, T2, H4, or T4) _6 B
. ;0 H
S 10 =5 {shaded} spinner
=

12
3 4 5
ainy
408 Chapter 10 (Probability) Exercise 10D

10 dii 2 a P(two 3s) b P(a 5 and a 6)


g =P((3,3)) =P((5, 6), (6, 5))
e
43 =135 _2
36
2 — L
1 18

123456 det
< die2 d die2
6 6
5 5
4 4
3 3
2 2
1 1

123456 %€l 123456 %€l


20
P(a 5 or a6 or both)= % P(at least one 6) = 3%
_5 9
e die2 f o die2
A
6 6
5 5
4 4
3 3
2 2
1 1

1234506 del 123456%€t


10 25
P(exactly one 6) = T P(no sixes)= 25
5
18

11 a A chlld B child
A
A A
B B

red green ticket red red green ticket

Both grids show the sample space correctly, although B is more useful for calculating
probabilities.
b P(child B selects green ticket) = 1 {using grid B}

12 a die 2
A
oD

—r—eiflg—»@—
=N
W

-0
[

» die1
D
Chapter 10 (Probability) Exercise 10D 409

b There are 36 outcomes in the sample space.


i die2 il die2
A
6 6 T T 10 ‘1 2
5 € 5 8 1t
4 8 4 8 0
3 3 €
2 2 8
1 2 1 % T
[ » die 1 f 1 » die 1
1 2 3 4 5 6 2 3 4 5 6

Two of the outcomes are 11. Five of the outcomes are 6.


. _ 5
P(sum of dice is 11) = 3_26 P(sum of dice is 6) = 35
_ 1
— 18

il die2 v die2
A
6 8 1 112 6
5 6T 011 5
4 o )1 4
3 8 3
2 4 2
1 3 1 1 1
» die 1 » die 1
1 2 3 4 5 6 1 2 3 4 5 6

Nine of the outcomes are 8 or 9. Ten of the outcomes are less than 6.
Lo L. 1
P(sum of dice is 8 or 9) = 3% P(sum of dice is less than 6) = 3—2
_1 _ 5
-1 — 18

Vo die2 vi o die2
A
6 6 1
5 5 f 0
4 4 )1
3 3
2 2 K
1 1 1
» die 1 » die 1
1 2 3 4 5 6 1 2 3 4 5 6

Ten of the outcomes are greater than 8. 26 of the outcomes are no more than 8.
P(sum of dice is greater than 8) . P(sum of dice is no more than 8)
_10 _2
T 36 T 36
518 118
410 Chapter 10 (Probability) Exercise 10D

13 a die 2

=N
W oo

die 1

b There are 36 outcomes in the sample space.


I die2 i die2
A
6 2 6
514 ) 5
4132 0 2 4
3 23 3
2 0 2 2
1 f 1

23456 ¢! 2345064°!
Six of the outcomes are 0. Eight of the outcomes are 2.
P(resulting value is 0) = % P(resulting value is 2) = %
=1 6 =2
9
iii die2 iv die2
A A
6 654
5 51432101
4 41321 2
3 3 312 ( 23
2 2 —1—2—
y 1 [
1

23 4 56>d|e1 23 4 56>dle1

18 of the outcomes are 1 or 2. Six of the outcomes are more than 3.


P(resulting value is 1 or 2) = 18 . P(resulting value is more than 3)
36 6
_ 1 =—
-2 36
_1
=5
VvV die2 24 of the outcomes are less than 3.
i ) 24
P(resulting value is less than 3) = i
=N
W Ot

> die1
23456
Chapter 10 (Probability) Exercise 10D 411

14 a spinner 1
A

3 € 12

2 2 &1

1 2

—T—> spinner2
2 3 4 5 g

b There are 15 outcomes in the sample space.


i spinner 1 il spinner 1
A A

3 1 5 3 1

2 0 2 &1

1 == 1 3

spinner 2 spinner 2
1 2 4 P 1 2 3 4 5 P

Two of the outcomes are 6. Seven of the outcomes are less than 5.
P(result is 6) = & P(result is less than 5) =
ili spinner 1
A

3 12—15—

» spinner 2

Six of the outcomes are odd.


6
P(result is odd) =
15
") 5

15
B/

(14)

a P(black wool) = n(B) b P(not black wool)


n )
3
7
412 Chapter 10 (Probability) Exercise 10D

16 n(U)=49+13 =62, n(R)=13


a P(red car) _= n(®)

_1362

b P(not red car) _= R))


. @ R
U

17 n(U)=15+9+37+4=65 n(B)=15+9=24, n(S) =9+ 37 =46


n(BNS)
a P(likes both activities) = @
n
B S


b P(likes neither activity) = %
— 465 4)
15+ 37
¢ P(likes exactly one activity) = .
_ 52
65
4
=%

18 n(U)=5+17+4+3=29, n(G)=5+17=22, n(H)=17+4=21


n(GNH)
a P(studies both subjects) =
n(U)
17 G H
— 29
n(GUH)
b P(studies at least one subject)
n(U)
54+17+4
29
sL®
=26
29

n(GNH')
¢ P(studies only Geography) =
n(U)
kst
- 29

19 n(U)=8+43+5+2+94+0+7+10=44

fo)
! ’ S R
a S NA)
P(plays only rugby) = RN
n(U)
_ 911
b P(plays both soccer and archery) = S0 4)
n(U)
542
(050N
4
4 (10)
Chapter 10 (Probability) Exercise 10D 413

/ !
¢ P(does not play soccer or rugby) = A T)
n(U)
7410
T
_ 1T
~ 11

20 n(U) =50
a P(interested in music) = e e
n(U) music
_=5081
b P(interested in music, sport, and computers)
20
()
20
T 50
=1
—2 ; (0) computers

¢ P(not interested in computers) = Suac OO


n(U)
_ 17
— 50

21 a Let F represent the event “the man gave flowers”


and C represent the event “the man gave chocolates”. F, C

n(FNC)=5
L n(FNC)=12-5=7
and n(F'NC)=31-5=26 - (12
n(F'NC)=50—-5-7-26=12
b i P(Cor F) ii P(C but not F) iii P(neither C' nor F)
_ T+5+4+26 26 12
R ) 50
_= %38 =13 5 -5
25

— 19
5
22 Let T represent the event “a student plays tennis”
and N represent the event “a student plays netball”. 0 N
n(T) =19, n(N)=20, n(I'NN')=8, n(U)=40
n(TNN')=19—n(T NN)
n(T'NN)=20-n(T'NN) & (8)
n(TNN)+n(TNN)+n(T' NN)+n(T' NN")=n(U)
19-n(TNN)+n(TNN)+20-n(TNN)+8=40
47 —n(T'NN) =40
n(ITNN)=T7
a P(plays tennis) = 13 b P(does not play netball) = 124;; c
20
)
s
Nl=
414 Chapter 10 (Probability) Exercise 10D

¢ P(plays at least one) = 12+4—70+13 d P(plays exactly one) = 12;]13

_ 32 _ 2
T 40 =1
_4 _s
5 8
13
e P(plays netball, but not tennis) = =

23 Let Me represent the event “a child had measles”


and Mu represent the event “a child had mumps”.
Me Mu
n(Me) =24, n(Men Mu) =12, n(MeU Mu) = 26,
n(U) =30
n(MeNMu') =24 —-12=12
n(Me'N Mu) =26 —24 =2 (4)
n(Me'
N Mu') =30—26=4

a P(child has had mumps) = 1233 ) b P(child has had mumps but not measles)
_ 1 -2
T30 B
=L =1

¢ P(child has had neither mumps nor measles) = %


_z
=15

24 a nU)=12+847+3+14+4+k+7=60
k+55 = 60
k=5
B SN
(oL
. . 14
b 1 P(member likes only Italian) = i
— L
30
il P(member likes Italian and Thai) = T
_u ™
= %0 U
i P(member likes none of these foods) = 7
iv. P(member likes at least one of these foods) = 1 — P(member likes none of these foods)
-1
60
_ 5360
v P(member likes all of these foods) =
vi P(member likes Chinese and Italian, but not Thai) = 6%
., 2l15

vil P(member likes Thai or Italian) = Gl b R


60
1
o N
ol
Chapter 10 (Probability) Exercise 10D 415

1241445
viii P(member likes exactly one of these foods) = N
31
60

English only 17
French only 7
Spanish only 12
English and French only
English and Spanish only
French and Spanish only
English, French, and Spanish

a Let F represent the event “a delegate had a conversation in English”,


F represent the event “a delegate had a conversation in French”,
and S represent the event “a delegate had a conversation in Spanish”.
n(EUFUS)=17T+3+14+6+7+4+12
=50 B F
So, every delegate had a conversation in at least fi
one language.
n(E' NF'AS)=0 @v&

s (0)
U

b i P(delegate had a conversation in English) = W


27
~ 50
n SA 3+7+4+1
ii P(delegate had a conversation in French) = %
15
50
=3
10
e r . . g 1244
iii P(delegate had a conversation in Spanish, but not in English) = N
16
"~ 50
-8
=25
. ol . i 347
iv. P(delegate had a conversation in French, but not in Spanish) = =coL

10
50
1
5
416 Chapter 10 (Probability) Exercise 10D

v P(delegate had a conversation in French, and also one in English) = ——

&=
g
26

(o fo) n
(NI
© 0
U

a 4+24+1+a=10 {10 watched a movie}


L a=3
442+4+143+64+124+9+b=40 {40 individuals in total}
37+b=140
b=3

b i P(watched sport) = W il P(watched drama and sport) = %

_12 Sk
T 40 T 40
_ 3 — L
10 10
iii P(watched a movie but not sport) iv P(watched drama but not a movie)
443 _ 1243
) a0
=T _ 15
T 40
3
=%

12434+3+14+4+2
v P(watched drama or a movie) = N
Chapter 10 (Probability) Investigation 4 The addition law of probability 417

1 U ={x |« is a positive integer less than 100},


A = {multiples of 7 in U}, B = {multiples of 5 in U}
a i A={T7 14, 21, 28, 35, 42, 49, 56, 63, 70, 77, 84, 91, 98}
There are 14 elements in A.
it B = {5, 10, 15, 20, 25, 30, 35, 40, 45, 50, 55, 60, 65, 70, 75, 80, 85, 90, 95}
There are 19 elements in B.
iii AN B = {35, 70}
There are 2 elements in AN B.
iv AuB = {5,7,10, 14, 15, 20, 21, 25, 28, 30, 35, 40, 42, 45, 49, 50, 55, 56, 60, 63,
65, 70, 75, 77, 80, 84, 85, 90, 91, 95, 98}
There are 31 elements in AU B.

b n(A)+n(B)—n(ANB)=14+19-2
=31
=n(AUB)

2 n(A)+n(B)—n(ANB)=(a+b)+
(b+c)—b
=a+b+ec A B
=n(AUB)

- @d
3 a P(4) _ n(4)
@)

_ a+b
T atbtctd

b
.i P(B)__n(U)
_ n(B) !ii P(AflB)_—n(U)
_ n(ANB)

- b+c . b
T atbtctd T atbtetd
_ n(AUB)
iii P(AUB)_—n(U)

_ a+b+ec
T a+btctd
iv P(A)+P(B)7P(AMB)fa7—fb+b’—icf;
T atbtctd atbtctd atbtctd
_ _atb+c
T atbtetd
=P(AUB)
418 Chapter 10 (Probability) Exercise 10E

1 P(AUB) =P(A)+P(B) -P(ANB)


=0.2+04-005
=055
2 P(AUB)=P(A) + P(B) — P(AN B)
. 09=04+P(B)—
P(B) = 0.6
3 P(XUY)=P(X)+P(Y)-P(XNY)
. 09=06+05-P(XNY)
L P(XNY)=02

4 a P(AUB) =P(A) +P(B) —P(ANB)


o 0.7=025+0.45—-P(ANB)
P(ANB)=0
b Since P(ANDB) =0, A and B are mutually exclusive events.

5 P(AUB)=P(A)+P(B) {since A and B are mutually exclusive}


. 0.8=P(A)+0.45
P(A) =0.35

6 a It is impossible for a number to be both greater than 11 and less than 8.


A and B are mutually exclusive.
b i P(A)=P(number drawn is 12, 13, 14, or 15)
_154
il P(B) = P(number drawn is 1, 2, 3, 4, 5, 6, or 7)
_z
15

iii P(AUB)=P(A)+P(B) {since A and B are mutually exclusive}


=1+
|

Gk
o

1
==

7 aPF)=1
oo
|~' EZI H

I
o

~
Il

Il
n
=

d P(C)= 2
|

f P(FUS)=P(F)+P(S) {since F" and S are mutually exclusive}


1,
=4, 1212
_2
i EO5

g P(FUD) cannot be found as we do not know how many students are both 15 and own a dog.
h P(CUN)=P(C)+P(N) {since C' and N are mutually exclusive}
=L+ 4
25 T 35
_u
— 25
Chapter 10 (Probability) Investigation 5 Independent events 419

i P(CUD) cannot be found as we do not know how many students own both a cat and a dog.
] P(DUN)=P(D)+P(N) {since D and N are mutually exclusive}
—8 44
_2+25
1

8 P(AUB)=P(A)+P(B) {A and B are mutually exclusive}


P(AUB")=PA)+P(B) {4’ and B’ are mutually exclusive}
Now, P(A')=1-P(A) and P(B')=1-P(B)
. P(AUB)=1-P(A)+1-P(B)
=2—(P(4) +P(B))
=2-P(AUB)
. P(AUB)+P(AUB)=2
P(AUB) =1 {since P(X)<1 and 1+1=2}

1 a The outcomes of the coin toss and dice roll have no effect on each other. The events are
independent.
b coin
A
T
H
die
123456

< A B P(A) | P(B) | P(ANB)


i | head 4 1 i =
ii | head odd number 1 =1l =1

iii | tail | number greater than1 | 3 s 3


iv | tail number less than 3 1 =1 =1}

2 a The outcome of the draw from either box does not affect ee
the outcome of the other. The events are independent. \fi
A
X Y

b boxY
A
w
R
R
R
box X
B B GG
420 Chapter 10 (Probability) Exercise 10F

c A B P(B) | P(ANB)
i | green from box X | red from box Y

slw

Sle
Nl

ol
R

=
ii | green from box X | white from box Y

Bl=

ool
Sl
R

Rl
iii | blue from box X red from box Y

ool
Bl
Dl

sl
B

Sl
iv | blue from box X | white from box Y

ool
Bl
Bl

o=

sk
Hlo
3 From our answers to 1 and 2, we can see that if A and B are independent events, then
P(ANB) =P(A) x P(B).

Spinner 1 Spinner 2

a P(green with 1 and blue with 2) b P(red with both)


= P(green with 1) x P(blue with 2) = P(red with 1) x P(red with 2)
{events are independent} {events are independent}
—lg1 %% —2y2
1756
_ L — 4
= 24
—1 6

2 a P(H, then H, then H) b P(T, then H, then T)


=P(HNHNH) =P(TNHNT)
= P(H) x P(H) x P(H) = P(T) x P(H) x P(T)
{events are independent} {events are independent}

=ix4x} ==1ixixi
=1 8 8

3 Let A be the event of photocopier A malfunctioning, and B be the event of photocopier B


malfunctioning.
a P(both malfunction) b P(both work effectively)
=P(ANB) =P(A'NB)
=P(A) x P(B) =P(A") x P(B')
{events are independent} {events are independent}
=0.08 x 0.12 =0.92 x 0.88
= 0.0096 = 0.8096
Chapter 10 (Probability) Exercise 10F 421

4 a P(B, then G, then B, then G) b P(not (B, then G, then B, then G))
=P(BNGNBNQG) =1—P(B, then G, then B, then G)
= P(B) x P(G) x P(B) x P(G) =1-+%
{events are independent} =13
=4ixixixi
1
16

5 Let J be the event of Jiri hitting the target, and B be the event of Benita hitting the target.
a P(both hit target) b P(both miss target)
=P(JNB) =P(J'NB)
=P(J) x P(B) =P(J') x P(B’)
{events are independent} {events are independent}
=0.7x0.8 =0.3x0.2
=0.56 = 0.06

< P(Jiri hits but Benita misses) d P(Benita hits but Jiri misses)
=P(JNB) =P(BNJ)
=P(J) x P(B’) =P(B) x P(J')
{events are independent} {events are independent}
=0.7x0.2 =0.8x0.3
=0.14 =0.24

6 Let H be the event the archer hits the bullseye.


P(H)=2, PH)=2
a P(3 hits) b P(2 hits then a miss) ¢ P(all misses)
=P(HNHNH) =P(HNHNH') =PH'NH NH
=P(H) x P(H) x P(H) =P(H) x P(H) x P(H") =P(H")
x P(H') x P(H")
{events are independent} {events are independent} {events are independent}
2 202 vl Nonds — 333
_ 8 _ 12 _ 27
125 125 125

7 a Let P be the event that the rubbish bin is full, and @ be the event that the recycling bin is full.

Rubbish Recycling outcome probability


bin bin
0.5 Q Pand Q 0.9x0.5=0.45
0.9 P <
i 0.5 Q' Pand Q' 0.9x0.5=0.45

e
05 Q P'and Q 0.1 x0.5=0.05

0.5 Q' P'and Q' 0.1 x0.5=0.05


. . total 1.00
b 1 P(both bins are full) = P(P N Q) ot
=09x0.5
=0.45
422 Chapter 10 (Probability) Exercise 10F

i P(recycling bin is full but rubbish bin is not) = P(P' N Q)


=0.1x0.5
=0.05

8 a Let R be the event that Celia chooses a red apple, and G be the event that Celia chooses a
green apple.
1st basket 2nd basket outcome probability
2 R Rand R Ixi=2
5 R
z 2 G Rand G ixi=1

3 3 R Gand R Zx5=18
7 G
Z G Gand G Zxi=2
total 2=1

b 1 P(Celia chooses two red apples) il P(Celia chooses one red and one green apple)
=P(RNR) =P(RNG)+P(GNR)
_5y5 _ 5.2 2.5
=7Xx7 =gX7tFX3
_ 2 _ 10, 10
= “ntn
— 20
1
9 a Let B represent the spinner landing on black, R represent the spinner landing on red, and
Y represent the spinner landing on yellow.
1st spin 2nd spin

b i P(both black) =P(BNB) i P(both yellow) =P(YNY)


~3x4
=1
~1xk
e 18
1 16
iii P(both different)
PBNR)+P(BNY)+PRNB)+PRNY)+P(YNB)+P(YNR)
1 1 1 1
=gxgtaxgtixgtaxg+tixg
Xy
_ 4, 2
=5t16
o),
oolut
Chapter 10 (Probability) Exercise 10F 423

iv P(black appears on either spin)


P(BNB)+PBNR)+PBNY)+PRNB)-+P(YNB)
Fhxh b hxdb dxh1 X

rol—=
2

Nl

ol
X

e
dsl=

ol
+
wsloo

10 Let A be the event that a 4 is rolled on the first roll, and B be the event that a 4 is rolled on the
second roll.
1st roll 2nd roll

i _.B
' <8 B’
%< 1
7 8 B
s A'<
T B’
8

a P(exactly one 4 is rolled) b P(at least one 4 is rolled)


=P(ANB)+P(A'NB) =1—P(no 4 is rolled)
IxI
17 4+ Ixt
71 =1-P(A'NB)
_ IRl
7,7 _ 77
51T 51 =l-gxg
14 —1_49
64 61
=L — 15
32 6

11 a Let B represent a blue ball being drawn, and G represent a green ball being drawn.
bag X bagY bagZ
3
1 g<:B
2 B 3 G
B 35

;
5 B
y o=3
1
G
35

; g -B="__
5 B bag X bagY
23 2 2 G
G S05
5 B
3 G <
2 G

b 1 P(3 blue balls are drawn) = P(BNB N B)


=1 1 3
X53X3
Sl gl
424 Chapter 10 (Probability) Exercise 10F

1 1 2 1 2
=3xX3 t3xg5X3
2 4
=3t
_ 830
_1 5
iii P(at least one blue ball is drawn) = 1 — P(no blue balls are drawn)
=1-P(GNGNGQG)
2 1 2
=l-gxg3x3
s

i 4
1-35
26
30
_13
15

12 —
path 1
A B
. path 2

a In order for the current to flow from A to B, both switches on the top of the network (path 1),
or both switches on the bottom of the network (path 2), need to be closed.
P(current flows from A to B)
= P(path 1 is closed U path 2 is closed)
= P(path 1 is closed) + P(path 2 is closed) — P(both paths 1 and 2 are closed)
=PXPpHPXPp—pXpXpXp
—p2yp_ph
—op?
b We need to solve 2p* —p* > 1
Let X =p?
2X —X?> 4%
L X?P-2x<g-4
X7 —2X +(-1)’< =3+ (~1)> {completing the square}
LX-1Pk3
1 1
s X-lsg
LT%<X<1W%
1 2 1
lffiép gl”fi

1—%<p< 1+fi {as p >0}

Lp=4/1- % ~ 0.541 is the least value for which 2p? —p* > 12

13 Kane should choose to play Penny - Quentin - Penny


To win 2 matches in a row, Kane must win the middle match, so he should play against the weaker
player in this match.
Chapter 10 (Probability) Exercise 10G 425

1 a P(both are red)= P(first is red N second is red)


P(first is red) x P(second is red given that the first is red)

/—\
T %6
10
_=90a2
- L
=15
b P(first is green and second is red)
= P(first is green) x P(second is red given that the first is green)
3l I
=10%s
2
=90
S
=30

2 a P(both are blue) = P(first is blue and second is blue)


P(first is blue) x P(second is blue given that the first is blue)
& i1y o) 3
9
sl SIS =l

b P(first is blue and second is white)


P(first is blue) x P(second is white given that the first is blue)
A

— 4 6
=15
X §
— 24
[ “:’l
1
ISt

3 a P(all strawberry creams)


1st is a strawberry cream N 2nd is a strawberry cream N 3rd is a strawberry cream)

none are strawberry creams)


= P(1st is not a strawberry cream N 2nd is not a strawberry cream
N 3rd is not a strawberry cream)
426 Chapter 10 (Probability) Exercise 10G

4 a P(wins first prize) = 125


b P(wins 1st and 2nd prize)
= P(wins 1st prize) x P(wins 2nd prize given that he won 1st prize)
_ 3 2
= 100 % 99
~ 0.000606

¢ P(wins all 3 prizes) = P(wins 1st prize N wins 2nd prize N wins 3rd prize)
_ 3 2 1
= 100 X 99 * 98
~ 0.000006 18
d P(wins none of the prizes)
= P(does not win 1st prize N does not win 2nd prize N does not win 3rd prize)
97 96, 95
= X 99
10 0X 98
~ 0.912

5 a P(does not contain captain)


= P(1st player selected is not the captain N 2nd player selected is not the captain
N 3rd player selected is not the captain)
L6y 54
=7X6%X5%
— 120
=2
— 4
=7
b P(does not contain captain or vice captain)
= P(1st player selected is neither the captain nor vice captain
N 2nd player selected is neither the captain nor vice captain
N 3rd player selected is neither the captain nor vice captain)
_5
-7
X
4
6
><5
3

6 a P(two boys) = P(first selected is a boy N second selected is a boy)


5

5.4
X%
5 Bl
o

b P(eldest two students) = P(either of the two eldest students N the remaining eldest student)
2 1
7 X%
2
12
L
21
Chapter 10 (Probability) Exercise 10G 427

0.2 violin b i P(male and does not play violin)


F < =P(M N not violin)
% 0.8 not violin
=04 x0.7
0.3 violin =0.28
04> <
not violin i P(plays the violin)
0.7
= P(F N violin) +P(M N violin)
=0.6x024+04x0.3
=0.24

Navy 0.19 officer

i i\ il
0. 81 other rank
0.22
0.47 0. 15 officer
Army
0.85 other rank
0.31 021
i officer
Air Force
0.79 other rank

P(officer)
= P(NNO)+P(ANO) +P(AFNO) {where N represents Navy,
A represents Army,
AF represents Air Force, and
O represents officer}
=0.22 x 0.1940.47 x 0.15 4+ 0.31 x 0.21
=0.1774

ii P(not an officer in the navy) i P(not an army or air force officer)


=1 — P(officer in the navy) =1 — P(army or air force officer)
=1-P(NNO) —1— (P(ANO) +P(AFNO))
=1-0.22x0.19 =1—(0.47 x 0.15 + 0.31 x 0.21)
= 0.9582 = 0.8644

. . b P(Mudlark wins)
i win
rain P(rain N win) + P(no rain N win)
5T
il

loss

no rain <
il

loss
Sl

-
10 spoiled P(next bottle is spoiled)
Machine A = P(from A N spoiled) + P(from B N spoiled)
04 not spoiled
=0.4x0.05+0.6 x 0.02

<
spoiled =0.020 +0.012
0.6 Machine B
not spoiled
=0.032
428 Chapter 10 (Probability) Exercise 10G

2 W P(red) = P(A Nred) + P(B Nred)


"
I A< 43
—1y 1yl
aw| |3w B 3R 275 T 271
=3 41
\fl‘ Ifl‘
5
3 16173
B 1 1 w 1245
A
2 B< =
TR M
4 =1

12 Tottenham is in 8th place, so there are 7 teams above Tottenham and 12 teams below Tottenham.
Opponent Result . P(draw) = P(above N draw) + P(below N draw)
1 =il 7] (O3 12,1
5 w =15 X1t X3
% above 3it~ D _ 21
=2L+ 224

1 L 45
190
1 w =2
12 8
19 below é D

3 L

13

a P(blue) = P(A N blue) + P(B N blue + P(CNblue)


alee =

X5 T ExXs tgX
_ 3 2 2 1 1

64+24+3
30
=1
30

b P(red) =1 — P(blue)
_u30
_ 1 30
Chapter 10 (Probability) Exercise 10G 429

14 Let F represent a student who participates in extra-curricular activities, and E’ represent a student
who does not participate in extra-curricular activities.

first student second student third student

EV

o
&
10 E
E' v

als
N
alo
¥s
-
=

|~

24 E'

|
5W

alo sl
S

<
=
rof=

=
2=
E'
i
Sl

oo
Bl
E

N
Gl
24 E’
B

Sho
IS
P(at least two students selected also participate in extra-curricular activities)
=P(EFEE) +P(EEE') + P(EE'E) + P(E'EE)
(g « 10 1) Jr(11 10 1@) + (% x i 19) (Xl 19)
25 © 24 © 23 35 X2 X3 35 X214
X 23 35 X231 X33
{branches marked v}
990 + 1540 + 1540 + 1540
13800
5610
13800
187
160
~ 0.407
430 Chapter 10 (Probability) Exercise 10G

15 Let R represent drawing a red card and B represent drawing a black card.

first card second card third card fourth card

cnll\?
SIE

&
o |
ol = O
=

=
=ls

l\/\b
o

o
S

B
= Ol S
o |

=
/\%Ia -@m/\a
=
218

s
o
s

=
<
o

&
] o
ol =
o
=

W ®
g/\gs
[&

=
50

<
k|
o
3

52
v
D
218

fi/\g
gl&

&

W
o e
CRZ]

<
=
218

=
&

51
&
S o
o


g/\@
gl

o
|
(2

a P(two red cards are drawn)


= P(RRBB) + P(RBRB) + P(RBBR) + P(BRRB) + P(BRBR) + P(BBRRE)
=BxBxBxB)+(BxExBxP)+ (B xBExFxH)
L (26 26 25 . 25\ 4 (26 . 26 .X 25 . 25\ . (26 . 25 . 26 . 25
FExBExBxR)+(BExRxBxR)+(ExHxBxH)
X

{branches marked v}
16250 + 16 250 + 16 250 + 16 250 + 16 250 + 16 250
249900
97 500
249900
325
833
~ 0.390
Chapter 10 (Probability) Exercise 10H 431

b P(at least one black card is drawn) = 1 — P(no black cards are drawn)
=1-P(RRRR)

Sl-FZxHExHxEH
— 1 _ 13800
249900
_ 236100
249900
_ 18T
833
~ 0.945

1 a HA|B)=”§;f) b P(AU B) = P(A) + P(B) — P(AN B)


o1 " 05=03+04—P(ANB)
T 04 o P(ANB) =02
_1
=1 Now P(A|B)=rAND)
P(B)
_02
T 04
—1
¢ P(A|B)=
P(AN
>0B)
B) :

__0 : )
=B {since A and B are mutually exclusive}

=0
2 Let C represent a cloudy day and R represent a rainy day.
P(C)=04, P(CNR)=02
P(R|C) = P(RNC)

The probability that it will be rainy on a day when it is cloudy is %

3 a Let M represent a student who studies Mathematics, and P represent a student who studies
Physics.
n(M) =40, n(P)=32, n(M'NP)=0, n(U)=>50
n(U) =n(M U P) +n(M' NP
50 =n(MUP)+0 M P
- n(MUP)=50
n(M U P) =n(M)+n(P)—n(MnNP)
50 =40+ 32— n(MNP) L (0)
oM P)=22
n(MNP)=40-22=18 and n(M'NP)=32-22=10
So, 22 students study both subjects.
432 Chapter 10 (Probability) Exercise 10H

b i PMOP)
= i p(p\]\l):%
=2 22
25 =m

&
_T 240
—u
20

L a Let D represent a boy who has dark hair, and B represent a boy who has brown eyes.
n(D) =23, n(B)=18, n(BUD) =26, n(U)=40
n(B'ND")=n(U) —n(BUD)
=40-26 P o
=14
n(BUD) =n(D)+n(B) —n(BND)
s 26=23+18-n(BND) U (14)
. n(BND)=15
n(BND')=18—-15=3 and n(B'ND)=23—15=8
b i P(dark hair and brown eyes) ii P(brown eyes given dark hair)
=P(BND) =P(B|D)
_15 _ P(BND)
T 40 T P(D)
=3 15
8 @
=z10
15
|
S

5 a Let S represent a hiker who was sunburnt and B represent a hiker who was bitten by ants.
n(S) =23, n(B)=22, n(SNB)=5, n(U)=50
. n(SNB)=23-5=18
and n(S'NB)=22-5=17 Sunburnt Bitten

n(S'NB') =n(U) —n(SUB)


=50-5—-18-17
=10 U (10)
b i P(hiker avoided being bitten) = 1 — P(B)
a2z
)
28
T 50
_u
— 25
Chapter 10 (Probability) Exercise 10H 433

ii P(hiker was bitten or sunburnt or both) = P(S U B)


1845417
- 50
40
50
4
5

iii P(hiker was bitten given he or she was sunburnt) = P(B | S5)
_ P(BNS)

iv. P(hiker was sunburnt given he or she was not bitten) = P(S | B’)

6 Let T represent a family who had a TV set, and C represent a family who had a computer.
Let the proportion of families in 7’NC' be z.
the proportionin TNC’ is 0.9— 2 and T c
the proportion in 7/ NC is 0.8 — .
The proportionin 77N C” is 0.
09—2)+z+(08—2)=1 ©)
1l7—z=1 U
=07

p(r|0) = 2L_P(TNC) 7 c

0.7

(0)
434 Chapter 10 (Probability) Exercise 10H

7 Let A represent a person who reads newspaper A,


B represent a person who reads newspaper B,
A
and C represent a person who reads newspaper C'.
The proportion of people in:
AnNBNC is 0.02 fl&
ANBNC is 0.08 —0.02 = 0.06
A'NBNC is 0.04—0.02=0.02
ANB'NC is 0.05-0.02=0.03
ANB'NC" is 0.2—0.06 —0.02 — 0.03 = 0.09 C
A'NnBNC is 0.16 —0.06 —0.02 — 0.02 = 0.06 U
A'NB'NC is 0.14—-0.02 —0.02 — 0.03 = 0.07
A'NnB'NnC" is 1-0.09 —0.06 —0.02 - 0.03 — 0.06 — 0.02 — 0.07 = 0.65

a P(person reads none of the papers) = 0.65


1
- 20

b P(person reads at least one of the papers) = 1 — P(person reads none of the papers)
=1-1
20
.
20
¢ P(person reads exactly one of the papers) = 0.09 + 0.06 + 0.07
=0.22
— 11
50

d P(person reads A or B or both) = 0.09 4 0.06 + 0.02 4 0.03 4 0.06 + 0.02


=0.28
¥
- 25

e P(person reads A, given that person reads at least one paper)


=P(A|(AUBUC(C))
P(AN(AUBUC))
P(AUBUCQC)
0.2
0.35
4
7

f P(person reads C, given that person reads either A or B or both)


=P(C | (AUB))
_ P(CN(AUB))
~ P(AUB)
0.03 + 0.02 + 0.02
0.09 + 0.06 + 0.02 + 0.03 + 0.06 + 0.02
0.07
0.28
_1
I
Chapter 10 (Probability) Exercise 10H 435

8 Let A represent urn A, B represent urn B, R represent a red marble, and Bl represent a blue marble.
urmn marble

: R @
A<
Z2REZ4RS ~n

o=
381 ) (_1BI
1
A B

®
=

QIA
&

&
a P(R)=P(ANR)+P(BNR) b P(BlR)zP(BfiR)
® eP(R
)
branch @ branch
:z>'<5 branch @
—1ly2 4 14
% ~—— from a
_3 s 2
-5

9 Let S represent Greta going shopping and I represent Greta having an ice cream.
a P(I)=P(SNI)+P(S'
NI
e —_—— —
I @ branch @ branch @
EI°/\5

I
2,7
=5X1w t
4343
5%
2
>
|
EI‘/\VSIw

b P(S|I)= P(SNI)
P( )
2 x 15 «<— branch (D)
23
55— from a
_u23
10 Let X represent machine X, Y represent machine Y, and M represent a machine malfunctioning.

X Y

P(X N exactly one malfunctioned)


a P(X | exactly one malfunctioned) = D (exactly one malfnctioned)
115 X 155
93 —— branch 3)
L3 9w T« branch d
0 X 100 T 10 % 100 ranches 3) and 2)
93
93
+ 63
93
156
31
52
436 Chapter 10 (Probability) Activity The Monty Hall problem

b P(Y | at least one malfunctioned)


P(Y Nat least one malfunctioned)
P(at least one malfunctioned)
7 9 7
X o0 + fixm<—branches®and@
o6 + 10T X 29
93 + 159 X 105% < branches
(D), 3). and0)

11 Let G represent a green ticket, Y represent a yellow ticket, H represent the result heads, and
T represent the result tails. . . .
coin toss first ticket second ticket

Wity

Q
©
H
1
2 iy
3 Y
1
2 ¢ Q@
1 3
3 3 v ®
T 2
0 3 ¢ @
3 Y

() 6) =15
P(HNG)
P(H|G)=—7—
0 Y

_ %3 branch (D
%x% + %x%x% + %x%x% + %x%x%*branches@,@,@,and@
1
B 3
T 1,1 .11
3tststs
_2
-3

ACTIVITY

1 If the contestant does not switch his or If the contestant does switch his or her
her choice, then the tree diagram is: choice, then the tree diagram is:

1st choice 2nd choice 1st choice 2nd choice


Q
Q

-
W

Q
Q
i
o

oo

LR
Q
Q

Q
Q

2 a The contestant can choose 1 out of 3 doors and the car is behind one of these.
P(contestant’s first choice has the car) = %1
Chapter 10 (Probability) Exercise 101 437

b Given that the contestant changes their guess, we consider the second tree diagram in 1.
P(contestant’s second choice has the car | change their guess)
=P(C'NC) {in 2nd tree diagram}
N2
=3x1

who

3 a The audience member sees one of the incorrect doors open, so must choose between the two
remaining closed doors, one of which has the car behind it.
P(audience member chooses the car) il
-2

b The contestant has the ability to switch or not switch from their original choice, unlike the
audience member.
The door chosen by the contestant is never one of the doors opened by the host, so the contestant
has a % chance of winning the car if they switch their choice, as shown in 2 b.
The audience member only sees two doors and has no other information, so the audience
member’s chance of guessing correctly is %

1 P(RNS)=P(R)+P(S)—P(RUS) Also, P(R) x P(S) = 0.4 x 0.5


=04+05-07 =02
=02
So, P(RNS) = P(R) x P(S) and hence R and S are independent events.

2 a i P(ANB)=P(A)+PB)—PAUB) ii P(B|A):w
2411 <
T5T3 T2 =z
_ 1 T
~ 30 25
- 13
_ P(ANB)
iiil P(A|B)= 5@

-
_ 30
- T3
-z10
b A and B are not independent as P(A | B) # P(A).

3 a As X and Y are independent,


then P(XNY)=P(X)xP(Y) X Y
=0.5x0.7
=0.35
P(both X and Y') = 0.35 U (0.15)
438 Chapter 10 (Probability) Exercise 101

b P(X orY or both) =P(XUY)


= P(X)+P(Y)—P(XNY)
=0.540.7—-0.35
=0.85

¢ P(neither X norY)=1-P(XUY) d P(X butnotY)=0.15


=1-0.85
=0.15

e P(X |¥) = PEOY)


P(Y)
0.35
o7
=05

4 A and B are independent, so P(ANB) = P(A4) P(B) ... ()


Now P(A'NnB")
=1-P(AUB)
=1-[P(A) +P(B) —P(ANB)]
=1-P(A)—P(B)+P(ANDB) vl A B
=1-P(A)—P(B)+P(A) P(B) {using (+)}
A'nB’
=1-P(4) —P(B)[1 - P(4)]
— [1— P(A4)] [1 - P(B)]
=P(A") P(B)
A" and B’ are also independent.

5 For events A and B to be independent, P(A N B) = P(A) x P(B).


For events A and B to be mutually exclusive, P(ANB) =0
o 0=2xP(B)
. P(B)=0

6 P(A)=P(ANB)+P(ANB')
B B =01+04
. P(A) =05
and P(ANB)=P(A) xP(B) {A and B are independent}
7 - 0.1=05xP(B)
- P(B)=0.2
Now P(AUB')=P(A)+P(B')-P(ANB’)
=05+08-04
=0.9
Chapter 10 (Probability) Investigation 6 Making predictions 439

7 P(DNC)=P(D[C)P(C)
=1 X 55 =25
Similarly, P(DNC’)=P(D|C)P(C") =4 x & =L
the Venn diagram is:

9 11 _ 89
a P(D)7%+m7m

b P(D)#P(D|C), soC and D are not independent events.

8 If X and Y are independent events, then P(XNY) =P(X) x P(Y)


if ANB and AUB are independent,
P((ANB)N(AUB)) =P(ANB) x P(AUB)
. P(ANB)=P(ANB)xP(AUB) {since (ANB)C (AUB)}
. P(ANB)[1-P(AUB)]=0
. P(AUB)=1 or P(ANB)=0

INVESTIGATION 6

1 a There are 6 possible outcomes when rolling an ordinary die.


we expect & of the rolls to be a “1”.
b We expect 1 in 6 rolls to be a “1”.
we expect 10 out of the 60 rolls to be a “1”.
¢ Note: These are example results only, your results will differ.

Theoretical probability | Predicted frequency Tally


1
6
1
6
1
6
1
6
1
6
1
6

d i We predict that the frequency of each outcome will be % % 60000 = 10000.


il We will almost never get exactly the outcome we predicted, but all of the frequency values
will be very close to 10 000.
440 Chapter 10 (Probability) Exercise 10J

2 a die2
A
6 T8 1
5 & { 1 1
4 ¢ 10
3
2 8
1 2 3 4 f 7
die 1
2 3 4 5 6

L x 360 = 10
36
L x 360 = 20
8
L x 360 = 30
12
3 x 360 =40
1

= x 360
= 50
1

25 x 360
=- 50
4
©l—=
| |H
|
- =
|
ol

All of the frequency values were close to the predicted frequencies.

3 We expect the event to occur np times.

1 n =90 attempts
p = P(saving a penalty attempt) = 1%
The goalkeeper would expect to save np = 90 x = = 27 penalties.

2 n =68 attempts
p = P(scores a goal) = 0.23
Brayden would expect to score np = 68 x 0.23 =~ 16 times.

3 a The chance of obtaining a red on any roll is % = 2.


b n =3 times
p = P(rolling a red) = 2
For the three rolls, you would expect to roll a red np =3 x % = 2 times.
Chapter 10 (Probability) Exercise 10J 441

4 a P(one coin falls heads)


P(both coins fall heads)

ol
Il

X
PR
b n =200 times
p = P(both coins fall heads)
We would expect the 2 coins to both fall heads on np = 200 x % = 50 occasions.

5 n=>5x7=35days
p = P(snow falling on any particular day) ==7 2
Udo could expect to see snow falling on np = 35 x % = 15 days.

6 n =180 times
p = P(rolling a double with two dice)
= P(rollling two 1s or two 2s or two 3s or two 4s or two 5s or two 6s)
=3 {6 of the possible 36 outcomes}
1
6
You would expect to get a double on np = 180 x %) = 30 occasions.

7 Total number of voters in poll = 165 + 87 + 48 B

=300 s
a 1 P(voter will vote for A) ~ i ii P(voter will vote for B) ~ L
300 300
~ 0.55 ~ 0.29
e . 48
ili P(voter will vote for C) ~ —
300
~ 0.16
b n = 7500 people
i We would expect np = 7500 x 0.55 ~ 4125 people to vote for A.
il We would expect np =~ 7500 x 0.29 ~ 2175 people to vote for B.
iii 'We would expect np =~ 7500 x 0.16 ~ 1200 people to vote for C.

8 Let C represent a person who has cancer, and P represent a positive result.
P(CNP
a 095 _» P P(C|P):W
C <
0.02 005~ p' _ 0.02 x 0.95
0.02 x 0.95 + 0.98 x 0.03
003 _~P ~ 0.393
0.98 o <
P/
0.97

b 7 =>5000 x 0.02 =100 {the number of people with cancer out of 5000 people}
p = P(person with cancer is correctly diagnosed) = 0.95
We would expect np = 100 x 0.95 = 95 people to be correctly diagnosed.
442 Chapter 10 (Probability) Review set 10A

1 Total frequency =2+5+9+5+4+4+1 Nl afemazls


=30
a P(Katie will send 5 emails) = i~
~0.13
b P(Katie will send less than 3 emails)
= P(Katie will send 0, 1, or 2 emails)
24549
30
16
30
~ 0.53

2 a i U={ABCDEFGHILJKLMNOPQRSTUYV,WXY,Z}
i C={CEH,S}

¢ (' is the event that a letter randomly selected from the English alphabet is not in the word
CHEESE.

d P(C)= % and P(C')=1-P(C)


=2 ) -23
—ui

3 a coin
A
T
H
» spinner
A B C D

b i P(a head and consonant) il P(atail and C) = %


=P(HNB)+P(HNC)+P(HND)

=3
brird
8
ili P(a tail or a vowel or both) = P(T) + P(A) — P(TN A)
4,2
RT8
1 78
Chapter 10 (Probability) Review set 10A 443

L a Two events are independent if the occurrence of either event does not affect the probability that
the other occurs. For A and B independent, P(AN B) = P(A) x P(B).
b Two events A and B are mutually exclusive if they have no common outcomes.
P(AU B) = P(A) + P(B)
5 Let A represent student A solving the problem, B represent student B solving the problem, and
C represent student C solving the problem.
P(4) =01, P(B)=0.2, P(C)=03
P(at least one student solves it) = 1 — P(no-one solves it)
=1-P(A'NnB'nC")
=1 (0.9%0.8x0.7)
=0.496

6 a Rain Wind
0.36_» W
R <
0.2 0.64 W’

0.75 R/
%: w
0.64™ W’

b 1 P(rain and wind) = P(RN'W)


=0.25 x 0.36
=0.09
i P(rain or wind or both) = P(RN W) +P(RNW’) +P(R'N'W)
=0.25x0.36 + 0.25 x 0.64 + 0.75 x 0.36
=0.52
¢ It is assumed that the events rain and wind are independent.

7 P(A)=2z and P(B’) =043


a P(AUB)=P(A) +P(B) {mutually exclusive events}
=P(A) +[1-P(B)]
=z+1-0.43
=z
+ 0.57

b P(AUB)=2z+0.57=0.73
.o =0.16

8 P(Y)=0.35 and P(XUY) =08


a P(XNY)=0 {X and Y are mutually exclusive events}
b P(XUY)=P(X)+P(Y)
. 0.8=P(X)+0.35
" P(X) =045
¢ P(X orY but not both) = P(X or Y) {X and Y mutually exclusive}
=P(XUY)
=0.8
444 Chapter 10 (Probability) Review set 10A

9 a die2
A
6
5
4
3
2
1

1234506 del
b i die2 i die2
A A
6 6
5 5
4 4
3 3
2 2
1 1

123456 4t 1234506 4!
P(sum of 7 or 11) = % {shaded} P(sum of at least 8) = g {shaded}
=2 9 -5
12
10 a Let E represent the event that a student studies Economics, and L represent the event that a
student studies Law.
n(E)=22, n(L)=25 n(E'NL)=3 nlU)=40
n(EUL)=n(U)—n(E'NL")
=40-3 E L
=37
n(EUL)=n(E)+n(L)—n(ENL)
s 3T=22+25—-n(ENL) . 3)
- n(ENL)=10
n(ENL)=22-10=12 and n(E'NL)=25-10=15
b 1 P(ENL)= i il P(at least one) = jarier
40 40
=1 1 — 37
10
_P(ENL)
iii P(E|L)= D)

_w
T 25
el 5

11 n = 5000 seeds
p = P(tomato seed will germinate) = 0.87
np = 5000 x 0.87 = 4350 tomato seeds are expected to germinate.

12 Total number of marbles = 3+ 4+ 5 =12

a P(both are blue) = 7 5 2>< %


254
Chapter 10 (Probability) Review set 10A 445

b P(they are the same colour) = P(2 reds) + P(2 yellows) + P(2 blues)

= (ExfH)+(xH)+(HxH)
9+16+25
144
50
144
25
=72
< P(at least one is red)
= P(2 reds) + P(1 red, 1 yellow) + P(1 yellow, 1 red) + P(1 red, 1 blue) + P(1 blue, 1 red)
5
=@ExH) (G (ExB)+(Exn)+(Hxn)
9+12412415+15
144
63
T 144
- T
16
d P(exactly one is yellow)
= P(1 red, 1 yellow) + P(1 yellow, 1 red) + P(1 yellow, 1 blue) + P(1 blue, 1 yellow)

(Bxwm)+(Exw)+(Hxs)+(Sxm)
12412420+
20
o 144
64

13 a
| Smoker | 20 | 40 | 60 |
| o | 90 [ 110 | 200 |
. 70 - 70
b i P(a female non-smoker) = — ii P(male | non-smoker) = —
200 140
_ _1
=120 S 2
¢ ii P(both are non-smoker females) _= 10
555 . X 69
155
_ 4830
= 39800
~0.121

ii 95t oX 140
P(one isi smoker and other is- non-smoker) =_ 60 155 355 .X 60
L+ 140 195
_ 16800
= 39800
~ 0.422
446 Chapter 10 (Probability) Review set 10B

14 P(A)=%2, P(B)=+, P(B|A)=1%


a P(B\A):PE(Q)A) b P(B|A)=1+P(B)
1 _P(BNA) A and B are not independent.
1= 2
P(BNA)=

REVIEW SET 10B

1 a n(T)=10, n(M)=17, n((TUM))=5, nU)=30


n(T'UM)=n(U) —n((TUM)")
=30-5
=25
n(TUM)=n(T)+n(M)—n(TNM)
. 25=10+
17— n(T N M)
on(TNM)=2
n(TNM)=10-2=8 and n(T'NM)=17—2=15
b
.i P(TfiM)_300] Bit P((T'NM)|M)
y )
=21 2
15 _ 30
=T30
_z2 17

2 Let W represent the photocopier working.


P(works on at least one day)
W%:W =P(WNW)+P(WnW)+P(W NW)
0.95 005~ W =0.95x0.95 + 0.95 x 0.05 + 0.05 x 0.95
= 0.9975
0.05 W’% i or P(works on at least one day)
0.0~ W’ =1 — P(does not work on either day)
=1-10.05x 0.05
=1-0.0025
= 0.9975
Chapter 10 (Probability) Review set 10B 447

3 P(A)=04 and P(B)=0.7


a P(ANnB)=P(A) xP(B) {A and B are independent events}
=0.4x0.7
=0.28
For A and B to be mutually exclusive, P(AN B) must be equal to 0. This is not the case, so
A and B cannot be mutually exclusive.
b P(AUB) =P(A) +P(B) — P(ANB)
=04+0.7T—-0.28
=0.82

L Let L; be the event that the salesman leaves his sunglasses in store i.
store 1 store 2
il
i

~ ™~

P(salesman left sunglasses in first store given that he left them in one of the stores)
=P(Ly | Ly or Lo)

P(Ly N (L1 U Ly))


P(L1ULy)
P(L1)
P((L1 N LQI) U (Lll N La))
[on

il =
ey

X
il
OlUT

5 a 15 men prefer tea, so 50 — 15 = 35 men prefer coffee.


n(CNM)=35 and n(C'NM)=15
24 women prefer tea, so 50 — 24 = 26 women prefer coffee.
n(CNM') =26
n(C'NM") =n(U) —n(CUM) C M
=100—-26—-35—-15
=24
(24)
P(MNC)
b P(M|C)= P(C)
35
100
26435
100
35
61
448 Chapter 10 (Probability) Review set 10B

6 a LetR represent Rolf winning a set and N represent Niklas winning a set.
1st set 2nd set 3rd set

0.6 R
R < 0.6 » R
0.6
04> N <
04" N <-——

0.6 _» R
0.4
0.6 - R <
N < 0.4™ N <—
0.4 N <—

b P(Niklas will win the match) = P(RNNNN) +P(NNRNN)+P(NNN)


{branches marked <— }
=06x04x04 4+ 04x06x04 + 04x04
=0.352

7 P(A)=08 and P(B)=0.65


a P(AUB)=P(A) +P(B)-P(ANB)
=0.840.65 — [P(A) x P(B)] {A and B are independent events}
=145 — (0.8 x 0.65)
=093
b P(A|B)=P(4) {A and B are independent events}
=0.8
! /
¢ P(A|B)= % d P(B|A)=P(B) {Aand B are independent events}
P((AU BY) =0.65

P(B’)
1-0.93
1-0.65

T
007
035
=0.2

8 a P(win first 3 prizes) = P(win 1st prize) x P(win 2nd prize given that you won 1st prize)
x P(win 3rd prize given that you won 1st and 2nd prizes)
4
= Foo X
3199 X
2108
~ 0.000000193
b P(win at least one of first 3 prizes)
=1 — P(do not win any of first 3 prizes)
=1- [P(do not win 1st prize) x P(do not win 2nd prize given that you did not win 1st prize)
x P(do not win 3rd prize given that you did not win 1st or 2nd prizes)]

=1-
(496 o 405 . 494
(53 x 208 X %)
~ 0.0239
Chapter 10 (Probability) Review set 10B 449

9 Let M represent a male student, F' represent a female student, and S represent a student who
participates in the survey.
a P(student will participate in the survey)
0.16 . S =P(MNS)+P(FNS)

0.48
M < )
=048 x 0.16 + 0.52 x 0.35
084S =0.2588
05 0.35_» S b P(student is female | will participate in survey)

N r 0.65 >S5
—P(F|S)
p(FnS)
T P(S)
_0.52x0.35
~ 7 0.2588
~ 0.703

11 Let C represent Jon going cycling, and E represent Jon having eggs for breakfast.
a P(Jon has eggs for breakfast) = P(C' N E) +P(C' N E)
5|u/\5|\1

om

1= Sl

=l
.I 5 X

X
=
Q
e

IQ‘
om
Nc/\hh—-

|
Bom

=
o
e

;Ix
Il

b P(Jon goes cycling | he has eggs for breakfast) = P(C' | E)


_P(CNE)
P(E)
3 -
7 10
X
3
w|= M=
450 Chapter 10 (Probability) Review set 10B

12 After 2 pregnancies the woman has given birth to 3 children.


she must have had twins and then a single baby, or a single baby and then twins.
The probabilities of having twins or a single baby remain constant. The probability of having twins
first is the same as having twins second.
P(woman had twins first) = 1

13 We extend the table to include totals for each row and column.

a | There are 100 balloons in total in the pack.


il There are 33 medium balloons in the pack.
b 1 P(balloon is not yellow) = P(balloon is red or blue)
_ 51+25
~ 100
_6 100
_ L25

il P(balloon is either medium or small) = 3313041

_n 100
_ 8150
P(both balloons are red)
n

= P(first balloon is red) x P(second balloon is red given that first is red)
51 50
= 100 X 99
_ 5L
198

_ 1766

ii P(neither of the balloons are large)


= P(first balloon is medium or small)
x P(second balloon is medium or small given that first is medium or small)
_ 33441 334411
100 99
74 73
o —
100 99
5402
~ 9900
_ 2701
4950
Chapter 10 (Probability) Review set 10B 451

iii P(exactly one balloon is blue)


= P(first balloon is blue and second balloon is red)
+ P(first balloon is blue and second balloon is yellow)
+ P(first balloon is red and second balloon is blue)
+ P(first balloon is yellow and second balloon is blue)
25 51 4 25 ><24 . 51 25 4 24 25
X X X
100 99 T 100 99 T 100 99 " 100 99
1275 4 600 + 1275 4 600
s 9900
3750
= 9900
_662
iv P(at least one of the balloons is blue)
= 1 — P(neither of the balloons is blue)
=1- [P(first balloon is red and second balloon is yellow)
+ P(first balloon is yellow and second balloon is red)
+ P(both balloons are red) + P(both balloons are yellow)]
(5L
=1-(Tp
24 L
%% + B
24 51 L 51,50 , 24,
X% + 100X 53 + o5 X
2359)
— 1 _ 5550
= 9900
_ 4350
= 9900
2
= 56
d i P(all three balloons are small and By yellow
= P(first balloon is small and yellow)
x P(second balloon is small and yellow given that first is small and yellow)
x P(third balloon is small and yellow given that first two are small and yellow)
| 1 10 9
100 X 59 X 58
=10
9
i P ( exactly two balloons are medium and red)
=P ( first two are medium and red, third is not)
+ P(first and third are medium and red, second is not)
+ P(second and third are medium and red, first is not)
15 14,8 _ 15 . 85 . 14 , 85 15 _ 14
To0
X< 99 X 98 T To0 X 99 X 95 T o0 X 99 X 8
_ 53550
= 570200
_ar
= 308

14 b Of 100000 females born, 98 956 are still alive at age 15.


Of 100000 males born, 98 555 are still alive at age 15.
98956 + 98 555
P(reaching the age of 15) =
200000
197511
~ 200000
=0.987555
~ (0.988
452 Chapter 10 (Probability) Review set 10B

¢ i There are 98 555 boys alive at age 15, and 53942 still alive at 75.
53942
P(15 year old boy will reach age 75) =
98 555
~ 0.547
ii There are 98956 females alive at age 15, and 72656 still alive at age 75.
72656
P(15 year old girl will not reach age 75) =1 —
98 956
26300
98956
~ (0.266
d In general, females live longer.
e A 20 year old is expected to live much longer than 30 more years, so it is unlikely the insurance
company will have to pay out the policy. A 50 year old however is expected to live for only
another 26.45 years (males) or 31.59 years (females), so the insurance company may have to
pay out the policy.
g For “third world” countries with poverty, lack of sanitation, and so on, the tables would show
a significantly lower life expectancy.
Chapter 11
SAMPLING AND DATA

1 The sample size is only 7 patients which is far too small to draw reliable conclusions about the
drug’s effectiveness for all patients.
The sample size is very small and may not be representative of the whole population.
The sample was taken in a Toronto shopping mall. People living outside of the city are probably
not represented.

The sample is likely to under-represent full-time weekday working voters.


The members of the golf club may not be representative of the whole electorate.
Only people who catch the train between 7 am and 9 am such as full-time workers or students
will be sampled.
The voters in the street may not be representative of those in the whole electorate.

The sample size of only 10 sheep from a population of 2000 is far too small, so this may
produce a coverage error.
With only 10 sheep being weighed, any errors in the measuring of weights will have more
impact on the results, so this may produce a measurement error.

The journalist’s question is worded in such a way as to lead the respondents to answer in a
certain way, which may produce a measurement error.
The question could be worded differently, such as “What are your views about the Government’s
proposed plan to move funding from education to health?”.

The whole population is being considered, not just a sample. There will be no sampling error
as this is a census.
Two of the sons have used a different method of counting the number of apples from the other
two sons. This is likely to produce a measurement error.

Many of the workers may not return or even complete the survey, which may produce a
significant non-response error.
There may be more responses to an online survey as many workers would feel that it is easier
to complete a survey online rather than on paper and mailing it back. Responses would also be
received more quickly however some workers may not have internet access and will therefore
be unable to complete the survey.

Yes, the non-response error in this situation is likely to produce a biased sample. Members
with strong negative opinions regarding the management structure of the organisation are more
likely to respond.
No, the feedback from the survey is still valid. Although it might be biased, the feedback might
bring certain issues to attention.
454 Chapter 11 (Sampling and data) Exercise 11B
c
=

Note: The solutions given for questions 1 and 2 are sample solutions only - many solutions are possible.
The random numbers generated in the solutions to these questions are different from those
generated in the answers given in the back of the book.

1 a B HfRadfon) /R
RanInt#(5,25,6)
{13,12,24,7,15,8}
0

[Ran#| Int [Norm| Bin [List [Samp| [Ran#] Int [Norm| Bin [List [Samp]
The numbers randomly generated are The numbers randomly generated are
13, 12, 24, 7, 15, and 8. 14, 22, 13, 16, 24, 25, 3, 4, 23, and 5.

< B ERdlon) @oRal


RanInt#(1,45,6) Int#(100,499,5)
i {18,43,38,1,33,14} 4,348,481,127, 236>

[Rant#| Int [Norm| Bin [List [Samp| [Ran#| Int [Norm[ Bin [List [Samp|
The numbers randomly generated are The numbers randomly generated are
18, 43, 38, 1, 33, and 14. 164, 348, 481, 127, and 236.

We use the following calendar for 2019 to answer question 2.


Chapter 11 (Sampling and data) Exercise 11B 455

CALENDAR 2019
[Tu (1)
1 RFr (32) [ e
Fr (60)
[ Mo
ea
(91)
[ W
We (121)
[ Sa (152)
]
We (2) Sa (33) Sa (61) Tu (92) Th (122) Su (153)

W N

W N

W N

W N
Th (3) Su (34) Su (62) Fr (123) Mo (154)
OO

We (93)

NG

NG

NG
W

U W
Mo (35)

WO
Sa (124) Tu (155)

0N
Fr (4) Mo (63) Th (94) Wk 23
Sa (5) Tu (36) Tu (64) Fr (95) Su (125) We (156)
Su (6) We (37) We (65) Sa (96) Mo (126) Th (157)
Mo (7) Th (38) Th (66) Su (97) Tu (127) Wk 19 Fr (158)
0N

Tu (8) Wk2 ® Fr (39) Fr (67) Mo (98) We (128) Sa (159)

®
9 We (9) Sa (40) Sa (68) Tu (99) Wk 15 Th (129) 9 Su (160)
10 Th (10) Su (41) Su (69) We (100) Fr (130) 10 Mo (161)
11 Fr (11) Mo (42) Mo (70) Th (101) Sa (131) 11 Tu (162) Wk 24
12 Sa (12) Tu (43) Tu (71) Fr (102) Su (132) 12 We (163)
13 Su (13) We (44) We (72) Sa (103) Mo (133) 13 Th (164)
14 Mo (14) Th (45) Th (73) Su (104) Tu (134) Wk 20 14 Fr (165)
15 Tu (15) Wk 3 Fr (46) Fr (74) Mo (105) We (135) 15 Sa (166)
16 We (16) 5 Sa (47) 5 Sa (75) 5 Tu (106) 5 Th (136) 16 Su (167)
17 Th (17) Su (48) Su (76) We (107) Fr (137) 17 Mo (168)
18 Fr (18) Mo (49) Mo (77) Th (108) Sa (138) 18 Tu (169) Wk 25
19 Sa (19) Tu (50) Tu (78) Fr (109) Su (139) 19 We (170)
20 Su (20) We (51) We (79) Sa (110) Mo (140) 20 Th (171)
21 Mo (21) Th (52) Th (80) Su (111) Tu (141) Wk 21 21 Fr (172)
22 Tu (22) Wk4 Fr (53) Fr (81) Mo (112) We (142) 22 Sa (173)
23 We (23) Sa (54) Sa (82) Tu (113) Th (143) 23 Su (174)
24 Th (24) Su (55) Su (83) We (114) Fr (144) 24 Mo (175)
25 Fr (25) Mo (56) Mo (84) Th (115) Sa (145) 25 Tu (176) Wk 26
26 Sa (26) 5 Tu (57) 5 Tu (85) 5 Fr (116) 5 Su (146) 26 We (177)
27 Su (27) We (58) We (86) Sa (117) Mo (147) 27 Th (178)
28 Mo (28) Th (59) Th (87) Su (118) Tu (148) Wk 22 28 Fr (179)
29 Tu (29) Wk5 Fr (88) Mo (119) We (149) 29 Sa (180)
30 We (30) Sa (89) Tu (120) Wk 18 Th (150) 30 Su (181)
31 Th (31) Su (90) Fr (151)

Mo (182) Th (213) Su (244) Tu (274) Wk 40 Fr (305) 1 Su (335)


Tu (183) Wk 27 Fr (214) Mo (245) We (275) Sa (306) 2 Mo (336)
G R W N

Th (276)
WN

We (184) Sa (215) Tu (246) Wk 36 Su (307) 3 Tu (337) Wk 49


G R W

Fr (277) Mo (308) 4 We (338)


0N

0N

0N

Th (185) Su (216) We (247)


Fr (186) Mo (217) Th (248) Sa (278) Tu (309) Wk 45 5 Th (339)
G

Sa (187) Tu (218) Wk 32 Fr (249) Su (279) We (310) 6 Fr (340)


Su (188) We (219) Sa (250) Mo (280) Th (311) 7 Sa (341)
Mo (189) Th (220) Su (251) Tu (281) Wk 41 Fr (312) 8 Su (342)
Tu (190) Wk 28 Fr (221) Mo (252) We (282) Sa (313) 9 Mo (343)
©

We (191) Sa (222) Tu (253) Wk 37 Th (283) Su (314) 10 Tu (344) Wk 50


- O

We (254) Fr (284) Mo (315) 11 We (345)


A WN RO

Th (192) Su (223)
TIOTRARINROOCOTIO
FSOXISCNRUNR,OORIDN

R IO RARNBNRO©ETID
A WO

e e e e
e
e

WWINNNNNNNNNNE

Fr (193) Mo (224) Th (255) Sa (285) Tu (316) Wk 46 12 Th (346)


Sa (194) Tu (225) Wk 33 Fr (256) Su (286) We (317) 13 Fr (347)
N A W

Su (195) We (226) Sa (257) Mo (287) Th (318) 14 Sa (348)


e e

Mo (196) Th (227) Su (258) Tu (288) Wk 42 Fr (319) 15 Su (349)


Tu (197) Wk 29 Fr (228) Mo (259) We (289) Sa (320) 16 Mo (350)
2 SR

Th (290)
e

We (198) Sa (229) Tu (260) Wk 38 Su (321) 17 Tu (351) Wk 51


Th (199) Su (230) We (261) Fr (291) Mo (322) 18 We (352)
Fr (200) Mo (231) Th (262) Sa (292) Tu (323) Wk 47 19 Th (353)
Sa (201) Tu (232) Wk 34 Fr (263) Su (293) We (324) 20 Fr (354)
Mo (294) Th (325) 21 Sa (355)
N NN

Su (202) We (233) Sa (264)


DN NN

Mo (203) Th (234) Su (265) Tu (295) Wk 43 Fr (326) 22 Su (356)


Tu (204) Wk 30 Fr (235) Mo (266) We (296) Sa (327) 23 Mo (357)
We (205) Sa (236) Tu (267) Wk 39 Th (297) Su (328) 24 Tu (358) Wk 52
NN

Th (206) Su (237) We (268) Fr (298) Mo (329) 25 We (359)


Fr (207) Mo (238) Th (269) Sa (299) Tu (330) Wk 48 26 Th (360)
NN

Sa (208) Tu (239) Wk 35 Fr (270) Su (300) We (331) 27 Fr (361)


W WM

Su (209) We (240) Sa (271) Mo (301) Th (332) 28 Sa (362)


Mo (210) Th (241) Su (272) Tu (302) Wk 44 Fr (333) 29 Su (363)
W
SO

Tu (211) Wk 31 Mo (273) We (303) Sa (334) 30 Mo (364)


FOO

Fr (242)
We (212) Sa (243) Th (304) 31 Tu (365) Wk 53
456 Chapter 11 (Sampling and data) Exercise 11B

2 We select 5 random numbers between We select a random number between 1 and


1 and 365 inclusive. 52 inclusive.

Bl WatiRadforn]) (b/cReal
RanInt#(1,365,5
D(98,263,121,178,241)

[Ran#| Int [Norm| Bin [List [Samp}

The randomly generated numbers are The randomly generated number is 37.
96, 263, 121, 178, and 241. Looking at the calendar, we take the week
Looking at the calendar, these numbers which starts on the Monday that lies in the
correspond to the dates 6th April, 20th 37th week. This corresponds to the week
September, 1st May, 27th June, and 29th Monday 16th September to Sunday 22nd
August. September.

We select a random number between 1 and We select 3 random numbers between


12 inclusive. 1 and 12 inclusive.

E EatRadforn]) @b/dReal
RanInt#(1,12) RanInt#(1,12,3)
n {9,2,3}
0

[Ran#| Int [Norm|


Bin [List]| [Ran#| Int [Norm[
Bin [List [Samp

The randomly generated number is 6. The The randomly generated numbers are
6th month of the year is June, so the 9, 2, and 3.
sample is the month of June. The 9th, 2nd, and 3rd months of the year
are September, February, and March.

We select a random number between 1 and We select 4 random numbers between


10 inclusive for the starting month. 1 and 52 inclusive.

B EstRadMorn]) [b/dReal HatRedForn]) [ab7dReal


RanInt#(1,10) RanInt#(1,52,4)
a {50,26,35,48}
O

[Ran#| Int [Norm| Bin [List [Samp| [Ran#| Int [Norm[ Bin [List [Samp}

The randomly generated number is 8. The randomly generated numbers are


The 8th month of the year is August, 50, 26, 35, and 48.
so the sample is the months of August, Looking at the calendar, we take the
September, and October. Wednesday that lies in these weeks.
This corresponds to the dates
11th December, 26th June, 28th August,
and 27th November.
Chapter 11 (Sampling and data) Exercise 11B 457

Lo W,
2(%7100750
So, every 50th block of chocolate will be sampled.
The first block to be sampled is the 17th block.
So, the first 5 blocks to be sampled are the 17th, 67th, 117th, 167th, and 217th blocks.

Total number of blocks sampled = 2% of 80 000


=0.02 x 80000
= 1600 blocks of chocolate

The sampling method used is convenience sampling, as the first 40 people through the gate are
more convenient to sample than every 80th person for example.
The first 40 people through the gate will probably spend more time at the show, and so are
more likely to spend more than €20. Also, the sample size is relatively small, being about 1.1%
of the total number of visitors.
We could use a systematic sampling technique in which every 10th person through the gate is
surveyed. The sample size would therefore be 10% of the total population.

The sampling method used is systematic sampling as people have been selected at regular
intervals.
1 year = 365 days

_ 368 lots of 28 days


28
= 13 lots of 28 days + 1 day remaining
So, including the first Monday sampled, there will be 13 + 1 = 14 days in her sample.
28 days = 4 x 7 days, and the first day sampled is a Monday. So the sample consists of every
fourth Monday in the year. Only visitors who use the library on Mondays will be counted.
Mondays may not be representative of all the days in a week, so the sample may be biased.

Total number of members = 80 + 60 + 20


= 160 members
For the sample, we want:
number of tennis members = 2% x 40 = 20
number of lawn bowls members = %00 x40=15
number of croquet members = 2% x 40 =5
So, the club should sample 20 tennis members, 15 lawn bowls members, and 5 croquet members.
458 Chapter 11 (Sampling and data) Exercise 11C

7 Total number of staff = 10 + 24 + 65 + 98 4 28 = 225


For the sample, we want:
number of departmental managers = % x30~133~1
number of supervisors = % x30=32~3
number of senior sales staff = £= x 30 ~ 8.67 ~ 9
number of junior sales staff = % x 30~ 13.07 = 13
number of shelf packers = % x30~3.73~4

Now 143494 13+4 =30 which is the required sample size.


So, 1 departmental manager, 3 supervisors, 9 senior sales staff, 13 junior sales staff, and 4 shelf
packers should be selected for the sample.

8 a It is easier for Mona to survey her own home room class, so this is a convenience sample.
b Mona’s sample will not be representative of all of the classes in the school. Mona’s survey
may be influenced by her friends in her class. Mona’s sample will therefore be biased.
¢ A stratified sample of students from every class may be a more appropriate sampling method.

9 a Not all students selected for the sample will be comfortable discussing the topic, so it may not
be practical for Lucian to use a simple random sample or systematic sample.
b Lucian should use a quota sample so that individuals may be specifically selected rather than
randomly selected as in a stratified sample.

10 a This is considered to be a census because all of the Year 11 and Year 12 students were asked,
not just a sample of them.
b 96 students said they had smoked.
Proportion of all students who said they had smoked =poibeghrsaid they had Smoked
total number of students
96
"~ 200
=048
¢ i The sample size would be too small to be representative of the whole population.
il The sample size would be too small to be representative of the whole population.
ili This sampling technique would be valid but at 50% of the population, it is an unnecessarily
large sample size.
iv This is a valid sampling technique with an appropriate sample size.
v This is a valid sampling technique with an appropriate sample size.
vi This is a valid sampling technique with an appropriate sample size.
d v is simple random sampling, fii and iv are systematic sampling, and vi is stratified or quota
sampling.

1 a The number of brothers a person has takes exact number values.


this is a discrete variable which could take the values 0, 1, 2, 3, ....

b The colours of lollies in a packet is a categorical variable which could have the categories red,
yellow, orange, green, and so on.
Chapter 11 (Sampling and data) Exercise 11C 459

¢ The time children spend brushing their teeth each day is a numerical variable which can be
measured. The data can take any value between certain limits.
this is a continuous variable which could take any value from 0 to 15 minutes.
d The height of the trees in a garden is a numerical variable which can be measured. The data
can take any value between certain limits.
this is a continuous variable which could take any value from 0 to 25 metres.
e The brand of car a person drives is a categorical variable which could have the categories Ford,
BMW, Renault, and so on.

f The number of petrol pumps at a service station takes exact number values.
this is a discrete variable which could take the values 1, 2, 3, ....
g The most popular holiday destinations is a categorical variable which could have the categories
Australia, Hawaii, Dubai, and so on.

h The scores out of 10 in a diving competition take exact number values.


this is a discrete variable which could take the values 0.0, 0.5, 1.0, ...., 9.5, 10.0.

The amount of water a person drinks each day is a numerical variable which can be measured.
The data can take any value between certain limits.
this is a continuous variable which could take any value from 0 to 4 litres.
i The number of hours spent per week at work is a numerical variable which can be measured.
The data can take any value between certain limits.
this is a continuous variable which could take any value from 0 to 80 hours.
k The average temperatures of various cities is a numerical variable which can be measured. The
data can take any value between certain limits.
this is a continuous numerical variable which could take any value from —20°C to 35°C.
| The items students ate for breakfast before coming to school is a categorical variable which
could have the categories cereal, toast, fruit, rice, eggs, and so on.
m The number of televisions in each house takes exact number values.
this is a discrete variable which could take the values 0, 1, 2, ....

2 The player’s name is a categorical variable.


The player’s age can be measured and can take any value between certain limits, so it is a continuous
variable.
The player’s height can be measured and can take any value between certain limits, so it is a
continuous variable.
The player’s country is a categorical variable as it describes which country the player is playing for.
The player’s fournament wins can be counted, so it is a discrete variable.
The player’s average serving speed can be measured and can take any value between certain limits,
s0 it is a continuous variable.
The player’s ranking can be counted, so it is a discrete variable.
The player’s career prize money can be counted, so it is a discrete variable.
460 Chapter 11 (Sampling and data) Exercise 11D

1 a The variable being considered is the number of goals scored in a game.


b The data is discrete as only a whole number of goals can be scored. The variable is counted,
not measured.

¢ | Goals scored | Tally | Frequency | Relative Frequency

d 10 frequency

6
4

214

0 | >
0 1 2 3 4 5 6
goals scored

e The modal score for the team is 1 goal.


f The data is positively skewed with one outlier (6 goals).
g The Flames failed to score in % x 100% =~ 20.8% of games.

2 a 0 frequency

01 2 3 4
I5 6 7 8
number of students on detention
9

b The modal number of students on detention in a week is 1 and 2 students.


¢ The data is positively skewed with one outlier (9 students).
Chapter 11 (Sampling and data) Exercise 11D 461

d There were more than 4 students on detention in 34141 50100% = 124% of weeks.

b
12y frequency
10
O

-lll.l
N
O

1 2 3 4 5 6
number of previews

The mode of the data is 3 previews.


The data is symmetrical with no outliers.
10+94+5+3
At least 3 previews were shown on x 100% =~ 79.4% of occasions.

8+12+9+7+4+2+241
=45 people 154 frequency
were surveyed.
The mode of the data is 1 time.
8 people did not eat out at all last week.

dr2+241 100% = 20% of people


surveyed ate out more than three times last
week. o 1 2 3 4 5 6 7
number of times
The data is positively skewed with no outliers.
462 Chapter 11 (Sampling and data) Exercise 11E

b There were less than 10 people at the station on 2 days.

¢ There were at least 30 people at the station on THa 100% =~ 36.7% of days.

d
frequency
N
==
N0 O
O

0 10 20 30 40
people waiting

e The modal class of the data is 20 - 29 people.

2 a 24+5+7+8+9+6 = 37 businesses were Number of employees


surveyed. 10 frequency
b The modal class is 40 - 49 employees. 8
¢ The data is negatively skewed. i

d 24547 0 100% ~ 37.8% of businesses 2 —


surveyed had less than 30 employees. 0 0 10 20 30 40 50 60
e No, it is not possible to determine the highest number of employees
number of employees a business had. We can only
say that it was in the interval 50 - 59 employees.

I
I
I
I
Chapter 11 (Sampling and data) Exercise 11F 463

b 164 frequency
14
12
10
8
6 [ (| e IR LT

4 { (- (] L R

2 (] [ [ e el s

0 >
0 10 20 30 40 50
number of houses

¢ The modal class is 30 - 39 houses.

d SHHANE s 100% = 67.5% of the streets contain at least 20 houses.

170 < H < 175


/

175 << H < 180


< H <185
<
< H <195
< H <200
< H < 205

a Height is a continuous variable as it is measured on a continuous scale.


b Heights of a volleyball squad
12 frequency
10
8
6
4 e Rl

2 '—
0 -
170 175 180 185 190 195 200 205
height (cm)

¢ The modal class 185 < H < 190 cm occurs most frequently. More volleyball players have
heights in this interval than in any other interval.
d The data is slightly positively skewed.
464 Chapter 11 (Sampling and data) Exercise 11F

: R
The data is discrete, so a column graph should be 4 frequency
used. 15

10

47 48 49 50 51 52 53 54 55
number of matches

b The data is continuous, so a frequency histogram


0 < should be used.
0 < A frequency
ol
15 I I R e
0 <
0< B
0 < 5
0
120 130 140 150 160 170
height (cm)

3 a Travel time is a continuous variable, even though times have been rounded to the nearest minute.

b
<t<10 M
SE<20 | T M|
St<30 | ML
<t<40 | M
0

<
Travel times to school

0
0 10 20 30 40 50
Travel time (min)
d The data is positively skewed.
e The modal travelling time is 10 < ¢ < 20 minutes. More students have travel times in this
interval than in any other interval.
Chapter 11 (Sampling and data) Exercise 11F 465

4 a The variable distanceis continuous, even b | Distance (m) | Tally


though distances have been rounded to
the nearest 10 cm. h
The shortest distance is 7.4 m and the 0<
longest is 42.9 m, so we will use class 0 <
intervals of width 10 m: 0 < d < 10,
10<d <20, ... 40<d< 50. 0<
0 <

< Javelin throwing distances


104 frequency
8
6
4
2
0
0 10 20 30 40 50
distance (m)

d The modal class is 20 < d < 30 m. More athletes achieved distances in this interval than in
any other interval.

e % x 100% = 36% of athletes threw the javelin 30 m or further.

5 | Height (h mm) | Frequency


300 < h <325 12
/

325 < h < 350 18


NN

350 0 < h < 375 42


5 h < 400 28
INCINCIN

0 h < 425 14
5 h < 450 6

@ Heights of 6-month old seedlings at a nursery

50 frequency
40
30
20
07 _'j‘
0 > height (mm)
300 325 350 375 400 425 450

b 14 + 6 = 20 seedlings are 400 mm or higher.


¢ 12+ 18 442+ 28 + 14 + 6 = 120 seedlings have been measured.
42 + 28
x 100% = 58.3% of the seedlings are between 350 mm and 400 mm high.
120
466 Chapter 11 (Sampling and data) Review set 11A

12+ 18 +42+28
d x 100% = 83.3% of the seedlings are less than 400 mm high.
120
So in a population of 1462 seedlings, we would expect
83.3% of 1462 = 0.833 x 1462
~ 1218 seedlings to be less than 400 mm high.
28414
ii x 100% = 35% of the seedlings are between 375 mm and 425 mm high.
120
So in a population of 1462 seedlings, we would expect
35% of 1462 = 0.35 x 1462
~ 512 seedlings to be between 375 mm and 425 mm high.

6 a The variable weight is a continuous b Weight (g) Tally


variable, even though weights have been 100 <w
< <125
recorded to the nearest gram.
The lowest weight is 100 g and the 125 S w <150
highest is 295 g, so we will use class 150 < w < 175
intervals of width 25 g: 100 < w < 125,
125 < w < 150, ..., 275 < w < 300. Liois =200
0 <w <225
5 < w < 250
0 <w <275
5 < w < 300

< Weights of laboratory rats


124 frequency
10
8
6 [N el "|
4
) —’:
0V >
100 125 150 175 200 225 250 275 300
weight (g)

5+5+11+4
d x 100% = 50% of the rats weigh less than 200 grams.

1 a Students studying Italian may have an Italian background so surveying these students may
produce a biased result.
b Andrew could survey a randomly selected group of students as they entered the school grounds
one morning. This should ensure that the results will be more representative of the whole
population of interest.
Chapter 11 (Sampling and data) Review set 11A 467

2 a As th.ere are 1800'members m'the club, it would be too expensive


and time consuming to question all members.
under 18
b For the sample, we want: 18 - 39
number of under 18s _= 7555
257 % 350 -~ 50.0 ~. 50 40 -- 54
number of 18 - 39s = £2- x 350 ~ 81.9 ~ 82 55 - 70
number of 40 - 54s = 52 x 350 ~ 122.9 ~ 123 over 70
number of 55 - 70s = 356 x 350 ~ 69.2 ~ 69
number of over 70s = 3% x 350 ~ 26.1 ~ 26
Now 50+ 82+ 123 469 + 26 = 350 which is the required sample size.
So, the club should survey 50 members aged under 18, 82 members aged 18 - 39, 123 members
aged 40 - 54, 69 members aged 55 - 70, and 26 members aged over 70.

3 a The number of pages in a book takes exact number values.


this is a discrete variable.
b The distance travelled by hikers in one day is a numerical variable which can be measured.
The data can take any value between certain limits.
this is a continuous variable.
< The attendance figures for a music festival take exact number values.
this is a discrete variable.

4 a It is easier for the police officers to test drivers on a major road due to higher volumes of
traffic, so this is convenience sampling.
b Yes, the sample will be biased as people are more likely to be drinking on a Saturday night.
It is sensible for this sample to be biased since drink-driving is illegal.

5 a The number of rounds won takes exact number Afrequency


values. 8
this is a discrete variable.
5 6
b The modal number of rounds won is 1 round.
< The data is positively skewed with no outliers. 4

2
0
0 1 2
| | =
3 4
>

rounds won

6 a

140
< h < 150
%=

150
< h < 160
%=

8
4
h < 170
3 15

8
S
[
o3
468 Chapter 11 (Sampling and data) Review set 11A

<
50 frequency
40

30

20

10

0
140 150 160 170 180 190
height (cm)
d The modal class is 160 < h < 170 cm. More boys have heights in this interval than in any
other interval.
e The data is slightly negatively skewed.

7 a | Number of tickets | Tally

¢ The data is symmetric


with no outliers.
DY
=
N

| | | [ ] »numberof
S

4 5 6 tickets

8 a The diameter of bacteria colonies is a numerical variable which can be measured. The data
can take any value between certain limits.
.. this is a continuous variable.

49 o 20 we wil e clas
® longe1 st <
0<d<1 3
intervals of width 1 cm.
1< d<2
2 d<3
3< d<4
4< d<5
Chapter 11 (Sampling and data) Review set 11B 469

Diameters of colonies
4y frequency
12
10

o
O
e
N

>
S

0 1 2 3 4 5 diameter (cm)

d The modal class is 3 < d < 4 cm. More bacteria have diameters in this interval than in any
other interval.
The data is slightly negatively skewed.

The number of pages in a daily newspaper takes exact number values.


this is a discrete variable.
The maximum daily temperature in a city is a numerical variable which can be measured. The
data can take any value between certain limits.
this is a continuous variable.
A person’s favourite flavour of ice cream is a categorical variable.
n

The position taken by a player on a lacrosse field is a categorical variable.


The time it takes to run one kilometre is a numerical variable which can be measured. The
data can take any value between certain limits.
this is a continuous variable.
The length of a person’s feet is a numerical variable which can be measured. The data can take
any value between certain limits.
this is a continuous variable.
A person’s shoe size takes exact number values.
this is a discrete variable.
The cost of a bicycle takes exact number values.
this is a discrete variable.

The houses have been selected at regular intervals, so systematic sampling has been used.
A house will be visited if the last digit in its number is equal to the random number chosen by
the promoter, with the random number 10 corresponding to the digit 0. Each house therefore
has a 1 in 10 chance of being visited.
Once the first house number has been chosen, the remaining houses chosen must all have the
same second digit in their house number, that is, they are not randomly chosen. For example,
it is impossible for two consecutively numbered houses to be selected for the sample. So this
is not a simple random sample.

Petra’s teacher colleagues are quite likely to ignore the emailed questionnaire as emails are easy
to ignore. So, Petra’s questionnaire may produce a high non-response error.
470 Chapter 11 (Sampling and data) Review set 11B

b It is likely that the teachers who have responded will have strong opinions either for or against
the general student behaviour. These responses may therefore not be representative of all
teachers’ views. Petra may therefore be likely to encounter a coverage error.

4 a The data is negatively skewed. 16§ frequency


b 34+ 7+11+14+5 = 40 students sat the test. 14

45 o 100% = 47.5% of the students E


scored 13 or more marks. 8

< 4% x 100% = 7.5% of the students scored 2


less than 5 marks. 2
0
20 score

a
frequency

10 20 30 40 50
winning margin (points)

. 13+ 35 .o ) 8
b i n 700 x 100% = 48% of games, the winning margin was 20 points or less.

. 1847 . . .
i In 00 x 100% = 25% of games, the winning margin was more than 30 points.

¢ No, it is not possible to tell from the table what the lowest winning margin was, only that it
was in the interval 1 - 10 points.

6 a The mass of a horse, m kg, is a quantitative variable which can be measured. The data can
take any value between certain limits.
this is a continuous quantitative variable.

0<m
0os<m
N
NN

0<m
®

290 < m
0<m
=)
@

os<m
0<m
Chapter 11 (Sampling and data) Review set 11B 471

¢ The modal class is 300 < m < 310 kg. More horses have a mass in this interval than in any
other interval.
d
54 frequency
4

260 270 280 290 300 310 320 330


mass (kg)

e The data is slightly negatively skewed.

a The lengths of yachts is a numerical variable which can be measured. The data can take any
value between certain limits.
this is a continuous variable.
b The shortest length is 10.1 m and the longest is 27.4 m, so
. i )
we will use class intervals of width 2 m.
Length (I m
T
Frequency
<l
S <12 3
8

NN
<16 8
1< 18 4

IN
1<20 2

N
1 <22 3
INCINCINCIN <24 1
<26 0
1< 28 1

10 frequency
8

10 12 14 16 18 20 22 24 26 28
length (m)

d The data is positively skewed with one outlier (27.4 m).


472 Chapter 11 (Sampling and data) Review set 11B

8 a | Films watched
)
0
1

Olwwo
2
3
4
5

N
6
7 =
O

8
H

b frequency

2,

0 > number of films

¢ The mode of the data is 0 films.


d There were 28 students in the class.
28—17
i x 100% = 75% of the students saw at least one film in the last month.

ii 7%?3 x 100% =~ 57.1% of the students saw less than 3 films in the last month.
Chapter 12
STATISTICS
COETTS——
1 a 1 is the data value which occurs most often, so the mode is 1 cup.
n+1
b As n=15, =38

The ordered data setis: 6—6-—60—++++ 2 223334+

8th value
median = 2 cups

24341+1+...4+1+4<—
sum of all the data values
¢ mean=—-———/
15 15 data values
27
15
= 1.8 cups

. 2+4+3+4+3+....4+8+9+9
< sum of all the data values
2 a i mean="~"—""7-4064—/@
— —
23 23 data values
_1»
sl 23
= 5.61
n+1
i As n=23, “2-=12
The ordered data set is:
2333 4445555666667
7+88899
f
12th value
median = 6

iii 6 is the data value which occurs most often, so the mode is 6.

. 10 4+ 12+ 12 + ... + 19 + 20 + 21 <— sum of all the data values


b i mean= —"7"——— — — —
15 15 data values
245
T
~ 16.3
e 1
ii As n=15, %:8

The ordered data set is:


021245151616
17 1818181819202t
f
8th value
median = 17

iii 18 is the data value which occurs most often, so the mode is 18.
474 Chapter 12 (Statistics) Exercise 12A

. 22.4424.6 + 21.8 4 .... + 25.3 + 29.5 + 23.5 = sum of all the data values
¢ | mean=
11«——————— 11 data values
213
11
~ 24.8

As n=11, "T“=6
The ordered data set is:
2+8-224235235246 24.9 25:0-25326-1+264295
!
6th value
median = 24.9
ili 23.5 is the data value which occurs most often, so the mode is 23.5.

sum of all data values


3 mean =
the
number
of data values
63
~
=9

160 + 175+ .... + 175 4 155 157+ 181 + ... + 168 4 148
L Gordon’s mean = Ruth’s mean =
10 10
_T 1%10 _ 101010
=159 =164

So, Ruth had the higher mean score.

3+4+4+5+....4+10 3+4+4+5+...+15
5 a mean ofset A= mean of set B =
13 13
_s _T 8 13
T 13
~ 6.46 ~ 6.85
1341
b As n =13 for both data sets, the median is the ( >th data value.

the median is the 7th data value for each data set.
median of set A = 7, median of set B =7
¢ The data sets are the same except for the last value, and the last value of set A is less than that
of set B. So, the mean of set A is less than that of set B.
The middle value of both data sets is the same, so the median is the same.

6 a i mean number of motichoor ladoo mean number of malai jamun


_ 62476+55+65+ ... +54 _37T+52+71+59+
...+ 76
B 31 i 31
2079 1663
T Bl X _3ig
~ 67.1 motichoor ladoo ~ 53.6 malai jamun
Chapter 12 (Statistics) Exercise 12A 475

31+1
il As n =31 for both data sets, the median is the < )th data value.

the median is the 16th data value for each data set.
16th value
For the motichoor ladoo, the ordered data set is: l
47—48—49—5H0—54—55—5H6—H8—60—61+—62—63—63—65—67
69
O——72— 56— 67— 78— 7981828285
median = 69 motichoor ladoo
. . 16th value
For the malai jamun, the ordered data set is: |
BT383839 443454647 4849565051 52
53—54—55—56—-57—59—66—61—63—67—68—7F1—72—73—76
median = 52 malai jamun
b The motichoor ladoo were more popular as the mean and median are both higher for motichoor
ladoo than for malai jamun.

29
40.5
48
70

mean = 39.7 passengers mean ~ 49.1 passengers


median = 40.5 passengers median = 49 passengers
b The tram data has a higher mean and median, but since there were more bus trips on the day
and more people travelled by bus in total, we conclude the bus is more popular.

q 43 4+ 55+ 41+ 37
a mean number of points = —41+
25 37
4
176
T4
= 44 points

b Let the score for the next match be = points.


. 4 41
mean number of points for first 5 matches = e i e
5
SOLE 176 + =
5
220 =176 +=z
xr =44
So the team needs to score 44 points in their next match.
476 Chapter 12 (Statistics) Exercise 12A

¢ i If the team scores only 25 points in the fifth match, this will decrease their overall mean
score since 25 is lower than the mean of 44 for the first four matches.
43 455441+ 37425
il mean number of points =
5
= 40.2 points

total sales for the year


= €15467
12
total sales for the year = €15467 x 12
= €185604

10 total dlstir;ce driven — 962 km

total distance driven = 262 km x 12


= 3144 km

- =1
1 T="1

total number of goals for first 14 matches


12 =16.5
14
total number of goals for first 14 matches = 16.5 x 14
=231
231+21+24
netballer’s average for whole season =
16

T
s16
= 17.25 goals per game

5+9+11+12+13+14+ 17+ —19 14 3+0+a+a+4+a+6+a+3


13 4
8 9
81;—1 —19 4a;16
=4

81 +x =96 oo da+16 =36


r=15 oo 4a =20
a=5
15 Let Aruna’s eighth test mark be z.
29436 +32+38+35+34+39+
=35
8
243+ —35
8
243
+ & = 280
x =37
So, Aruna scored 37 marks out of 40 for the eighth test.
Chapter 12 (Statistics) Investigation 1 Effects of outliers 477

sum of sample of 10 measurements


16 =15.7
10
sum of sample of 10 measurements = 15.7 x 10
=157
sum of sample of 20 measurements —143
20
sum of sample of 20 measurements = 14.3 x 20
= 286

Mean of all 30 measurements = W

_© i30
~ 14.8

17 As n=09, nTH =5, so the median is the 5th ordered data value.

The median is 12, so 12 must be one of the unknown measurements. Let the other unknown
measurement be a.
the measurements are 7, 9, 11, 12, 13, 14, 17, 19, and a.

Now, 7+9+11+12+1§+14+17+19——a — 19 {since R 12}

102 4+ a = 108
a=6
So, the other two measurements are 6 and 12.

1 The data set is: 4,5,6,6,6,7,7,8,9, 10


44+54+64+....4+494+10
a mean =
10
68
10
=6.8
b 6 is the data value which occurs most often, so the mode is 6.

¢ Asn=10,
2l _55
The ordered data setis: “4—5—6—+6- ~—
6 7 +—8—9—10-

two middle data values


the median =
6+7_ 6.5

2 The data set is: 4,5,6,6,6,7,7,8,9, 10, 100


4+5+6+4
...+ 10+ 100
a mean =
11
168
11
~ 15.3
478 Chapter 12 (Statistics) Exercise 12B

b 6 is the data value which occurs most often, so the mode is 6.

¢ As n=11, "T“=6
The ordered data setis: 4—5—6—6—6 7 +8—9—10—166-
!
6th value
the median = 7

3 a The presence of the extreme value increases the mean by more than double.
b The presence of the extreme value has no effect on the mode.
¢ The presence of the extreme value increases the median slightly. If the two middle values in
1 ¢ were the same, then the median would not have changed.

4 The mean is most affected by the inclusion of an outlier.

$346 400 + $327 600 + .... + $331 400 + $362 500


1 a 1li
mean selling ice =
price 0
$3 637 700
10
= $363770
n+1 —55
Since n = 10,

So the median is the average of the 5th and 6th ordered data values.
The ordered data set is:
327600—329-500—331406—332466- 346400
————
348000 -362-500—392-500—411-000—456406
two middle data values

$346 400 4 $348 000


median selling price =
2
= $347200
The mean has been affected by the extreme values (the two values greater than $400 000).
b [ If you were a vendor wanting to sell your house, you would use the mean as it is higher,
and you want to sell at the highest price possible.
il If you were looking to buy a house in the district, you would use the median as it is lower,
and is more representative of a typical selling price in the area.

2 a $33000 is the data value which occurs the most often, so the modal salary is $33 000.
$33000 + $56 000 + .... 4+ $33 000 + $42 000
mean salary = i
~$393000
T10
= $39300
Chapter 12 (Statistics) Exercise 12B 479

. 1
Since n = 10, % =55

So the median is the average of the 5th and 6th ordered data values.
The ordered data set is:
-33000—33000—33000—33000- 33000
————
34000 -42000—48000—48000—56000-
two middle data values

$33 000 + $34 000


median salary = 3

= $33500
b The mode is the lowest value and it does not take the higher values into account. So the mode
is an unsatisfactory measure of centre in this case.
¢ No, the median does not take the higher values into account. It is too close to the lower end
of the distribution. So it is not a satisfactory measure of centre for this data set.

a Fedlorat
1-Variable
S}nX =0

Med
OhWOO

Q3
In

maxX
N

Mod

So the mean is ~ 3.19 mm, the median is 0 mm, and the mode is 0 mm.
b The data is very positively skewed which means the median is not in the centre. Therefore the
median is not the most suitable measure of centre for this data set.
¢ The mode is the lowest value, and it does not take the higher values into account. So it is not
the most suitable measure of centre for this data set.
d There are two outliers. They are 21 mm and 42 mm.
e No, the outliers should not be removed as they are genuine data values.

242424424342
a mean number of children = 0
61
© 30
=~ 2.03

Since n = 30, "T“ —155


So the median is the average of the 15th and 16th ordered data values.
The ordered data set is:
1222 2 2 222222333
34444
—~
two middle data values
: . 242
median number of children = %

=2
Both 1 and 2 are the data values which occur the most often, so the modal number of children
per family is 1 and 2.
480 Chapter 12 (Statistics) Exercise 12C

b Yes, the mode is a useful statistic in this case as Esmé can then offer a “family package” to
match the most common number of children per family.
¢ Esmé should include 2 children per family in the package, since this is one of the modes; it is
also the median and is very close to the mean.

2 8 16 21
3 4 12 25
4 5 20 30
Total $r=30 | Tar=61
a Looking down the frequency column, the highest frequency is 13. This corresponds to 1 person,
so the mode is 1 person.
b n+1
There are 30 data values, so n = 30. S 15.5, so the median is the average of the 15th
and 16th ordered data values.
From the cumulative frequency column, the 14th to 21st ordered data values are 2 people.
the 15th and 16th ordered data values are 2 people.

median = % = 2 people

7=
>f
ol
"~ 30
~ 2.03 people

2 | Number of phone calls (x) | Frequency (f) | Product (xf) | Cumulative frequency
0 5 0 5
1 8 8 13
2 13 26 26
3 8 24 34
4 6 24 40
5) 3 15 43
6 3 18 46
7 ) 14 48
8 1 8 49
9 0 0 49
10 0 0 49
11 1 11 50
Total P50 Saf =148
Chapter 12 (Statistics) Exercise 12C 481

= 2.96 phone calls

ii There are 50 data values, so n = 50. "TH = 25.5, so the median is the average of the
25th and 26th ordered data values.
From the cumulative frequency column, the 14th to 26th ordered data values are 2 phone
calls.
the 25th and 26th ordered data values are 2 phone calls.

median = 2—;2 = 2 phone calls

iii Looking down the frequency column, the highest frequency is 13. This corresponds to
2 phone calls, so the mode is 2 phone calls.

b Phone calls in a day


A frequency,
12

0
4 :l_':
o 1 2 3 4
o5 6 7 8 9 10

11
mode, median (2)T T mean (2.96) number of phone calls

The distribution is positively skewed, with one outlier (11 phone calls).
n

d The mean is larger than the median as the mean is affected by outliers and larger data values,
unlike the median.
e The mean would be the most suitable measure of centre for this data set as it best represents
all of the data.

Number of matches (x) | Frequency (f) | Product (xf) | Cumulative frequency


235 5
192 9
539 20
300 26
153 29
30

a i Looking down the frequency column, the highest frequency is 11. This corresponds to
49 matches, so the mode is 49 matches.
482 Chapter 12 (Statistics) Exercise 12C

n+1
il There are 30 data values, so n = 30. = 15.5, so the median is the average of the
15th and 16th ordered data values.
From the cumulative frequency column, the 10th to 20th ordered data values are 49 matches.
the 15th and 16th ordered data values are 49 matches.
. 49 + 49
-. median =

= 49 matches

il 7= 22t
> f
_umn
BED
~ 49.0 matches
b No, the results do not support the company’s claim that there are 50 matches in each box on
average as all of the measures of centre are around 49 matches.
¢ The sample size of 30 match boxes is not a large enough sample size. The company could
have won its case by arguing that a larger sample would have found an average of 50 matches
per box.

L | Number of children (x) | Frequency (f) | Product (xf) | Cumulative frequency


1 5 5 5
28 56 33
3 15 45 48
4 8 32 56
5 2 10 58
6 1 6 59
Total | Tr=59 | Tar=154]

a i z= 2=
>f
154
~ 9
~ 2.61 children
ii Looking down the frequency column, the highest frequency is 28. This corresponds to
2 children, so the mode is 2 children.

iii There are 59 data values, so n = 59. Tily= 30, so the median is the 30th ordered
data value.
From the cumulative frequency column, the 6th to 33rd ordered data values are 2 children.
the 30th data value is 2 children.
the median is 2 children.
b As the mean of this data set is ~ 2.61 children, this school has more children per family than
the average British family.
The data is positively skewed.
n

The values at the higher end of the data have increased the mean more than the mode and
Q

median.
Chapter 12 (Statistics) Exercise 12C 483

5 Column graph of pocket money


10 A frequency

R
N
S

b There are 29 children in total.

2of
>f
92
T 29
~ €3.17
n+1
il There are 29 data values, so n = 29. = 15, so the median is the 15th ordered
data value.
From the cumulative frequency column, the 14th and 15th ordered data values are €3.
the 15th data value is €3.
the median is €3.
iii Looking down the frequency column, the highest frequency is 9. This corresponds to €2,
so the mode is €2.
d The mode can be found easily using the graph only, as it is represented by the highest column
on the graph.

6 The 31 measurements in order are:


{15 values below 10}, 10.1, 10.4, 10.7, 10.9, {12 values above 11}

There are 31 data values, so n = 31. nTH =16, so the median is the 16th ordered data value.
the median is 10.1 cm.

7[R 56 000
o o7 560000
70000 420000
84000 252000
100000 100000
S f=20 | Saf=1332000
484 Chapter 12 (Statistics) Exercise 12C

1 T A
i There are 20 data values, so n = 20. s 10.5, so the median is the average of
the 10th and 11th ordered data values.
From the cumulative frequency column, the first 10 ordered data values are $56 000, and
the 11th to 16th ordered data values are $70 000.
the 10th data value is $56 000 and the 11th data value is $70000.
$56 000 + $70 000
-, the median =
2
= $63 000
ii Looking down the frequency column, the highest frequency is 10. This corresponds to a
salary of $56 000, so the mode is $56 000.

W oz= 2
>f
81332000
20
= $66 600
b The boss would use the mean value to argue against a pay rise, as it is the largest of the
measures, and takes all salaries into account.

0+2+3+5+x+4+1
6x + 79
z+ 15
5.45(x +15) = 62 + 79
5.45x + 81.75 = 6z + 79
. 0.55z =2.75
=5

b There were 5+ 15 = 20 students in total.


ber of students wh d
Percentage of students who passed = JUmPer
SCen’so7 WO PASEC x 100%
total number of students

= 5+4+1
100%
_55
= x 100%g
=75%
Chapter 12 (Statistics) Investigation 2 Mid-interval values 485

Marks | Frequency (f) | Lowest possible result (x) | Product (zf)


0
310
1460
2550
1120
>z
f = 5440

~ 24.8
The mean Physics examination mark must be at least 24.8.

7411
219
~33.8
The mean Physics examination mark must be at most 33.8 .

S af = 6425.5
486 Chapter 12 (Statistics) Exercise 12D

b The result in a is halfway between the lower and upper limits in 1 and 2.
¢ The mean Physics examination mark was approximately 29.3.
L The accuracy of the mid-interval estimate will depend on how the data values are distributed in
each class interval. For example, the estimate will be more accurate if the data is uniformly or
symmetrically distributed in each class interval.
Using a greater number of narrower class intervals will also improve the accuracy of the estimate.
This is because the mid-interval value will be more likely to be close to the actual data values.

Frequency f) Mzdpomt (z) | Product


(z )

the mean score was


about 31.7.

3 | Number of children Product(x h

a The playground was used by more than 40 children on 14 + 12 = 26 days.


b Looking down the frequency column, the highest frequency is 16. This corresponds to
31 - 40 children, so the modal class is 31 - 40 children.
Chapter 12 (Statistics) Exercise 12D 487

2000 < P < 3000 10000


3000 < P < 4000 14000
/

4000 < P 5000 40500


IN

5000 < P 6000 77000


NN

6000 < P 7000 149500


7000 < P 8000 120000
IN

a There were 70 service stations involved in the survey.


b The total amount of petrol sold was about 411000 L, or 411 kL.

>zl
> f
411000
T 10
=~ 5870
the mean amount of petrol sold for the day was about 5870 L.
d The modal class is 6000 < P < 7000 L. This is the most frequently occurring amount of
petrol sold at a service station in one day.

a | Runs scored idpoi Product (x f)


0-9 3 49.5
10 - 19 e 116

20 - 29 s 196
69

¢ exact mean number of runs scored


1745422413+ ....+25+9
29
432
29
the mean number of runs scored ~ 14.9
was about 14.8. The exact mean ~ 14.9 is very close to
the estimated mean in b, which means the
estimate was very accurate.
488 Chapter 12 (Statistics) Exercise 12D

6 Wamng time (t min) Frequency f) Mtdpomt (z) | Product (zf)


<t<1 =24 112
63
125
273
270
165
104
| S af = 1012
a Total number of customers = p + 42 + 50 + 78 + 60 + 30 + 16
300 = p+ 276
p=24

b T= %flfff ¢ 3016 100% ~ 15.3% of customers


1012 waited for at least 5 minutes.
~ 300
~ 3.37
the mean waiting time is about
3.37 minutes.

7 Aptitude test results


50, frequency

40

30

20

10

0
80 90 100 110 120 130 140 150 160
score

Frequency f) Mm’pomt (z) | Product(z )


0 < 5 <90
90 < s < 100
100< s < 110
110< s < 120
120< s < 130
130< s < 140
140< s < 150
150< s < 160

a 125 people took the test.


Chapter 12 (Statistics) Exercise 12E 489

the mean score was about 119.


5+ 10 15 3
t0_ 5 3 ofthe people scored less than 100 for the test.
125 125 25

d % = % = 28% of the people scored at least 130 for the test.

1 a The ordered data set is: 569 10 11 13 15 16 18 20 21 (11 data values)


n+1
i Since n =11, =6 .. the median is the 6th data value.

569101t 13 15—16—1820—2%
median = 13
ii Since the median is a data value we now ignore it and split the remaining data into two:
lower half upper half
—_—N——
569 10 11 15 16 18 20 21
Q; = median of lower half =9
Q3 = median of upper half = 18

il range = maximum — minimum iv IQR=Q3—0Q,


=21-5 =18-9
=16 =9
b The ordered data set is:
7 7 10 13 14 15 18 19 21 21 23 24 24 26 (14 data values)

i Since n = 14, nTH =17.5 .. the median is the average of the 7th and 8th data values.

1013141518 19 212123242426
Tth value + 8th value
. median =
2
18419
T2
=18.5

il We have an even number of data values, so we include all data values when we split the
data set into two:
lower half upper half
r—— e r— e,
7710 13 14 15 18 19 21 21 23 24 24 26

Q; = median of lower half = 13


Qs = median of upper half = 23
490 Chapter 12 (Statistics) Exercise 12E

il range = maximum — minimum iv IQR=Q3—-Q;


=26—7 =23-13
=19 =10
¢ The ordered data set is: 15 19 21 24 29 32 38 43 (8 data values)

i Since n=8, —— =4.5 .. the median is the average of the 4th and 5th data values.

51921
24 29 323843
4th value + 5th value
. median =
2
24429
e
= 26.5

il We have an even number of data values, so we include all data values when we split the
data set into two:
lower half upper half
e =
15 19 21 24 29 32 38 43

Q; = median of lower half = 42 _ 9

Q3 = median of upper half = 5211, 35 . 35

iii range = maximum — minimum iv IQR=Q3—Q;


=43-15 =35-20
=28 =15

d The ordered data set is: 20 26 28 32 33 41 45 52 57 69 (10 data values)


1 . 1
i Since n =10, nt = 5.5 . the median is the average of the 5th and 6th data values.

20262832
33 41 45525769
5th value + 6th value
. median =
2
33441
LD
=37

il We have an even number of data values, so we include all data values when we split the
data set into two:
lower half upper half
[rm— | ee
20 26 28 32 33 41 45 52 57 69

Q; = median of lower half = 28


Qs = median of upper half = 52

il range = maximum — minimum iv IQR=Q3—Q


=69 — 20 =52-28
=49 =24
Chapter 12 (Statistics) Exercise 12E 491

2 The ordered data sets are:


Natalie: 26 28 29 29 34 36 39 41 46 46 48 49 (12 data values)
Karen: 12 20 21 22 24 25 28 38 44 47 48 50 (12 data values)

range = maximum — minimum


=49 -26
= 23 goals
We have an even number of data values, so we include all data values when we split the
data set into two:
lower half upper half
r— e e,
26 28 29 29 34 36 39 41 46 46 48 49
29+29
1 = median of lower half = 29

Q3 = median of upper half = 6146 _ 46


IQR=Q3 —Q:
=46 —29
= 17 goals
range = maximum — minimum
=50—-12
= 38 goals
We have an even number of data values, so we include all data values when we split the
data set into two:
lower half upper half
—_———
12 20 21 22 24 25 28 38 44 47 48 50
21+ 22
Q; = median of lower half = =215

Q3 = median of upper half = MF47_ 455

IQR =Q3 —Qu


=45.5—-215
= 24 goals

b The range and IQR are much lower for Natalie than for Karen.
Natalie was the more consistent netball player.

3 The ordered data sets are:


Jane: $29 $29 $29 $31 $34 $35 $36 $36 $38 $40 $42 $47 (12 data values)
Ashley: $19 $19 $23 $24 $24 $26 $26 $32 $35 $40 $42 $59 (12 data values)
a Jane:
$29 + $29 + $29 + ... + $40 + RAT
$42 + O$47
mean — - T2&7 AT e PP T
12
_ %426
B L2
= $35.50
492 Chapter 12 (Statistics) Exercise 12E

Since n =12, ntl =6.5 .. the median is the average of the 6th and 7th data values.

$29-$29—$29—$31-5$34 $35 $36 $36—$38—$40—$42-$47


6th value + 7th value
. median =
2
_ $35+536
T2
= $35.50
Ashley:
$19 + $19 + $23 + ... + $40 + $42 + $59
mean =
12
_ $369
12
= $30.75

Since n=12, Z Rt 6.5 .. the median is the average of the 6th and 7th data values.

$15-519-523—$24—$24 $26 $26 -$32—$35—$46—542—$59-


6th value 4 7th value
. median =
2
$26 + $26
e

= $26
b Jane:
range = maximum — minimum
= $47 —$29
= $18
We have an even number of data values, so we include all data values when we split the data
set into two:
lower half upper half
e e e e
$29 $29 $29 $31 $34 $35 $36 $36 $38 $40 $42 $47

Q; = median of lower half = $29+831 $30

540 _ $39
Q3 = median of upper half = $3+ 8

IQR=0Q3 — Q1
= $39 — $30
=39
Chapter 12 (Statistics) Exercise 12E 493

Ashley:
range = maximum — minimum
= $59 — §19
= $40
We have an even number of data values, so we include all data values when we split the data
set into two:
lower half upper half
e, e ee e,
$19 $19 $23 $24 $24 $26 $26 $32 $35 $40 $42 $59

Q; = median of lower half = $23 4824 _ g9350

Q3 = median of upper half = 3354540 _ 3750

IQR=Q3 — Q1
= $37.50 — $23.50
=$14
¢ The mean and median are much higher for Jane than for Ashley.
Jane generally pays more for her telephone bills.
d The range and IQR are much higher for Ashley than for Jane.
Ashley has the greater variability in telephone bills.

& The ordered data set is:


779 10 11 11 12 13 14 14 15 15 18 18 19 20 20 22 25 67 (20 data values)
a range = maximum — minimum
=67-7
=60
We have an even number of data values, so we include all data values when we split the data
set into two:
lower half upper half
e e, e e
77910 11 11 12 13 14 14 15 15 18 18 19 20 20 22 25 67
11411
Qi = median of lower half = =11

Q3 = median of upper half = B+20 _195

IQR=Q3 - Q1
=19.5-11
=85
b The outlier is the value 67.

¢ If the data value 67 is removed, then range =25 —7


=18
n+1
Since n=20—-1=19, =10 .. the median is the 10th data value.

T91H0—H—H121314 14 +51518181920—26—2225
median = 14
494 Chapter 12 (Statistics) Exercise 12E

Since the median is a data value, we now ignore it and split the remaining data into two:
lower half upper half
—_——
11 12 13 14
7791011 15 15 18 18 19 20 20 22 25

Q; = median of lower half = 11


Q3 = median of upper half = 19

IQR
= Q3 —Q
=19-11
=38
d The range is much more affected by the outlier than the IQR.

5 The ordered data sets are:


Derrick: 210 380 400 415 420 420 425 425 430 435 440 445 445 450 450
(15 data values)
Gareth: 330 340 340 360 370 420 430 450 460 460 470 480 480 490 500
(15 data values)
a Derrick:
range = maximum — minimum
=450
— 210
= 240 minutes
n+1
Since n = 15, 8 .. the median is the 8th data value.

210—380—<400—415—420—420—425 425 430—435—440—445—445—450—450-


median = 425
Since the median is a data value, we now ignore it and split the remaining data into two:
lower half upper half
ee ———— e,
210 380 400 415 420 420 425 430 435 440 445 445 450 450

Q; = median of lower half = 415


;5 = median of upper half = 445

IQR=0Q3 - Q1
=445 — 415
= 30 minutes

Gareth:
range = maximum — minimum
=500 — 330
= 170 minutes
n+1
Since n = 15, 8 .. the median is the 8th data value.

-330—346—340—360—376—420—430- 450 460—460—470—480—480—490—566-


median = 450
Chapter 12 (Statistics) Exercise 12E 495

Since the median is a data value, we now ignore it and split the remaining data into two:
lower half upper half
e e e e,
330 340 340 360 370 420 430 460 460 470 480 480 490 500
Q; = median of lower half = 360
;5 = median of upper half = 480

IQR=Q3 —Q
= 480 — 360
= 120 minutes

b i Gareth’s data has the lower range.


ii Derrick’s data has the lower interquartile range.
¢ The interquartile range is more appropriate than the range for determining who is generally the
more consistent sleeper as it is less affected by outliers.

6 The ordered data set is: a b cde f g hij k1l m (13 data values)
n+1
a Since n =13, =7 .. the median is the 7th data value.

——b—e—d——f
gy g
median = g

b i range = maximum — minimum


=m-—a
ii Since the median is a data value we now ignore it and split the remaining data into two:
lower half upper half
—_——~
abede f hijklm

Q; = median of lower half = C; d

Q3 = median of upper half = #

IQR=0Q3 —Q;

=G
;[ Original value 7 13 6
New value (max + 2) — (min + 2)
= max — min
=13
(2 x max) — (2 x min)
= 2(max — min)
=2x13
=26
496 Chapter 12 (Statistics) Exercise 12F

< Lol Lo b s e b b b b ) > poimsscored

a | median = 35 points il maximum value = 78 points


iii minimum value = 13 points iv upper quartile = 53 points
v lower quartile = 26 points
b i range = maximum — minimum il IQR=Q3—Q,
=78—-13 =53 —26
= 65 points = 27 points

) - Lo b b b b b b b b b b b
I N b b b b b b b bl >testscores

0 10 20 30 40 50 60 70 80 90 100

a i The highest mark scored for the test was 98, and the lowest mark was 25.
ii Half of the class scored a mark greater than or equal to 70 marks.
ili The top 25% of the class scored at least 85 marks.
iv. The middle half of the class had scores between 55 and 85 marks.
b range = maximum — minimum ¢ IQR=Q3—-Q
=98 —-25 =85—-155
= 73 marks = 30 marks

3 a 1 The ordered data set is:


34555 6 6 6 77889 10 (14 data values)
| | |
Q=5 median=6 Q3=38

minimum = 3 Q=5
So the five-number summary is: median = 6 Q3 =38
maximum = 10
ii . .

234567
8 9101112

ili range = maximum — minimum iv IQR=Q3—Q


=10-3 =8-5
=7 =3

b i The ordered data set is:


0123456
6 7 7788888 8 9 9 (19data values)
| } |
Q=4 median = 7 Q3 =38

minimum = 0 Q=14
So the five-number summary is: median = 7 Q3 =38
maximum = 9
Chapter 12 (Statistics) Exercise 12F 497

-1
T L |

0123456738910

il range = maximum — minimum iv IQR=Q3—Q


=9-0 =8—-4
=9 =4

< i The ordered data set is:


17 20 23 26 26 28 30 31 31 31 33 35 44 47 47 49 49 51 (18 data values)
} | |
Q1 =26 median = 31 Qs =47
minimum = 17 Q=26
So the five-number summary is: median = 31 Qs =47
maximum = 51


T
15 20 25 30 35 40 45 50 55

ili range = maximum — minimum iv IQR=Q3—Q


=51—17 =47 - 26
=34 =21

L a The ordered data set is:


233444455555556666667777888999101213
| | | (33 data values)
Q=5 median = 6 Q3 =38
So, median = 6 beans, Q; =5 beans, Q3 = 8 beans.

b IQR=Q3 - Q1
=8-5
= 3 beans
<
-— L

2 4 6 8 10 12 14
number of beans in a pod

5 a of bolts | 33 | 34 | 35 | 36 | 37 | 38 | 39 | 40
Number
Frequency 13

The ordered data set is:


minimum = 33 Q=35 median = 36
f f f
33 34 34 34 34 34 35 35 35 35 35 35 35 36 36 36 36 36 36 36 36 36 36 36
36 36 37 37 37 37 37 37 37 37 37 37 37 37 38 38 38 38 38 38 38 38 40
| |
Q3 =37 maximum = 40
(47 data values)
498 Chapter 12 (Statistics) Exercise 12G

minimum = 33 Q=35
So the five-number summary is: median = 36 Qs =37
maximum = 40
b i range = maximum — minimum ii IQR=Q3—Q
=40-33 =37-35
= 7 bolts = 2 bolts

c D]
- S e

number of bolts

T a IQR=Qs-Q
=43.5-315
=12

b lower boundary = lower quartile — 1.5 x IQR


=315-15x12
=135
upper boundary = upper quartile + 1.5 x IQR
=435+15x12
=615
¢ 13 is below the lower boundary, so it is an outlier.
20, 52, and 55 are all within the two boundary values, so none of these data values are outliers.

4 v T
10 15 20 25 30 35 40 45 50 55 60

2 a The ordered data set is:


356778899910
10 10 11 11 12 12 13 13 13 14 14 16 18 22
| | |
Q=8 median = 10 Q3 =13 {n =25}
So the median is 10, the lower quartile is 8, and the upper quartile is 13.
b IQR=Q3—Q
=13-38
=5
< lower boundary upper boundary
= lower quartile — 1.5 x IQR = upper quartile + 1.5 x IQR
=8-15x5 =13+15x%x5
=05 =20.5
22 is above the upper boundary, so it is an outlier.
Chapter 12 (Statistics) Exercise 12G 499

number of birds

The data displayed in this graph has a


8 4 frequenc
q y
minimum of 2 and a maximum of 8.
6 There are no outliers.
4 These characteristics match box plot A.
2
0
1 2 3 4 5 6 7 8 @ ) —T

o The data displayed in this graph has a


o
o minimum of 1 and a maximum of 13.
o
o o o
13 is clearly an outlier, and there are no outliers
e o o at the lower end of the data set.
e o o o
© ©o o 0 0 0 0 0 O o These characteristics match box plot D.
2 4 6 8 10 12 =« D
o—‘ |— * *

12345678 910111213

frequency The data displayed in this graph has a


W W minimum of 1 and a maximum of 13.
1 and 13 are clear outliers.
O

These characteristics match box plot €.


C
. O -
N
O

12345678910111213 ¢
12345678 910111213

frequency The data displayed in this graph has a


minimum of 1 and a maximum of 8.
There are no outliers.
NW

These characteristics match box plot B.


O

Properties sold by a real estate agent


A frequency
24
20
16
12

2
= 3 4 5 6 7
> properties sold
500 Chapter 12 (Statistics) Exercise 12H

b From the column graph, 7 properties sold appears to be an outlier.


¢ Since n = 52, we have an even number of data values, so we include all data values when
we split the data set into two groups of 26 data values.

For the lower half of the data set, n = 26, so HTH =13.5

Q) is the average of the 13th and 14th data values.


Q; = median of lower half
_ 13th value 4 14th value
e

=— {from a, the 4th to 16th values are 1}


=1
For the upper half of the data set, we need to find the average of the 26 + 13 = 39th and
26 + 14 = 40th data values.
Q3 = median of upper half
_ 39th value + 40th value
. 2
. 3—? {from a, the 38th to 45th values are 3}
=3
IQR =Q3 —Q
=3-1
=2
lower boundary upper boundary
= lower quartile — 1.5 x IQR = upper quartile + 1.5 X IQR
=1-15x2 =3+15x2
=-2 =6
7 properties sold is above the upper boundary, so it is an outlier.

L L L L L L L >

Year 9
Year 12
L+ | » time (min
120 )

Year 12
minimum 6 36
Q1 30 60
median 45 84
Qs 60 96
maximum 72
Chapter 12 (Statistics) Exercise 12H 501

b i Year9: range=72-6 Year 12: range = 105 — 36


= 66 min = 69 min
il Year 9: IQR=Q3—Q; Year 12: IQR = Q3 — Qg
=60 —30 =96 — 60
= 30 min = 36 min

¢ i We cannot tell if Year 12 students spend about twice as much time on homework as Year 9
students since the mean was not calculated.
il It is true that over 25% of Year 9 students spend less time on homework than all Year 12
students since the lower quartile for the Year 9 students is less than the minimum value
for the Year 12 students.

Saturday

T iy
0 50 100 150 200 amount (€)
a Friday: min = €20, Q; = €50, median = €70, Q3 = €100, max = €180
Saturday: min = €40, Q; = €80, median = €100, Q3 = €140, max = €200

b i Friday: range = €180 — €20 Saturday: range = €200 — €40


= €160 = €160
ii Friday: IQR=Q3—Q Saturday: IQR = Q3 —Q;
= €100 — €50 = €140 — €80
= €50 = €60

a i The highest mark was in class 1 (96%). ™ percentage


il The lowest mark was in class 1 (37%).
iii Class 1 had the larger spread of marks. 9
b IQR ofclass 1 = Q3 — Q
=T70% — 52% 80
=18%
70
¢ range of class 2 = maximum — minimum
=93% — 38% 60
=55%
d i 70% is the upper quartile of class 1. o
25% of the students in class 1 received an achievement N
award.
il 70% is the median for class 2. 30 class1 class 2
50% of the students in class 2 received an achievement
award.
e 1 The marks in class 1 were slightly positively skewed.
il The marks in class 2 were negatively skewed.
f The students in class 2 generally scored higher marks.
The marks in class 1 were more varied.
502 Chapter 12 (Statistics) Exercise 12H

L a Kirsten:
Radlforn Radlforn]
1-Variable 1-Variable
n. =25 n. =25

The five-number summaries are:


Kirsten: minimum = 0.8 min Erika: minimum = 0.2 min
Q; = 1.3 min Q1 = 2.2 min
median = 2.3 min median = 3.7 min
Q3 = 3.3 min Q3 = 5.7 min
maximum = 6.9 min maximum = 11.5 min

b Phone call duration

s
'—-—< Kirsten

L L | | ! | L | | L | L 1 l 5 . .
0 2 4 6 8§ 10 12 14 1 tme(min)
¢ Both sets of data are positively skewed. Erika’s phone calls were more varied in their duration,
but tended to be longer than Kirsten’s.

5 a The number of goals scored is an exact number value.


this is a discrete numerial variable.

12‘ frequency Emil

8
4
0

goals

d Emil’s distribution is approximately symmetrical. Aaron’s distribution is positively skewed.


Chapter 12 (Statistics) Exercise 12H 503

Aaron:
a Redforn] a (dZc)Real
1-Variabl 1-Variable
.88888888 X =2.66666666
2
04 Ix 9 6
66 zx2 3 90

O 0o
3 34943975 ox 1 .92 930615

I
. 3685817 SX 1 .95667356
n 3

(=2}

@
a (&) Real
1-Variable .

BDD =
o
OQO
Emil: mean ~ 2.89 goals, median = 3 goals, mode = 3 goals
Aaron: mean =~ 2.67 goals, median = 2.5 goals, mode = 2 and 3 goals
Emil’s mean and median are slightly higher than Aaron’s, and Emil has a clear mode of 3 goals,
whereas Aaron has two modes of 2 and 3 goals.
f Emil: Aaron:
a Radforn] (d7c)Real a (GZ)Real
1-Variable 1-Variable
=36 n =36
=0 i =
=2 =
=3 =
=4 =
=6 =

range =6 — 0 IQR=Q3 —Q range = 8 — 0 IQR=Q3 — Q


=6 goals =4-2 = 8 goals =4-1
= 2 goals = 3 goals
The range and IQR are lower for Emil than for Aaron. So Emil’s data set demonstrates less
variability than Aaron’s.

Aaron

0 1 2 3 4 5 6 7 8 9
goals

h Emil generally scores more goals than Aaron and is a more consistent goal scorer than Aaron.

6 a The lifespan of the globes is a numerical variable which is measured.


this is a continuous variable.
Chapter 12 (Statistics) Exercise 121

B Fellom] @Cfa R
1-Variable 1-Variable
o =107 X =134
=4280 =5360
=465448 =728170
=13.6821051 =15.7559512
=13.8564064 =15.9566721
=40 =40

E)fe) 8 E)fe
1-Varial
n =40 =40
minX =75 =107
=97.5 =124.5
=110.5 =132
=116.5 =143
=131 =191

Old type: mean = 107 hours New type: mean = 134 hours
median = 110.5 hours median = 132 hours
range = 56 hours range = 84 hours
IQR = 19 hours IQR = 18.5 hours
The “new” type of light globe has a higher mean and median than the “old” type.
The IQR is relatively unchanged going from “old” to “new”, however, the range of the “new”
type is greater, suggesting greater variability.
[4

60 80 00 120 140 160 180 200


lifespan (hours)

d The “old” type data is negatively skewed. The “new” type data is positively skewed.
The “new” type of light globes do last longer than the “old” type. From ¢, both the mean
and median for the “new” type are close to 20% greater than that of the “old” type. The
manufacturer’s claim appears to be valid.

<z <20
/

<
< r <40
< r < 50
< r < 60
< <70
< z < 80
< r <90
< r < 100
Chapter 12 (Statistics) Exercise 121 505

cumulative frequency

) 150 | 61l ies i7s . examination scores


0 10 20 30 40 50 60 70 80 90 100
b The median is the 50th percentile. As 50% of 150 is 75, we start with the cumulative
frequency 75 and find the corresponding examination score.
The median &~ 61 marks.
¢ Approximately 92 students scored 65 marks or less.
d From the table, 36+ 40 = 76 students scored at least 50 but less than 70 marks.
e Approximately 23 students scored 45 marks or less.
approximately 23 students failed the examination.
f As 16% of 150 is 24, we start with the cumulative frequency 150 — 24 = 126 and find the
corresponding examination score.
The top 16% of students scored approximately 75 marks or more.
the credit mark was approximately 75 marks.

Heights of seedlings
4 cumulative frequency
60
55 ES4 L
50
45
40 43

35
30 o
25
20
15
10 |eemimmmrmmmrmisy
5
559 710 8i838 |10 height (cm
0
2 3 4 5 6 7 8 9 1011 12 13
a Approximately 9 seedlings have heights of 5 cm or less.
b Approximately 60 — 43 = 17 seedlings have heights of more than 8 cm.
é—g x 100% = 28.3% of seedlings are taller than 8 cm.
506 Chapter 12 (Statistics) Exercise 121

¢ The median is the 50th percentile. As 50% of 60 is 30, we start with the cumulative frequency 30
and find the corresponding height.
The median ~ 7.1 cm.
d Qq is the 25th percentile. As 25% of 60 is 15, we start with the cumulative frequency 15 and
find the corresponding height.
Qi ~5.9cm
Q3 is the 75th percentile. As 75% of 60 is 45, we start with the cumulative frequency 45 and
find the corresponding height.
Q3 ~ 83 cm

IQR=Q3 - Qy
~83—-59
~ 2.4 cm

e As 90% of 60 is 54, we start with the cumulative frequency 54 and find the corresponding
height.
The 90th percentile &~ 10 cm which means that 90% of the seedlings are shorter than
approximately 10 cm.

Age (x years) | Number of accidents | Cumulative frequency


59
141
184
205
224
235
259
300

350& cumulative frequency


300

250

200

150

100 11

50
3
0
20 26 30 40 50 60 70 80
16 age (years)

b The median is the 50th percentile. As 50% of 300 is 150, we start with the cumulative
frequency 150 and find the corresponding age.
The median ~ 26 years.
¢ Approximately 113 drivers involved in accidents had an age of 23 or less.
é%g x 100% = 37.7% of drivers involved in accidents had an age of 23 or less.
Chapter 12 (Statistics) Exercise 121 507

d i Approximately 162 drivers involved in accidents were aged 27 years or less.


P(driver involved in an accident is aged 27 years or less) ~ 12
~ 0.54
ii Approximately 150 drivers involved in accidents were aged 26 years or less, from b.
162 — 150 = 12 drivers involved in accidents were aged 27 years.
P(driver involved in an accident is aged 27 years) ~ %
~ 0.04

a| Length (cm) Cumulative frequency


24 < x <27 1 1
27T< 2z <30 2 3
30<z<33 5 8
33< <36 10 18
36< <39 9 27
39< <42 2 29
42<x <45 1 30

b 354 cumulative frequency


30

25

20

15

10

0 >
24 27 30 33 36 39 42 45 48
trout length (cm)

¢ The median is the 50th percentile. As 50% of 30 is 15, we start with the cumulative frequency
15 and find the corresponding length.
The median ~ 35 cm.
n+1
d There are 30 data values, so n = 30. = 15.5, so the median is the average of the 15th
and 16th ordered data values.
The ordered data set is:
2427283031+ 3132323333 33333434 34
35 3535363636363 7 3538383 5—40—40—44
15th value + 16th value
median =
2
34435
D)
= 34.5 cm
The median found from the graph is a good approximation for the actual median.
508 Chapter 12 (Statistics) Exercise 121

Cross-country race times

,\I 20 25 282 30 35

a The lower quartile is the 25th percentile. As 25% of 80 is 20, we start with the cumulative
frequency 20 and find the corresponding time.
Q; ~ 27 min

b The median is the 50th percentile. As 50% of 80 is 40, we start with the cumulative frequency 40
and find the corresponding time.
The median ~ 29 min.
¢ The upper quartile is the 75th percentile. As 75% of 80 is 60, we start with the cumulative
frequency 60 and find the corresponding time.
Q3 ~ 31.3 min

d IQR=Q3 —Q
~31.3-27
~ 4.3 min

e As 40% of 80 is 32, we start with the cumulative frequency 32 and find the corresponding
time.
The 40th percentile ~ 28.2 min.
f From the cumulative frequency curve we can obtain the following cumulative frequency table:

Time (tmin) | 21 <t <24 | 24 <t <27 [27<¢t<30|30<t<33|33<t<36


Cumulative 5 2 50 70
frequency

So, the table is:

Number af
15 [ 50 —20 =30 | 70 20| 80 —"70=10
competitors
Chapter 12 (Statistics) Exercise 121 509

Life (I hours) Number of globes | Cumulative frequency


0< 1< 500 5 5
500 < 1 < 1000 17 22
1000 < 1 < 2000 46 68
2000 < I < 3000 79
3000 < 1 < 4000 27
4000 < 1 < 5000 4

a 2004 cumulative frequenc;

1500§ 2280; 227 0 .


0 1000 2000 2500 3000 4000 5000
globe life (hours)

b The median is the 50th percentile. As 50% of 178 is 89, we start with the cumulative
frequency 89 and find the corresponding globe life.
The median ~ 2280 hours.
¢ Approximately 126 globes had a life of 2700 hours or less.
128 % 100% ~ 70.8% of globes had a life of 2700 hours or less.
d Approximately 42 globes had a life of 1500 hours or less.
Approximately 109 globes had a life of 2500 hours or less.
approximately 109 — 42 = 67 globes had a life between 1500 and 2500 hours.

a Lengths are rounded to 20 cm if they are in the range 19.5 <1 < 20.5 cm.
510 Chapter 12 (Statistics) Exercise 12J

b Cumulative frequency
19.5 <1< 205 1
205 <1< 215 2
21.5<1<225 2
225 <1< 23.5 5
23.5<1<245 10
24.5 <1< 25.5 23
25.5 <1< 26.5 40
26.5 <1<275 47
27.5<1<285 49
28.5 <1<295 49
29.5 <1< 305 50

¢ 604 cumulative frequency


50

40

0\ >
20 22 24 26 28 30 32
foot length (cm)

d i The median is the 50th percentile. As 50% of 50 is 25, we start with the cumulative
frequency 25 and find the corresponding foot length.
The median foot length ~ 25.6 cm.
il Approximately 32 people had a foot length of 26 cm or less.
approximately 50 — 32 = 18 people had a foot length of 26 cm or more.

10+74+5+8+10
1 a The mean of data set A = =8

44+12+11+144+1+6
The mean of data set B = =38
6
So, each data set has mean 8, as required.

b Data set B appears to have a greater spread than data set A, as data set B has more values
which are a long way from the mean, such as 1 and 14.
Chapter 12 (Statistics) Exercise 12] 511

¢ Data set A4:


The population variance o =Z(z

n

_1s
E1i5
=3.6

The population standard deviation o = v/3.6


~ 1.90

Data set B:
_ )2
The population variance o2 Z(z
= 2
n

_16
~21.7

The population standard deviation o ~ v/21.7


~ 4.65
Total 130

a Using technology: b Using technology:


et Redforn]) (d7c)Real

2.539940828

The population standard deviation The population variance o2 & 2.54.


o =~ 1.59 pets.

22 425423428429+ 21420+ 26
a Themean pp=—"—""—-—~
— ~ ~ °
8
= 24.25 years 5 0625

The population standard deviation o = 4/ Z(IT_M)? '. ?ggzg


: 14.0625
=, /B : 22.5625
s 25 | 10.5625
~ 3.07 years R 18.0625
3.0625
512 Chapter 12 (Statistics) Exercise 12J

b 4 years later, each team member will be 4 years older, so the ages of the members will be:
26, 29, 27, 32, 33, 25, 24, 30.
26 + 29 + 27 + 32 4 33 + 25 + 24 + 30
The new mean p =
8
= 28.25 years 5 0625
. - - /Z(‘” — )2 ; 0.5625
The new population standard deviation o = e ) 1.5625

e . 14.0625
Sl e g 22.5625
10.5625
~ 3.07 years
18.0625
3.0625

¢ If each data value is increased or decreased by the same amount, then the mean will also
be increased or decreased by that amount, however the population standard deviation will be
unchanged.

& Using technology:

The population standard deviation o =~ 2.64 glasses.

5 Using technology:

The mean =~ 30.0 years and the population standard deviation o =~ 14.3 years.

6 a Danny:
mean number of hours spent on homework
_35+35+4+25+3+35+3+15+3+4+25+4+4+3
- 14
~ 3.21 hours

Jennifer:
mean number of hours spent on homework
_25+41425+2+42425+15+2+2+25+2+2+2+15
14
= 2 hours
Chapter 12 (Statistics) Exercise 12J 513

b Danny’s mean is higher than Jennifer’s, so Danny generally studies for longer.
¢ Using technology:
Danny: Jennifer:
a (97c)Real (37l
1-Variable 1-Variable
e =3 X =
=4
=1
=0.
=0.
=1

The population standard deviation The population standard deviation


o ~ 0.700 hours. o =~ 0.423 hours.
d Jennifer’s standard deviation is lower than Danny’s, so there is less deviation from the mean
for her data set. Jennifer therefore studies more consistently than Danny.

32.2426.4435.6 +.... +38.9 +29.0 + 31.3


a Boys’ mean time =
10
=32.02s
L, . 36.2433.5+28.1+....
+36.0 + 39.7 4 29.8
Girls” mean time = 0
=34.7Ts
The ordered data set for the boys is:
26:4—273—285—29:6- 30.8 31.3 322356389402 {n=10}
n+1
Since n = 10, =5.5 .. the median is the average of the 5th and 6th data values.
5th value + 6th value
median of boys’ data =
2
_==
30.8+31.3

=31.05s

The range of the boys’ data = maximum value — minimum value


=40.2 - 26.4
=138s

The ordered data set for the girls is:


28+—298—3+6—335 35.7 36.0 362373397398 {n=10}
n+1 -
Since n = 10, =5.5 .. the median is the average of the 5th and 6th data values.

median of girls’ data = valucss


DUl Gih value
2
_ 35.7+36.0
- 2
=35.85s
The range of the girls” data = maximum value — minimum value
=39.8—-28.1
=11.7s
514 Chapter 12 (Statistics) Exercise 12J

Using technology:
Boys: Girls:
e Badfforn]) (dFe)Fes) R
1-Variable
X =32.02
Zx 3 20.2
zx2 1 0457.28
ox 4 .52190225
SX 4 .76650349
n 10

The population standard deviation The population standard deviation


o~ 4.52s. o~ 3.76 s.

So, the table is:

s | ois
31055 | 35.85s
Standard deviation o

b i The mean and median are lower for the boys, so the boys generally swim faster.
ii The standard deviation and range are higher for the boys, so the boys have the greater
spread of swimming speeds.
¢ Tyson could improve the reliability of his findings by increasing his sample size.

8 | Rockers| 0 | 10 1191108 |5]|6]|7


[Bullers | 4 [3 [a [1]a]u]7]6]12]5]
0+10+14...+5+6+7
a Rockets’ mean number of runs =
10
= 5.7 runs

4+3+44+....4+6+1245
Bullets’ mean number of runs =
10
= 5.7 runs

Range of Rockets” data = maximum — minimum


=11-0
=11 runs

Range of Bullets’” data = maximum — minimum


=12-1
= 11 runs
So, the two teams have the same mean (5.7 runs) and range (11 runs) of runs scored.
Chapter 12 (Statistics) Exercise 12J 515

b We suspect the Rockets’ performance is more variable over the period since they twice scored
ZEro runs.
Using technology:
Rockets: Bullets:
a [d7c)Real
1-Variable l—Variab%e
X 7
P4 7
77
cX . 9
SX B 1 1096095
n

The population standard deviation The population standard deviation


o = 3.9 runs. o =~ 3.29 runs.
The standard deviation is higher for the Rockets which confirms our suspicion that the Rockets’
performance is more variable.
¢ The standard deviation gives a better indication of variability as it takes all data values into
account, not just the lowest and highest values.

a i Museum:
_ 1108 + 1019 + 850 + 1243 + .... 4 1084 + 981
Mean number of visitors
31
28963
31
= 934 visitors
Art gallery:
°r 1258 + 1107 + -1179774 1302 T+ ....
e + AT
T 1259 + 0
1366
Mean number of visitors = ——~ — - T
31
38197
T o
~ 1230 visitors
ii Using technology:
Museum: Art gallery:

4.7286E+07
84.6339734
g? 0329769

The population standard deviation The population standard deviation


o ~ 208 visitors. o =~ 84.6 visitors.
b The standard deviation was higher for the museum data, so the museum had the greater spread
of visitor numbers.
i “0”is an outlier in the Museum data.
i This outlier corresponds to Christmas Day, so the museum was probably closed which
meant there were no visitors on that day.
Yes, it is reasonable to remove the outlier when comparing the numbers of visitors to these
places. Even though the outlier is not an error, it is not a true reflection of the visitor
count for a particular day.
516 Chapter 12 (Statistics) Exercise 12J

28963
iv. New mean number of visitors to the museum =
30
~ 965 visitors
Using technology:
RedFornD) (dFc]Resl
1-Variable
X =965.433333
XX 8 963
zx2? W»—-»—ANM
.8398E+07
CX 2 0.589574
SX 2 2.651084
n 0

The new population standard deviation o ~ 121 visitors.


v The outlier had greatly increased the population standard deviation.

Using technology:
a Redfornd) (d7c)Res)
1 Variable
=5
Zx
_-0
\1\"\70
MOOO‘I»—A

zx2?
oX 7459666
SX 9471941
n
o

The population standard deviation o ~ 0.775.

n[ Frequency
el s ol
Using technology:

The mean age of squash players p = 14.48 years, and the population standard deviation
o ~ 1.75 years.
Chapter 12 (Statistics) Exercise 12J 517

12 Data Set A Data Set B


e g eT T e e
o o
8 8

6 6

4 4

2 2

0456789101112 0456789101112

a By looking at the graphs, data set A appears to have a wider spread.


b Using technology:
Data set A: Data set B:
Redfiorn]) (dc)Real
—Variable
8
2 0
oo

1 i
o

2
2 .04124145
2
o

The mean of data set A is 8. The mean of data set B is 8.


¢ Using technology:
Data set A: Data set B:
Redforn]) (d7c)Real
=1-Variable
=
= = = N) 0O

XX
RO

Zx2
X
SX
-

n
DN

The population standard deviation is The population standard deviation is


o=2. o~1.06.
The population standard deviation is higher for data set A than for data set B which confirms
that data set A has a wider spread.

a A 8 3
B 4 2

The range only takes into account the maximum and minimum values.
The IQR only takes into account the upper and lower quartiles.
The standard deviation however is calculated using all of the data values, so it gives a better
description of how the data is distributed than the range or IQR.
518 Chapter 12 (Statistics) Exercise 12J

12 0 1

WO
KR
N
O
=
O

a The female students” marks are in the range 16 to 20 whereas the male students’ marks are in
the range 12 to 19.
i The females appear to have scored better in the test.
il The males appear to have a greater spread of scores.
b Using technology:
Females:
Redforn] (d7c)Real
a7.g333333
=B N

The females’ mean score p ~ 17.5 marks, and the population standard deviation
o ~ 1.02 marks.

Males:
a8 Redfornd) (d7c)Real
1-Variable
X 1 5.5384615
2
3
1
?
1
1

The males’ mean score o~ 15.5 marks, and the population standard deviation
o ~ 1.65 marks.

14 Jess’ question is worded so that the respondent will not include themselves.
the results for the mean will differ by 1, but the results for the standard deviation will be the
same.
Chapter 12 (Statistics) Exercise 12J 519

Class interval | Mid-interval value | Frequency

B Radforn]) (d7c)Rea)
1-Variable 1-Variable
X =48.2666666 X =48.2666666
1 448 1 448
7 0102 7/ 0102
2 65748419
1 2 .65748419
2 70291456
5 2 .70291456
30 3 0

a Using technology: b Using technology:


Redforn]) ([dTc)Real
1-Varia

The mean ~ 17.45 vehicles. The standard deviation


~ 7.87 vehicles.
520 Chapter 12 (Statistics) Exercise 12J

720 < W < 740


/
W < 760

a Using technology: b Using technology:


a8 Radformd) _(d7c)Real B RadFormd) _(d7e)Real
1-Variable 1-Variable
X NG
feiles)
Il
N 6O 6O 1= 1=~
NOHO

Ix 120
Zx2 206E+08
oxX . 7433457
sX .8230029
o

n
S

The mean ~ $780.60. The standard deviation ~ $31.74.

18 a Using technology: 3 ot
1-Variable
X =40.85
P 4 =1614
Xx2 =65840
oX =4.22817927
SX =4,2820436
n =40

The mean T = 40.35 hours and the standard deviation o =~ 4.23 hours.

2
5

Using technology: 5 e
1-Variable
Xy =40.6
P4 =1624
Xx2 =66606
oX =4.09756024
SX =4.14976057
n =40

The mean 7 = 40.6 hours and the standard deviation ¢ ~ 4.10 hours.
The mean is slightly higher for the class interval data set than for the raw data. The standard
deviation is slightly lower for the class interval data set than for the raw data. The values for the
mean and standard deviation for the class interval data set are therefore good approximations
for the mean and standard deviation of the raw data.
Chapter 12 (Statistics) Investigation 3 Transforming data 521

1 For the data


set: 4 23 35 297335
2 15366 3367

The mean = 2+
2+3+
.. +6+7
20
_8520
=4.25
Using technology, the standard deviation ~ 2.05. R
—Variable
=4.25
=85
=445
oX =2.04633819
=2.09949868
n =20

2 a The new data set is: 9 7 8 8 10 7 14 12 8 10


7T 6 10 8 11 11 8 8 11 12
The mean — 9+74+84+....+11+12
20
_ s20
=9.25
=4.25+5
Using technology, the standard deviation ~ 2.05.
—Varia

b If k is added to each data value, then k will be added to the original mean but the standard
deviation will not change.
c i The new data set is: 15 13 14 14 16 13 20 18 14 16
13 12 16 14 17 17 14 14 17 18
15+134+144....
417+ 18
The mean =
20
30520
=15.25
=4.25+11
Using technology, the standard deviation ~ 2.05.
522 Chapter 12 (Statistics) Investigation 3 Transforming data

ii The new data set is: 1 -1 00 2 -1 6 4 0 2


-1 -2 2 0 3 3 0 0 3 4
14 (1) +0+....+3+4
The mean =
20
_»20
=1.25
=4.25-3
Using technology, the standard deviation ~ 2.05.

3 a The new data set is: 16 8 12 12 20 8 36 28 12 20


8 4 20 12 24 24 12 12 24 28
16+8+12+....+24
428
The mean =
20
_T 3020
=17
=425x4
Using technology, the standard deviation ~ 8.19
1-Var i e
~2.05 x4
D000~=
e

0
W—NO

8535277
9799474
o

b If each data value is multiplied by a, we expect the mean and standard deviation will be
multiplied by a.
¢ i The new data set is: 36 18 27 27 45 18 81 63 27 45
18 9 45 27 54 54 27 27 54 63
36+ 18427+ ....4+54+63
The mean =
20

T
o20
=38.25
=425x9
Using technology, the standard deviation ~ 18.4
~2.05 x9
Chapter 12 (Statistics) Investigation 4 523

il The new data set is: 1 05 075 075 1.25 05 225 1.75 0.75 1.25
05 025 125 075 1.5 1.5 075 0.75 1.5 175
1+054+0.75+....+1.54+1.75
The mean =
20
2125
20
= 1.0625
_=425x% 71

Using technology, the standard deviation ~ 0.512


1 1-Variabl
~2.05 % 7 X =1.0
DR =21.
Zx2 =27.
oX =0.5
sX =0.5
n =2

4 a mean = ap, standard deviation = ac b mean = p+ k, standard deviation = o


¢ mean = ap + k, standard deviation = ao

1 a| Time (t minutes) | Frequency (f frequency

W
(=]
ot
V/

= 16 40
0< 51 30
5 <t <20 53
0<
5<
1l
w0
~

S
A

0
0 5 10 15 20 25 30
o
S
~
WV

t (minutes)
The distribution of the data is approximately symmetrical.
b From the spreadsheet, the true population standard deviation ~ 4.521.
" true population variance = (true population standard deviation)?
~ (4.521)°
~ 20.439

2 a Sample I:
Using technology,
the sample standard deviation s =~ 4.351
s~ (4.351)2
~ 18.933
524 Chapter 12 (Statistics) Investigation 4

Repeating this process with the remaining samples, we get:

¢ To help us judge which estimates are closer to the true values, we calculate the absolute
difference of each estimate from the true population values:

So, on average s is closer to the true standard deviation and s? is closer to the true variance.
d Yes, the formulae for the sample statistics s and s> generally produce estimates which are closer
to the true standard deviation and variance respectively.

3 From the spreadsheet:

s ~ 4.981 o=~ 4.833


5% ~ 24.806 02 ~23.358

Based on these results, the sample estimates are generally closer to the true values o = 5 and
0% =25. This agrees with our answer to 2 ¢.

4 Note: The following answers are examples only.


Changing the true mean p = 40, we obtain:

s ~ 1907
5% a2 24.970 02 ~ 23.532

The sample estimates are still generally closer to the true values.
Changing the true standard deviation o = 10, we obtain:

5%~ 100.127
Again, the sample estimates are generally closer to the true values.
Considering the above results, changing y or o does not affect the conclusion.
Chapter 12 (Statistics) Review set 12A 525

5 Having accurate estimates of the variance and standard deviation of a population is important when
we use these statistics in our inference.
For example, suppose we wanted to simulate the population using the standard deviation or variance
as one of the parameters. We would want our estimates to be as close to the actual values as possible,
so that our simulation matches what we observe in reality.

REVIEWSET 124
0+2+34+34+4+5+54+6+6+74+7+438
1 a | mean=
12
_ 5612
=~ 4.67
il As n=12, —n; L 6.5, so the median is the average of the 6th and 7th ordered data
values.
The ordered data set is: 62334 5 5 66778
6th value + T7th value +5
median = =—=5
2 2

. 29+31+4+37+384+39+39+404+45+474+54
b i mean=
10
399
ET)
=3.99
o 1 ) )
il As n=10, % = 5.5, so the median is the average of the 5th and 6th ordered data
values.
The ordered data set is: 2:9—31+—3738 3.9 3.9 40454754
5th value + 6th value ~ 3.9+43.9
median = 3.9

Number of cats per house


Do
o

10

0 —_— — >
0 1 2 3 4 5 number of cats
526 Chapter 12 (Statistics) Review set 12A

b The data is positively skewed.


C i Looking down the frequency column, the highest frequency is 36. This corresponds to
0 cats, so the mode is O cats.

ii T= >zt
> f
55
T 63
~ 0.873 cats
n+1
iii There are 63 data values, so n = 63. = 32, so the median is the 32nd ordered
data value.
From the cumulative frequency column, the 1st to 36th ordered data values are O cats.
the 32nd ordered data value is O cats.
median = 0 cats
d The mean is the most appropriate measure of centre for this data as it does at least suggest that
some people have cats, whereas the mode and median are both 0 which suggests that no one
has any cats.

Girls Boys
equency 4 frequency
IO

5
OFNWE

] R 4

3
- 2 N

by 32 33 34 35 36 37 38 39 40 41
- ;oy r
32 33 34 35 36 37 38 39 40 41
-
time (s) time (s)

We first organise the data into tables:


Girls:

Frequency (f) | Midpoint (x) | Product (xf) | Cumulative frequency


32.5 - 33.5 1 33 33 1
33.5 - 34.5 3
34.5 - 35.5 5
35.5 - 36.5 4
36.5 -37.5 4
37.5 - 38.5 1
38.5 -39.5 1
39.5 - 40.5 0
40.5 -41.5 1
Total S f=20 Saf="720
Chapter 12 (Statistics) Review set 12A 527

Boys:

Time (s) | Frequency (f) | Midpoint (x) | Product (zf) | Cumulative frequency
315 - 32.5 1 32 32 1
32.5 - 335 4 33 132 5
33.5 - 34.5 5 34 170 10
34.5 - 35.5 6 35 210 16
35.5 - 36.5 3 36 108 19
36.5 - 37.5 1 37 37 20
Tl | X f=20 Y af =689 |

a There are 20 data values for each data set, so n = 20. = il = 10.5, so the median is the
average of the 10th and 11th ordered data values.
From the cumulative frequency column for the girls’ data set, the 10th to 13th ordered data
values are 36 s.
the 10th and 11th ordered data values are both 36 s.
36 + 36
median =

=36s
From the cumulative frequency column for the boys’ data set, the 6th to 10th data values are
34 s and the 11th to 16th data values are 35 s.
the 10th ordered data value is 34 s and the 11th ordered data value is 35 s.
. 34435
. median =

=345s

The mean of the girls’ data setis T = L=


>f
20
-

20
6 s
o
Il

The mean of the boys’ data setis T = >ef


> f
689
0
=34.45s
Looking at the histograms, the highest column for the girls’ data is a frequency of 5 which
corresponds to the interval 34.5 - 35.5 s. So the modal class is 34.5 - 35.5 s.
Similarly, the highest column for the boys” data is a frequency of 6 which corresponds to the
interval 34.5 - 35.5 s. So the modal class is 34.5 - 35.5 s.
So, the table is:

median 36 s 34.5 s
mean 36 s 34.45's
modal class | 34.5-35.5s | 34.5-35.5s
528 Chapter 12 (Statistics) Review set 12A

b The girls’ distribution is positively skewed and the boys’ distribution is approximately
symmetrical. The median and mean swim times for boys are both about 1.5 seconds lower than
for girls. Despite this, the distributions have the same modal class because of the skewness in
the girls’ distribution.
The analysis supports the conjecture that boys generally swim faster than girls with less spread
of times.

L If the mode is 6, then one of the unknown numbers must be 6.


Suppose the other unknown number is x.

%66*3”“” =6 {since mean = 6}


28+ =36
r=28
Since a > b, then a =8 and b=6.

(k—2)+k+ (E+3)+(k+3)
5 a mean=
4
_ dk+4
ol A
_Ak+D)
T4
=k+1
b If each number in the data set is increased by 2, then the data set becomes
k, k+2, k+5, k+5.
k+(k+2)+ (k+5)+ (k+5)
new mean —
4
4k + 12
4
4(k
+ 3)
4
=k+3

6 a We do not know each individual data value, only the intervals they fall in, so we cannot calculate
the mean winning margin exactly.

b | Margin (points) | Frequency (f) | Midpoint (z) | Product (xf)


13 5.5

the mean winning margin is about 22.6 points.


Chapter 12 (Statistics) Review set 12A 529

Guinea Pig | Mass (g) at birth | Mass (g) at 2 weeks

75+ 70 4 80 + 70 + 74 + 60 + 55 + 83
a mean birth mass =
8
_ 518
~709¢g
210 4+ 200 + 200 + 220 + 215 + 200 + 206 + 230
b mean mass after 2 weeks =
8
1681
T8
~210g
. 1681
¢ mean increase over the 2 weeks = %8 - %

1114
8
~139¢g

a The ordered data set is:


37810 11 13 14 14 14 15 15 16 18 18 19 19 19 22 28 31 (n =20)
| | | | |
min = 3 Q=12 median = 15 Q3 =19 max = 31

minimum = 3 Q=12
So the five-number summary is: median = 15 Q3 =19
maximum = 31

b range = maximum — minimum


=31-3
=28

IQR =Q3 - Q:
=19-12
=7
< —|

ol b b e b Ly
0 5 10 15 20 25 30 35
530 Chapter 12 (Statistics) Review set 12A

n+1
9 a Since n =20, = 10.5, so the median is the average of the 10th and 11th ordered data
values.
The ordered data set is:
-81-84-96-—95-98—98—99—100—16%+ 101 102 103—164—104—105—106—106—107108—112
(20 data values)
10th value + 11th value
. median =
2
1014102
o 2
=101.5

b We have an even number of data values, so we include all data values when we split the data
set into two:
lower half upper half
—_—m—m—m—_—_—_—_—m_m<m_l—m—AoA AL, e—————————————————
81 84 90 95 98 98 99 100 101 101 102 103 104 104 105 106 106 107 108 112
98 +98
Q1 = median of lower half = 98

Q3 = median of upper half = 005, 10078 105.5

IQR=Q3 - Qy
=105.5—-98
=75
90 + 106 + 84 + ... + 102 4+ 98 + 101
¢ mean= —m—o0—o0
00—
20
2004
20
=100.2
d Using technology:

The standard deviation is ~ 7.59.

I
po—
T +— -l b bt 1o
11 12 13 14
time (seconds)

Reading from the box plot, the five-number summaries are:


A: min=11s, Q; =11.6s, median=12s, Q3 =12.6's, max = 13 s
B: min=11.2s, Q; =125, median=12.6s, Q3 =13.2s, max = 13.8 s
Chapter 12 (Statistics) Review set 12A 531

b A: range =13 —-11 IQR=Q3 — Q;


=25 =126 -11.6
=1s

B: range = 13.8 — 11.2 IQR=Q3 —Q;


=26s =132-12
=12s

i The members of squad A generally ran faster because their median time is lower.
ii The times in squad B are more varied because their range and IQR are higher.

1 80

60

40

20

0
50 53 55 58.5 60 61.5 65

The median is the 50th percentile. As 50% of 80 is 40, we start with the cumulative frequency
40 and find the corresponding time.
The median ~ 58.5 s.

Q; is the 25th percentile. As 25% of 80 is 20, we start with the cumulative frequency 20 and
find the corresponding time.
Q1 ~55.5s
Qs is the 75th percentile. As 75% of 80 is 60, we start with the cumulative frequency 60 and
find the corresponding time.
Q3 ~61.5s

IQR=Q3 —Q:
~61.58—55.5s
~6s

As 10% of 80 is 8, we start with the cumulative frequency 8 and find the corresponding time.
The top 10% of runners took less than approximately 53 s.
532 Chapter 12 (Statistics) Review set 12A

12
200

180

160

140

120

100

80

60

40

20

00 5 10 15 20 25 30 35 40

a Approximately 20 students took 10 minutes or less to travel to school by bus.


Approximately 108 students took 20 minutes or less to travel to school by bus.
approximately 108 — 20 = 88 students took between 10 and 20 minutes to travel to
school by bus.
b As 30% of 200 is 60, we start with the cumulative frequency 200 — 60 = 140 and find the
corresponding time.
Approximately 30% of the students spent more than 24 minutes travelling to school.
m =~ 24
¢ From the cumulative frequency graph we can obtain the cumulative frequency table:

Time (t min) | Cumulative frequency


5<t<10
10<t<15
15 <t <20
20Kt <25
25 <t <30
30 <t <35
35 <t <40

t<10 ~20—-0~20
10<t<15 ~ 60 — 20 ~ 40
15<t<20 | ~108—60~48
20 <t <25 | =150 — 108 ~ 42
25 <t <30 | =178 — 150 ~ 28
30<t<35 | =195 -178 = 17
35 <t <40 | =200—-195~5
Chapter 12 (Statistics) Review set 12A 533

13 a Using technology:
ot Rad Forn)
1-Variable ox2
21.545454 62.97520661
g [

The population standard deviation The population variance o2 = 63.0.


o794,
b Using technology:
i ad et
0.96859375

The population standard deviation The population variance o2 = 0.969.


0 ~0.984.

Class interval | Mid-interval value | Frequency


<L<20 5
/

13
NN

L <30 17
L <35 29
CIN

L <40 27
NN

L <45 18
L <50 7
IN

b Using technology:

1-Varia

The mean ~ 33.6 L. The standard deviation ~ 7.63 L.

15 a Extreme values will have less effect on the standard deviation of a larger population than on a
smaller population.
no, you would not expect the standard deviation for the whole population to be the same
for one day as it is for one week.
b i The mean would be used to check that an average of 250 g of biscuits goes into each
packet.
ii The standard deviation would be used to check the variability of the mass going into each
packet.
534 Chapter 12 (Statistics) Review set 12B

¢ A low standard deviation means that the weight of biscuits in each packet is, on average, close
to 250 g.

REVIEW SET 12B )

1 a WeekI:
16.4 +15.2+16.3 4+ 16.3 + 17.1 + 15.5 4 14.9
mean = Z
1117
T
~ 16.0 s

As n=1, n;rl =4

The ordered data set is: +49—152—155 16.3 163—164—17t+


f
4th value
median = 16.3 s

Week 2:
149+ 15.7+15.1 4+ 15.1 + 14.7 + 14.7 4+ 15.3
mean = -
1055
7
~15.1s
1
As n=1, ntl_ 4

The ordered data set is: +H7—H7—1+£9 15.1 45+—153—15-7F


f
4th value
median = 15.1 s

Week 3:
14.34+14.2 +14.6 + 14.6 + 14.3 + 14.3 + 14.4
mean = -
1007
T
~14.4s

As n=1, el =4
2

The ordered data set is: +42—+43—1+4£3 14.3 446146

4th value
median = 14.3 s
Chapter 12 (Statistics) Review set 12B 535

Week 4:
14.0+414.0+13.9+14.04+ 14.1 +13.8 + 14.2
mean =
7
_% 7
=14.0s

As n=71, "T“ =4
The ordered data set is: 13
8—13:9—140- 14.0 H40—H1—142
!
median = 14.0 s e

Yes, Heike’s mean and median times have gradually decreased each week which indicates that
her speed has improved over the 4 week period.

The mode is 5 as this is the data value which occurred most frequently.

Value (x) | Frequency (f) | Product (zf) | Cumulative frequency

There are 50 data values, so n = 50. Aaal = 25.5, so the median is the average of the 25th
and 26th ordered data values.
From the cumulative frequency column, the 18th to 25th ordered data values are 3 and the 26th
to 30th ordered data values are 4.
the 25th ordered data value is 3 and the 26th ordered data value is 4.

-, median = 3+4
2
=35

2 3 6 3
5 2 10 &

7=
>f
57— 5x + 22
10
57 =bx + 22
5 =35
536 Chapter 12 (Statistics) Review set 12B

b There are 10 data values, so n = 10. nTH = 5.5, so the median is the average of the 5th
and 6th ordered data values.
From the cumulative frequency column, the 4th and 5th ordered data values are 5 and the 6th
to 9th ordered data values are 7.
the 5th ordered data value is 5 and the 6th ordered data value is 7.
. 5+7
-. median = -

=6

4 If the mode is 7, then one of the unknown numbers must be 7 as there are currently an equal number
of 6s and 7s in the list.
Suppose the other unknown number is .
6+8+7+7+5+7+6+84+6+9+6+7+7+x
= =7 {since mean = 7}
89+
4
89 +x =98
=9
p=7ad ¢=9, or p=9 and ¢=17.

250- 299
300 - 349 . 11033
350 - 399 . 25466
400 - 449 . 30564
450 - 499 b 25623
500 - 549 . 12063.5
550 - 599 . 4021.5
Saf=112614

the mean number of patrons per day is about 414.

6 The ordered data set is:


11 12 12 13 14 14 15 15 15 16 17 17 18 (13 data values)
| | | | |
min =11 Q; =125 median = 15 Q3 =16.5 max = 18

minimum = 11 Q=125
The five-number summary is: median = 15 Q3 =16.5
maximum = 18

So, the box and whisker diagram is: '_:D—’

11 12 13 14 15 16 17 18
Chapter 12 (Statistics) Review set 12B 537

Using technology:

The standard deviation is o ~ 11.7.

The ordered data set is:


93 116 118 120 122 127 128 132 135 (9 data values)
| | |
Qi =117 median = 122 Q3 =130

So, Q; =117 and Q3 = 130.


IQR=Q3 — Q1
=130 —-117
=13
lower boundary upper boundary
= lower quartile — 1.5 x IQR = upper quartile +- 1.5 x IQR
=117-15x%x 13 =130+1.5x 13
=975 =149.5

93 is below the lower boundary, so it is an outlier.

* -
“T90 100 110 /120N 130 140
117 122

=30
=87
=88
=89
=90
=91

IQR = Q3 — Q
= 910 — 898
=2

So, the table is:


538 Chapter 12 (Statistics) Review set 12B

- . ! . L . . . . . >
870 880 890 900 910 920 930

c i The median is higher for brand Y than for brand X, so we would expect brand Y to have
more peanuts per jar.
ii The IQR is lower for brand Y than for brand X, so we would expect brand Y to have a
more consistent number of peanuts per jar.

9 120

90

60

30

60 70 80 90

a The median is the 50th percentile. As 50% of 120 is 60, we start with the cumulative
frequency 60 and find the corresponding time.
The median ~ 77 days.
b The lower quartile is the 25th percentile. As 25% of 120 is 30, we start with the cumulative
frequency 30 and find the corresponding time.
Qq =~ 71 days
The upper quartile is the 75th percentile. As 75% of 120 is 90, we start with the cumulative
frequency 90 and find the corresponding time.
Qs ~ 83 days

IQR=0Q3 — Q1
~ 83 — 71 days
~ 12 days
Chapter 12 (Statistics) Review set 12B 539

Scores (x) | Frequency | Cumulative frequency


0<z<10

V/
0< z <20
0 < z < 30
0<
0<
a7 A cumulative frequency
60

50
40
30
20

10
O v Y 5 >

0 5 10 15 20 25 30 35 40 45 50 55
scores

b 1 The median is the 50th percentile. As 50% of 60 is 30, we start with the cumulative
requency 30 and find the corresponding score.
The median ~ 26.
il The lower quartile is the 25th percentile. As 25% of 60 is 15, we start with the cumulative
frequency 15 and find the corresponding score.
Q1 ~20
The upper quartile is the 75th percentile. As 75% of 60 is 45, we start with the cumulative
frequency 45 and find the corresponding score.
Q3
~ 32
IQR=Q3
— Q1
~32—20
~ 12

the mean of the data set is about 26.


540 Chapter 12 (Statistics) Review set 12B

iv Using technology:

7 4 m
8 7 13
9 P 25
10 5 30

a 24+4=m and 134+p=25


m =6 cop=12

b The highest frequency is p = 12. This corresponds to a score of 9, so the mode is 9.


There are 30 data values, so n = 30. nTH = 15.5, so the median is the average of the 15th
and 16th ordered data values.
From the cumulative frequency column, the 14th to 25th ordered data values are 9.
the 15th and 16th ordered data values are 9.

median Kt
=

=9
range = maximum — minimum
=10-6
=4
_ _ sum of all data values
¢ Themean T= ——8——
number of data values

_ X
> f
= % {there are 30 data values}
127
15
Chapter 12 (Statistics) Review set 12B 541

Completion time (t min) | Number of players | Cumulative frequency


1

/
0< 5
0< 17
0< 35
0 < 42
0< 44

a There were 44 players surveyed.


b The modal class is 90 < ¢ < 120 min as this is the completion time which occurred most
often.

B 504 cunulative frequency


40

30 |17

0 Y v {mn)
e
0 30 60 90 120 150 180 210

d i The median is the 50th percentile. As 50% of 44 is 22, we start with the cumulative
requency 22 and find the corresponding time.
The median ~ 98 min.

ii | Completion time | Frequency (f) | Midpoint (z) | Product (xf)


0<
30 <
60 <
90 <
120 <
150 <
Total

7 5
s zf

_ 426044
~ 96.8 min
iii The game is considered too easy if either the mean or median completion time is below
90 minutes. Since both the median and mean are both above 90 minutes, then the game
is not considered to be too easy.
542 Chapter 12 (Statistics) Review set 12B

e The lower quartile is the 25th percentile. As 25% of 44 is 11, we start with the cumulative
frequency 11 and find the corresponding time.
Q; ~ 78 min
The upper quartile is the 75th percentile. As 75% of 44 is 33, we start with the cumulative
frequency 33 and find the corresponding time.
Q3 ~ 116 min
The middle 50% of players completed the game in times between 78 and 116 minutes.

13 Number | 47 | 48 | 49 | 50 | 51

a Using technology:
Redfornd) (JFc)Rea)
1-Variable
i =49.5681818
Zx =8724
Ix2 =432882
oX 1.59755107
SX 1.60210899
n 176

The mean number of matches in a box p ~ 49.6 matches, and the standard deviation
o =~ 1.60 matches.

b Yes, this result does justify the claim that the average number of matches per box is 50, because
the mean o ~ 50 matches.

14 | Class interval Mid-interval value | Frequency


140 < b < 160 150 27
160 < b < 180 170 32
180 < b < 200 190 48
200 < b < 220 210 25
220 < b <240 230 37
240 < b < 260 250 21
260 < b < 280 270 18
280 < b < 300 290

a Using technology: b Using technology:


Redforn]) (d7c]Real a Radfornd) (d7c]Real
1-Variable 1-Variable
X =207.023255 X =207.023255
Ix =44510 Ix =44510
xx2 9.5383E+06 zx2 .5383E+06
cX 38.8015154 ox 8.8015154
SX 38.8920675 SX .8920675
n 215

The mean ~ €207.02. The standard deviation ~ €38.80.


Chapter 12 (Statistics) Review set 12B 543

15 Kevin:
The mean time T taken by Kevin to complete a crossword puzzle
37+53+47+334+39+
... 39 +41
20
824
20
= 41.2 minutes

Felicity:
The mean time T taken by Felicity to complete a crossword puzzle
_ 33+36+41+26+52+
... +50+ 31
20
790
T 20
= 39.5 minutes

Using technology:
Kevin: Felicity:

The population standard deviation The population standard deviation


o ~ 7.61 minutes. o ~ 9.22 minutes.
Felicity’s mean time is lower than Kevin’s, so Felicity generally solves crossword puzzles faster.
Kevin’s population standard deviation is lower than Felicity’s, so Kevin is more consistent in
his time taken to solve the puzzles.

You might also like